You are on page 1of 361

Casos

Clnicos en Ginecologa
y Obstetricia
__________________





Dr. Fernando San Martin

Internos : Camila Antivilo.
Diego Gonzlez.
Ignacia Hinojosa.
Ral Mera.
Roman Minieri.
Paulina Sandoval.
Alfredo Sandoval.
Catalina Verdugo.

2015

Temas Obstetricia .
TEMA PAGINA

1. Embarazo Fisiolgico. 5
2. Hemorragia primer trimestre (Amenaza de aborto) 11
3. Hemorragia primer trimestre (Aborto incompleto ) 15
4. Hemorragia primer trimestre (Aborto Retenido) 20
5. Hemorragia primer trimestre (Mola) 26
6. Hemorragia del primer trimestre (Embarazo ectpico) 33
7. Hemorragia del 2 trimestre ( Aborto ) 40
8. Embarazo patolgico (Hipermesis gravdica) 44
9. Infecciones de embarazo (Rubeola) 48
10. Infecciones en el embarazo (estreptococo grupo B ) 51
11. Embarazo patolgico (Incompetencia cervical) 54
12. Embarazo patolgico (Anemia) 57
13. Embarazo patolgico (Apendicitis Ag ) 62
14. Embarazo patolgico (Pielonefritis Ag ) 65
15. Embarazo patolgico (Hipertension arterial Cronica) 73
16. Embarazo patolgico (Preclampsia) 76
17. Embarazo patolgico (Eclampsia) 81
18. Embarazo patolgico (Sndrome de HELLP) 86
19. Embarazo patolgico (Diabetes Pre gestacional) 92
20. Embarazo patolgico (Diabetes gestacional) 99
21. Embarazo Patolgico (Enfermedad Hemoltica Perinatal) 106
22. Embarazo Patolgico (Embarazo gemelar ) 114
23.Embarazo Patolgico (Sntomas de parto prematuro) 118
24. Embarazo Patolgico (Rotura prematura de membranas) 127
25. Embarazo Patolgico(Corioamnionitis) 134
26. Embarazo Patolgico (Oligohidroamnios) 138
27. Embarazo Patolgico (Polihidroamnios) 144

2
28. Embarazo Patolgico (Placenta Previa ) 146
29. Embarazo Patolgico (DPPNI) 150
30. Embarazo Patolgico (RCIU) 154
31. Parto Patolgico (Cesarea) 161
32. Parto patolgico (Induccin de parto) 165
33. Parto patolgico (Retencion de Hombros) 172
34. Puerperio fisiolgico 177
35. Puerperio patolgico (Hemorragia post parto) 181
36. Puerperio patolgico (Restos placentarios) 189
37. Puerperio patolgico (Mastitis) 192
38. Anticoncepcin post parto 195


Temas Ginecologa
______________________________________________________________________
TEMA PAGINA

42. Patologa vulvar (Vaginosis) 200


43. Flujo vaginal (Candida ) 203
44. Flujo vaginal (Tricomonas) 206
45. Infecciones ginecolgicas (Herpes) 207
46. Infecciones ginecolgicas ( VPH ) 213
47. Sndrome premenstrual 217
48. Quiste ovrico (Funcional) 222
49. Dolor plvico (Dismenorrea primaria) 225
50. Dolor plvico (Dismenorrea por Diu) 231
51. Dolor Plvico Crnico (Endometriosis) 235
52. Dolor plvico agudo (Quiste ovrico torcido) 243
53. Amenorrea (SOP) 247
54. Amenorrea (hper prolactinemia ) 251
55. Mioma 253
56. Dolor plvico agudo (PIP) 257
57. Amenorrea (Primaria) 261

3
58. Amenorrea ( SOP ) 269
59. Amenorrea (HPOTALAMICA) 278
58. Pareja infrtil (Estudio bsico) 284
59. Menopausia Sintomtica 294
60. Sangrado uterino anormal 298
61. Sangrado uterino anormal (SUA O ) 308
62. Sangrado uterino anormal (Plipo) 314
63. Hiperplasia endometrial 316
64. Metrorragia post menopausia 320
65. Prolapso Organos Pelvicos 325
66. Incontinencia urinaria de Urgencia. 333
67. Incontinencia urinaria de Esfuerzo. 339
68. Patologa mamaria benigna (Mastodinia) 346
69. Patologa mamaria benigna (MFQ) 350
70. Patologa benigna de mama (Fibroadenoma) 352
71. Cncer de mama 354
72. Cancer de Cuello Uterino 358

1. CONTROL EMBARAZO NORMAL



Mujer de 27 aos que acude a control el da 26 de marzo, refiriendo que su ltima regla
fue el 18 de febrero y que ayer en la tarde se realizo un test de embarazo en orina que
di + .
Antecedentes familiares : madre hta , padre diabtico

4
Antecedentes personales : apendicetoma a los 18 aos
Antecedentes GO : menarquia 12 aos, RM IV/30 , casada desde hace 3 meses y
usaba anillo vaginal hasta hace 2 meses G1P0A0
Hbitos : no fuma no bebe
Profesin : profesora de educacin fsica.
1,60 mts 75 kg

1. Diagnstico: -Primigesta
-Gestacin de 5+1 semanas


Definicin: Se entiende por embarazo el periodo comprendido desde la fecundacin
del ovulo hasta el parto. Su duracin aproximadamente es de 280 das. (40 semanas)

2. Fisiopatologa:
La mantencin del feto in tero, proporcionndole un medio ambiente
favorable, y el apropiado aporte de nutrientes y oxgeno para su desarrollo,
requieren una serie de cambios adaptativos maternos.(1)

Sistema Msculo-Esqueltico
- Hiperlordosis lumbar
- Hiperextensin de la musculatura paravertebral
- Separacin de las ramas pubianas en la snfisis pubiana (produce inflamacin de
la snfisis pubiana).

Piel y Fanreos
-Eritema palmar y araas vasculares
-Estras de distencin
-Hiperpigmentacin: Ocurre hasta en el 90% de las embarazadas. Ocurre
principalmente en
pezones, areola, vulva, regin perianal, lnea parda umbilical y el melasma (en la
mejilla).

Sistema Endocrino:
a. Hipfisis: aumento de volumen y produccin de lacttropos, encargados de la
produccin de prolactina.
b. Tiroides: los sntomas del embarazo pueden simular un hipo o hipertiroidismo.
La paciente se mantiene eutiroidea, sin embargo existe aumento de TBG, T3 y
T4. La T4 libre se mantiene en rangos normales. El control debe hacerse
siempre con TSH y T4 libre.
c. Suprarrenal: Existe produccin de ACTH placentaria, que estimula la produccin
de cortisol.
d. Pncreas: En el primer trimestre existe hiperplasia de islotes pancreticos,
aumento de la secrecin de insulina, y aumento de la utilizacin perifrica de

5
glucosa. Esto ocasiona una reduccin de la glicemia de ayuno. En el 2do y 3er
trimestre se produce aumento de la resistencia perifrica a la insulina (en
respuesta al lactgeno placentario). Las mujeres sanas compensan la resistencia
a la insulina mediante niveles mayores de esta hormona.

Sistema Cardio-Circulatorio
- En el examen fsico hay taquicardia sinusal, desplazamiento del choque de la punta
hacia lateral, puede existir soplo sistlico eyectivo precordial. El pulso venoso
yugular puede ser ms visible, pero la onda de presin venosa yugular se mantiene.
- La presin arterial disminuye durante el primer y segundo trimestre y luego
aumenta en el ltimo trimestre.
- Tambin es posible observar edema de extremidades inferiores durante el tercer
trimestre.
- Aumenta el volumen plasmtico y en menor medida la masa eritrocitaria por lo
que se genera una anemia dilucional o tambin llamada fisiolgica del embarazo.
- La resistencia vascular sistmica disminuye debido a una vasodilatacin perifrica
favorecida por la presencia de xido ntrico y prostaglandinas.
- Luego del parto, estos cambios adaptativos vuelven a niveles previos al embarazo,
de manera paulatina, incluso pueden tardar meses.

Sistema Respiratorio

- La mayora de las embarazadas refieren sensacin de falta de aire de manera
progresiva desde el primer trimestre.
- Aumenta la sensibilidad al CO2, lo que estimula el centro respiratorio, lo que
genera un estado de hiperventilacin que conduce a un aumento de la PaO2 y cada
de PaCo2.
- Dentro de los cambios anatmicos se encuentra la elevacin diafragmtica hasta
de 4 cm y el aumento del dimetro de la caja torcica.
- El VEF1 y CVF se mantienen estables.

Sistema Nefrourolgico

- Hasta el 95% de las embarazadas refiere aumento de la frecuencia miccional,
causa que aparentemente sera multifactorial.
- La nicturia ocurre por el aumento de excrecin de agua y sodio nocturno.
- El edema es debido a retencin de sodio y agua y disminucin de la capacidad de
excrecin la -carga de sodio.
- Aumenta el flujo plasmtico y la filtracin glomerular (clearence de creatinina
mayor a 120). Esto ltimo explicara por qu algunas pacientes presentan glucosuria
(sin ser diabticas) o proteinuria con capacidad de reabsorcin tubular disminuida
(normal hasta 300 mg/24 hrs).

Sistema Digestivo

6
- Tambin en relacin a los cambios en los niveles de progesterona, ocurre una
disminucin de la motilidad del estmago, intestino delgado e intestino grueso.
- Son sntomas digestivos frecuentes: distensin abdominal; reflujo gastroesofgico,
pirosis y constipacin.
- Las mujeres pueden notar el desarrollo de eritema palmar o araas vasculares
durante el embarazo. Estos son por lo general el efecto de los altos niveles de
estrgeno, y se resolvern despus del parto.


Sistema Hematolgico
- El volumen Plasmtico aumenta en un 60% y la mayor parte ocurre antes de las
32- 34 semanas. En el embarazo se considera indispensable esta hipovolemia para
cubrir las necesidades de oxgeno y soportar el aumento de la demanda sangunea
del tero con su sistema vascular hipertrofiado y de baja resistencia
- El riesgo de trombo embolismo en la poblacin general es de 1/10.000 y en
mujeres embarazadas 1/1.000. El periodo de mayor riesgo es el puerperio.(1)

Glndula Mamaria
-Aumento de la irrigacin mamaria
-Aumento del volumen mamario
-Secrecin lctea
-Pigmentacin de la areola.

a. Factores de riesgo:

Sociales:
- Edad materna las edades extremas es un factor de riesgo <16 o >40 aos
- Educacin: NSE bajo es de mayor riesgo
- Dependencia econmica
- Estado civil
- Red de apoyo
- Desnutricin

Biolgicos:
- Historia obsttrica previa: antecedente de parto prematuro, muerte fetal in
tero, confiere uno de los elementos de riesgo mas importantes para una nueva
gestacin, 3 o mas abortos sucesivos, anomalas congnitas, feto de nacimiento
<2500gr o >4500gr , SHE o eclampsia.
- Enfermedades pre-gestacionales
- Enfermedades inducidas por el embarazo

Ambientales:
- Hbitos: drogas, alcohol, tabaco
- Actividad laboral: exposicin a bioqumicos

7
- Contaminacin ambiental: (aguas, alcantarillas, etc.) en ciudades cercanas a
faenas como la minera, se ha evidenciado riego de exposicin a plomo o arsnico.

b. Epidemiologia: El 2011 se presenta una tasa de natalidad de 14,4 por 1.000
Habitantes (cedip2014)

c. Diagnostico clnico:
Determinar edad gestacional:
a. fecha de ltima menstruacin (FUR): siempre que sea segura y confiable (
asumir que a los 14 das ovul y se embaraz ).
La fecha de ultima menstruacin no es confiable en las siguientes situaciones:
primera menstruacin post-parto, post aborto, en los extremos de la vida
reproductiva, consumo de aco en los 3 meses previos, durante la lactancia y mujer
con SOP

b. tamao uterino: altura uterina

c. ecografa: Tamao del saco gestacional (1er trimestre) Visible por eco TV
desde las 4-5 semanas
til para el diagnstico de EG:
A las 5 semanas: 10 mr.
A las 6 semanas: 20 mm.
A las 7-8 semanas: 30 mm.

d. Anamnesis:

a. Fecha de ltima menstruacin(FUR):primer da de la ltima menstruacin.
b. Clculo de Edad Gestacional (EG)
c. Fecha probable de parto (FPP) : FUR + 7 DIAS MENOS 3 MESES
d. Sntomas gestacionales normales: nuseas vmitos, cansancio, sueo,
sensacin de mareo, distensin abdominal, dolor pelviano, congestin mamaria,
antojos, reflujo Gastro-esofgico.
e. Sntomas patolgicos: sangrado genital, dolor pelviano intenso, flujo genital que
produce mal olor, prurito genital.
f. Inicio de control prenatal y exmenes prenatales (averiguar si ya inici control y
si trae exmenes de rutina).

e. Examen fsico:
Examen fsico del primer control:
a. Examen fsico general: especial nfasis en PA, peso, talla, clculo ndice
peso/talla (medir en todos los controles).
Hiperpigmentacin:
Examen ginecolgico del primer control: examen mamario, espculoscopia (PAP si
procede), tacto vaginal.

8

b. Examen Obsttrico:
Examen del abdomen de la mujer embarazada.
Antes de las 12 semanas: el tero an se encuentra intraplvico, por lo que el
examen abdominal de este periodo es similar al de una mujer no embarazada.
Entre 12-20 semanas: recin a las 12 semanas el tero est a nivel de la snfisis
pbica.
Palpacin
12 semanas: supra pbico (no confundir con vejiga).
16 semanas: entre pubis y ombligo.
20 semanas: umbilical.

Auscultacin LCF: se pueden auscultar con equipo de ultrasonido(12 sem), y con
Pinard(20sem).

Despus de las 20 semanas:
1. Maniobras de Leopold (desde las 28 semanas)
2. Auscultacin LCF (ultrasonido y Pinard):normal 120-160lpm
3. Altura uterina (AU): Crecimiento fetal.
4. Estimacin peso fetal (EPF): clnicamente, 10% de error en manos entrenadas

f. Estudio de la paciente
a. Hemograma Evidencia si paciente est cursando con anemia
b. Grupo-Rh y Coombs indirecto Identifica a las pacientes con grupo sanguneo
Rh negativo y permite establecer su estado de sensibilizacin.
c. VDRL RPR portadoras de sfilis
d. VIH. Acceso universal del test de deteccin de VIH a las gestantes sin
diagnstico conocido de VIH en el primer control prenatal. Si el resultado del
examen es negativo, se debe repetir entre las 32 y 34 semanas de gestacin en
aquellas mujeres que tengan mayor riesgo de adquirir VIH. Se debe realizar
consejera abreviada pre y post test con nfasis en el diagnstico y tratamiento.
e. Glicemia Diagnstico de Diabetes Gestacional
f. Sedimento urinario y urocultivoPermite la deteccin de pacientes portadoras
de infecciones urinarias, y en especial de bacteriuria asintomtica
g. TSH ( Valor normal < 2.5 en embarazo )
h. Citologa de cuello uterino (Papanicolaou)De gran importancia para el
diagnstico y manejo de las lesiones pre-malignas y malignas del cuello del
tero. Se debe realizar en el primer control antenatal si no cuenta con examen
segn norma vigente..
i. Ecografapermite diagnosticar o confirmar la edad gestacional, el nmero de
fetos y su viabilidad. Solicitar primera ecografa a las 6-7 semanas de amenorrea
( si no hay dolor ni sangrado uterino )

g. Tratamiento:

9
a. La dieta balanceada
b. Se recomienda que la mujer tome un suplemento de acido flico (0,4-0,8 mg por
da) 3 meses antes y 12 semanas despus de la concepcin para prevenir el
riesgo de defectos del tubo neural. Si ha tenido hijos previos con estos defectos,
la dosis debe ser 1 mg da
c. Evitar el consumo de carnes, pescados, aves y huevos sin cocer para evitar el
contagio de enfermedades tales como: listeriosis, brucelosis y toxoplasmosis .
d. En mujeres sin anemia, se recomienda un consumo de fierro de alrededor de 15
a 30 mg al da

h. Riesgos para la paciente y el feto:

Morbimortalidad perinatal
a. Prematurez: primera causa de mortalidad perinatal.
b. Bajo peso al nacer Asfixia perinatal
c. Restriccin del crecimiento intrauterino (RCIU): peso del recin nacido bajo el
percentil 10 en relacin a su edad gestacional.
d. Malformaciones congnitas Infeccin perinatal
e. Dao neurolgico
f. Muerte fetal
g. Muerte neonatal
h. Muerte infantil: en el primer ao de vida

Morbimortalidad materna
a. Sndrome hipertensivo del embarazo (SHE):
b. Enfermedades maternas pre-gestacionales
c. Diabetes gestacional
d. Infeccin peri parto Cicatriz de cesrea
e. Muerte materna

Referencias
1. Manual Obstetricia y Ginecologa Dr. Jorge Carvajal 4ta Ed. PUC

2. Common symptoms during Pregnancy. Sarah Germain. Obstetrics, gynecology
and reproductive medicine 21:11
3.Gua Perinatal 2014. MINSAL

2. HEMORRAGIA PRIMER TRIMESTRE


(AMENAZA O SINTOMA DE ABORTO)

Paciente de 33 aos cursando embarazo de 8 semanas que consulta por dolor
hipogstrico y sangrado vaginal escaso desde hace 5 horas

10

Antecedentes familiares: Madre Hipertensa y padre Ca de prstata
Antecedentes personales: alergia a penicilina , IMC 32
Historia obsttrica: menarquia 11 ao , RM V/30 G2P2A0 ultimo parto 12 aos
PTVE peso Max 3800 grs .Embarazo actual sin control
Hbitos: cigarro 10 al da, alcohol social

1. Diagnostico Multpara de 2
Embarazo de 8 semanas
Metrorragia del primer trimestre (amenaza de aborto)

2. Diagnostico diferencial: Aborto
Embarazo ectpico
Enfermedad trofoblastica
Causas ginecolgicas (plipos, cervicitis, neoplasia)

3. Definicin:

- Aborto: Interrupcin espontanea o inducida del embarazo antes de la viabilidad
fetal. Se puede considerar viable un feto desde dos puntos de vista:
a. Anatmico, a las 24 semanas que es el momento donde el pulmn es apto para
realizar su funcin
b. Cronolgico, que se considera hasta las 20 semanas de gestacin o 22 sem de
amenorrea o un peso fetal de 500 grs ( OMS )

4. Clasificacin

Amenaza de aborto:
o Presencia de contracciones uterinas dolorosas y frecuentes. A la
espculoscopia no se constata sangrado, en el tacto vaginal no se aprecian
modificaciones cervicales y el tero es de tamao acorde al tiempo de
amenorrea.

Sntomas de aborto:

o Se considera a las contracciones uterinas asociadas a sangrado vaginal
escaso a moderado, a la espculoscopia se descartan lesiones del cuello y
se aprecia la salida de sangre por el OCE, pero sin modificaciones
cervicales.


Trabajo de aborto o Aborto en evolucin

11
Embrin sin LCF con contracciones uterinas o modificaciones cervicales o
sangrado por canal cervical

Aborto Incompleto

Eliminacin de partes ovulares en forma parcial , an quedan restos
ovulares en cavidad uterina .
Al examen fsico podramos observar a la espculoscopia un cuello dilatado
con salida de tejido trofoblastico por canal cervical y al tacto un tero cuyo
tamao es menor al que le corresponde para para la edad gestacional.
En la ecografa se observa un endometrio engrosado mayor a 15 mm o de
aspecto irregular.

Aborto Completo

Eliminacin de completa del trofoblasto y partes fetales
Al examen fsico podramos observar a la espculoscopia un cuello cerrado
o poco dilatado con salida de escaso sangrado ( o sin sangrado ) por canal
cervical y al tacto un tero cuyo tamao es de tamao normal o mucho
menor al que le corresponde para para la edad gestacional.
En la ecografa se observa un endometrio menor a 15 mm de aspecto
regular.


Aborto Provocado

Hay una accin por parte de la paciente o un tercero para abortar.

Aborto Espontaneo

No Hay una accin por parte de la paciente o un tercero para abortar.

Aborto sptico

Hay infeccin intrauterina en paciente con aborto incompleto ,puede ser
provocado o espontaneo

Aborto retenido

Paciente asintomtica con Embrin sin LCF .No hay sangrado ni
modificaciones cervicales .Al examen fsico no auscultamos LCF , cuello
cerrado y tero de tamao normal para EG o menor a la EG que le
corresponde por amenorrea. Habitualmente es un hallazgo ecogrfico

12
Aborto Inevitable

Es la existencia de un embrin vivo intrauterino con una o mas de las
siguientes condiciones :

Rotura ovular.
Dilatacin cervical
Hemorragia que compromete hemodinamia materna
Infeccin ovular


5. Etiologa:

Desprendimiento placentario
Coagulo retro placentario
Insuficiencia ltea

6. Epidemiologia:

El 20% de los embarazos tendr amenaza o sntomas de aborto.

7. Diagnostico clnico:

El diagnstico es clnico y se confirma por ecografa.
Puede existir una hemorragia y/o dolor abdominal tipo clico. El dolor puede
manifestarse como una lumbalgia persistente, asociada a una sensacin de presin
plvica, o como un dolor sordo en la lnea media por encima del pubis.
Tamao del tero adecuado sin modificaciones cervicales.
Es requisito que exista LCF (+)

8. Anamnesis:
Es necesario realizar una buena anamnesis preguntando dirigidamente sobre las
caractersticas del sagrado (cuanta, color, frecuencia, sntomas asociados), adems
de preguntar por dolor y/o contracciones uterinas.
Adems preguntar por traumas o relaciones sexuales previo al sangrado ya que es
una causa importante de sangrado en una mujer embarazada.

9. Examen fsico:
Efectuar una evaluacin de signos vitales de la paciente, en sentido de verificar
hemodinamia.
Segundo, verificar LCF, mediante auscultacin de latidos y medicin de altura
uterina
Tercero verificar procedencia del sangrado, mediante la espculoscopia,
diferenciando bien procedencia del sangrado

13
Cuarto, realizar tacto vaginal, buscando modificaciones cervicales.

10. Estudio de la paciente:

Ecografa transvaginal para visualizar gestacin viva intra uterina.

11. Tratamiento y conducta:

Una vez realizada la eco es necesario tranquilizar a la paciente y explicar los
hallazgos y su pronostico
- nica indicacin es el reposo y un control clnico y ecogrfico a la semana
- Antiespasmdicos sin utilidad, porque no tienen ningn efecto sobre la
evolucin de la enfermedad porque no tratan la causa, si hay dolor indicar
paracetamol.
- No es necesario hospitalizar, pero si es recomendable el reposo a domicilio.
- Abstinencia sexual
- Control ecogrfico en 15 das

Bibliografa: Alfredo Prez Snchez, obstetricia, cuarta edicin, pagina 530 539,
editorial mediterrneo






3. HEMORRAGIA PRIMER TRIMESTRE : ABORTO


INCOMPLETO.

Paciente de 35 aos con amenorrea de 7 semanas que consulta por dolor plvico
hipogstrico tipo clico moderado a severo en hipogastrio y sangrado vaginal con
cogulos escasos y tejidos desde hace 6 horas.
Antecedentes familiares: madre con miomas uterinos
Antecedentes personales: antecedente de PIP y IMC 32
Hbitos:
- 10 cigarrillos diarios
- alcohol social
Historia obsttrica
- Menarquia a los 12 aos

14
- RM V/30-60
- G2 P1 A1
- Ultimo parto hace 12 aos PTVE (3.500 grs)
- Aborto espontaneo de 8 semanas hace 4 aos
- MAC: no utiliza

1. Diagnostico mas probable:

Multpara de 2.
Probable embarazo de 8 semanas
Metrorragia primer trimestre ( Aborto en evolucin o trabajo de aborto )
PIP antiguo
Tabquica
Obesidad

2. Diagnsticos diferenciales:

Aborto en evolucin (incompleto, completo, provocado )
Embarazo ectpico
Sangrado uterino anormal
Embarazo molar
Anormalidades cervicales (trauma, plipo, malignidad)

Trauma vaginal

3. Definicin:

Se define aborto a la perdida del producto de la concepcin antes de la viabilidad
fetal, es decir < 22 semanas o < 500 grs o LCN < 25 cm.

4. Etiologa:

Anomalas genticas (trisomas 52 % ) 50-60%
Pat. Endocrina materna 10-15%
Incompetencia cervical 8-15%
Placentacin anormal 5-15%
Separacin coro-amnitica 5-10%
Infecciones 3-5%
Alteracin anatmica uterina 3-5%
Trauma


Causas Fetales y Ovulares

15

Anomalas cromosmicas
Anomalas genticas
Alteracin desarrollo embrin y placenta ( cardiopatas , desprendimientos)
Rotura ovular

Causas Maternas

Enfermedades Crnicas ( Lupus, diabetes, hipotiroidismo )
Causas uterinas ( malformaciones , miomas , incompetencia cervical,
sinquias )
Inmunolgicas ( Lupus ,sndrome antifosfolipdico ,incompatibilidad grupo )
Trombofilias
Endocrinas ( hipotiroidismo, diabetes , insuficiencia Ltea )
Infecciones ( clamidia , sfilis. Listeria , toxoplasmosis , rubeola, parotiditis )
Trauma
Iatrognica ( amniocentesis , biopsia vellosidades coriales , cirugas ,
radiacin)

5. Factores de riesgo:

Edad materna (principal factor de riesgo para aborto espontaneo, ya que a
mayor edad es mayor el riego de anomalas genticas)
Nmero de abortos previos (principal factor de riesgo)
Tabaco, alcohol, cafena, drogas
AINES
Peso materno (IMC < 18,5 o >25)
Malformacin tero-cervicales
Estrs

6. Epidemiologa:

20-25% de los embarazos clnicos abortan
50% de todos los embarazos abortan
80% de los abortos ocurren < 12 semanas aborto temprano
12 % de los abortos ocurren entre las 12-22 semanas aborto tardo

7. Clasificacin del Aborto

Amenaza de aborto:
o Presencia de contracciones uterinas dolorosas y frecuentes. A la
espculoscopia no se constata sangrado, en el tacto vaginal no se aprecian

16
modificaciones cervicales y el tero es de tamao acorde al tiempo de
amenorrea.

Sntomas de aborto:

o Se considera a las contracciones uterinas asociadas a sangrado vaginal
escaso a moderado, a la espculoscopia se descartan lesiones del cuello y
se aprecia la salida de sangre por el OCE, pero sin modificaciones
cervicales.


Trabajo de aborto o Aborto en evolucin

Embrin sin LCF con contracciones uterinas o modificaciones cervicales o
sangrado por canal cervical

Aborto Incompleto

Eliminacin de partes ovulares en forma parcial , an quedan restos
ovulares en cavidad uterina .
Al examen fsico podramos observar a la espculoscopia un cuello dilatado
con salida de tejido trofoblastica por canal cervical y al tacto un tero cuyo
tamao es menor al que le corresponde para la edad gestacional.
En la ecografa se observa un endometrio engrosado mayor a 15 mm o de
aspecto irregular.

Aborto Completo

Eliminacin de completa del trofoblasto y partes fetales
Al examen fsico podramos observar a la espculoscopia un cuello cerrado
o poco dilatado con salida de escaso sangrado ( o sin sangrado ) por canal
cervical y al tacto un tero cuyo tamao es de tamao normal o mucho
menor al que le corresponde para la edad gestacional.
En la ecografa se observa un endometrio menor a 15 mm de aspecto
regular.


Aborto Provocado

Hay una accin por parte de la paciente o un tercero para abortar.

Aborto Espontaneo

No Hay una accin por parte de la paciente o un tercero para abortar.

17

Aborto sptico

Hay infeccin intrauterina en paciente con aborto incompleto ,puede ser
provocado o espontaneo

Aborto retenido

Paciente asintomtica con Embrin sin LCF .No hay sangrado ni
modificaciones cervicales .Al examen fsico no auscultamos LCF , cuello
cerrado y tero de tamao normal para EG o menor a la EG que le
corresponde por amenorrea. Habitualmente es un hallazgo ecogrfico

Aborto Inevitable

Es la existencia de un embrin vivo intrauterino con una o mas de las
siguientes condiciones :

Rotura ovular.
Dilatacin cervical
Hemorragia que compromete hemodinamia materna
Infeccin ovular

8 Cmo hacer el diagnstico clnico?:

Dolor intenso tipo clico en hipogstrico
Metrorragia abundante
Modificacin de cuello uterino l, con dilatacin de este en que se puede
palpar o visualizar a la espculoscopia , incluso observar restos ovulares en
el canal cervical o vagina .
Altura uterina disminuida para la edad gestacional.

9 Que es importante preguntar en la anamnesis?

Preguntar por factores de riesgo:

Edad materna (mayor riesgo a mayor edad)
Antecedentes de abortos previos
Malformaciones tero-cervicales
Tabaquismo
Uso de medicamentos
Aborto provocado
Fur segura y confiable

18
Uso de MAC

10 Qu buscar dirigidamente al examen fsico?

Espculoscopia : sangrado cervical. modificaciones cervicales. Restos
ovulares
Altura uterina disminuida
Dolor a la palpacin de hipogstrico
Presencia de sangrado por OCE (metrorragia)
Modificaciones cervicales (permeabilidad y dilatacin de cuello uterino)
CSV ( T , Pa y pulso )


11 Cmo estudiar el cuadro? qu resultados espera encontrar en los exmenes?

Eco TV: endometrio engrosado > 15 mm.
Niveles de beta-HCG ( segn EG )

12 Tratamiento:

Evaluar si es una emergencia medica, ya que si hay un sangrado activo,
puede evolucionar a shock hipovolmico, CID y finalmente a la muerte.
Hospitalizar: va venosa perifrica, control signos vitales, hematocrito,
recuento GB, grupo y RH. Analgesia
Se puede intentar uso de Misoprostol 200 600 mcg para intentar evacuar
cavidad uterina. Siempre informar a la paciente que puede requerir legrado
si no se logra evacuacin completa o hay sangrado importante.
Tratamiento definitivo Legrado uterino bajo anestesia
Profilaxis antibitica

13 Posibles consecuencias de esta patologa para la madre y feto:

Muerte materna en caso de emergencia medica: sangrado activo, shock,
compromiso de conciencia, muerte. El shock en estos casos no tarda en
aparecer si no se hace el tratamiento adecuado en forma oportuna.
Infeccin ovular , endometritis.
Histerectoma .
Infertilidad.
Secuelas sicolgicas.



Bibliografa:

19

1. Charles R. B. Beckmann, MD., M.H.P.E. Obstetricia y Ginecologa. Sexta edicin.
Capitulo 13. Embarazo ectpico y aborto. Paginas 141 150.
2. Drs. Jorge Carvajal C., Constanza Ralph T. Manual obstetricia y ginecologa. Cuarta
edicin 2013. Capitulo 43. Paginas 346 446.

4. HEMORRAGIA PRIMER TRIMESTRE : ABORTO


RETENIDO
Mujer de 29 aos cursando embarazo de 9 semanas por amenorrea y ecografa precoz de la
semana 6, que consulta en urgencia por escaso sangrado oscuro genital. Refiere que desde hace
10 das no siente las molestias tpicas del embarazo que estaba sintiendo. Al examen
ginecolgico a Ud le parece que el tamao uterino es menor al esperado para su edad
gestacional.

Antecedentes familiares: no refiere
Antecedentes personales: asma bronquial
Historia obsttrica: menarquia 13 aos, RM IV/30, G2P1A0, ltimo parto hace 12 aos
PTVE no usa MAC
Hbitos: cigarro 3 al da.

1. Diagnsticos
Multpara de 1
Embarazo 9 semanas
Metrorragia del primer trimestre
Obs. Aborto retenido
Asma Bronquial.

2. Diagnstico diferencial:

a. Aborto espontneo
b. Hemorragia subcorinica (desprendimiento ovular)
c. Alteraciones cervicales: Malignidad, plipos, trauma
d. Sangrado idioptico con embarazo viable
e. Vaginosis
f. Cervicitis
g. Trauma vaginal

3. Definicin:

El aborto retenido corresponde a la muerte fetal (feto sin latidos cardiofetales) sin
dinmica uterina ni modificaciones cervicales que permitan su expulsin. Cabe destacar
esta definicin corresponde a una subcategora de aborto espontneo. Otras
subcategoras corresponden a las siguientes:

20
1. Aborto completo: donde se eliminan todos los productos de la concepcin sin
necesidad de manejo mdico ni quirrgico.
2. Aborto incompleto: Slo se eliminan algunos productos de la concepcin y se retiene
otros como placenta, partes fetales o membranas ovulares.
3. Aborto inevitable: crvix dilatado, pero no se han eliminado los productos de la
concepcin.
4. Aborto recurrente: tres o ms abortos consecutivos.
5. Aborto sptico: aborto espontneo que se complica con infeccin intrauterina y
compromiso sptico materno.
6. Amenaza de aborto: embarazo complicado con sangrado antes de 20 semanas de
gestacin.

4. Etiologa: Las causas de aborto espontneo son transversales para todas las subcategoras
y segn frecuencia se describen las siguientes:
1. Anormalidades genticas (49%) trisoma autosmica (52%), poliploida y monosoma X,
alteraciones numricas, traslocaciones, etc.
2. Alteracin endocrina (10-15%)
3. Separacin cori amnitica (5-10%)
4. Incompetencia cervical (8-15%)
5. Infecciones (3-5%)
6. Placentacin anormal (5-15%)
7. Alteracin inmunolgica (3-5%)
8. Malformaciones uterinas (1-3%)
9. Desconocidas (<5%)

Adems, destacan los siguientes factores de riesgo de aborto espontneo:
1. Edad materna avanzada.
2. Consumo de alcohol
3. Anestesia (xido ntrico)
4. Uso de cafena en grandes cantidades
5. Enfermedad materna crnica: diabetes mal controlada, enfermedad celiaca, sndrome
anti fosfolpido
6. Consumo de tabaco
7. Consumo de cocana
8. Concepcin luego de 3 a 6 meses del ltimo parto
9. Dispositivos intrauterinos
10. Infeccin materna: vaginosis bacteriana, mycoplasmosis, herpes simple,
toxoplasmosis, listeriosis, chlamydia, VIH, sfilis, parvovirus B19, malaria, gonorrea,
rubola, CMV.
11. Medicamentos: uso de Misoprostol, retinoides, metotrexato, AINES.
12. Abortos electivos previos
13. Abortos espontneos previos
14. Toxinas: arsnico, etilenglicol, poliuretano, metales pesados, solventes orgnicos
15. Alteraciones uterinas: malformaciones congnitas, adhesiones, leiomiomas.

5. Epidemiologa:

21
20% de los embarazos tendrn algn tipo de sangrado antes de las 20 semanas de
gestacin y slo la mitad de ellos corresponder a un aborto. En la actualidad, se han
realizado estudios de seguimiento con b-HCG y se ha determinado que la tasa de aborto
real corresponde a un 31% de abortos espontneos. El 80% de los abortos ocurren antes
de las 12 semanas (aborto temprano) y el resto entre las 12 y 20 semanas (aborto tardo).


15% 0

1
19%
2
35%
3
47% Abortos previos

Probabilidad de recurrencia del aborto segn


nmero de abortos previos.



6. Diagnstico clnico:

El diagnstico se puede sospechar con la clnica: metrorragia del 1 o 2 trimestre (antes
de las 12 semanas de gestacin para abortos tempranos y entre las 12 y 20 semanas para
aborto tardo) con sangrado oscuro escaso, altura uterina adecuada para la edad
gestacional o ligeramente disminuida y desaparicin de sntomas iniciales de embarazo.
Sin embargo el diagnstico se confirma con ecografa transvaginal cuando es antes de las
12 semanas (sensibilidad 90-100%, Especificidad 80-92%) que muestra saco gestacional
mayor o igual a 25 mm sin saco vitelino presente o la visualizacin de feto con LCN
mayor o igual a 7 mm sin latidos cardiofetales. (Si el saco gestacional es de menos de 25
mm y la LCN es menor a 7 mm, se hace diagnstico de viabilidad incierta, ya que no se
puede confirmar la presencia o no de LCF o saco vitelino y en ese caso, se debe repetir la
ecografa transvaginal).
En paralelo se solicita b-HCG en plasma, esta puede hacerse positiva a los 9 das de
gestacin. Con niveles > 1500-1800 UI la gestacin es visible in tero con ecografa
transvaginal. Con valores mayores a 3500 U se debe visualizar la gestacin con ecografa
abdominal. Se estima un incremento de b-HCG mayor a 60-66% a las 48 horas, si el
incremento es menor o hay un estancamiento, se debe sospechar aborto, pero primero se
debe descartar embarazo ectpico en caso de no ver feto en ecografa o un embarazo
viable con menor edad gestacional.
Cuando se detecta un embarazo no viable o de viabilidad incierta se recomienda realizar
una segunda ecografa en 10-14 das siguientes para confirmar los hallazgos o descartar la
posibilidad de embarazo viable con menor edad gestacional.
La progesterona en plasma cuando es menor a 25 nmol/L indica un embarazo no viable,
pero es de ayuda limitada y no se utiliza para tomar decisiones en el manejo.

22
7. Anamnesis y examen fsico:

Es importante consignar la desaparicin de los sntomas iniciales del embarazo como son
nuseas y vmitos principalmente, con estancamiento o disminucin de la b-HCG. Adems
la paciente podra relatar un sangrado uterino escaso rojo oscuro.
Corresponde realizar una evaluacin hemodinmica de la paciente y temperatura para
descartar compromiso infeccioso. Luego en el examen ginecolgico, consignar la altura
uterina acorde o levemente disminuida, realizar la espculoscopia para objetivar el
sangrado proveniente del orificio cervical externo y observar cuello cerrado sin
modificaciones cervicales.

8. Estudio de la paciente:

Siempre Ecografa transvaginal, b-HCG, es de eleccin y dependiendo de las condiciones
generales de la paciente solicitar progesterona, hemograma, pruebas de coagulacin,
Grupo sanguneo y Rh, cultivo cervicovaginal, para descartar anemia, compromiso
infeccioso o necesidad de Rhogan en caso de incompatibilidad de grupo Rh.

9. Tratamiento y conducta:

Determinacin de Grupo y Rh , Hemograma
Uso de inmunoglobulina anti-D (Rhogan) 50 mcg debe ser administrada si la paciente es
Rh negativo.
Uso de antibiticos debe ser individualizado (no se realiza de rutina).
2 posibilidades de manejo: Se ha demostrado que no hay mejores resultados entre una y
otra tcnica, pero si la eleccin de la paciente de forma informada mejora los resultados
en cuanto a depresin y ansiedad despus del aborto.
1. Expectante: slo con conducta activa en tiempo determinado en caso de no expulsin.
El seguimiento con ecografa se puede extender por 2 a 6 semanas. (xito del 82-96%
sin necesidad de intervencin el promedio en 9 das cuando es un aborto incompleto,
pero slo 16-76% de xito en aborto retenido),Bhcg debe ser negativa.
2. Activo:
1. Mdico: Uso de prostaglandinas va oral o vaginal. Tasa de xito del 80% para
aborto retenido (es la eleccin). Misoprostol 800mcg vaginal (anlogo de
prostaglandina E1), repetir despus de 4 horas de ser necesario (otras fuentes
avalan dosis 1200-1400 mcg vaginal), o uso de Mifepristona 200 mg va oral. Si
no se eliminan los productos de la concepcin en la segunda evaluacin, se
recomienda manejo quirrgico.
2. Quirrgico: legrado aspirativo con anestesia general o local, indicado en
mujeres que lo prefieran, sangrado persistente excesivo, inestabilidad
hemodinmica, evidencia de infeccin, enfermedad trofoblastica. Cuando se
sospecha infeccin se debe diferir en 24 hrs el procedimiento. Algunas
complicaciones (<1-2%) corresponden a perforacin, falsa va, traumatismo
abdominal, evacuacin incompleta, riesgos anestsicos. El tejido obtenido

23
debe enviarse a anatoma patolgica para estudio histolgico y descartar
enfermedad del trofoblasto.

10. Consecuencias para la paciente y el feto:




RAM Misoprostol con altas dosis: nuseas, diarrea, vmitos, fiebre.
La recurrencia de un aborto es variable y depende de las causas y factores de
riesgo de la paciente.
Es importante educar a la paciente y su pareja en que el aborto es una condicin
normal dentro de la seleccin natural para evitar la depresin, culpa, y ansiedad
posterior.
Mortalidad materna 0,7 x 10.000
Siempre se debe enviar a biopsia el trofoblasto para descartar una ETG.

24
EVALUACION ECOGRAFICA.

EMBRION CON LCF REPOSO PROGESTERONA ?

EMBRION SIN LCF VACIAMIENTO UTERINO CON 2


> 5 MM ECOS IGUALES.

> 18 MM HUEVO ANEMBRIONADO


SACO
GESTACIONAL
VACIO REPOSO
USTV < 18 MM REPETIR ECO 14 DIAS

SIN SACO SOSPECHA BHCG SERIADA


GESTACIONAL ECTOPICO REPETIR ECO

SUGERENTE DE ESTUDIO VACIAMIENTO UTERINO


MOLA MOLA







Bibliografa:
1. Craig P. Griebel, MD. et al. Management of Spontaneous Abortion. American Family
Physician. University of Illinois College of Medicine at Peoria, Peoria, Illinois. October
1,2005. Vol 72. Number 7.
2. Shreelata Data, Kumar Kunde, Tom Bourne. Managing early pregnancy loss: diagnosing
miscarriage is not as easy at you might think. Obstetrics, Gynaecology and Reproductive
Medicine. 22:9. Elsevier 2012.
3. Protocol: Manejo de la prdida gestacional temprana. Protocols: Medicina fetal i perinatal,
SERVEI de Medicina Maternofetal, ICGON, Hospital Clinic Barcelona.
4. Harriet Pugsley, Judith Moore. Management of early pregnancy complications. Obstetrics,
Gynaecology and Reproductive Medicine 22:3. Elsevier 2012.

25
5. HEMORRAGIA PRIMER TRIMESTRE (MOLA)
Mujer de 42 aos , cursando embarazo no controlado de 11 semanas por amenorrea
que consulta por sangrado genital abundante , nuseas y vmitos intensos que han
aumentado en forma progresiva, dolor hipogstrico importante ,taquicardia , PA
140/90 y tero sensible de mayor edad que la EG que corresponde.
- Antecedentes familiares: Madre hipertensa.
- Antecedentes personales: no refiere.
- Historia obsttrica : menarquia 13 aos , RM V/38 35 , G3P2A0 2 ptve
- 3200 grs peso mx., uso ACO por 4 aos y luego condn hace 2 aos
- Hbitos :Cigarro 4 al da, alcohol social

1) Diagnstico:
- Multpara de 2
- Embarazo de 11 semanas
- Metrorragia 1 trimestre
- Emabarazo EGD
- Obs ETG
- Tabquica

2) Diagnsticos diferenciales:

- Sangrados Ginecolgicos de origen :
Vulvar
Vaginal
Cuello tero
- Obsttricos:
Aborto
Mioma
Embarazo Multiple
Embarazo de mayor edad gestaciinal
Mola

3) Definicin :
Enfermedad trofoblastica gestacional: Cojunto de enfermedades del embarazo,

malignoi oprogresivo.
relacionadas con proliferacion anormal del trofoblasto y que tienen un potencial
Mola hidatiforme: es un tipo de ETG, en que las
vellosidades coriales normales han sido reemplazadas por mltiples estructuras
qusticas. El embrin y anexos o membranas ovulares pueden estar presentes o no,
dependiendo del tipo de embarazo molar.

26

4) Clasificacin ETG:

Mola hidatidiforme.
Total BENIGNAS

Parcial

Neoplasia Trofoblastica Gestacional.


Mola Invasora
Coriocarcinoma
MALIGNAS
Tumor trofoblastico placentario
Tumor trofoblastico epiteliode.





5) Etiologa:

Mola Completa : (67%) Existe
proliferacin del trofoblasto difusa
a moderada y degeneracin
hidrpica de las vellosidades y stas
carecen de vasos. Hay ausencia de
embrin y genticamente son
diploides 46XX (por fertilizacin de
un huevo vaco por un espermio
23X). En este caso se produce un fenmeno de regresin temprana y completa del
embrin y de los anexos en las primeras etapas del desarrollo (involucin). El juego
cromosmico es exclusivo paterno composicin cromosmica andrognica 85 %
son 46 XX

27

Mola Parcial: Histolgicamente son
similares a la total pero los cambios son
focales y menos intensos. Hay embrin que
suele morir en el primer trimestre aunque
raramente puede llegar al segundo o tercer
trimestre. Su cariotipo es triploide XXX, XXY,
o XYY en que huevo normal es fertilizado
por dispermio y un cromosoma materno .20 % progresan a NTG .

6) Fisiopatologa:

Mola completa: sobrexposicin de oncogenes (bcl-2) y defectos en los genes
supresores de tumores (p53). Se ha visto que hay una marcada sobrexposicin de
receptores de factor de crecimiento epidrmico en el tejido trofoblstico.
Mola completa y mola parcial: sobrexposicin de metaloproteinasas (protenas que
participan en la invasin y metstasis tumorales). Por otro lado hay disminucin en
la concentracin de inhibidores de las metaloproteinasas.

7) Factores de riesgo:
Completa y parcial: historia de aborto e infertilidad
- Completa: Edad (<20 y > 35)
Dieta con poca grasa animal y poco caroteno
- Incompleta: no hay relacin con la edad

8) Epidemiologia:
La incidencia no est muy clara, muchas molas son tratadas solo como abortos y
no son llevadas a biopsia. Si se tiene claro que hay una frecuencia variable
geogrficamente aunque tampoco est claro cul es la razn de esta variabilidad.
Chile aprox 1 mola completa x 1000 embarazos 3 molas paciales x cada 1000
embarazos
Asia 4.5 por cada 1000 embarazo
Riesgo de repetir 1 - 2 % 15 % con 2 molas previas
Coriocarcinoma 1:50.000 partos
Tumor placentario 0. 2 % de las ETG
Mas frecuente en menores de 15 aos y mayores de 45 aos

9) Diagnstico clnico:

Dg: En general es clnico + ECO (60%)
Biopsia: es el dg de certeza, se realiza en 30% de las veces
Legrado: dg solo en un 10%

28

1.- Metrorragia 1 mitad del embarazo: es lo ms frecuente al igual que dolor
hipogastrico
2.- Altura uterina > para la EG , la BhcG es mucho mayor que nibvel que
corresponde a EG.
3.- Quistes tecaluteinicos: tumor anexial bilateral en general de gran tamao (6
12 cm) frecuentemente seroso-sanguinolento. Salen por hiperestimulacin ovrica
debida a la -hcg es auto limitada en semanas a meses. La complicacin es que se
pueden torcer (2%) y se debe realizar ciruga.
4.- Hiperhemesis gravdica: debido a los altos niveles de (no comprobado), tener
cuidado con la deshidratacin y alteraciones de ELP (hipoNa, K, Cl, HCO3)
5.- Preeclampsia de la primera mitad del embarazo
6.- Hipertiroidismo: la -hcg tiene estructura similar a TSH, por lo que puede dar
tormenta tiroidea (usar -bloqueo)
7.- Embolia trofoblastica: muy grave, es lo que puede matar a la mujer.
Radiografa trax: infiltrados pulmonares bilaterales, asociada a hipoxia y alcalosis
respiratoria. Puede ser auto limitada a las 72 horas manejando el cuadro
cardiorrespiratorio, solo se puede hacer apoyo cardioresperiratorio.

La mola parcial tiene los mismos sntomas anteriores, pero en general son mucho
menos sintomticos, incluso en algunos casos se hace solo un hallazgo de la mola.
Para la mola parcial metrorragia sigue siendo lo ms frecuente (72%) despus le
sigue la desproporcin del tamao uterino (4%), preeclampsia (3%) y luego le
siguen los quistes tecaluteinicos, hiperemesis, hipertiroidismo.

10) Anamnesis:
Lo importante es determinar la EG, inicio de los sntomas y factores de riesgo. En
este caso al preguntar es necesario descarta otras causas de metrorragia del 1
trimestre.
Inicio de cuadro, si haba pasado antes.
Sntomas generales: compromiso del estado general, fiebre, dolor abdominal,
vmitos, diarrea.
Sntomas locales: dolor, disuria, prurito, salida de fluidos uretral y vaginal
(sangre o leucorrea)
FUR, Conducta sexual (nmero de parejas sexuales, proteccin), Embarazos
anteriores, partos anteriores.
Ciruga. abdomino-pelvica.
Uso de frmacos o drogas.

11) Examen fsico:
CSV
Examen fsico general, buscando signos de deshidratacin

29
Examen abdominal: sensibilidad abdominal, signos de irritacin peritoneal,
Altura uterina mayor de lo esperado (si mola completa) o menor de lo esperado
(si mola parcial).
Especuloscopa: sangrado por el OCE
TV: Palpacin de quistes teca luteinicos

12) Estudio de la paciente:
Laboratorio:
Hemograma: buscando anemia, signos de infeccin, plaquetas
ELP: por los vmitos y diarrea hiponatremia, hipokalemia, hipocloremia
grupo Rh: pensando en posible incompatibilidad
BUN Crea, Funcin heptica: pensando en uso de MTX, evaluar el nivel de
deshidratacin.
-hcg.: en la mola completa los niveles esperados son > 100000, en la
parcial los niveles se encuentran aumentados pero por lo general <100000.
Gases arteriales: si la paciente se encuentra en malas condiciones puede
deberse a embolia trofoblastica. Hipoxia, alcalosis respiratoria
Ecografa TV:
Mola completa: mltiples reas hipoecogenicas pequeas de 3 10 mm con
refuerzo acstico posterior (panal de abeja o racimo de uva o copos de nieve), sin
rastros fetales.
Mola Parcial: desde una ecografa normal hasta muy alterada, placenta gruesa,
feto con alteraciones morfolgicas, cambios hidrpicos en la placenta.
Radiografa Trax, Eco abdomina y TAC cerebro : (si se sospecha embolia
trofoblstica): infiltrados pulmonares bilaterales
13) Tratamiento:
Lo antes posible:
1. Estabilizar condicin mdica existente
2. Aspiracin
3. Legrado suave
4. Seguimiento -hcg
Antes del vaciamiento: hemograma, grupo Rh, funcin heptica, funcin renal, -
hcg, RxTx.
Si el tero mide < 14 semanas: dilatar cuello y aspiracin con cnula de 12-14 mm
o se puede hacer con tubo endotraqual 6-7 seguido por un legrado muy suave
bajo infusin continua de retractor uterino (oxitocina)
tero >14-16 semanas: mucho cuidado con la evacuacin uterina ya que hay
mayor riesgo de hemorragia y alteraciones metablicas.

30
No est recomendado inducir el vaciamiento con oxitocina o con prostaglandinas
ya que aumenta el riesgo de mayor sangrado y de enfermedad trofoblastica
persistente. Posterior al vaciamiento si se puede usar oxitocina ya que en este
momento si controla y disminuye la hemorragia.

Se usa quimioterapia profilctica post vaciamiento (Metotrexato) solo en pacientes


de alto riesgo ya que disminuye la ETG persistente.
Alto riesgo:

-hcg previo tratamiento > 100000


tero > 14-16 semanas
ovario >6 cm
edad >40 aos
Al usar MTX igual se debe hacer seguimiento de -hcg

histerectoma: si paridad cumplida. (igual se hace seguimiento con )


profilaxis anti D si Rh (-) no sensibilizada posterior al vaciamiento.
Seguimiento: (riesgo de persistencia Mola completa 8-30%, mola parcial 3-5%)

Se mide -hcg posterior al vaciamiento, luego cada 2 semanas hasta tener 3


resultados seguidos (-), luego se controla cada 1 a 2 semanas hasta completar 6
meses (que es el tiempo de mayor riesgo de persistencia)
Anticoncepcin: muy importante, la -hcg es la forma que tenemos de poder hacer
el seguimiento a la paciente, si se embaraza perdemos esa opcin. Se recomienda
el uso de ACO ya que se ha visto que ayuda a disminuir la tasa de persistencia.
14.Consecuencias para la paciente y feto:
La fertilidad posterior es normal
Recurrencia: 1% (significa que tiene riesgo 10 veces mayor a la poblacin normal),
por esta razn en embarazos posteriores se realizan eco precoces y evaluacin de
-hcg por 6 meses al termino del parto. No es necesario enviar la placenta a
anatoma patolgica si es que no se observan anomalas.
En el caso inmediato la mujer tiene riesgos que van desde la muerte hasta no
presentar ningn problema, la muerte se produce principalmente por la embolia
trofoblastica.
Existe el riesgo de presentar neoplasia trofoblastica gestacional, siendo la mola
invasora la ms frecuente. La mola invasora es la evolucin natural de la mola
hidatiforme, ya que cuando una mola invade el miometrio subyacente se
considera mola invasora.

31
Mola completa Mola parcial
Incidencia 1/1500 1/750
Vellosidades corinicas Edema difuso Algunas edema, en mosaico
con vellosidades y tejido fetal
normal
Capilares Escasos, sin eritrocitos fetales Numerosos con eritrocitos
fetales
Hiperplasia del trofoblasto Difuso de moderada a severa Focales, Mnimos a
moderados
Atipia del trofoblasto Frecuente, difusas Infrecuentes, leves y focales
Embrin-feto En general (-) (+)
Cariotipo Diploide, solo genoma Triploide, genoma materno y
paterno paterno
Tamao tero En general > para edad gesta En general < para edad gesta
Quiste tecaluteinico Presente < 25% Raros
Mola invasora 15% 5%
Coriocarcinoma 3% 0.1%

Bibliografa
- Horn, L. (2014). Clasification and Morphology of gestational. Trophoblastic
disease. Cur Obstet Gynecol Rep. 3. 44-54
- Kobrich, S. (2013). Enfermedad del trofoblasto [41]. ppt.

6. HEMORRAGIA PRIMER TRIMESTRE : EMBARAZO


ECTPICO

Mujer de 32 aos consulta por dolor en FID y sangrado vaginal escaso, rojo y sin
cogulos desde hace 1 semana. FUR hace 10 das escasa y le lleg con 5 das de atraso.

- Antecedentes familiares: Madre diabtica.
- Antecedentes personales: Hospitalizada por PIP hace 2 aos por 5 das.

32
- Historia obsttrica: menarquia 13 aos, RM V/38 35, G0P0A0, uso ACO por 4
aos y luego condn ocasional.
- Hbitos: Cigarro 4 al da , alcohol social .

1. Diagnsticos :
- Primigesta
- Sangrado Uterino Intermenstrual
- Tabquica
- PIP antiguo tratado

2. Diagnsticos diferenciales:
- Ginecolgicos:
EIP
Vaginitis
Cuello tero
Sangrado por ovulacin
- Obsttricos:
Aborto
Embarazo Ectopico
Sintomas De Aborto
Embarazo ectpico
Mola

3. Definicin: Implantacin del ovocito fecundado fuera de la cavidad uterina. Su
localizacin se da principalmente en la trompa (98%): ampular (79.6%),
Istmo (12.3%), Fimbrias (6.2%) e Intersticial (1.9%); seguido de los
extratubricos (1.7%): abdominal (1.4%), luego ovrico (0.15%) e
intersticial (0.15%)

4. Etiologa: Est determinada por aquellos factores que impiden o dificultan la
migracin normal del huevo hacia la cavidad uterina. Se pueden distinguir
entre:
1. Factores tubulares:
1) Secuela de PIP
2) Ciruga tubria previa.
3) Endometriosis tubrica
4) Anomalas congnitas (hipoplasia, trompas muy largas o tortuosas)
5) Alteraciones de la movilidad o de la actividad ciliar

2. Factores ovulares:
1) Nidacin precoz del huevo (por huevos muy pesados que maduran
rpido y no pueden progresar o desprendimiento precoz de la capa
pelcida del huevo capacitando la implantacin)

33
2) Transmigracin: el huevo fecundado se traslada a la trompa
contralateral.
3) Terapia de reproduccin asistida por hiperestimulacin ovrica y
mltiples implantaciones.

5. Fisiopatologa: Lo normal es que el ovocito se fertilice en la trompa y viaje al tero
para implantarsen en estado de blastocito , lo cual ocurre solo durante una
breve fase de receptividad endometrial llamada ventana de implantacin
( dia 5 7 post fertilizacim ). El epitelio tubario normal puede tener esa
misma ventana de implantacin con una duracin de 5 a 7 das. Por otro
lado, el movimiento de la capa muscular de las trompas est muy
determinado por los niveles de estrgeno y progesterona, mientras ms
hormonas menos movimiento.
Una vez implantado el huevo en la trompa, las vellosidades invaden
rpidamenteel endosalpinx, alcanzando la pared tubrica y el peritoneo.
Esta penetracin va acompaada de una proliferacin vascular y de un
hematoma peritubrico o hematosalpinx, que a menudo afecta la trompa
contralateral, y que se propaga hacia el extremo distal de la trompa, entre
la pared y la serosa. A partir de aqu, puede evolucionar hacia:

1) Aborto tubrico: el huevo carece de vellosidades suficientes. Se atrofia luego se
desprende y es expulsado por el orificio peritoneal de la trompa. Esta expulsin
suele ir acompaada de hemorragia moderada, que ocupa el fondo de saco de
Douglas formando hematocele. En raras ocasiones puede convertirse
secundariamente, en embarazo abdominal.
2) Rotura tubarica: las vellosidades son suficientes y enrgicas, pero el aumento de
la presin intratubrica llega a dificultar el riego sanguneo de la trompa, lo que
produce necrosis tisular que conduce a la ruptura por corrosin vascular (no por
estallido). Esta rotura suele acompaarse de hemorragia intensa, que da lugar al
hemoperitoneo y al shock hipovolmico, ya que suele afectar la anastomosis de
arterias tubrico con la uterina ascendente. La rotura tubrica se da con ms
frecuencia en la implantacin stmica e intersticial de la trompa.
La metrorragia se manifiesta generalmente antes de la rotura, debido a la
insuficiencia placentaria (esteroidognesis reducida), que precede las
manifestaciones clnicas. Es generalmente escasa y se acompaa a menudo de
una proliferacin endometrial atpica, denominada Reaccin Arias Stella,
constituida por una decidua que carece de vellosidades.
3) Ectpico viable: si es el caso de un embarazo ectpico abdominal, aunque igual
tiene altas tasas de morbimortalidad materna y fetal.

6. Factores de riesgo:

RR
- Riesgo alto: Cirugia Tubaria ant. 21 x

34
Esterilizacin 9.3 x
Ectpico previo 8.3 x (1 ectpico 10% y con 2 o ms ectpicos 25%)
Uso de DIU 4.5 x
Patologa tubaria demostrada
Exposicin del tero a dietiletilbestrol

- Riesgo moderado: Infertilidad: Fertilidad asistida (in vitro)
Infeccin genital previa (PIP)
Mltiples parejas sexuales

Riesgo leve: Cirugia de abdomen pelvis
Tabaco
Duchas vaginales
Coito antes 18 aos

7. Antecedentes y epidemiologia:

Causa importante de morbimortalidad materna y eventual causa de
disminucin del potencial reproductivo.
En general se diagnostica entre las 6 y 10 semanas
En las ltimas dcadas la incidencia ha aumentado principalmente por los
tratamientos infertilidad.
Ms del 50% se presenta sin factores de riesgo
10 a 15% es en nuligestas
Mortalidad materna (0,5 muertes por cada 1000 embarazos)
Ocurre en el 1-2% de los embarazos, asociado a morbimortalidad.
Chile tasa global de 10 x 10000 mujeres en edad frtil

8. Diagnstico:
Triada clsica: dolor abdominal + amenorrea + Sangrado uterino , esta triada
puede acompaarse de compromiso hemodinmico ( si la trompa est rota ) y se
presenta entre las 5 a 9 semanas (por lo general).
Historia: antecedentes de irregularidades menstruales, SUA (infrecuente o
amenorrea) , con dolor abdominal de inicio brusco o paulatino que aumenta segn
la edad gestacional. En algunas ocasiones puede ser asintomtico hasta que se
rompe. El SUA es el signo ms frecuente, tpicamente intermitente leve, con sangre
fresca o roja oscura y raramente excede el flujo de una menstruacin normal. Es
por degeneracin el cuerpo lteo.
Si se rompe los sntomas se intensifican, puede haber compromiso hemodinmico,
irritacin peritoneal, irritacin del diafragma ( omalgia debido al nervio frnico ).
Puede haber sensacin de pujo x compresin rectal debido a coagulo en el saco de
Douglas

9. Anamnesis:

35
Lo importante es determinar la EG, inicio de los sntomas y factores de riesgo:
Inicio de cuadro, si haba pasado antes
Sntomas generales: compromiso del estado general, fiebre, dolor abdominal,
vmitos, diarrea.
Sntomas locales: dolor, disuria, prurito, salida de fluidos uretral y vaginal
(sangre o leucorrea)
FUR, Conducta sexual (nmero de parejas sexuales, proteccin), Embarazos
anteriores, tratamiento de fertilidad.
Ciruga. abdomino-pelvicas.
Uso de frmacos o drogas.

10. Examen fsico:

Signos generales: alteracin hemodinmica: taquicardia, hipo hipertensin,
fiebre
En general es variable. Lo habitual es observar sensibilidad a la movilizacin del
cuello, tero sin aumento de volumen,blando , sensible pudiendo estar
lateralizado debido al tumor anexial.


11. Estudio de la paciente:
-hcg: para confirmar embarazo. Recordar que es detectable en orina si > de 25
MUI/ml y en sangre si > 5 MUI/ml. En embarazo ectpico la est presente pero
en menores cantidad a lo esperado, y su aumento es < de 66% en 48 hrs. Si se
sospecha embarazo ectpico se debe hacer seguimiento cada 48 hras .
Lo normal es que la se duplique en 48 hrs (mnimo 66%) y eso es signo de
viabilidad.
Si el incremento es < 66% en 48 hrs hay solo 53% de viabilidad en 48 hrs.

Ecografa TV: Patognmico ver saco gestacional fuera de la cavidad uterina, sobre
todo si LCF (+). Se puede observar un saco gestacional como una imagen ovoidea
adyacente a la lnea endometrial. Si 1500 se debera verse en la eco ( 4 % de
las >1500 tiene embarazo intrauterino y no se visualiza saco gestacional a la

36
ecotv ). Si la eco es abdominal se debera ver con de 6500. Si tiene 5 Sem.
Casi 100% debera verse intrauterino.
Otros: (no de uso rutinario)
Concentracin de Progesterona: solo ayuda. Si >25 asociado a viabilidad, si < 5
no viable (independiente de la ubicacin)
Culdocentesis: se extrae sangre del fdo de saco Douglas por puncion a travs
de la pared vaginal. Sirve solo de forma tarda si se ve hemoperitoneo (
embarazo complicado ) .
Signo de Arias Stella: biopsia de endometrio por legrado, se ve presencia de
vellosidades coriales ( Indica embarazo )
Biopsia: Presencia de embrin da diagnostico definitivo de embarazo ectpico.

12. Tratamiento:
Tratamientos de EE: expectante, medico, Cd
1.- Expectante:
1. Si la mujer asume los riesgos y logra entenderlos, hay que tener la seguridad
de que va a responder a los controles rigurosos.
2. Requisitos : Ttulos de en descenso < 1000 mUI
Certeza de ectpico en trompa
Sin evidencia de complicaciones
Tu anexial 4 cm
Paciente viva cerca de centro asistencial
3. Si hay duda razonable de que sea aborto espontaneo evaluar seriada
(mientras ms alto el valor menos posibilidades de resultado exitoso)
< 1750 xito 95%
>2500 xito 66%
4. Hacer seguimiento de hasta que sea (-)

2.- Medico: (Metotrexato)
1. Requisitos: Pcte confiable que se le pueda hacer el seguimiento
No complicado
Embarazo normotpico 100% descartado
Tu anexial 5 cm
< 5000
Ausencia de LCF en ECO TV (o si no sera un aborto provocado)

MTX: Es un antagonista de Ac flico que bloquea la enzima dihidrofolato
reductasa (DHFR), enzima clave en el metabolismo del cido flico que regula la
cantidad de folato intracelular disponible para la sntesis de protenas y cidos
nucleicos, afectando la sntesis de ADN y produciendo apoptosis del tejido
trofoblastico, el MTX se puede usar en embarazos fuera de la trompa.

Dosis: 50 mg/m2 IM x 1 vez, en general se usa 75 mg IM x 1 vez. Si a los 7 das
no disminuye un 15-20% se puede repetir la dosis.

37

Protocolo MTX:

Pre tratamiento: Hcg, Hemograma, grupo, test Coombs indirecto, funcin
heptica, renal ECO TV
Da 0 MTX 50 mg/m2 IM x 1 vez, + RhoGAM si Rh (-), suspender Ac
flico, no actividad sexual y no ejercicio.
Da 7 -hcg + eco tv.
Segunda dosis MTX si no se logra disminucin de 25% en
Semanal -hcg seriadas hasta que sea (-) + ECO TV
Cualquier momento Laparoscopia si dolor abdominal severo o abdomen agudo o ECO
TV >100 ml de sangre en abdomen

o Factores asociados a fracaso de MTX: -hcg > 5000 (ms importante)
- hcg > 50% a las 48 hrs.
Presencia de Hemoperitoneo
LCF (+)
Puede haber leve aumento al 4 dia de
tratamiento

Contraindicaciones de MTX:
- Absolutas: LCF (+) en ECO, embarazo intra uterino, inmunodeficiencias,
anemia moderada, leucopenia, trombocitopenia, ulcera pptica activa,
disfuncin heptica o renal, lactancia, OH

- Relativas: > 5000, tamao > 4 cm, rechazo a aceptar transfusin,
incapacidad de seguir a la paciente.

Reaccion Adversa al Metotrexato : neutropenia, alopecia, gastrointestinales,
alteracin pruebas hepticas, supresin de la medula sea.

3. Imposibilidad de seguimiento de

3.- Quirrgico: (laparotoma o laparoscopia)
Se opta por cirugia si hay fracaso, rechazo o est contraindicado el
tratamiento mdico o si la paciente esta hemodinamicamente inestable. Lo
ideal es hacer laparoscopia ya que tiene menos costo, tiempo de
hospitalizacin, prdida de sangre, analgesia y menos adherencias, la
recuperacin es ms rpida.
Conservador: Salpingostoma lineal y se extrae el trofoblasto tratando de
conservar la anatoma de la trompa:
Radical: Salpingectoma. Si hay mucho dao, sangrado no controlable o si
EE recurrente en la misma trompa.

38
La decisin de si se hace conservador o radical en general se hace durante
la ciruga dependiendo del tamao, indemnidad de la trompa, deseo de
fertilidad futura o antecedente de EE anterior.
Post cirugia se hace igual seguimiento de -hcg ya que pueden quedar
clulas trofoblsticas en la trompa y requerir MTX.
13. Consecuencias para la madre/feto:
Las consecuencias maternas pueden ser desde ser completamente
asintomtica si se reabsorbe hasta la muerte si se produce shock
hemodinmico. La fertilidad va a depender del tratamiento y del grado de dao
que se haya producido.
o Con MTX 88 % tuvo embarazo uterino posterior.
o Salpingectoma: 66% embarazo uterino.
o Salpingostoma: 89% embarazo uterino.

Bibliografa
o Madhra, M. Home, A. (2014). Ectopic pregnancy. Obstetrics, Gynaecology
and Reproductive Medicine. Article in press. 1-6.
o Addi, M. (sin fecha). Embarazo ectpico. Manual de urgencias y
emergencias. 1-20

7. HEMORRAGIA 2 TRIMESTRE : ABORTO SEGUNDO


TRIMESTRE.
Mujer de 17 aos, G1P0A0, cursando embarazo de 19 semanas controlado en consultorio que
consulta de urgencia por dolor tipo clico y contracciones en hipogastrio , adems refiere flujo
vaginal sanguinolento desde hace 4 horas, Al examen fsico, usted no logra auscultar LCF .

Antecedentes familiares: madre Diabetes Mellitus.
Antecedentes personales: no refiere
Historia obsttrica: Menarquia 13 aos, RM V/30-60, G0P0A0, no usa MAC.
Hbitos: cigarro 4 al da, OH social

1. Diagnsticos:

Primigesta

39
Gestacin de 19 semanas
Metrorragia 2 trimestre, Obs. aborto 2 trimestre (aborto tardo).

2. Diagnstico diferencial:

a. Hemorragia subcorinica (desprendimiento ovular)
b. Alteraciones cervicales: Malignidad, plipos
c. Sangrado idioptico con embarazo viable
d. Trauma vaginal

3. Definicin: Aborto del 2 trimestre se define como la prdida de embarazo que se produce
entre las 13 y 26 semanas de gestacin.

4. Etiologa: Las causas de aborto espontneo son generalmente de causa materna y segn
frecuencia se describen las siguientes:


1. Anormalidades genticas
2. Anomalas congnitas no cromosmicas
3. Alteracin endocrina
4. Separacin corioamnitica
5. Incompetencia cervical
6. Infecciones
7. Placentacin anormal
8. Alteracin inmunolgica
9. Malformaciones uterinas
10. Embarazo con DIU
11. Desconocidas


Adems, destacan los siguientes factores de riesgo :

1. Edad materna avanzada.
2. Consumo de alcohol
3. Anestesia (xido ntrico)
4. Uso de cafena en grandes cantidades
5. Consumo de tabaco
6. Consumo de cocana

5. Epidemiologa:

20% de los embarazos tendrn algn tipo de sangrado antes de las 20 semanas de
gestacin y slo la mitad de ellos corresponder a un aborto. En la actualidad, se han
realizado estudios de seguimiento con b-HCG y se ha determinado que la tasa de aborto
real corresponde a un 31% de abortos espontneos. El 80% de los abortos ocurren antes
de las 12 semanas (aborto temprano) y el resto entre las 12 y 20 semanas (aborto tardo).
Slo 6,2% de los abortos tardos ocurren entre la semana 13 y 15 de gestacin, un 4%
ocurre despus de la semana 16 de gestacin o despus, y un 1,3% ocurre despus de la

40
semana 21 de gestacin. La frecuencia de estos abortos aumenta en adolescentes,
mujeres de 35 aos y ms, mujeres de raza negra no hispnica, hispnicas, nativas indias
americanas y nativas de Alaska.

6. Diagnstico clnico:

El diagnstico se puede sospechar con la clnica: sangrado uterino anormal, altura
uterina menor a la que corresponde por edad gestacional, ausencia de movimientos
fetales y ausencia de latidos cardiofetales.
Sin embargo se confirma con ecografa transvaginal donde se objetiva la ausencia de
latidos cardiofetales y pueden observarse algunas malformaciones fetales.

7. Anamnesis y examen fsico:

Es importante consignar dolor hipogstrico, la cuanta del sangrado, ausencia de
movimientos fetales, factores de riesgo y ecografas previas con determinacin de riesgo
de aneuploidas.
Corresponde realizar una evaluacin hemodinmica de la paciente y examen fsico general
en busca de anemia, hipotensin, y temperatura para descartar compromiso infeccioso.
Luego en el examen ginecolgico, evaluar la altura uterina acorde o levemente disminuida,
presencia de modificaciones cervicales , buscar evidencia de RPO , verificar ausencia de
LCF con doppler, y en la espculoscopia se debe objetivar el sangrado proveniente del
orificio cervical externo y observar cuello por si hay presencia de dilatacin y/o
modificaciones cervicales.

8. Estudio de la paciente:

Siempre Ecografa para determinar nmero de fetos, viabilidad, ausencia de LCF,
posicin de placenta. Adems se solicitan exmenes generales por si la paciente requiere
manejo de complicaciones: hemograma (hematocrito, hemoglobina, plaquetas,
leucocitos), pruebas de coagulacin, Grupo sanguneo y Rh, Cultivo cervicovaginal (no est
ampliamente validado) y creatinina en caso de que requiera embolizacin.

9. Tratamiento y conducta:

a. Dilatacin y Evacuacin: Este procedimiento es de eleccin, debido que en esta etapa
del embarazo el feto tiene algunas estructuras seas que pueden lesionar en canal del
parto. Requiere anestesia y/o sedacin, preparacin cervical y su dilatacin para
aspiracin de lquido amnitico y remover el feto con el uso de pinzas frceps hacia el
canal cervical dilatado y el canal vaginal. Luego se realiza legrado para asegurar que la
cavidad uterina haya sido evacuada. Existe una tcnica modificada que se denomina
Dilatacin y Evacuacin Intacta, esta requiere ms das de preparacin cervical para
extraer el feto completo (no sus partes) minimizando el uso de frceps. La preparacin
cervical se realiza con uso de Misoprostol 400 800 mcg por va oral o vaginal
asociado a dilatador osmtico sonda Foley durante uno o ms das previos al
procedimiento.

41


b. Mdico: incluye el uso de anlogos de prostaglandinas, mifepristona, dilatadores
cervicales osmticos, cateterizacin con sonda Foley, oxitocina. Sin embargo, es
menos costo efectiva que la anterior, ya que se asocia a mayor riesgo de aborto
incompleto. Los esquemas son los siguientes:
i. Mifepristona 200 mg va oral, seguida en 24-48 horas por Misoprostol 800mcg
va vaginal, seguida de 400mcg de Misoprostol va vaginal cada 3 horas por
mximo 5 dosis (es el ms efectivo).
ii. Si mifepristona no est disponible se usa Misoprostol 600-800mcg va vaginal
como dosis inicial, seguida de 400mcg va vaginal cada 3 horas por mximo de
5 dosis.
iii. Si Misoprostol no est disponible se utiliza oxitocina 20-100 U en infusin Ev
diluida, cada 3 horas seguida de 1 hora sin oxitocina, se puede incrementar la
dosis hasta mximo 300 U cada 3 horas. Sin embargo, altas dosis de oxitocina
no son usadas ampliamente ya que no hay suficiente expresin de receptores
de oxitocina en tero durante el segundo trimestre del embarazo.

10. Ciruga abdominal: histerectoma o histerotoma, est indicada cuando han fallado dos
procedimientos previos para extraccin fetal o cuando hay contraindicacin de ellos.

a. No hay evidencia de que inductores de muerte fetal (como la digoxina) aumenten
la seguridad del tratamiento mdico o quirrgico.
b. Se puede usar tratamiento anticonceptivo inmediatamente despus del
procedimiento quirrgico.

11. Consecuencias para la paciente y el feto:

Est descrita una mortalidad de 8,9 por 100.000 procedimientos a las 21 semanas (el
riesgo aumenta con mayor edad gestacional).

42
El procedimiento de dilatacin y evacuacin se asocia a 4% de complicaciones,
mientras el tratamiento mdico se asocia a 29% de complicaciones (lo ms frecuente
es retencin de placenta).
Hemorragia postaborto: corresponde a una respuesta clnica exagerada con
requerimiento de hemoderivados, o sangrado uterino mayor a 500 mL. Factores de
riesgo incluyen la edad materna avanzada, dilatacin cervical insuficiente, uso de
anestesia general, historia de cesrea previa. Algunas de las causas de la hemorragia
postaborto corresponden a retencin de productos de la concepcin (<1-8%), atona
uterina (52%), laceracin cervical (12%), perforacin o rotura uterina (7%), alteracin
placentacin (17%), coagulacin intravascular diseminada (5%).
productos de concepcin retenidos: tienen mayor riesgo con tratamiento mdico
laceracin cervical: disminuye su riesgo con adecuada dilatacin. Algunos factores
de riesgo son dilatacin mecnica, nuliparidad, edad gestacional avanzada, falta
de experiencia del operador.
perforacin uterina: se asocia con edad gestacional avanzada, multiparidad,
inexperiencia del operador.
Rotura uterina: El riesgo aumenta con cesrea anterior.
Alteracin placentacin: riesgo de placenta acreta en cicatriz de cesrea previa.
Coagulacin intravascular diseminada: se debe evaluar la hemoglobina,
hematocrito y pruebas de coagulacin, grupo y Rh. Est indicada la transfusin de
plasma fresco congelado o crio precipitado y hemoderivados en caso necesario.
Infecciones: 0,1-4%. La administracin profilctica de antibiticos antes del
procedimiento quirrgico disminuye el riesgo de infecciones. El tratamiento ms
efectivo corresponde a Doxicilina 100 mg va oral una hora antes del
procedimiento quirrgico seguido de 200 mg despus del procedimiento. (sin
embargo no est recomendado el uso de antibiticos en todos los casos)
Embolia: TEP 10-20/100.000 abortos, embolia de lquido amnitico 1/80.000
embarazos, con 80% mortalidad.

Bibliografa:
1. Clinical Management Guidelines for Obstetrician-Gynecologists. Practice Bulletin. The
American College of Obstetricians and Gynaecologists. Vol. 121, No. 6, June 2013.
2. Craig P. Griebel, MD. et al. Management of Spontaneous Abortion. American Family
Physician. University of Illinois College of Medicine at Peoria, Peoria, Illinois. October
1,2005. Vol 72. Number 7.

8. HIPERMESIS GRAVDICA

1) Definicin :
Se define hipermesis gravdica, como vmitos persistentes en ausencia de otras
patologas que los expliquen con prdida ponderal importante (superior al 5% del

43
peso inicial).Las nuseas y vmitos son una situacin frecuente durante el
embarazo, afecta al 75-80 % de las gestantes, principalmente en el primer
trimestre. La hipermesis gravdica, es mucho menos frecuente, y afecta
aproximadamente entre el 0,5 al 2% de las gestantes.

2) Etiologa
La etiologa es desconocida, pero se han postulado diferentes posibles causas.
- De manera adaptativa, para la eliminacin adecuada de toxinas.
- Aumento de -Bhcg, la cual tiene una conformacin similar a la TSH, por lo que
produce su estimulacin, y a su vez esta estimula la glndula tiroides, aumentado
la produccin de hormonas tiroideas, produciendo un hipertiroidismo transitorio.
- Aumento de la masa placentaria, situacin que puede ser explicada que en
embarazos mltiples y enfermedad trofoblastica presentan mayor sintomatologa.
- Aumento de la progesterona, la cual disminuye la motilidad gstrica.
- Factores psicolgicos y familiares.

3) Factores de riesgo
- Primigestas
- Bajo nivel de escolaridad
- Adolescentes
- Sobrepeso/obesidad
- Embarazo gemelar
- Enfermedad trofoblastica gestacional
- Antecedentes previos de hipermesis gravdica
- Antecedentes de trastorno alimentario
- Embarazo no deseado.

4) Diagnostico
El diagnstico es clnico, las pacientes deben presentar ms de tres episodios de
nuseas y/o vmitos al da, acompaado de:
- Cetonuria (Alteraciones Hidroelectroltica)
- Prdida de peso 3 kg o 5 %
- Exclusin de otras causas (tabla 1)

5) Diagnsticos diferenciales :

44

Diagnsticos diferenciales (Tabla 1)
Trastornos Gastrointestinales Gastroenteritis
Acalasia
Afectacin biliar
Hepatitis
Obstruccin intestinal
lcera pptica
Pancreatitis
Trastornos metablico-endocrinos Cetoacidosis diabtica
Porfiria
Enfermedad de Addison
Hipertiroidismo
Alteracin tracto genito-urinario Pielonefritis
Uremia
Torsin anexial
Clico renal
Degeneracin mioma
Lesiones del SNC Pseudotumor cerebral
Lesiones vestibulares
Migraa
Tumor en SNC
Otros Toxicidad farmacolgica

6) Anamnesis
Dentro de la historia clnica es importante destacar:
- El peak de sintomatologa se presenta en la semanas 9 y en general se resuelve a
las 14 semanas, y es poco frecuente que se mantenga sobre la semana 16. Por lo
tanto es importante detallar el momento de aparicin, ya que si los sntomas
aparecen ms all de la semana 9 se debe valorar otras causas.
- Evaluar si los vmitos presentan relacin o no con las comidas, detallar frecuencia
e intensidad de los sntomas, para valorar gravedad y por tanto intervencin.
- Valorar si presenta intolerancia total o parcial a la ingesta.
- Consultar por diagnsticos diferenciales (tabla 1), y por lo tanto preguntar referida
mente sintomatologa, como fiebre, dolor abdominal, cefalea, alteraciones
neurolgicas, etc.
- Siempre evaluar prdida de peso, signos de deshidratacin, taquicardia,
hipotensin. Y valorar viabilidad fetal.
7) Estudio de la paciente

45
Podemos solicitar
- Hemograma, donde podemos pudiese estar normal o hemoconcentracin.
- ELP y gases venosos, para valorar deshidratacin. Segn gravedad podemos
encontrar alteraciones electrolticas (hiponatremia, hipopotasemia, hipocloremia y
alcalosis metablica)
- Perfil heptico, donde pudiese estar normal o presentar elevacin leve de GOT y
GPT < 300U/L, y Bilirrubina < 4 mg/dl.
- Pruebas tiroideas (TSH y T4L) donde pudiesen estar normal o aumento de T4L y
TSH.
- Orina competa : cuerpos cetonicos
- Ecografa: Las pacientes con embarazos mltiples o enfermedad trofoblastica
presentan vmitos con mayor frecuencia.
8) Tratamiento y conducta

Prevencin y ttm. no
farmacologico
Siempre evaluar
severidad
Tratamiento HG Ttm. Farmacologico

Hospitalizacin


1. Estrategias de prevencin de las nuseas y los vmitos durante el
embarazo:
- Ingesta de complejos multivitamnicos periconcepcionales han demostrado
disminuir la incidencia de nuseas y vmitos del embarazo. Se recomienda su
administracin en aquellas pacientes que han presentado nuseas y vmitos en
gestaciones anteriores.
- Comidas frecuentes, poco abundantes (repartir la ingesta en un mnimo de 5
comidas de menor cantidad), slida, evitar comidas de alto contenido en grasas y
condimentadas.
2. Tratamiento farmacolgico:
- Doxilamina 10 mg + Piridoxina 10 mg (Pluriamin): La dosis habitual es un
comprimido cada 6-8 horas (dosis mxima de 70 mg/d). Hay que tener en cuenta

46
que ante la persistencia de sintomatologa en una franja horaria, se debe aumentar
la dosis en la franja anterior (Ejemplo si persisten las nuseas y vmitos matutinos
hay que aumentar la dosis de la noche). Efecto adverso Doxilamina somnolencia.
- Aadir Dimenhidrinato 50-100 mg cada 4-6 horas (Mareamin)(mxima 400 mg
/d. vo o vr)
- Aadir Metoclopramida 5-10 mg cada 8 horas. Puede causar sintomatologa
extra piramidal (espasmos en cara, cuello y lengua).
- Si persiste la clnica a pesar de la asociacin de 2 ms tratamientos y la paciente
presenta intolerancia total a la ingesta, nos encontraremos ante el espectro ms
severo de la enfermedad y ser necesario un ingreso hospitalario para
rehidratacin endovenosa.
3. Hospitalizacin
- Exmenes, rgimen cero.
- Rehidratacin Ev.
- Aadir Metoclopramida 5-10 mg/8 horas Ev.
- En casos resistentes, se puede asociar: Ondansetrn 8 mg/12 h Ev. O
Metilprednisolona 16 mg/8h va oral o endovenosa durante 3 das, seguido de
dosis decrecientes durante 15 das hasta desaparicin de los sntomas.
Puede indicarse clorpromazina como antiemtico
9) Consecuencias para la madre :
Es importante tener un control seguido y evaluar signos de severidad, como la
deshidratacin, para lograr una intervencin precoz, previniendo posible
consecuencias para la madre y el feto, como: Encefalopata de Wernicke,
hiponatremia y mielolisis pontina, Mallory Weiss, desnutricin, alteraciones
psicolgicas, y RCIU.

Bibliografa
- Gua clnica Hipermesis gravdica. . S Fernndez, M Palacio. Servei de Medicina
Materno-Fetal. Instituto Clnico de Ginecologa, Obstetricia y Neonatologa,
Hospital Clnico de Barcelona. Abril 2009.
-

9. INFECCIONES EN EL EMBARAZO : RUBEOLA


Mujer de 28 aos cursando embarazo 9 semanas que consulta por erupcin
maculopapular que comenz en la cara y ahora tambin en tronco y extremidades.

47
Refiere fiebre, cefalea, conjuntivitis en los das previos a la erupcin, adems de ndulos
dolorosos retro auriculares y cervicales.
Antecedentes familiares: madre HTA
Antecedentes personales: hernia inguinal operada
Antecedentes gineco obsttrico: menarquia 12 aos, G1P0A0, RM IV/28
Hbitos: no refiere
Profesin: profesora (refiere que varios nios del jardn lo presentaron)

1. Diagnstico apropiado:
Primigesta,
Embarazo de 9 semanas,
Rubeola
2. Diagnstico diferencial:
Exantemas virales,
- Parvovirus B19, CMV, VEB o Virus Herpes 6.

3. Definicin: Es una enfermedad infecciosa producida por un virus ARN, que se
caracteriza por exantema de curso benigno que ocurre predominantemente
durante la infancia.
4. Etiologa: El virus de la rubeola, es un virus de ARN, que se transmite por va
respiratoria (Gotitas). Tiene un perodo de incubacin de 14 a 21 das. Este
virus se excreta en las secreciones farngeas desde 5 das antes y hasta 6 das
despus de la aparicin del exantema.
5. Epidemiologa: Tasa de 30,5 por 100.000 hbtes. 70% de casos entre 10-29 aos 18
casos de SRC (14 con manifestaciones) Nuevo brote 2007. 90% de los nios a
los 5 aos de edad y el 95% de las embarazadas presentan
seropositividad (Tras campaas de vacunacin).
6. Diagnstico clnico: No ms del 30-40% de los casos de rubeola son
diagnosticados clnicamente. Este diagnstico no es confiable
porque las lesiones pueden ser similares a las de otras infecciones virales.
7. Anamnesis y examen fsico: El cuadro clnico se caracteriza por fiebre baja, cefalea y
compromiso del estado general, seguido por aparicin de exantema mculopapular
que comienza en la cara y se extiende luego al tronco y extremidades. El
exantema dura en promedio 3 das, y con frecuencia se acompaa de
adenopatas cervicales. Tambin se pueden observar artralgias o artritis
transitorias.
8. Estudio de la paciente: El diagnstico materno se debe realizar por mtodos
serolgicos como deteccin de anticuerpos (ELISA: IgG e IgM especifica anti-

48
rubeola: Sensibilidad (S) 73% y Especificidad (E) 92,6%, desde 4- 5 das antes del
exantema hasta 6 meses despus), aislamiento viral o PCR.
Para el RN, el diagnstico prenatal, se realiza por PCR en lquido amnitico (S
100% y E 83-95% (15 semanas de gestacin), PCR en biopsia de corion (11
semanas de gestacin), IgM en sangre fetal por cordocentesis (22
semanas de gestacin).

9. Tratamiento y conducta:
- Notifico caso sospechoso
- Confirmo mediante IgM especfica anti rubola
- Hospitalizo con aislamiento
La rubola no tiene un tratamiento especfico, slo es necesario aislar
relativamente al paciente para evitar el contagio de contactos, especialmente
mujeres embarazadas.
- Se explica a la paciente riesgos de SRC, por 9 semanas de gestacin (Explicado
abajo)
- Paracetamol SOS en caso de fiebre
- No s a demostrado el papel de inmunoglobulinas en SRC.
Prevencin Las mujeres en edad frtil debieran conocer su estado inmune
determinando la IgG especfica anti rubola, antes de embarazarse, de ser
seronegativas (susceptibles), deben ser vacunadas abstenindose de embarazo en
los 3 meses posteriores a la vacunacin.
10. Consecuencias para la paciente y el feto:
Sndrome de Rubola Congnita (SRC) (Parte del sndrome de TORCH)
Ocurre en el curso de una infeccin primaria, donde se puede infectar a travs de
diseminacin hematgena o infeccin de la placenta y en forma secundaria al feto.
Puede:
- No producir infeccin del feto
- Reabsorcin del feto (primeras semanas del embarazo)
- Muerte fetal aborto espontneo o muerte intrauterina.
- Infeccin placentaria sin infeccin del feto
- Infeccin del feto y de la placenta SRC
En embarazada durante el primer trimestre de gestacin, existe entre un 80-90%
de probabilidad de que el feto nazca con una malformacin congnita, ya que el
virus de la rubeola es un agente teratognico, produce agresin cromosmica. El
mayor dao se produce cuando la infeccin se contrae durante la organognesis
(8-10 semanas), traducindose en importantes malformaciones cardacas y
oculares. Luego el cerebro, al continuar el desarrollo estructural, es blanco de
alteraciones morfolgicas hasta las 16 semanas. El riesgo de malformaciones en

49
infecciones adquiridas durante las primeras 8 semanas de gestacin es de 50 a
80%; entre las 9 y 12 semanas, 40 a 50%; y entre las 13 y 16 semanas, 16%. Las
infecciones que se producen despus de las 17 semanas excepcionalmente se
asocian a malformaciones congnitas.
Se confirma por laboratorio (IgM), que en secreciones bucofarngeas y orina se
mantiene + por 1 ao.
Las manifestaciones, se dividen en transitorias, permanentes o secuelas. Las
transitorias incluyen: retraso del crecimiento intrauterino, lesiones seas,
neumonitis, hepatitis, miocarditis, meningoencefalitis, trombocitopenia y anemia
hemoltica. Los defectos permanentes que se pueden observar son: sordera
bilateral, malformaciones cardacas (ductus persistente, estenosis artica,
estenosis de la arteria pulmonar y tetratologa de Fallot), lesiones oculares
(cataratas, retinopata pigmentosa, microftalmia, glaucoma), encefalopata con
retraso mental, alteraciones conductuales y trastornos del aprendizaje. Las
secuelas que se han descrito son hipoacusia, diabetes mellitus, hiper o
hipotiroidismo y pan encefalitis progresiva.

10. EMBARAZO PATOLGICO : INFECCIONES POR


ESTREPTOCOCO GRUPO B

Mujer de 26 aos cursando embarazo de 40 semanas que consulta por haber presentado
flujo vaginal claro con aspecto de orina que no puede controlar desde hace 3 horas y
desde hace hora contracciones cada 5 minutos. Su ltimo control fue en la semana 34
de gestacin.
Antecedentes familiares: madre depresiva , padre hipertenso.
Antecedentes personales: apendicetoma.
Antecedentes go : menarquia a los 13 aos , rm iv/28 g2p1a0 ptve 3100 grs
Embarazo controlado en consultorio de evolucin fisiolgica ,exmenes
prenatales normales , urocultivo + a estreptococos agalactie que se trato con
ampicilina oral y uro control negativo .en semana 32 cultivo vaginal a
estreptococo grupo b , resto de exmenes normales .
Al ingreso:
salida de la claro por vagina
cuello borrado, blando , centrado, 3 cm de dilatacin, ceflica en espinas+1
rnbs : lcf 140 x reactivo du cada 5 minutos
pa 125/70
pulso 80 x minuto
t 36,7

50

1. Diagnostico: -Multpara de 1
-Gestacin 40 semanas
-Trabajo de parto: Fase latente
- Presentacin Ceflica
- Obs RPO

2. Diagnsticos diferenciales:

Diagnstico diferencial Caracterstica
Leucorrea Flujo genital blanco amarillento,
infeccioso, asociado a prurito.
Incontinencia urinaria Frecuente en segunda mitad del
embarazo, especialmente en multparas
(por relajacin perineal y recto cistocele).
Descartar ITU.
Eliminacin tapn mucoso Fluido mucoso, a veces algo
sanguinolento.
Rotura de quiste vaginal Prdida brusca de lquido cristalino por
vagina. Al examen se observa cavidad
pequea en paredes vaginales (raro).
Hidrorrea decidual Primera mitad del embarazo. Lquido claro
con tinte amarillo, a veces sanguinolento.
Desde espacio entre decidua parietal y
refleja, que se fusiona entre las 16 y 18
semanas.
Rotura de bolsa amniocorial Raro; de espacio virtual entre corion y
amnios; se produce por de laminacin de
este ltimo.

3. Definicin:
El estreptococo grupo b es un diplococo Gram positivo, aerobio y anaerobio
facultativo que coloniza de forma habitual el tracto gastrointestinal inferior y
vagina.

4. Etiologa:
La infeccin es producida por el estreptococo grupo b mas frecuente en chile
serotipos IA,II y III.

5. Fisiopatologa:
El EGB forma parte habitual de la flora del tracto gastrointestinal, colonizando de
manera transitoria, intermitente o crnica. Sus factores de virulencia son
adhesinas, b-hemolisina la cual es inhibida por el surfactante pulmonar, y el
pigmento carotenoide, los cuales permiten la colonizacin vaginal que es la que
finalmente se trasmite e infecta al feto durante el trabajo de parto.

51

6. Factores de riesgo:
a. bacteriuria por estreptococos agalactiae en embarazo actual
b. colonizacin recto vaginal materna
c. parto de pre trmino o RPO a edad gestacional menor a 37 semanas
d. fiebre materna durante trabajo de parto ( 38c) antecedente de recin nacido
con sepsis precoz por estreptococos agalactiae
e. perodo de latencia entre RPO y parto prolongado ( 18 horas)

7. Epidemiologia:
La prevalencia de colonizacin por SGB mundial 2-35% en chile es de 20% entre las
35-37 semanas de gestacin. Un 2% de pacientes colonizadas desarrollar
infeccin neonatal y de estos sepsis neonatal en un 50%.

8. Diagnostico clnico:
El diagnstico se realiza a las 35-37 semanas a la mujer asintomtica como mtodo
de screening tomando un cultivo( Todd-hewitt) por trula en el tercio inferior de la
vagina y regin anal, no se necesita la utilizacin de especulo. En las pacientes con
factores de riesgo se recomienda tomar el cultivo a las 15 semanas para prevenir
el riesgo de aborto, 28 semanas por parto prematuro y 35-37 semanas para evitar
la infeccin perinatal.

9. Anamnesis:
Preguntar a la mujer embarazada para descartar factores de riesgo :
a. antecedentes de sepsis neonatal o muerte neonatal precoz por infeccin en
embarazos previos
b. bacteriuria asintomtica a EGB en el embarazo actual
c. parto prematuro o rpo a edad gestacional menor a 37 semanas
d. antecedente de recin nacido con sepsis precoz por estreptococos agalactiae
e. flujo vaginal actual

10. Examen fsico
En las pacientes embarazadas habitualmente es asintomtico o podra ser ausante
de Sntomas del tracto urinario, flujo vaginal, fiebre( infeccin perinatal).

11. Estudio de la paciente:
La colonizacin en las mujeres embarazadas es asintomtica se realiza entre las 35
y 37 semanas de gestacin un cultivo (Todd-Hewitt) obtenido por trula de la
regin vaginal y anal enriquecido con cido nalidxico y Gentamicina, para
aumentar la sensibilidad al eliminar agentes Gram negativos.

12. Tratamiento:
1. penicilina sdica 5 millones UI Ev por una vez, luego 2 millones UI Ev cada 4
horas hasta el parto

52
2. ampicilina 2 gr Ev por una vez, luego 1 gr Ev cada 4 horas hasta el parto

Terapia alternativa
1. clindamicina 900 mg Ev, cada 8 horas hasta el parto
2. eritromicina 500 mg Ev, cada 6 horas hasta el parto

No administrar antibiticos antes del inicio del trabajo de parto, ya que el riesgo de
recurrencia y resistencia es mayor al 50% en mujeres sexualmente activas.

13. Consecuencias para la madre y feto:
Madre
bacteriuria asintomtica
infeccin intrauterina
parto prematuro
aborto de segundo trimestre
infecciones puerperales

Feto
Muerte
Infeccin neonatal
sepsis neonatal
neumona congnita con natal

Bibliografa

Tapia JL, Perret C: Infecciones Bacterianas. En Ta- pia JL, Gonzlez A. Manual de
Neonatologa. III Edi- cin, Santiago, Editorial Mediterrneo, ao 2008.
Sepsis neonatal por Streptococcus Grupo B Rev Chil Pediatr 2008; 79 (5): 462-470
Guia perinatal 2013
Manual de Obstetricia y Ginecologa. Dra. Constanza Ralph T. - Dr. Jorge A. Carvajal

11. EMBARAZO PATOLOGICO :INCOMPETENCIA


CERVICAL.

Mujer de 32 aos cursando embarazo de 23 semanas que consulta en maternidad por flujo
vaginal aumentado
Antecedentes familiares: madre CA mama.
Antecedentes Personales: Aprendicectomia.
Antecedentes Ginecobsttricos: menarquia 13 aos, Aborto 12 semanas con dilatacin y
legrado, parto prematuro sin contracciones semana 25 gestacion.

53
Embarazo fisiolgico hasta ahora, no refiere DU. Al examen se observa cuello
borrado 50%, blando, 2 cm dilatacin y se visualiza que las membranas protruyen
por el OCI.

1) CUAL ES EL DIAGNOSTICO MAS PROBABLE DE ESTA PACIENTE?


Multipara 1
Aborto 12 semanas
Embarazo 18 semanas
Trabajo de parto prematuro
Incompetencia cervical

2) CUALES SON LOS DIAGNOSTICOS DIFERENCIALES?
Sintomas de parto prematuro
Infeccin intrauterina / Corioamnionitis

3) COMO SE DEFINE ESTA PATOLOGIA?
Incapacidad del cuello uterino de mantener el producto de la concepcin por dilatacin
pasiva de este.
Dilatacin indolora del cuello, que lleva a protrusin de las membranas y la consecuente
expulsin fetal si no se maneja oportuna y adecuadamente.

4) CUAL ES/SON LA/S ETIOLOGIA DE ESTE CUADRO CLINICO?
Si bien la etiologa de la incompetencia cervical (disfuncin cervical) no es clara, se pueden
observar ciertos asociaciones a la aparicin de esta; Secundaria a conizacion, traumticas,
dilatacin cervical manual y congnita.Trastornos del colageno

5) CUAL ES LA FISIOPATOLOGIA DE ESTE CUADRO?
La fisiopatologa de la incompetencia cervical no est definida. Principalmente es
traumtica por manipulacin medica o instrumental y/o por predisposicin gentica.
Tiende a ser recurrente en los embarazos, produciendo abortos tardos y partos
prematuros principalmente.
La estimulacin directa del cuello uterino provocara una dilatacin de este, tal y como
sucede durante el parto.

6) QUE FACTORES DE RIESGO BUSCARA?
Lo ms importante para el diagnostico de la incompetencia cervical es la historia previa de
abortos (2 o ms). Por esto es primordial una buena anamnesis, verificando posibles
abortos, y partos prematuros previos. Principalmente historia de Abortos tardos (cada vez
ms precoces en gestaciones sucesivas) y partos prematuros.Es importante preguntar por
instrumentalizacion previa en abortos o partos .

7) CUAL ES LA EPIDEMIOLOGIA DE ESTE CUADRO?
Corresponde al 8-15% de los abortos, siendo la segunda causa de estos
Genticas 50-60%
Incompetencia cervical 8-15%

54
Endocrinas 5-10%
Separacion corioamniotica 5-10%
Infecciosas 3-5%
Inmunolgicas 3-5%
Anomalas uterinas 1-3%

8) COMO HACEMOS EL DIAGNOSTICO CLINICO ?
Por la historia clnica, Antecedentes previos de aborto y parto prematuro sin
contracciones. Sintomatologa caracterstica, principalmente flujo, asociado o no a dolor
y/o sangrado.
Al examen fsico se encuentra un cuello blando, borrado y dilatado (2 cms o mas), con o
sin flujo/sangre. Se puede observar la protrusin de membranas ovricas por el OCE.
Clnica de presin plvica + dilatacin cervical 2 cm + Ausencia de contracciones
regulares uterinas + Presencia o no de bolsa amnitica en vagina.

9) QUE NOS INTERESA PREGUNTAR EN LA ANAMNESIS DE ESTA PACIENTE ?
Antecedentes previos de abortos/partos prematuros, tratamientos/instrumentalizacin de
cuello y tero, traumatismos, TV, actividad sexual, ecografas previas (ver desarrollo
cervical anmalos).

10) QU BUSCARA DIRIGIDAMENTE EN EL EXAMEN FISICO DE ESTA PACIENTE ?
A la especuloscopia se puede puscar cuello borrado y dilatado, con o sin flujo/sangre. Se
puede observar la protrusin de membranas ovricas por el OCE. Todo esto, por lo
general, no acompaado de dolor ni contracciones.

10) COMO ESTUDIAMOS ESTE CUADRO Y QUE RESULTADOS ESPERARIA ENCONTRAR EN ESTOS
EXAMENES ?
La especuloscopia y la ecografa son parte del estudio de esta paciente, donde se
confirmara el borramiento y dilatacin del cuello, y se observara el bienestar fetal. El
diagnostico es principalmente a travs de los antecedentes previos y el examen fsico, con
dilatacin/borramiento y protrusin de las membranas ovricas.

11) CUAL ES EL TRATAMIENTO DE ESTA PACIENTE ?

El tratamiento es quirrgico consiste en el reforzamiento del cuello mediante un cerclaje,
el cual se retira a las 38 semanas o antes, si comienza el trabajo de parto o existe
infeccin. Existen tres tipos de cerclaje:
El cerclaje se considera profilctico o electivo (o primario) cuando se realiza de forma
electiva por historia previa de incompetencia cervical antes de evidenciar cambios en el
crvix y generalmente suele realizarse entre las 13 y 16 semanas de gestacin.
1) 3 o ms prdidas fetales en el segundo o inicios del tercer trimestre en contexto
de cuadros clnicos compatibles con incompetencia cervical.
2) 2 prdidas fetales en el segundo o inicios del tercer trimestre documentadas
(sin evidencia, se recomienda realizarlo).
3) Puede considerarse en pacientes con prdida fetal en el segundo o inicio del
tercer trimestre con historia de conizacin.

55
El cerclaje teraputico secundario que se realiza tras la deteccin, en el seguimiento
obsttrico, de modificaciones en el crvix antes de las 26 semanas de gestacin. Se
realiza
en pacientes con un riesgo potencial de parto pretrmino (historia previa; cuello <25mm).

El cerclaje teraputico terciario, en caliente, de rescate o emergent cerclage que se
realiza en pacientes que presentan la membrana amnitica visible a travs del orificio
cervical externo o en vagina.




12) QUE CONSECUENCIAS PUEDE TRAER ESTA PATOLOGIA A LA MUJER / MADRE/ Y AL
FETO ?
Las consecuencias principales de la incompetencia cervical son el aborto
(generalmente 2do trimestre), parto prematuro, infecciones (relacionadas con el
cerclaje principalmente) y traumas maternos por la ciruga del cerclaje , adems de
la RPO .


Bibliografa
- Obstetricia; Perez Sanchez, 4ta edicin (2011), Capitulo "Parto Prematuro" (J. Carvajal, J. Poblete); pag. 722-723.

- Protocolos de medicina fetal y perinatal Hospital Clinic-Hospital Sant Joan De Du - Universidad de Barcelona - Protocolo:
Indicacin del cerclaje; T. Cobo, S. Ferrero, M. Lpez, M. Palacio- Ultima actualizacin y revisin: 15/02/2014.

- Up to date; Cervical insufficiency; Vincenzo Berghella, MD; Ultima actualizacin Dec 24, 2014.

12. EMBARAZO PATOLOGICO : ANEMIA Y EMBARAZO



Mujer de 18 aos cursando embarazo de 14 semanas a quien la matrona le pide IC con
gineclogo por presentar en los exmenes de rutina pre natales Hb de 9.5 g/L , anemia
microctica hipocrmica. No refiere molestias.
Antecedentes familiares: Madre con miomas.
Antecedentes personales: Sana
Antecedentes Gineco obsttricos: menarquia 12 aos, RM VII/28 G1P0A0
La evolucin del embarazo ha sido fisiolgica.

56

1. Diagnstico apropiado: Primigesta
Embarazo de 14 semanas
Anemia en el Embarazo
2. Definicin: Disminucin de la masa de la hemoglobina durante el perodo grvido
puerperal mas all de lo ocurrido por la hemodilucin relativa.
OMS: Hb 11 g/dL - Hto 33%
CDC: I y III Trimestre (Hb 11 g/dL Hto 33%), II Trimestre (Hb 10,5 g/dL Hto
32%)

Clasificacin:

Anemia Hemoglobina Hematocrito
(g/dL) (%)
Leve 9 a 11 27 a 33
Moderado 7 a 9 21 a 26
Severo 7 20
Tabla extrada de Manual Alto Riesgo Obsttrico PUC


3.Etiologa:






4. Fisiopatologa:

1. Anemia por dficit de Fe

57
Anemia nutricional ms comn en el ser humano (1)
Demanda de hierro:
- Requerimiento total en el embarazo es de 1000 mg (1000-1400 mg)
- 300 mg para el feto y la placenta
- 500 mg para el incremento de la masa de Hb.
- 200 mg en las prdidas basales (intestino, orina, piel)
- 250 mg por las prdidas durante el parto (se duplica en la cesrea)
Requerimientos diarios:
- 6 mg/da a partir del cuarto mes
- Fe diettico no supera los 2 mg/da
Dieta balanceada:
- Aporta 20 mg/da de sales de Fe, se absorbe el 10% (2 mg/da).
- La embarazada necesita 1000 mg de Fe en todo su embarazo (40
semanas= 280 das): 3.57 mg/da
- Por lo tanto hay un dficit de 1.57 mg/da


2. Anemia por Prdidas agudas
Etiologa:


Tabla extrada de Manual Alto Riesgo Obsttrico PUC

Es comn con el aborto, embarazo ectpico y mola hidatidiforme.

3. Anemia Megaloblstica

Aumento de necesidades de Acido flico y Vitamina B12, para la sntesis de ADN y ARN.
Diagnostico: macrocitosis, determinacin de acido flico <4 ug/L
Consecuencias: aumento de complicaciones infecciosas de la madre, abortos, partos
prematuros, RPM.

5. Factores de riesgo:

58
- Multparas
- Intervalos intergensicos cortos
- Antecedente de menstruaciones abundantes
- Dietas de baja disponibilidad de hierro
- Parasitosis anemizantes

6. Epidemiologa:
-Prevalencia (2):
frica (57,1%)
Asia Sudoriental (48,2)
Mediterrneo Oriental (44,2%)
Pacfico Occidental (30,7%)
Europa (25%)
Amrica (24,1%).
Prevalencia mundial: 41,8%

7. Diagnstico clnico:
Suelen ser asintomticas.Depender de la reserva funcional, intensidad y causa de
anemia.

8. Anamnesis:
- Sndrome Anmico: laxitud, irritabilidad, astenia, nerviosismo, cefalea, anorexia.
- Severas: manifestaciones digestivas, circulatorias y de sistema neuro-muscular.
Alteraciones apetito, pirosis, ardor lingual, flatulencia, constipacin y glositis,
parestesias. Manifestaciones de insuficiencia cardiaca.
- Consumo de alimentos que permiten la mayor absorcin de fierro: carne de
vacuno, pescado y pollo, interiores (hgado, riones y embutidos de sangre)
- Consumo de alimentos que disminuyen la absorcin de fierro: tanatos del t y
caf.

9. Examen Fsico
- Taquicardia
- Palidez
- Sudoracin
- Uas opacas sin brillo
- Glositis, queilitis angular, coilinoquia, lengua depapilada.
- Disnea de esfuerzo.
- La inestabilidad hemodinmica se asocia a prdidas agudas e intensas de masa
eritrocitaria que obligan a su hospitalizacin

59
10. Estudio de la paciente:

- Hto <33% y Hb <11 g/dl (Microctica-Hipocrmica)
- VCM <82 (907)
- CHCM < 30
- TIBC >400 ug%
- Fe srico <60 ug%
- Saturacin de transferrina <10%
- Ferritina srica <10-15 ug/L
- Depsito de Fe:
- ausencia de ferritina en la tincin de mdula sea.

11. Tratamiento:

1. Anemia por dficit de Fe
Profilaxis:
- Dieta: carne vacuno, pescado, pollo, huevo, legumbres interiores.
o Aumentan absorcin: ctricos, azcares
o Disminuye absorcin: t, caf, legumbres, lcteos.
- Suplementacin oral diaria: 30 a 60 mg de hierro elemental en II y III trimestre y
400 mcg de acido flico
o Gluconato ferroso: 600 mg/ da (10% absorcin)
o Sulfato ferroso: 300 mg/da (10 20 % absorcin : 1 comp aporta 40 mg
Hierro elemental )
o Fumarato ferroso: 200 mg/da (30% absorcin)
Tratamiento:
- Duplicar dosis profilctica por 6 semanas(120 mg/da)
o Gluconato ferroso: 1200 mg/ da
o Sulfato ferroso: 600 mg/da
o Fumarato ferroso: 400 mg/da
- Control con Hto y Hb
- Anemia severa requiere hospitalizacin
Respuesta reticulocitaria a los 3 a 4 das de instaurada la terapia, y en un lapso de
un mes puede normalizar los parmetros eritrocitarios

Sulfato ferroso LCH 200 mg. Comprimidos, equivalente a 40 mg de Fe
elemental/comp.

2. Anemia por Prdidas agudas
- Hemorragia masiva exige tratamiento inmediato

60
- Si es anemia moderada, hemodinmica mente estable, capaz de deambular sin
sntomas adversos, y no es sptico, se indica terapia con hierro por al menos 3
meses.

3. Anemia Megaloblstica
- Profilaxis: cido flico 1mg/da. Dieta: frutas ctricas frescas, vegetales de hoja
verde, legumbres
- Tratamiento: cido flico 5-10 mg/da + Fierro

12. Consecuencias para la paciente y el feto:
- Aborto
- RCF
- Parto prematuro
- Prolongacin del perodo expulsivo
- RN bajo peso
- Anemia neonatal
- Infartos placentarios
- DPPNI
- Sufrimiento fetal (Hb <8 gr/dL)
- Muerte fetal

Bibliografa
1. Manual Alto Riesgo Obsttrico PUC, disponible en:
http://escuela.med.puc.cl/paginas/departamentos/obstetricia/altoriesgo/Indice
2. Organizacin Mundial de la Salud, disponible en:
http://www.who.int/vmnis/database/anaemia/anaemia_data_status_t3/es/

13. EMBARAZO PATOLOGICO : APENDICITIS AGUDA




Paciente de 28 aos cursando embarazo de 20 semanas que consulta por dolor desde
hace 12 horas que se inici en zona peri umbilical y luego se irradi a FID. Refiere
nuseas y deposiciones normales, examen obsttrico normal. Hemograma muestra 9500
leucocitos/mm3 que aumenta a 14.000 despus de 8 horas de observacin y PCR de 8.
Antecedentes familiares: no refiere
Antecedentes personales: no refiere
Antecedentes Gineco-Obsttrico: G0P0A0

61

1.Cul es el diagnstico ms probable de esta paciente?

Primigesta
Embarazo de 20 semanas
Dolor plvico agudo. Obs apendicitis aguda

2. Cules son los diagnsticos diferenciales?

Los principales diagnsticos diferenciales en una paciente con dolor abdominal y
test de embarazo positivo, son complicaciones obsttricas como embarazo
ectpico, quiste ovrico complicado ( hemorrgico o torcido ) ,mioma
complicado , trabajo de parto prematuro. Una vez descartadas las causas
obsttricas se deben diferenciar otras causas como gastroenteritis, clico renal,
infeccin urinaria alta, clico biliar o colecistitis aguda, colitis ulcerosa, adenitis
mesentrica .

3. Cmo se define esta patologa?

La apendicitis aguda se define como la inflamacin del apndice vermiforme.

4. Cul es/son la/s etiologa de este cuadro?

La causa inicial es una obstruccin de la luz del apndice, el 60% de los casos es por
hiperplasia linftica que puede ser congnita, secundaria a enfermedades
sistmicas o por enterocolitis bacteriana. Un 35% de los casos se deben a fecalitos,
tumores apendiculares 1%.

5. Cul es la fisiopatologa de este cuadro?

Al obstruirse la luz apendicular hay un aumento de la presin intraluminal
secundario a la continua secrecin de mucosa, con sobre-crecimiento bacteriano.
Al aumentar la presin el apndice se adelgaza y dificulta la circulacin linftica y
venosa, con expansin bacteriana a todas sus capas. Si el flujo vascular se
compromete se produce necrosis y perforacin del apndice.



6. Qu factores de riesgo buscara?

Los factores de riesgo para apendicitis son : dieta pobre en fibra, infecciones vricas
gastroenteritis reciente, y luego de un trauma abdominal.

7. Cul es la epidemiologia de este cuadro?

62

La apendicitis aguda se considera como la patologa quirrgica ms frecuente
durante el embarazo, alcanzando 1/1.500 a 1/2.000 embarazos (1). Es ms
frecuente durante el segundo trimestre

8. Cmo hacemos el diagnostico clnico?

El cuadro clnico caracterstico de dolor abdominal que comienza en zona peri
umbilical y luego migra hacia FID se presenta solo en un 75% de las pacientes
embarazadas. Y debido al ascenso fisiolgico del ciego y apndice durante el
embarazo por el crecimiento uterino la clnica tambin vara. Hasta el 72%
presenta signos de irritacin peritoneal, a dems de presentar nauseas, vmitos y
fiebre. Por efecto hormonal sobre la pared abdominal , es difcil realizar el
diagnostico de abdomen agudo en el embarazo .

9. Qu nos interesa preguntar en la anamnesis de esta paciente?

Se debe indagar en las caractersticas del dolor, forma de inicio , tiempo de
evolucin, sntomas asociados. A dems de preguntar por caractersticas de
bienestar fetal pesquisada por la madre, como movimientos fetales. Es necesario
conocer antecedentes quirrgicos de la paciente.

10. Qu buscara dirigida mente en el examen fsico de esta paciente?

Al examen fsico se debe evaluar el estado general de la paciente y compromiso
hemodinmico. La paciente puede presentar ruidos intestinales disminuidos,
taquicardia, palidez, fiebre, postura antilgica, y signos de irritacin peritoneal
como singo de Mc Burney (dolor a la compresin profunda), Blumberg (dolor de
rebote), Rovsing (dolor en FID al presionar FII).
Se debe realizar examen ginecolgico, movilizacin de crvix, dolor anexial ,
evaluar sensibilidad y masas en fondo de aco posterior.

11. Cmo estudiamos este cuadro y que resultados esperara encontrar en estos
exmenes?

Por las caractersticas del cuadro y su resolucin quirrgica, es importante
complementar el diagnostico con recuento de leucocitos, que nos indicaran la
presencia de infeccin. Durante el embarazo el nivel normal de leucocitos
aumenta (la elevacin de leucocitos hasta 16.000 puede ser normal), por lo que no
es una prueba especfica, pero si orientadora. Los niveles de PCR tambin
aumentan durante el embarazo.
Complementar con ecografa abdominal la cual tiene una sensibilidad cercana al
60% con alta especificidad, sin embargo, es un mtodo operador dependiente y no
siempre es diagnostica. Por la variacin de posicin anatmica del apndice, podra

63
ser til durante los dos primeros trimestres. El uso TAC no se recomienda como
rutinario durante el embarazo pero se podra usar de manera relativamente segura
en embarazos de ms de 20 semanas.
La Resonancia magntica puede ser til en embarazas con sospecha clnica de
Apendicitis Aguda con Ecografa abdominal dudosa (2)

12. Cul es el tratamiento de esta paciente?

El tratamiento de esta paciente es la resolucin quirrgica con la realizacin de una
apendicetoma. Se aconseja realizar laparotoma.

13. Qu consecuencias puede traer esta patologa a la mujer/madre/y al feto?

Hay riesgo fetal en este cuadro ya que la resolucin es siempre quirrgica. La
mortalidad fetal llega al 1,5% de las apendicitis no complicadas. 35% de muerte
fetal en apendicitis complicadas con perforacin o peritonitis difusa. La
laparoscopia aumenta el riesgo de muerte fetal de 3,1 a 5,6% en relacin a
laparotoma. Hasta un 15% de las pacientes presentaron parto prematuro luego de
la ciruga.
Las principales complicaciones son la infeccin de herida operatoria (15%),
evisceracin (4%).

Referencias
1. Kort B, Katz VL, Watson WJ. The effect of nonobstetric operations during
pregnancy. Surg Gynecol Obstet 1993; 177: 371-376
2. Yarmuch G, Julio. Ciruga y embarazo. Rev Chil Cir. 2010, vol.62, n.5 pp. 520-524 .

14. EMBARAZO PATOLGICO : PIELONEFRITIS AG.



Primigesta de 27 aos cursando embarazo de 24 semanas que consulta por disuria ,
aumento de la frecuencia miccional y tenesmo .El embarazo ha sido de evolucin normal
hasta la fecha. Consulta en centro de urgencia municipal donde le indican ampicilina 500
mg cada 8 horas por 7 das y solicita urocultivo . 7 das despus consulta por fiebre 38 ,
escalofros, dolor en zona renal y refiere endurecimiento del abdomen.
Antecedentes familiares : madre y padre sanos.
Antecedentes personales :no refiere.
Antecedentes gineco obsttricos : menarquia 13 aos , rm iv/28 g1p0a0
Hbitos : sana

1. Definicin:
-Primigesta

64
-Gestacin 24 semanas
-Pielonefritis aguda (PNA) o infeccin urinaria alta

2. Diagnsticos diferenciales:
a. Itu baja persistente: persistencia del patgeno inicial en el urocultivo a las
48 h de finalizado el tratamiento.
b. Cistitis Aguda:
Infeccin sintomtica de la vejiga
> 102-3 UFC
c. Estado gripal
3. Definicin:
Bacteriuria significativa asociado a inflamacin del parnquima renal, clices y
pelvis.

4. Etiologa:
a. escherichia coli (95%)
b. kiebsiella pneumoniae
c. proteus mirabilis, staphilococcus coagulasa(-)
d. streptoccus agalactiae
e. enterobacter sp (5%)

5. Fisiopatologa:
Cambios anatmicos y fisiolgicos del tracto urinario en el embarazo:
Meato uretral tiene cercana anatmica con recto y vagina propia de la mujeres
Hidroureteronefrosis fisiolgica del embarazo: El proceso de dilatacin
pieloureteral comienza durante el primer trimestre y se pronuncia hasta el
trmino, se caracteriza por un aumento del dimetro interior del urter
asociado a hipotona e hipomotilidad muscular, ms acentuado del lado
derecho.
Progesterona Relajacin msculo liso: que lleva a una disminucin de la
peristalsis en urteres y aumento en capacidad vesical y ectasia urinaria(relaja
la vejiga, aumenta el residuo urinario y disminuye su vaciamiento.)
Diferencias en pH (elevado) , os molaridad urinaria, Glucosuria y aminoaciduria
: la excrecin aumentada de bicarbonato y el aumento de la filtracin
glomerular con presencia de glucosa en orina, favorecen la multiplicacin
bacteriana.

Factores del Microorganismo:


Cepas de E. coli de mayor virulencia:
Provocadas por toxinas y factores de adhesin a uroepitelio
75% pielonefritis con cepas ms virulentas versus 22% en bacteriuria
asintomtica

6. Factores de riesgo:

65

a. Bacteriuria asintomtica previa
b. La edad, aumenta su prevalencia 1% por cada dcada de vida a partir de los 5
aos de edad
c. Historia de infecciones urinarias recurrentes
d. Abortos de segundo trimestre de causa no precisada
e. Parto prematuro de causa no precisada
f. Malformacin de la va urinaria o litiasis
g. Actividad sexual frecuente, facilita el movimiento de uro patgenos del introito
a la uretra
h. Diabetes mellitus
i. Anomalas estructurales del tracto urinario
j. Factores socioeconmico

7. Epidemiologia:
3-12% es la prevalencia de infeccin urinaria en la poblacin sin factores de riesgo
y un 30% en la poblacin con factores de riesgo.
La infeccin urinaria alta en un 0,5-2% de los embarazos y de estos un 5% de las
hospitalizaciones.
En un estudio retrospectivo de 24.000 embarazadas:
7% 1er trimestre; 67% 2do y 3er trimestre
8% intraparto y 19% post parto

8. Diagnostico clnico:
Fiebre 39-40C, sensibilidad en ngulo costo vertebral, dolor en flanco, calofros,
nuseas y vmitos. 3% debuta en shock sptico.
El diagnstico se realiza en base a un cuadro clnico compatible y ratificado por el
urocultivo.

9. Anamnesis:
Fiebre, sensibilidad en ngulo costo vertebral, calofros, nuseas y vmitos,
contractilidad uterina asociada y molestias urinarias( disuria, poliaquiria y
tenesmo).

10. Examen fsico:
Ver signos vitales, Fiebre, dolor en regin costo lumbar, buscar signos de
deshidratacin.

11. Estudio de la paciente:
a. hemograma( hto <30%, leucocitosis)
b. sedimento de orina ( bacterias presentes y nitritos positivos)
c. cultivo de orina con recuento de colonias >100.000/ml,
d. funcin renal (disfuncin renal transitoria creatininemia > de 1.4 mg/dl)

66
12. Tratamiento:
1. Hospitalizacin.
2. Hidratacin lquidos intravenosos: suero fisiolgico o lactato de Ringer.
3. Monitorizacin de diuresis y signos vitales.
4. Hemograma, creatinina srica, hemocultivos.
5. Antimicrobianos endovenosos:

1. cefazolina: 1 g cada 6h intravenoso por 3 a 7 das, luego se continua con cefradina
va oral 500 mg cada 6h hasta completar 14 das.

2. cefuroxima 750 mg cada 8h intravenoso por 3 a 7 das, luego 250 mg cada 12h va
oral hasta completar 14 das.

Con resistencia o fracaso clnico:

1. ceftriaxona: 1 g cada 12h intravenoso por 3 a 7 das, se continua con cefixima
400 mg va oral hasta completar 12 das.

2. cefotaxima: 1 g cada 6h intravenoso por 3 a 7 das, luego cefixima 400 mg va
oral hasta completar 12 das.

3. gentamicina (2 a 4 mg x kg da) 160 mg intravenoso x 3 das, luego 160 mg
intramuscular por 7 das.

13. Consecuencias para la madre y el feto:
Maternas :

Shock sptico y muerte materna : poco frecuente
Disfuncin renal: Cuando la concentracin de creatinina es mayor de 1
mg/dl despus de la hidratacin.
Alteraciones hematolgicas: La endotoxemia causa hemlisis. La
trombocitopenia grave es infrecuente, pero cuando aparece es un hallazgo
de mal pronstico.
Lesin pulmonar: Cerca del 2% de las pacientes con pielonefritis grave
desarrollan alguna manifestacin de insuficiencia respiratoria, es ms
probable en infecciones causadas por Klebsiella pneumonae.

Fetales:
Corioamnioitis
Ruptura prematura de membranas
Bajo peso al nacer
Menor edad gestacional

67
Infeccin neonatal
Fiebre post-parto
Aborto
Parto prematuro


Bibliografa
1. j. schnarr; f. smaill eur j clin invest 2008; 38 (s2): 50-57 asymptomatic
bacteriuria and symptomatic urinary tract infections in pregnancy.
2. cedip 2014
3. prez snchez a, donoso sia e. obstetricia. publicaciones tcnicas mediterrneo
ltda. Santiago de chile. segunda edicin. 1992


15. EMBARAZO PATOLGICO : COLESTASIA


INTRAHEPTICA DEL EMBARAZO.

Paciente de 30 aos cursando embarazo de 30 semanas que consulta por presentar
prurito generalizado de predominio nocturno palmo plantar.
Antecedentes familiares: madre colicistectomia.
Antecedentes personales: tonsilectomia 12 aos.
Antecedentes Gineco Obstetrico: menarquia 12 aos, RM V/30, uso ACO hasta 1
mes antes del embarazo. G2P1A0 obito anterior 35 semanas .
Embarazo de evolucin fisiolgica, en ultimo control se encontr transaminasas
elevadas. Refiere desde hace 2 semanas prurito palmo plantar de predominio
nocturno que ahora es generalizado. Hace 2 aos, sin estar embarazada tambin
presento prurito en palma de manos.

1) CUAL ES EL DIAGNOSTICO MAS PROBABLE DE ESTA PACIENTE?

- Multipara de 1
- Embarazo de 30 semanas de gestacion
- Colestasia intrahepatica del embarazo.

2) CUALES SON LOS DIAGNOSTICOS DIFERENCIALES?

Prurito Gravidico Alergias

68
Infecciones Hepatitis
Patologa dermatolgica Drogas
Coledocolitiasis Hemolisis

3) COMO SE DEFINE ESTA PATOLOGIA ?

Patologa obsttrica de la segunda mitad de la gestacin (ms frecuente en el 3er
trimestre) en la cual aparece colestsia reversible y acotada al curso del embarazo.
Por definicin no existe antes de las 20 semanas. Tambin llamada colestasia
obsttrica o colestasia gravdica.

4) CUAL ES/SON LA/S ETIOLOGIA DE ESTE CUADRO CLINICO ?
La etiologa de la enfermedad es an desconocida. Se postulan varias relaciones con la
aparicin de la CIE:
Predisposicin gentica (ej: > mapuches)
Factores hormonales (estrgenos; > gemelar)
Factores ambientales (estacional, alimentacin, no recurre en todos los
embarazos)

5) CUAL ES LA FISIOPATOLOGIA DE ESTE CUADRO?

Aunque la fisiopatologa no es clara, se ha postulado y observado una relacin
entre los niveles de estrgenos y la CIE (Reyes y Simon, 1993). Esta relacin se
establece principalmente por el aumento progresivo de estrgenos, siendo en el
3er trimestre, cuando los niveles de estrgenos son ms altos y cuando la CIE se
expresa clnicamente. Es ms frecuente en embarazos gemelares y en mujeres con
antecedentes de hepatitis inducida por estrgenos. Adems la enfermedad
desaparece post parto, cuando se normalizan los niveles hormonales.
Sin embargo no se observan niveles elevados de estrgenos en las mujeres con CIE,
por lo que se postula una sensibilidad elevada a los niveles normales de estrgeno
y/o existencia de metabolitos estrogenicos anormales en la CIE.
Otros postulan la participacin de la progesterona en la etiopatogenia de la
enfermedad (Meng y cols, 1997).
Existe un componente de factores ambientales en el origen de la CIE: dieta,
estacional, geogrfico. Adems del factor gentico: etnias, polimorfismo de
transportadores de sales biliares, etc.
Todos estos factores actuaran en el hgado donde produciran la colestasia,
acumulacin de sales biliares, siendo estos los causantes de los sntomas de la
paciente y del dao perinatal.

6) QUE FACTORES DE RIESGO BUSCARA ?

69
Raza (mapuches)
Antecedentes familiares
Embarazos gemelares
Estacin (invierno)
Alimentacin (aceites)
Lugar geogrfico
Antecedentes de CIE previo

70
7) CUAL ES LA EPIDEMIOLOGIA DE ESTE CUADRO ?

Mundialmente la incidencia es baja ( 0,5 % 1 % ), siendo en Chile un pas de
prevalencia elevada. El primer estudio en chile demostr que un 16% de los
embarazos presentaban CIE (Reyes y cols, 1968). Otros estudios describieron
aumento de prevalencia en poblaciones mapuches, llegando hasta el 25% de los
embarazos.
En Chile la incidencia a disminuido entre 1985 y el 2000, de 15% a 1-2 %,
principalmente por una mejora en el status de salud poblacional. La recurrencia
est entre 40-60%.

8) COMO HACEMOS EL DIAGNOSTICO CLINICO ?

El diagnostico de CIE es CLINICO. Fundamentalmente la aparicin durante el
embarazo de prurito persistente palmo-plantar, algunos casos se presenta
generalizado, de predominio nocturno, que interfiere en el reposo materno, con
frecuencia respeta la cara y cuello, no afecta mucosas, no hay lesiones cutneas
primarias, solo secundarias al grataje, generalmente moderada, algunas veces
invalidante (incluso llegando a la interrupcin del embarazo) y desaparece de
forma espontanea despus del parto. Por lo general no aparece antes de las 28
semanas; cuando es antes se deben sospechar otras patologas.
En un 10% de los casos se puede presentar con ictericia por hiberbilirrubinemia
conjugada que no suele ser mayor a 5 MG/ML y aumento de las fosfatasas
alcalinas ( que no llegan a triplicarse ) y puede haber una concentracion de acidos
biliares totales en plasma mayor a 11 micromoles/ml Los sntomas deben persistir
por al menos una semana.
Con poca frecuencia se manifiesta asociada a vmitos y nauseas, si estos aparecen
con gran intensidad, asociado a CEG debe sospecharse otra etiologa.

9) QUE NOS INTERESA PREGUNTAR EN LA ANAMNESIS DE ESTA PACIENTE ?
Raza (mapuches), Antecedentes personales/familiares de hepatopatas del
embarazo, Embarazos gemelares, bitos fetales, partos prematuros,
alimentacin.Prurito con uso de ACOS .

10)QU BUSCARA DIRIGIDAMENTE EN EL EXAMEN FISICO DE ESTA PACIENTE ?

Al examen fsico NO se encuentran signos que sean caracteristicos de la CIE. Las
araas vasculares y eritema palmar se presentan en embarazos normales por el
hiperestrogenismo, por lo que no son necesariamente caracteristicos de la CIE. Se
puede observar, en algunos casos, ictericia, coluria, acolia.

10)COMO ESTUDIAMOS ESTE CUADRO Y QUE RESULTADOS ESPERARIA ENCONTRAR EN
ESTOS EXAMENES ?

71

Aunque el diagnostico de la CIE es 100% clnico (cuadro que cede una semana post
parto), se pueden observar alteraciones de laboratorio, no siendo ninguna de estas
patognomnicas de CIE
Pruebas hepticas normales, transaminasas discretamente elevadas, sales biliares.
Para diagnsticos diferenciales y cuantificar ictericia: Bili, SGOT, FA, test de atopia
(parches, prick test, etc), infecciones, estudio enfermedades dermatolgicas.
*Bili total > 1,8 mg/dl define la forma ictrica de la CIE.

11)CUAL ES EL TRATAMIENTO DE ESTA PACIENTE ?

Segn las normas de la gua perinatal (CEDIP) :
Control semanal con evaluacin de movimientos fetales, evaluacin clnica de
coluria, ictericia y curso del prurito; evaluacin de eventual aparicin otras
enfermedades asociadas. Adems desde las 34 semanas se puede evaluar con
RBNE semanal o bisemanal.
Hospitalizacin en casos puntuales: prurito invalidante o asociacin con otra
patologa de importancia.

Tratamiento:
Sintomatico
o No se ha demostrado la utilidad del fenobarbital, SAMe, rifampicina,
epomediol, silamarina, bloqueadores H1, ni el carbn activado.
o La hidroxicina (25-50 mg/da) alivio transitorio de prurito.
o Algunas pacientes de benefician del uso de dexametasona y colestiramina
(8-16 g/da).
Mejorar flujo biliar
o Acido ursodeoxicolico (Palma y cols, 1996), generando alivio sintomtico
importante, normalizacin de pruebas hepticas, y disminucin de cidos
biliares plasmticos. Podra evitar los riesgos perinatales de la CIE (no
demostrado).

Interrupcin de embarazo (induccin parto vaginal):

Alteracin UFP
CIE anicterica 38 semanas ( cuestionable ) , por riesgo de obito fetal.
CIE ictrica 36 semanas

12) QUE CONSECUENCIAS PUEDE TRAER?

Maternas:
Elevacion de PRL
Alt funcin heptica
Alt funcin renal

72
Deficiencia de factores de coagulacin dependientes de vit K (>RR de hemorragia
postparto)
Fetales:
Alteracion de la oxigenacin fetal intraparto (estado no tranquilizador, meconio,
asfixia, distres): vasocontriccion de venas corionicas por aumento de acidos biliares
y acumulacin de estos en el parnquima pulmonar, adems estimularan la
peristalsis colonica.
Parto prematuro (4 veces mas riesgo): los acidos biliares podran estimular la
liberacin de protaglandinas. Ademas se a observado una sobreexpresin de
receptores de oxitocina en clulas miometriales de pacientes con CIE.
bito fetal: sin causa demostrada, ya que presenta monitorizaciones normales
previas, por lo que se postula como muerte sbita (sales biliares producen
arritmia ?)



Bibliografa
- Obstetricia; Perez Sanchez, 4ta edicion (2011), Capitulo "Colestasia intrahepatica del embarazo" (R. Zapata, J.C. Glasinovic);
pag. 920-929

- POMA, Pedro A.. Colestasis del embarazo. Rev. peru. ginecol. obstet. [online]. 2013, vol.59, n.3 [citado 2015-04-06], pp.
207-224

- Up to date; Intrahepatic cholestasis of pregnancy; Jan 23, 2015

16. EMBARAZO PATOLGICO : HIPERTENSIN


CRNICA.

Paciente de 35 aos cursando embarazo de 18 semanas que asiste a control prenatal de
rutina y se encuentra Pa 150/95 hace 1 semanas y luego 2 controles con 145/90 y
140/90.
Antecedentes Familiares : Padre hta
Antecedentes personales : Alergia a penicilina.
Antecedentes gineco obsttrico : menarquia 13 aos , RM IV/ 35 40 G2P1A0
Hbitos : fuma 2 cigarros da.

1. Diagnstico:

Multpara de 1
Embarazo de 18 semanas
Hipertensin crnica de la embarazada
Tabquica

73

2. Diagnsticos diferenciales: Hay cuatro grandes sndromes hipertensivos en las
mujeres embarazada (pre eclampsia-eclampsia; pre eclampsia-eclampsia sobre
agregada; hipertensin gestacional e hipertensin crnica que trataremos en el caso)
Pre eclampsia eclampsia: La pre eclampsia hace referencia a la aparicin de
hipertensin y proteinuria despus de las 20 semanas en una embarazada
previamente con presiones normales. La eclampsia se asocia a convulsiones.
Pre eclampsia-eclampsia sobre agregada: Se diagnostica cuando una mujer
hipertensa crnica desarrolla un deterioro de su hipertensin asociado a
nuevos episodios de proteinuria u otras caractersticas de pre eclampsia (ejm:
sd. Hellp)
Hipertensin gestacional: Es la hipertensin detectada despus de las 20
semanas, sin proteinuria u otras caractersticas de pre eclampsia y auto
limitada
3. Definicin: La hipertensin crnica (preexistente) se define como la Presin Sistlica
> o igual a 140 mmHg y/o PD > o igual a 90 mmHg que precede al embarazo, esta
antes de las 20 semanas o persiste durante ms de 12 semanas post parto.

4. Etiologa: HTA crnica
Hipertensin esencial: (sobre 90% casos)
Hipertensin secundaria: (o crnica de causa conocida)

5. Factores de Riesgo: Grupo de edades extremos, obesidad, historia familiar de
hipertensin arterial crnica, embarazo mltiple, antecedentes de
embarazo previo con alza de presin, enfermedad renal crnica, sndrome
anti fosfolpidos, diabetes mellitus, mola hidatiforme

6. Diagnstico clnico: Se realiza con valores de presiones elevadas, existiendo dos
categoras:
HTA leve: PS <150 mmHg o PD < 100 mmHg
HTA moderada: PS >= 1150 159 mmHg o PD >= 100 109 mmHg
HTA severa: PS >= 160 mm Hg o PD >= 110 mm Hg

7. Anamnesis: Interesante preguntar antecedentes familiares de HTA, sntomas
asociados y importante caractersticas del embarazo anterior. Por otro lado durante el
interrogatorio preguntar dirigida mente sobre signos y sntomas de una pre
eclampsia,pensando en una pre eclampsia sobre agregada.

8. Examen fsico:
Por el lado materno buscar signos y sntomas de una pre eclampsia para descartar
una pre eclampsia sobre agregada, control de PA en cada control adems de
exmenes de sangre, mencionados en la pregunta 9.

74
Adems muy importante el bienestar fetal, evaluacin de UFP, evaluacin de
movimientos fetales, monitorizacin de LCF, biometra fetal por ecotomografia, perfil
biofsico y dopler de arteria umbilical en ecotomografia de 22 24 semanas como
predictor de pre eclampsia.

9. Estudio de la paciente: Es importante descartar si posee una HTA crnica por lo que se
realiza un perfil de presin arterial o en su defecto holter de presin, asociado a
pruebas de laboratorio para vigilancia estricta para evitar que haga una pre eclampsia
sobre agregada. Adems de pruebas de laboratorio tales como: orina completa,
funcin renal, glicemia, pruebas hepticas, pruebas tiroideas y proteinuria en 24
horas. Junto con lo anterior realizar eco tomografas para evaluar crecimiento y
bienestar fetal.

10. Tratamiento y conducta: El tratamiento de las pacientes con hipertensin crnica
implica la vigilancia estrecha de la tensin arterial de la madre y la observacin para
detectar la superposicin de pre eclampsia o eclampsia, as como el seguimiento del
feto para comprobar que su crecimiento y su bienestar sean apropiados
1. Derivar a alto riesgo obsttrico
2. Administracin de antihipertensivos con PS sea de 150 160 mm Hg o PD de
100 110, utilizando metildopa, desde 250 mg cada 12 horas hasta 500 mg
cada 6 horas. Si la madre previa al embarazo utilizaba diurticos, se mantienen.
Cuyo objetivo de tratamiento en una mujer sin dao en rgano blanco es de
PA entre 140-150 / 90-100 y para mujeres con dao de rgano blanco es PA
bajo 140-90, ahora est en discusin si llevar a la madre a presiones normales
(120/80) es beneficioso para la madre y/o feto.
3. Utilizacin de aspirina, 100 mg da reduce el riesgo de PE (pacientes de alto
riesgo tales como: HTA crnica, antecedentes de PE en embarazo anterior,
enfermedad renal crnica).

11. Consecuencias para la paciente y el feto: El ser hipertensa previamente a un
embarazo aumenta el riesgo de desarrollar una pre eclampsia o eclampsia con lo que
lleva a los riesgos propios del SHE:
-Madre: desprendimiento prematuro de placenta normoinserta, insuficiencia
cardiaca y edema pulmonar agudo, insuficiencia renal, dao heptico,
coagulacin vascular diseminada, accidente vascular enceflico, necrosis sub
endocardica, eclampsia y muerte.
-Fetales y neonatales: retardo de crecimiento intrauterino, prematurez,
vasoconstriccin fetal, insuficiencia cardiaca intrauterina, muerte fetal, muerte
neonatal.

Bibliografa:
Alfredo Prez Snchez, obstetricia, cuarta edicin, pagina 530 539, editorial
mediterrneo

75
Uptodate, Management of hypertension in pregnant and postpartum women, Phyllis
August, MD,

17. EMBARAZO PATOLOGICO (PREECLAMPSIA)



Paciente de 30 aos cursando embarazo de 28 semanas quien es derivada por matrona
por Pa 145/95 y 150/100 en 2 tomas separadas por 2 das, adems presenta edema y
refiere aumento de peso de 5 kg en 15 das

Antecedentes familiares : Madre Hta.
Antecedentes personales : colecistectoma
Antecedentes gineco obsttrico : G3P2A0 , menarquia 12 aos RM IV/28
Hbitos : fumaba 12 cigarros da .

1. Diagnostico:

Multpara de 2
Embarazo de 28 semanas
Sndrome hipertensivo del embarazo : obs Preclampsia moderada

2. Diagnsticos diferenciales:
Hay cuatro grandes sndromes hipertensivos en las mujeres embarazada
(hipertensin crnica; pre eclampsia-eclampsia sobre agregada; hipertensin
gestacional y pre eclampsia - eclampsia que trataremos en el caso)
Hipertensin crnica: La hipertensin crnica (preexistente) se define como la
PS > o igual a 140 y/o PD > o igual a 90 que precede al embarazo, esta antes de
las 20 semanas o persiste durante ms de 12 semanas post parto.
Preclampsia-eclampsia sobre agregada: Se diagnostica cuando una mujer
hipertensa crnica desarrolla un deterioro de su hipertensin asociado a
nuevos episodios de proteinuria u otras caractersticas de pre eclampsia (ejm:
sd. Hellp)
Hipertensin gestacional: Es la hipertensin detectada despus de las 20
semanas, sin proteinuria u otras caractersticas de pre eclampsia y auto
limitada

3. Definicin:

Presin arterial sistlica mayor o igual a 140 mm Hg o diastlica mayor o igual a 90
mm Hg en 2 ocasiones separadas por 4 horas despus de las 20 semanas de
embarazo en una mujer previamente normo tensa
o

76
Presin arterial sistlica mayor o igual a 160 mm Hg o Presin arterial diastlica
mayor o igual a 110 mm Hg
o
Proteinuria mayor o igual a 300 mg en 24 horas o Protena/creatinina mayor o igual
a 0,3
Dipstick que marca 1+ ( si no hay otro mtodo cuantitativo )

o en ausencia de proteinuria , la aparicion de hipertensin y de cualquiera se los
siguientes :

Trombocitopenia , plaquetas < 100.000
Creatinina > 1.1 mg/dl o el doble de valores previos en ausencia de enfermedad
renal.
Aumento de transaminasas al doble de su valor normal
Edema pulmonar
Sntomas visuales o neurolgicos.

4. Fisiopatologa:

Los eventos fisiopatolgicos que llevan finalmente a una PE, incluyen: isquemia
placentaria absoluta o relativa y la activacin difusa de las clulas endoteliales, por
lo cual la pre eclampsia es una enfermedad sistmica.
La isquemia placentaria se relaciona con una penetracin o implantacin
trofoblastica superficial, aun en desconocimiento el porqu, pero debido a esto
persiste una vasculatura uterina de menor dimetro y mayor resistencia que
disminuye el territorio de sntesis de sustancias vasodilatadoras (PGS ON) y que
la placenta libera a la circulacin materna hipertensores (aun no conocidos). Estos
factores por si solos o asociados, poseen propiedades cito toxicas que daan el
endotelio, aumentan su permeabilidad y son responsables del edema; a nivel renal
causan la tumefaccin celular y favorecen la agregacin plaquetaria.

5. Factores de riesgo

Nuliparidad, grupo de edades extremos, obesidad, historia familiar de pre
eclampsia, embarazo mltiple, antecedentes de pre eclampsia previa, hipertensin
arterial crnica, enfermedad renal crnica, sndrome anti fosfolpidos, diabetes
mellitus, mola hidatiforme

6. Epidemiologia

En Hispanoamrica y el caribe la hipertensin del embarazo, representa la primera
causa de mortalidad materna (25.7%) Beltrn y cols. (Prez Snchez cuarta edicin,
pag 836)
En Chile 1992 representaba el 10% de las muertes maternas

77
Periodo 2000 2004 el 25% de las 208 muertes maternas, fueron por causadas por
SHE
Prevalencia del 7 10% de la poblacin gestante

7. Diagnstico clnico

HTA despus de las 20 semanas (2 cifras tensionales >140 sistlica o >90 diastlica,
separadas por 4 horas) y proteinuria > 300 mg/24 horas o creatinina/protena 0.3
o signos de disfuncin de rganos diana (recuento de plaquetas < 100.000,
creatinina srica > 1.1 mg/dl o duplicacin de la creatinina srica, transaminasas
sricas elevadas al doble de lo normal

8. Anamnesis

Establecida la HTA es necesario considerar si la paciente tiene antecedentes
familiares de HTA o PE y si ha tenido HTA antes del embarazo actual. Si se present
en embarazos previos, preguntar cul fue la conducta y si hubo compromiso del
peso del recin nacido. HTA antes de las 20 semanas apoya la hiptesis de HTA
crnica y no de una PE.
Alteraciones visuales, especialmente escotomas, o cefaleas graves o persistentes
son indicativos de vaso espasmo. El dolor en el cuadrante superior derecho puede
indicar afectacin heptica. Preguntar sobre cualquier antecedente de prdida de
conocimiento o convulsiones.

9. Examen Fsico

CSC
Evaluara el compromiso de conciencia,. La presencia de edema en la cara, manos y
regin lumbosacra (el edema persistente que no responde al reposo en decbito
supino no es normal, especialmente cuando tambin afecta a las extremidades
superiores, regin sacra y cara). El peso corporal (atencin con el aumento de peso
excesivo o demasiado rpido), la intensidad y la extensin del rea reflexogena
bicipital y patelar.
El fondo de ojo, la mayora de las pre eclampsias presenta vasoconstriccin
arteriolar y aumento del brillo retiniano, en hipertensiones severas pueden verse
hemorragias, exudados y edema de papila.
Mujer embarazada hipertensa, edematosa y con la cara hinchada es la imagen
clsica de la pre eclampsia.
Medir AU , evaluar cantidad de LA y auscultar LCF

10. Estudio de la paciente

Exmenes de Laboratorio

78
Hemograma: hematocrito elevado certifica hemoconcentracin propia de PE.
Alteraciones del recuento de plaquetas y alteraciones morfolgicas en los glbulos
rojos denotan dao endotelial y sealan severidad del cuadro (trombocitopenia,
esquistocitos, signos de hemolisis y/o aparicin de crenocitos)
Uricemia: valores > 5 mg/dl es caracterstico de PE
Proteinuria: cualitativa o cuantitativa (>300 mg/da en muestra aislada) indica
nefropata previa o PE
Clearence de creatinina: el deterioro de la funcin renal esta asociado a un
agravamiento de la enfermedad
Relacin protena/creatinina
ECG: presencia de hipertrofia ventricular es indicativo de HTA previa al embarazo
Enzimas hepticas: se agregan en la PE severa, buscando un sndrome de HELLP,
forma grave de PE
Orina completa: evaluar el compromiso renal previo o agudo y alteraciones
asociadas como infecciones urinarias, a travs de la presencia de cilindros,
hematuria, bacteriuria y/o piuria
Fondo de ojo si se sospecha HTA crnica

Evaluacin bienestar fetal (para determinar los efectos de la hipertensin arterial
relacionados con el embarazo sobre el feto)
Mediante exmenes que evaluaremos UFP, que permitir precisar las posibilidades
de prolongar el embarazo y decidir va de parto:
Evaluacin de movimientos fetales
Monitorizacin electrnica de LCF
Biometra fetal ultrasonografa
Perfil biofsico
Dopler fetal: evaluando arteria umbilical

11. Tratamiento y manejo
No existe tratamiento ptimo efectivo para PE. La solucin final es la interrupcin
del embarazo

Lo primero clasificar la PE en moderada o severa
Moderada Severa
PS <160 >= 160
PD <110 >= 110
Proteinuria (grs /24h) <5 >= 5
Diuresis (ml / 24h) >= 500 < 500
Edema Moderado Generalizado (anasarca, EPA)
Compromiso Ausente Irritabilidad SNC eclampsia (cefalea, hiperrreflexia,
neurolgico fotopsia, tinitus)
Compromiso Ausente Trombocitopenia, hemolisis microangiopatica
coagulacin
Compromiso heptico ausente Elevacin de enzimas hepticas

79

Objetivos del tratamiento:
Evitar compromiso materno secundario a la HTA
Prevenir aparicin de HELLP y de PE sobre agregada
Prevenir la eclampsia
Promover la mejora o detener el deterioro de la circulacin placentaria
Prolongar, con el menor riesgo posible, la estada fetal in tero

PE moderada:
Sin criterios de gravedad, se debe mantener manejo expectante e
interrupcin del embarazo 37 38 semanas
Monitoreo materno fetal bisemanal, en busca de criterios de severidad
- Hospitalizacin
o Reposo decbito lateral izquierdo
o Rgimen completo, normo sdico

- CSV maternos y LCF cada 4 6 horas
o Medicin del peso y diuresis diaria

- Antihipertensivos
o Con PD >=100, como: hidralazina, alfa metildopa, labetalol o
antagonista de calcio has lograr PD ente 90 y 100. Si existe
control adecuado y buen funcionamiento UFP, no necesario
interrumpir.

Antihipertensores en el embarazo

Frmaco Mecanismo accin Efectos
Tiazida Disminucin del Disminucin del GC; disminucin del FSR;
volumen plasmtico y hipovolemia materna; trombocitopenia neonatal
el GC
Metildopa Neurotransmisin GC inalterado; FSR inalterado; fiebre; letargo,
falsa; efecto sobre el hepatitis y anemia hemoltica materna
SNC
Hidralazina Vasodilatacin Aumento del GC; FSR inalterado o aumentado; crisis
perifrica directa vasomotora, cefalea, taquicardia, pseudo lupus
materno
Propanolol - bloqueante Disminucin del GC; disminucin del FSR; aumento
del tono uterino materno con posible disminucin
del riesgo sanguneo placentario; respiraciones
neonatales deprimidas
Labetalol - bloqueante GC inalterado; FSR inalterado; temblor, crisis
vasomotora, cefalea; respiraciones neonatales
deprimidas; contraindicado en asmticas y con
insuficiencia cardiaca

80
Nifedipino Antagonista del calcio GC inalterado; FSR inalterado; hipotensin arterial
ortostatica y cefalea materna; ningn efecto
neonatal conocido
FSR: flujo sanguneo renal; GC: gasto cardiaco tabla, obstetricia y geniecologia Beckmann 2010, 1 edicion


- Criterios de interrupcin
o Aparicin de signos de mayor dao materno o fetal, ya sea por
induccin o cesrea, de acuerdo a condiciones obsttricas de cada
paciente.
o Cuando evolucin del cuadro hipertensivo nos lleva a interrumpir un
embarazo < 34 semanas, con pulmn inmaduro, es conveniente
maduracin pulmonar con corticoides e interrumpir 48 hrs. De la
primera dosis.

12. Consecuencias para la madre y el feto

Madre: desprendimiento prematuro de placenta normo nserta, insuficiencia
cardiaca y edema pulmonar agudo, insuficiencia renal, dao heptico, coagulacin
vascular diseminada, accidente vascular enceflico, necrosis subendcardica,
eclampsia y muerte.
Fetales y neonatales: retardo de crecimiento intrauterino, prematurez,
vasoconstriccin fetal, insuficiencia cardiaca intrauterina, muerte fetal, muerte
neonatal.

Bibliografa:
Alfredo Prez Snchez, obstetricia, cuarta edicin, pagina 530 539, editorial
mediterrneo
Beckmann, obstetricia y ginecologa, primera edicin, paginas 175 181.
Guas perinatales, ministerio de salud

18. EMBARAZO PATOLGICO : ECLAMPSIA


Primigesta de 23 aos, cursando embarazo de 31 semanas a quien se le detecta PA
160/95 y proteinuria de +++ en cinta reactiva. Edema +++. Su mdico le indic reposo,
metildopa 500 mg cada 8 horas y se le solicit Dopler. Refiere desde hace 6 horas
cefalea, dolor hipogstrico. En la urgencia pierde conciencia y comienza con
movimientos tnicos y convulsiones y luego cae en coma profundo.

Antecedentes Familiares: No
Antecedentes personales: Rh -

81
Antecedentes ginecobsttricos: G1P0A0
Desarrollo:
1. Diagnstico apropiado: Primigesta, embarazo de 31 semanas, SHE, Eclampsia, Rh -
2. Diagnstico diferencial:
a. AVE
b. Epilepsia
c. Hemorragia subaracnodea
d. Hemorragia cortical o subcortical
e. Rotura de aneurisma
f. rotura de malformacin arteriovenosa cerebral
g. Sndrome de encefalopata reversible posterior (PRES): Corresponde a un desorden del
sistema de autorregulacin cerebral producido por el aumento de presin brusco.
h. Hipertensin maligna
i. Glomerulonefritis aguda
j. Drogas: Cocana, fenilpropanolamina
k. Feocromocitoma
l. Suspensin de hipotensores
m. Inhibidores de la MAO.
n. Hipertensin arterial reno vascular
o. Corticoides

3. Definicin: La Eclampsia es la complicacin ms severa de Pre eclampsia, se caracteriza
por aparicin violenta de convulsiones tnico-clnicas generalizadas que pueden ser
recurrentes y pueden llegar al coma y la muerte en contexto de Pre eclampsia y en
ausencia de otra condicin neurolgica que la explique.

4. Etiologa: Etiologa claramente identificable: HTA en embarazo por alteracin de la
placentacin. El trofoblasto invade parcialmente la decidua, las arterias espiraladas no
logran llegar a placenta con buen flujo y aumentan su presin (resistencia vascular)
para mantener el flujo placentario, lo cual produce un aumento de citoquinas
inflamatorias, tromboxanos y disminucin de prostaglandinas y xido ntrico favoreciendo
la vasoconstriccin perifrica, apareciendo la hipertensin.

Factores de riesgo:
Primiparidad
Desbalance entre vasoconstrictores y vasodilatadores
Diabetes
Embarazos mltiples
Predominio tromboxano sobre prostaciclina
Mola hidatidiforme
Dficit xido ntrico
Enfermedades inmunolgicas (LE, sndrome anti fosfolpidos)
Dficit sistema calicrena-cinina
Dficit del sistema renina-angiotensina
Hipertensin

82
Edades extremas <20 aos y >30 aos
Historia familiar

5. Epidemiologa: PE 3-6% de embarazos, 50% de SHE. Eclampsia corresponde a 1-5% de PE.
27,5 casos por 100.000 embarazos. Mortalidad 3,1%. (Datos UK). 50% de
Eclampsia aparece ante parto, 20% intraparto, 30% postparto.

6. Fisiopatologa: Encefalopata hipertensiva: Inicialmente se plante una sobrerregulacin de
la curva de autorregulacin del flujo sanguneo enceflico. As, sobrepasado el lmite
mximo de presin arterial que permite mantener la autorregulacin, se
producira una vasoconstriccin excesiva, con hipo perfusin del tejido enceflico .
Posteriormente, se estableci que la EHT se debe a una falla o quiebre de la
autorregulacin. Frente a un alza sbita de la presin arterial, la contraccin arterial es
incapaz de mantenerse, y se dilatan las arterias, con aumento del flujo sanguneo
enceflico. De los mecanismos que controlan el estado de vasodilatacin-
vasoconstriccin de los vasos enceflicos: metablico, neurognico y miognico,
este ltimo es el ms importante en la mantencin de la curva de autorregulacin del
flujo sanguneo enceflico. El fenmeno de vasodilatacin que ocurre en la
encefalopata hipertensiva, no es un fenmeno mecnico pasivo, sino un fenmeno
activo secundario a la vasoconstriccin extrema de las arteriolas enceflicas, que
determinan un ingreso excesivo de calcio a la fibra muscular, lo que a su vez gatilla la
activacin de canales de potasio, calcio dependiente, produciendo relajacin
muscular. Este aumento de la presin arterial y vasodilatacin, determina un
aumento de la presin hidrosttica capilar, favoreciendo la salida de lquido al intersticio.
Por otra parte, el aumento de la presin arterial va a determinar un
aumento de la permeabilidad vascular, principalmente por pinocitosis, sin alteracin
estructural de la pared, lo que es importante para entender la rpida y total
reversibilidad que tiene este cuadro clnico, al bajar la presin arterial. Lo anterior se
traduce en edema cerebral vaso gnico, que compromete de preferencia la sustancia
blanca. En estudios de autopsia, se ha encontrado edema y necrosis fibrinoide de la
pared arterial. Estos son los casos ms graves, en que el dao llega a su expresin
mxima en la pared vascular, como es la necrosis fibrinoide. sta a su vez produce
trombosis, con micro infartos y micro hemorragias, que pueden llegar a evolucionar a
hemorragias extensas. La mayora de las encefalopatas hipertensivas se presentan en
pacientes previamente normo tensos. Se estima que menos del 1% de los hipertensos
crnicos desarrolla una EHT. Esto se ha atribuido a una proliferacin de la capa
muscular de las arterias enceflicas secundaria a la HTA, con desviacin de la curva de
autorregulacin del flujo sanguneo cerebral a la derecha. Esto determina que slo con
presiones arteriales muy altas, superiores a 220/110 mmHg, habitualmente sobre
250/150 mmHg, se produzca un quiebre de la autorregulacin del flujo sanguneo
enceflico.

83

7. Diagnstico clnico:
Sntomas premonitorios: epigastralgia, dolor hipocondrio derecho, reflejos osteotendneos
aumentados, clonus, cefalea, escotomas, fotopsia, compromiso de conciencia.
Criterios diagnsticos:
Mujer con convulsiones durante el embarazo o dentro de 10 das postparto,
ms dos de los siguientes criterios dentro de las primeras 24 horas de la
convulsin:
Hipertensin: PAD mayor o igual a 90 o incremento mayor al 25.
Proteinuria: proteinuria + en toma aislada o ms de 300mg en proteinuria de
24 hrs.
Trombocitopenia menor a 100.000.
ALT o AST > 42 iu/l

8. Anamnesis y examen fsico: Investigar antecedente de PE, controles prenatales y


PA, sntomas premonitorios, convulsiones tnico clnicas generalizadas, coma. PA
materna cada 15 minutos, control de signos vitales, examen neurolgico
exhaustivo, ROT aumentados, observar convulsin, signos de hipertensin
endocraneana (cefalea, alteraciones visuales, compromiso de conciencia).
Monitorizacin materna y fetal continua.

9. Estudio de la paciente:
-Monitorizacin materna y fetal continua.
-Se solicita proteinuria, hemograma (trombocitopenia), pruebas hepticas, funcin renal,
grupo y Rh.
-TAC y/o RM estn indicadas ante sospecha de hemorragia subaracnodea.
-TAC muestra hipo densidad de la sustancia blanca, principalmente en el territorio
posterior de los hemisferios cerebrales.
-La RM muestra hiperintensidad en stas zonas lo que es consistente con edema vaso
gnico y no isquemia.
-No existe una contraindicacin absoluta para realizar un TAC durante el embarazo.

84
-El uso de medio de contraste yodado est contraindicado luego de las 14 semanas de
gestacin, por poder daar la tiroides del feto.
-En cuanto a la RM, se permite en el 2 y 3 trimestre, y no es recomendable en el primero,
por posible riesgo de desprendimiento placentario.
-No hay contraindicacin para el uso de gadolinio durante el embarazo.
-Neuroimgenes: edema de predominio en la sustancia blanca posterior reversible
en la mayora de los casos.

10. Tratamiento y conducta:
-Manejo expectante en ningn caso ha demostrado tener algn efecto positivo, por el
contrario, aumenta la mortalidad materna y perinatal.
- Se hospitaliza en sala aislada idealmente en cuidados intensivos, con disponibilidad de
sala de neonatologa y pabelln, sala oscura sin ruido.
-Monitorizacin materna y fetal continua.
- Va area y venosa permeable
- Yugulacin de la crisis convulsiva:
Sulfato de Magnesio trata y previene convulsiones, mejores resultados que
fenitona y diazepam.
Sulfato de Magnesio 4-6 gr Ev, seguidos de 1-2 gr/hora Ev en las siguientes 24
horas.
Objetivo de Magnesemia 4-8 meq/L.
Antdoto Sulfato de Magnesio en caso de intoxicacin, Gluconato de Calcio 1 gr Ev.
En la prctica habitual se colocan inicialmente 10 mg Ev de Diazepam y a
continuacin se usa sulfato de magnesio en las dosis de carga y mantencin.
Cuando se usa sulfato de magnesio se deben monitorizar frecuencia respiratoria,
reflejos osteotendneos y diuresis horaria.
Tambin puede usarse Diazepam en dosis de 10 mg iniciales y dosis de
mantencin de 2-4 mg/hora en infusin Ev posteriormente.
En casos extremos puede usarse Pentotal en dosis de 100 a 200 mg Ev.
- Disminucin de la HTA si la hipertensin es severa.
- Evaluacin hemodinmica y del equilibrio cido bsico:
Registro de pulso, PA, PVC, diuresis horaria, equilibrio cido bsico y observacin
de signos sugerentes de insuficiencia cardaca.
- Evaluacin neurolgica y tratamiento del edema cerebral:
1. Interesa investigar pares craneanos, nivel de conciencia y la presencia o no de paresias
y/o hemiplejias. Puede requerirse puncin lumbar para descartar o confirmar el
diagnstico de hemorragia subaracnodea (xantocroma en LCR).
2. El tratamiento del edema cerebral puede efectuarse con Dexametasona 10-20 mg Ev,
luego 6 mg cada 6 horas.(controversial)
- Resolucin del parto (interrupcin del embarazo):
Una vez controlado el cuadro convulsivo y recuperada la conciencia. La va de
parto debe ser preferentemente vaginal y debe intentarse, por tanto, la induccin
oxitcica. Las pacientes eclmpticas parecen ser especialmente sensibles a la
estimulacin oxitcica.
Siempre prima el riesgo vital materno, ante emergencia como esta, se interrumpe
el embarazo por la va ms expedita previa estabilizacin de la paciente.
Con estabilizacin de la paciente se refiere a yugular crisis epilpticas, mantener
PA 140/90, hemodinamia estable y recuperacin de conciencia,

85
Si persiste compromiso materno, interrupcin por riesgo vital materno
independiente a edad gestacional.
Si paciente logra estar estable se puede realizar maduracin pulmonar en las
siguientes 48 horas y luego interrupcin del embarazo.
-En paciente con PE severa se recomienda el uso de sulfato de magnesio intraparto y
postparto para disminuir el riesgo de eclampsia.
-En mujeres con PE en la interrupcin del embarazo por cesrea se recomienda el
uso intraoperatorio de sulfato de magnesio para disminuir el riesgo de eclampsia.

11. Consecuencias para la paciente y el feto:
-Consecuencias para la madre: Hemorragia subaracnodea, Coma, muerte, DPPNI, parto
prematuro, HTA crnica.
-Consecuencias para el feto: Prematurez, RCIU, muerte fetal in tero, muerte
neonatal

19. EMBARAZO PATOLGICO : SNDROME DE HELLP



Paciente de 36 aos, cursando embarazo de 35 semanas, consulta por dolor epigstrico
y en hipocondrio derecho acompaado de vmitos. Refiere que orina poca cantidad y
que se est hinchando progresivamente.
Antecedentes familiares: madre diabtica, padre hipertenso.
Antecedentes personales: anorexia a los 18 aos.
Antecedentes ginecobsttricos: menarqua 13 aos, RM IV/28, G2P10.
Hbitos: sana
Al Examen fsico destaca PA 160/100, en exmenes de laboratorio se evidencia
proteinuria +++ y la ecografa obsttrica muestra feto creciendo en percentil 5.

1. Diagnstico apropiado
Multpara de 1
Embarazo 35 semanas
Sndrome Hipertensivo del Embarazo
o Pre eclampsia severa
o Obs. Sndrome de HELLP
Restriccin del Crecimiento Fetal.

2. Diagnstico diferencial
a. Hgado graso agudo del embarazo.
b. Hepatitis viral.

86
c. Colangitis
d. Colecistitis
e. Pielonefritis Aguda
f. Gastroenteritis aguda
g. lcera gstrica
h. Pancreatitis aguda
i. Trombocitopenia benigna del embarazo.
j. PTI
k. Dficit de folato
l. Lupus Eritematoso Sistmico
m. Sndrome Anti fosfolpidos.
n. PTT
o. Sndrome hemoltico urmico.

3. Definicin
Corresponde a un sndrome clnico caracterizado por hemlisis (H), elevacin de
enzimas hepticas (EL), y trombocitopenia (LP) que se presenta en el contexto de
Pre eclampsia. No hay consenso si corresponde a una variante de Pre eclampsia
severa, a una complicacin de ella o a una entidad separada.
Segn los hallazgos bioqumicos se puede clasificar en:
Clase 1: PQT <50.000, AST o ALT mayor a 70, LDH mayor a 600.
Clase 2: PQT <100.000 o mayor a 50.000, AST o ALT mayor a 70, LDH mayor a
600.
Clase 3: PQT <150.000, AST o ALT mayor a 40, LDH mayor a 600.

4. Etiologa:
Se asocia a sndrome hipertensivo del embarazo.

5. Epidemiologa:
Se presenta entre 0.5 a 0.9% de todos los embarazos, 10-20% del total de casos de
Pre eclampsia. 70% de los casos se presentan antes del parto, la mayora entre las
27 y 37 semanas. Slo 10% ocurre antes de las 27 semanas. Se puede presentar
dentro de las primeras 48 horas postparto en pacientes con hipertensin y
proteinuria. El 50% de los casos se preceden de edema generalizado y aumento de
peso.

6. Fisiopatologa:

87
Como se menciona anteriormente, no hay consenso de que el Sndrome de HELLP
corresponda a una variante de Pre eclampsia severa, a una complicacin de ella o a
una entidad separada. Sin embargo, fisiopatolgicamente, este sndrome se
comprende por cada uno de los elementos que lo conforman en el contexto de una
paciente con pre eclampsia, como se presenta a continuacin:
a. La Hemlisis es debida a una anemia hemoltica microangioptica, la
fragmentacin de glbulos rojos causa un rpido pasaje de clulas al endotelio
produciendo dao en la ntima, disfuncin endotelial y depsitos de fibrina. La
presencia de glbulos rojos fragmentados (esquistocitos) refleja hemlisis.
Aumentan los reticulocitos, LDH y bilirrubina no conjugada y disminuye la
concentracin de hemoglobina manifestando anemia en el hemograma, se
produce hemoglobinuria que es detectable en el 10% de pacientes. La hemlisis
libera haptoglobina que es rpidamente aclarada por el hgado lo cual hace que
disminuya la haptoglobina en el plasma a valores inferiores a 1-4 g/L (marcador
ms especfico de hemlisis).
b. La Elevacin de enzimas hepticas manifiesta injuria heptica mediante elevacin
de transaminasas.
c. La Trombocitopenia es manifestacin del consumo de plaquetas debido a que se
activan y adhieren a las clulas del endotelio daadas. Slo el 12% de plaquetas
menores a 150.000 corresponden a HELLP, sin embargo, cuando el recuento es
menor a 100.000 siempre se debe descartar este sndrome.
7. Diagnstico clnico
El diagnstico es clnico y de laboratorio (bioqumico), habitualmente en pacientes
cursando el tercer trimestre de embarazo, con diagnostico de pre eclampsia,
clnica sugerente y estudio de laboratorio con hemograma con recuento de
plaquetas, LDH y enzimas hepticas y bilirrubina compatibles. Basta con la
presencia de uno de los elementos bioqumicos para hacer el diagnstico del
sndrome de HELLP, sin embargo, para que sea un sndrome de HELLP completo
requiere de los 3 elementos diagnsticos: hemlisis, elevacin de enzimas
hepticas y trombocitopenia. Si slo hay 1 o 2 elementos corresponde a un
sndrome de HELLP parcial o incompleto.
8. Anamnesis y examen fsico
La manifestacin clnica caracterstica corresponde a una paciente embarazada
hipertensa con dolor abdominal superior o localizado en hipocondrio derecho y
epigastrio, nauseas y vmitos, el dolor abdominal puede ser fluctuante o de tipo
clico. La mayora de los pacientes presenta malestares varios das antes de la
presentacin del cuadro. Un 30-60% de las pacientes sealan cefalea, y un 20%
manifiesta sntomas visuales. No obstante, la mayora de las veces se presenta con
clnica inespecfica. Los sntomas se exacerban en la noche y disminuyen en el da.
Puede presentarse hasta 6 das despus del parto

88
9. Estudio de la paciente
A toda paciente con sospecha clnica de sndrome de HELLP se debe solicitar
hemograma con recuento de plaquetas, pruebas hepticas (enzimas hepticas y
bilirrubina) y LDH.
Los resultados esperados son: hemograma con disminucin de hemoglobina y/o
hematocrito y trombocitopenia habitualmente menor a 100.000, pruebas
hepticas donde se evidenciar aumento de transaminasas mayor a 40, aumento
de bilirrubina mayor a 1,2 mg/dl, LDH aumentada mayor a 600.
Imgenes complementarias slo son tiles en caso de sospecha clnica de
hematoma subscapular heptico, donde la paciente presentar dolor epigstrico
intenso o en hipocondrio derecho o en hombros, hipotensin, CID, o evidencia de
ascitis severa), en este caso es til un TAC de abdomen o ecografa abdominal.
Imagen: TAC que muestra hematoma
subscapular heptico. Deepak Joshi,
Andrea James, Alberto Quaglia, Rachel
Westbrook, Michael Heneghan. Liver
disease in pregnancy. The Lancet.
Volume 375, No. 9714, p594-605, 13 Feb
2010

10. Tratamiento y conducta
En primer lugar se hospitalizar para
valoracin inicial:
Estatus materno: control de presin arterial cada 5 minutos hasta estabilizar con
posterior control horario, ECG, control de diuresis horaria con sonda Foley.
Bienestar fetal: RBNE, ecografa obsttrica con doppler.PFB
Edad gestacional
Presencia de trabajo de parto o test de Bishop (evala madurez cervical para
condiciones de induccin del parto, considera la dilatacin, borramiento, posicin
del cuello uterino, consistencia del cuello uterino y encajamiento. Con
puntuaciones menores a 6 se indica mtodos farmacolgicos de maduracin antes
de induccin, con valores entre 7 y 9 las condiciones de induccin son favorables y
con valores mayores a 9 un parto espontneo es altamente posible)
Laboratorio: Hb, Hcto, PQT, pruebas de coagulacin, transaminasas, bilirrubinemia,
LDH, haptoglobina, orina completa, acido rico. Dopler y cardiotocografo para
evaluacin fetal. Toma seriada de presin arterial.
Estabilizacin materna con fluidos suero fisiolgico o ringer lactacto para mantener
diuresis mayor a 30-40 ml/hr. El uso de antihipertensivos para manejo de pre

89
eclampsia severa con objetivo de PAD entre 90-105 en el preparto y menos de
90mmHg en el postparto (con uso de labetalol, nifedipino o hidralazina), y sulfato
de magnesio para control de convulsiones.
La interrupcin del embarazo es el tratamiento definitivo (va ms expedita)
En gestacin de 34 semanas o ms la interrupcin del embarazo es inmediata. (el
uso de anestesia est contraindicado en recuento de plaquetas inferior a 100.000,
por lo que en esta condicin se deben transfundir plaquetas).
Entre las 27 y 34 semanas de gestacin se indica estabilizacin materna, uso de
corticoides y posterior interrupcin tras 48 horas para maduracin pulmonar
(Betametasona 12 mg im cada 24 horas x 2 veces). Si la condicin materna no es
favorable tambin est validado el uso de una dosis de corticoides (Betametasona
12 mg im por 1 vez) antes de la interrupcin del embarazo.
Beneficios de corticoides: Slo se ha demostrado que dosis habitual para
maduracin pulmonar disminuye el riesgo de hemorragia interventricular,
enterocolitis necrotizante, sndrome distres respiratorio y mortalidad
neonatal. No se ha logrado demostrar el efecto beneficioso de uso de
corticoides en altas dosis para la madre.
En gestaciones menores a 27 semanas el manejo es expectante o conservador
por ms de 48-72 horas. Requiere monitorizacin con Dopler y ecografa y uso de
antihipertensivos. Si la condicin materna empeora, se debe interrumpir mediante
cesrea de emergencia.
Otras terapias emergentes: Uso de antitrombina mejora la hipercoagulabilidad y a
diferencia de heparina, disminuye eventos hemorrgicos. Otros estudios muestran
que la estabilizacin de niveles de glutatin disminuyen el efecto de peroxidasas
disminuyendo la activacin de plaquetas y la resistencia de arterias uterinas.
Ciruga en ruptura heptica y embolizacin selectiva en caso de hemorragia
heptica.
Para el manejo postparto
30% de HELLP se desarrolla en 48 horas postparto, por lo que requiere
monitorizacin continua. Se administran corticoides (Dexametasona 10 mg cada 12
horas o metilprednisolona 40mg cada 12 horas) para acelerar la recuperacin
durante el puerperio temprano (primeros 6 das) para lograr recuento plaquetario
mayor a 100.000 postparto o mayor a 150.000 en el preparto, esto no est
totalmente validado. En condiciones en donde aumenta bilirrubina y creatinina en
las primeras 72 horas del postparto, la madre se puede beneficiar del uso de
plasma fresco congelado. Si persiste la hemlisis, trombocitopenia e
hipoproteinemia, se debe administrar glbulos rojos, plaquetas y albmina. Si

90
persiste la falla renal, se debe administrar bolos endovenosos de suero fisiolgico
o ringer lactato de 250 ml y si requiere, monitorizacin con catter venoso central.
Respecto al uso de heparina en CID, PE y HELLP la mayora de los autores no lo
valida por el riesgo de hemorragias.

1. Consecuencias para la paciente y el feto
Las consecuencias para la madre:
Indicadores de alta morbimortalidad materna son (en un 75% de los casos): LDH
mayor a 1400, AST o ALT mayor a 100 o 150, cido rico mayor a 7,8.
Algunas de las complicaciones son: eclampsia, DPPNI, CID (5-56%), falla renal
aguda (7-36%), ascitis severa, edema cerebral, edema pulmonar, hematoma
subscapular heptico, hemorragia postparto, ruptura heptica (puede ocurrir en el
postparto), infarto heptico, trombosis recurrente, desprendimiento de retina,
ACV, hemorragia cerebral, muerte materna (1,1%). Las de mayor riesgo
corresponden a CID, DPPNI y hematoma subscapular heptico que se asocia a 50%
de mortalidad materna.
- CID: Con recuento de plaquetas menores a 100.000 ocurre en aproximadamente
38% de los casos debido a una activacin del endotelio vascular y plaquetaria. La
concentracin plasmtica de fibringeno es menor a 3 g/l, productos de
degradacin del fibringeno como el dmero D, aumentan a ms de 40 mg/l. La
disminucin de antitrombina (anticoagulante producido por el hgado) por la falla
heptica aumenta el consumo de los factores de la coagulacin, habitualmente la
CID se relaciona a DPPNI.
Las consecuencias para el Feto:
Puede ocurrir muerte perinatal (7-34%), RCIU (38-71%) por insuficiencia
placentaria, parto prematuro (70%), trombocitopenia neonatal (15-50%), sndrome
distres respiratorio (5-40%).
Tiene gran importancia la educacin preconcepcional para embarazos posteriores:
Los anticonceptivos orales son seguros en este tipo de pacientes. Es importante
destacar que la paciente que tuvo un sndrome de HELLP, tiene riesgo entre 5-52%
(promedio 20%) de hacer alguna forma de hipertensin gestacional en los
embarazos siguientes, de ah la importancia de la anticoncepcin.

Referencias:
a. Kjell Haram, Einar Svendsen, Ulrich Abildgaard. The HELPP syndrome: Clinical
issues and management. A review. BMC Pregnancy and Childbirth 2009, 9:8.

91
b. Drs. Alejandro Pattillo G, Enrique Oyarzn E. Sndromes Hipertensivos del
Embarazo. Captulo 13. Obstetricia. Departamento de Obstetricia y
Ginecologa Facultad de Medicina. Pontificia Universidad Catlica de Chile
c. F. Crispi. Pre eclampsia control antenatal. Curso Intensivo en Medicina
Materno fetal. Cardona (Barcelona), del 4 al 9 de Marzo del 2012. Pg. 27.
d. S Hernndez, J. Bellart. Complicaciones graves de la pre eclampsia. Curso
Intensivo en Medicina Materno fetal. Cardona (Barcelona), del 4 al 9 de
Marzo del 2012. Pg. 33.
e. Deepak Joshi, Andrea James, Alberto Quaglia, Rachel Westbrook, Michael
Heneghan. Liver disease in pregnancy. The Lancet. Volume 375, No. 9714,
p594-605, 13 Feb 2010


20. EMBARAZO PATOLGICO : DIABETES PRE -
GESTACIONAL

Paciente de 35 aos diabtica desde los 25 aos, insulino dependiente, cursando
embarazo de 31 semanas que consulta por glicemia en su domicilio de 200 mg/dL y Pa
140/100 mm Hg.

Antecedentes Familiares : madre hipertensa , padre diabtico


Antecedentes Personales: Diabetes tipo II diagnosticada hace 10 aos en
tratamiento con glibenclamida y dieta y ejercicio.
Antecedentes Gineco obsttricos : menarquia 13 aos , RM IV/28 G3P0A2 a las 8
y 10 semanas
Hbitos : sana
Antes del embarazo se realiz un examen metablico y clnico completo (incluyendo
fondo de ojo, PA, orina completa, clearence de creatinina, ECG) que fue normal y se
cambi de hipoglicemiantes a insulina. El control prenatal y los exmenes de rutina han
sido normales hasta el momento del control.
1. Diagnstico:
M2
Embarazo 31 sem
Abortos anteriores
Diabetes pre gestacional IR descompensada
Obs SHE

92

2. Diagnsticos diferenciales:
Preeclampsia
3. Definicin:
Aquella paciente que presenta una alteracin en el metabolismo de los
carbohidratos conocida previamente al embarazo. Su pronstico depender de los
daos a rgano blanco.
El sistema de clasificacin ms utilizado es el de Priscilla White White, el cual
categoriza a las pacientes en funcin de la estabilidad de la enfermedad y de la
presencia o no de lesin orgnica. Estos criterios son importantes para determinar
el pronstico del embarazo. ste sistema divide a las pacientes en grupos segn la
edad de comienzo y los aos de duracin de la enfermedad, as como la presencia
o ausencia de enfermedad vascular. Ha sido de vital importancia para el momento
de la programacin del embarazo, control, tratamiento y decisin momento parto.

93

4. Etiologa:

En su etiologa intervienen tanto factores tanto genticos como inmunolgicos que
condicionan un dao irreversible a las clulas beta de los islotes de Langerhans,
con ausencia o reduccin muy, importante de la secrecin de Insulina. Los islotes
pancreticos presentan insulitis, que se caracteriza por una infiltracin de linfocitos
T acompaada con macrfagos y linfocitos B, con la prdida de la mayora de las
clulas beta. Se cree que los mecanismos inmunitarios mediados por clulas
representan el principal papel en la destruccin de las clulas beta.
Algunos de los anticuerpos presentes en los Islotes pancreticos son citotxicos
para las clulas beta, pudiendo estos contribuir a su destruccin. En las
poblaciones de raza blanca existe una fuerte asociacin entre la DM tipo1
diagnosticada antes de los 30 aos y fenotipos HLA HLA-D especficos. Se cree que
uno o ms genes portadores de la susceptibilidad a la DM tipo1 estn localizados
en el locus HLA HLA-D o cerca de l en el cromosoma 6.
Todos los datos indican que la susceptibilidad gentica a la DM tipo1es
probablemente polignica.

5. Fisiopatologa:
1.- Aborto y Diabetes Mellitus tipo 1.
Varios estudios han demostrado que el Aborto espontneo se asocia con el mal
control de la Glicemia durante el primer trimestre de gestacin (15 15-30%, slo en
pacientes mal controladas.)
Se presenta en mujeres con una concentracin de hemoglobina glicosilada A1
inicial mayor del 12% o concentraciones preprandiales de glucosa persistentes por
encima de 120 mg/dL.

2.- Embriopata Diabtica.
Las edades gestacionales ms frecuentes para la induccin de malformaciones
varan entre la tercera y sexta semana postconcepcin. Aunque se ha establecido
un fuerte vnculo entre la hiperglicemia y las malformaciones, no se ha dilucidado
por completo el mecanismo preciso que se encarga del desarrollo fetal anormal. Se
han postulado varias teoras como son:
Antecedentes Genticos.
Hiperglicemia Hipercetonemia.
Inhibidores de la Somatomedina.
Deficiencia de cido araquidnico, mioinositol o ambos.
Generacin de radicales libres de Oxgeno.
El peso de las pruebas en distintas publicaciones apoya la hiptesis de que la
diabetes induce un estado aberrante de energticos metablicos, pudiendo causar
dao morfolgico al embrin en desarrollo.

94
La Hiperglicemia lleva a estados deficitarios de lpidos de membrana (Mioinositol y
cido Araquidnico Araquidnico), sta lleva a una mayor entrada de Glucosa a la
clula con dao mitocondrial, sobresaturacin de los sistemas enzimticos
rastreadores de radicales libres, ocurriendo as un exceso de estos radicales
potencialmente teratognicos.

3.- Muerte Fetal Intrauterina Fisiopatologa.
An se desconoce el mecanismo exacto de la muerte fetal intrauterina en
embarazos complicados por Diabetes Mellitus. No obstante, se ha aclarado que
cuando las cifras de Glicemia se mantienen dentro de lmites fisiolgicos, rara vez
ocurren bitos. A menudo se observa hematopoyesis extramedular en bitos
fetales, lo que indica que la posible causa de su muerte fue la hipoxia crnica
intrauterina.
Por otro lado, la hipoxia fetal crnica puede exacerbarse ms en pacientes mal
controladas, en quienes la desviacin a la izquierda de la curva de disociacin de la
oxihemoglobina puede causar una mayor afinidad de la hemoglobina y, por tanto
disminucin del aporte de oxgeno a los tejidos. Las lesiones placentarias
frecuentes en embarazos de diabticas, como inmadurez de vellosidades y
senescencia prematura, producto de dao hipxico y reparacin, tambin pueden
alterar el intercambio intervelloso. Por su parte, la respuesta fetal a la
hiperglicemia materna tambin contribuye a la asfixia intrauterina. La
hiperinsulinemia fetal puede aumentar la tasa metablica fetal y as tambin los
requerimientos de oxgeno.
Se postula que la muerte podria producirse por edema placentario que llevaria a
interrupcion del flujo sanguineo.

4.- SHE en diabetes pre gestacional (20-30%)
Es la principal causa de interrupcin de embarazo de pre trmino, tiene una
mortalidad perinatal aumentada en 20 veces. No tiene relacin con el control
glicmico.
En la mayora de los casos la funcin renal permanece estable, pero en algunas
ocasiones se puede producir una disminucin permanente o temporal de la
funcin renal, observndose una disminucin del clearence de creatinina. El
clearence de creatinina es el parmetro ms importante como indicador
pronostico. Los clearence < 50 ml/min se han asociado a una alta prevalencia de
SHE y perdida fetal.

5.- Infecciones:
80% de las diabticas insulinorrequirientes desarrollar al menos un episodio de
infeccin
Localizacin: tracto genital, tracto respiratorio, PNA. Son la principal causa de
descompensacin y de hospitalizacin.

95

6. Factores de riesgo:
Mal control metablico, mal uso de insulina, factores de riesgo para ITU y para
infeccin del tracto respiratorio, obesidad , embarazo

7. Epidemiologia:
1 a 14% de las embarazadas poseen diabetes
Sobre los 40 aos se reporta una prevalencia de hasta 10%
DM 1: 1/1000
Aumento de la frecuencia de DM2 y embarazo en aproximadamente, un 65% de
las Diabetes pre-gestacionales
Prevalencia en Chile:
o D. Pre-gestacional: 7,5% segn la encuesta realizadas a nivel nacional el ao
2006.
o D. Gestacional: entre 3 - 5 %, frecuencia que aumenta a un 10 14 % si se
consideran las embarazadas con factores de riesgo de diabetes
,
8. Diagnstico:

1. Dos glicemias en ayuna 126 mg/dL
2. Una glicemia 200 mg/dL, en cualquier momento del da, asociado a sntomas
clsicos de diabetes (4P)
3. Glicemia 200 mg/dL luego de 2 hrs post carga de glucosa de 75 mg durante una
PTGO

9. Anamnesis:
Control metablico, alteraciones en la dieta
Uso de la insulina, dosis, horario de uso, forma de uso,
Ecografas y Exmenes de laboratorio anteriores durante este embarazo
Perdida de flujo por la vagina, mal olor
Si ha sentido los movimientos fetales
Clnica de diabetes:
O Prdida de peso a pesar de un aumento del apetito
O Infecciones frecuentes, incluyendo las de vejiga, vagina y piel
O Fatiga
O Nuseas y vmitos
O Aumento de la sed
O Visin borrosa
O Incremento de la miccin
O Alteraciones de conciencia
Clnica de SHE: epigastralga, cefalea, tinitus, fotopsias, orina espumosa
Sntomas de infeccin, sobretodo urinario (disuria, poliaquiuria, tenesmo)

96
Historia de los abortos anteriores

10. Examen fsico:
Estado general: PA, FC, T, signos de infeccin tracto respiratorio
Evaluacin obsttrica: bienestar fetal, altura uterina, LCF,

11. Estudio de la paciente:
Una vez hecho el diagnstico de embarazo, debemos realizar exmenes sobre todo
en la primera semana para disminuir el riesgo de malformaciones congnitas.
- Orina completa + Urocultivo pensando en posible infeccin urinaria
(leucocituria, bacterias abundantes).
- Proteinuria de 24 hrs (Pensando en SHE).
- BUN/Crea
- ECG
- Fondo de ojo
Control de PA en 6 hrs mas

12. Tratamiento: caso 19
Rgimen con aporte de caloras adecuado ms hidratos de carbonos
Aporte calrico:
30-35 kcal/k de peso ideal.
Obesas: 25 kcal/k.
Mnimo 1500 kcal y 160 g de H de C
o Fraccionamiento de las comidas: 3 o 4 comidas y 2 colaciones
Insulinoterapia

Esquemas insulina:
1.- Glicemia de ayuno alterada:
NPH 0.1- 0.3 U/kg/d: 2/3 dosis maana y 1/3 dosis noche
NPH+IC
Obesas 0.5-0.7 U/K
Relacin Matinal NPH:IC 2:1
Relacin Vespertina 1:1

2.- Glicemia postprandial alterada con glicemia de ayuno normal:
IC 2-4 U antes del almuerzo y comida (antes desayuno si es necesario)

Autocontrol glicemia:
Glicemias en sangre capilar.
Si est con Dieta:
Control 3-4 veces por semana.

97
1 vez/sem con Glicemia en ayuno.
Si esta con Insulina: Control 3-4 veces al da.

Educar a la paciente, autocontrol
Actividad fsica
Control mdico c/2 semanas hasta las 28 semanas y luego semanal

En este caso hay que determinar cul es la causa de la descompensacin, y tratar la
causa. Las posibles causas son infecciosa, mal uso del medicamento, transgresin
alimentaria.

Momento de interrupcin del parto:
- Clase B-D White: 36-38 semanas
- Clase F-H White: 34-36 semanas
Via del parto:
- Va vaginal es la indicada o sin contraindicaciones.
- Cesrea si peso mayor a 4.3 Kgs.


13. Consecuencias maternas y fetales:
Fetales: aumenta la morbimortalidad
0-7 semanas: aborto, malformaciones (cardiacas: miocardiopata
hipertrfica; msculos esquelticas: regresin caudal; SNC: anencefalia;
Alteraciones genitourinarias).
8-40 semanas: PHA, Macrosoma, Retardo en el crecimiento, Muerte
fetal en tero, Parto prematuro.
Recin nacido: Hipoglicemia, Hipocalcemia, Policitemia,
Hiperbilirrubinemia, Luxacin hombro dao plexo braquial, Distress
respiratorio, Asfixia neonatal.
Maternos:
Pre eclampsia (40-50%). No depende del control metablico
Complicaciones metablicas de la diabetes
Agrava la retinopata.
Nefropata
Aumentan las infecciones

Bibliografa
1. Araya F., Dra. Rosa. (2009). Diabetes y Embarazo. REV. MED. CLIN. CONDES, 20(5),
614-629.
2. CEDIP. (2013). Diabetes y Embarazo. Gua Perinatal. 173-188

98
21. EMBARAZO PATOLGICO : DIABETES
GESTACIONAL.

Paciente de 36 aos cursando embarazo de 26 semanas que acude a control de rutina
prenatal con matrona y en exmenes se encuentra glicemia post carga de 75 gr de
glucosa a las 2 hrs de 156mg/dL
Antecedentes Familiares: madre diabtica insulino dependiente, padre
hipertenso
Antecedentes personales. Anorexia a los 18 aos.
Antecedentes Gineco obsttricos: menarquia a los 13 aos, RM IV/28 G2P1A0,
RNPT 36 sem 3800grs
Hbitos: sana

1. Diagnstico:
Multipara de 1
Emb 26 sem
Diabetes gestacional

2. Diagnsticos diferenciales:
Diabetes pre gestacional

3. Definicin:

El trmino diabetes mellitus gestacional (DMG) describe a las mujeres con
intolerancia a la glucosa de cualquier severidad, que se inicia o pesquisa por
primera vez durante el embarazo. Caracterizado por un dficit absoluto o relativo
de insulina que resulta en un estado hiperglicemico.

4. Etiologas:

Diabetes y Embarazo (2009), REV.MED.CLC

99


5. Fisiopatologa:
- 1 trimestre: Estrgenos y progesterona inducen hiperplasia de las clulas beta del
pncreas, esto se traduce en un aumento de la secrecin de insulina, aumento de
la utilizacin perifrica de glucosa y disminucin de los niveles de glicemia en
ayuno en un 10-20% con respecto a los niveles pre-gestacionales
- 2 trimestre: La demanda fetal por nutrientes aumenta, producindose la
movilizacin de los depsitos de glucosa materna, glucogenolisis heptica y
resistencia a la insulina mediada por el aumento de lactgeno placentario (el
principal), PRL, cortisol, resistina, por lo tanto, aumenta la glicemia post prandial.
Accin diabetognica leve
- Estrgenos: Cortisol libre
- Progesterona: Glicemia
- Prolactina: Cortisol
- Cortisol: Insulinemia
Accion diabetognica mayor
- Lactgeno placentario: lipolisis materna y accin de resistencia perifrica por
alteracin transportador
- Factor de necrosis tumoral: aumento de manera considerable en el 3Trimestre.
Como factor independiente de la resistencia a la insulina durante el embarazo

6. Factores de riesgo:
Maternos:

100
Antecedentes familiares de 1 grado de DM
Edad 30 aos
Obesidad IMC >30
SOP
Antecedente en embarazos anteriores:
DMG
PHA
Mortalidad perinatal inexplicada
RN GEG o macrosomico >4000 grs
Parto prematuro

Fetales:
Feto actual creciendo percentil >90 / Macrosoma
Malformaciones congnitas
Sndrome hipertensivo
Polihidroamnios


7. Epidemiologia:
Incidencia mundial 7% de todos los embarazos.
Incidencia chilena de 1 -4 %
Si hay factores de riesgo la incidencia aumenta a un 10-14%.
Mayor incidencia en la ltima dcada: por aumento de la obesidad intolerancia
a la glucosa y DM2, cada vez a edades ms tempranas.
Condicin de alto riesgo, generalmente asintomtica.
67% sern cesreas.

8. Diagnstico clnico:

El diagnstico es con laboratorio:

Glicemias en ayuna 105 mg/dl
Glicemia 140 mg/dl a las 2 hrs de 75 grs de TTGO
Glicemia 200 mg/dl

Actualmente tambin se est usando hemoglobina glicosilada 6.5, pero en chile
casi no se usa por un problema de estandarizacin de los laboratorios.

101



9. Anamnesis:

Caractersticas del embarazo anterior: va parto, pre trmino, glicemias,
infecciones durante el embarazo, polihidromaniois, macrosomia.
Ecografas y Exmenes de laboratorio anteriores durante este embarazo
Hay que preguntar por los factores de riesgo, caractersticas de la dieta, uso de
frmacos,
Clnica de diabetes:
- Prdida de peso a pesar de un aumento del apetito
- Infecciones frecuentes, incluyendo las de vejiga, vagina y piel
- Fatiga
- Nuseas y vmitos
- Aumento de la sed
- Visin borrosa
- Incremento de la miccin
- Alteraciones de conciencia


10. Examen fsico:
Estado nutricional (obesidad)
Altura Uterina compatible con polihidroanmios y/o macrosoma fetal
Bienestar fetal (LCF)
Signos de hiperglicemia: Acantosis nigricans, acrocordones
Edema y varices en EEII


11. Estudio de la paciente:

102
Solicitar orina completa, urocultivo buscando cetonuria, glucosuria o signos de
infeccin.
Ecografa obsttrica: evaluar polihidroamnios, madurez de placenta, biometra
fetal


12. Tratamiento:

Objetivos del tratamiento:
Obsttricos: Metablicos:
Disminucin de la macrosomia fetal Glicemia de ayuno entre 70 y 90 mg/dl
Lograr embarazo de termino Glicemias preprandiales 90 y 105
Glicemias post prandiales a las 2 hrs entre 90-120
Evitar trauma obsttrico Cetonurias negativas
Disminuir complicaciones metablicas del RN Glucosurias negativas

No se hospitaliza inmediato, solo si la glicemia sale > 200
Se deriva a alto riesgo obsttrico para:
2. Examen clnico obsttrico
3. La frecuencia de las consultas ser cada 2 semanas hasta las 32 semanas de
gestacin y luego 1 vez por semana hasta el parto.
4. En cada control evaluar
1. PA
2. Aumento ponderal
3. IMC
4. Deteccin de edema y vrices en miembros inferiores
5. Altura uterina
6. FCF
7. Movimientos fetales
5. Ecografa obsttrica cada 4 semanas para valorar el crecimiento fetal, volumen de
lquido amnitico, espesor y madurez de la placenta. Si presenta polihidroamnios
realizar estudio de vitalidad fetal (doppler y perfil biofsico)
6. Biometra fetal. Determina: dimetro biparietal (DBP), CC, longitud femoral (LF),
CA, para estimacin del peso fetal.
7. El tratamiento est basado en 3 pilares fundamentales: dieta, frmacos y
ejercicios.

1.- Dieta:
90% de pacientes con DMG se controlan
adecuadamente solo con dieta.
Plan de alimentacin segn estado nutricional y
actividad fsica.

103
Evitar alza desproporcionada de peso.
Aporte calrico:
30-35 kcal/k de peso ideal.
Obesas: 25 kcal/k.
Mnimo 1500 kcal y 160 g de H de C
Fraccionamiento de las comidas: 3 o 4 comidas y 2 colaciones.
Se indica dieta en toda paciente con glicemias de ayuno normales y postprandiales
120-200 mgs/dL. Por una semana antes de iniciar insulina
Luego de 7 das de tratamiento estricto: Reevaluar.
Si glicemias postprandiales 120-130 mgs/dL: Dieta por 7 das ms + Reforzar el
autocontrol y la educacin.

2.- Insulinoterapia:
Inicio insulina:
Luego de 7 das de tratamiento diettico: Glicemia >130 mgs/dL.
Luego de 14 das de tratamiento diettico: Glicemia postprandial (2hrs) >120
mgs/dL.
Glicemia de ayunas >/=105 mgs/dL. (En ms de una ocasin) o glicemia
postprandial >200 mgs/dL.

Esquemas insulina:
1.- Glicemia de ayuno alterada:
NPH 0.1- 0.3 U/kg/d: 2/3 dosis maana y 1/3 dosis noche
NPH+IC
Obesas 0.5-0.7 U/K
Relacin Matinal NPH:IC 2:1
Relacin Vespertina 1:1
2.- Glicemia postprandial alterada con glicemia de ayuno normal:
IC 2-4 U antes del almuerzo y comida (antes desayuno si es necesario)

Autocontrol glicemia:
Glicemias en sangre capilar.
Si est con Dieta:
Control 3-4 veces por semana.
1 vez/sem con Glicemia en ayuno.
Si esta con Insulina: Control 3-4 veces al da.

3.- Ejercicio:
Prescripcin individualizada
Ejercicio de msculos grandes
Duracin 30-45 minutos diarios

104
Tipo: bicicleta estacionaria, natacin, caminar, ejercicio de brazos (de
grandes articulaciones)
Monitoreo de: Glicemias, ganancia de peso y unidad feto placentaria

Interrupcin del embarazo:
Paciente diabtica sin otras patologas:
Diabtica sin insulinoterapia: 40 semanas
Diabtica con insulinoterapia: 38 semanas
Con patologa asociada:
38 semanas
Va del parto:
Va vaginal si no existe contraindicacin.
Cesrea electiva si EPF >4000 grs. (por el riesgo de distocia de
hombros

Manejo del Trabajo de parto:
Diabtica sin insulinoterapia:
o Infundir S. Glucosado 5% 125 cc/hrs.
Diabtica con insulinoterapia:
o Suspender insulina de la maana.
o S. Glucosado 5% 125 cc/hrs.
o Mantener glicemias 70-120 mgs/dL.
o Suspender Insulina luego del parto.
o Parto programado:
o Glicemia en ayunas y luego control cada 2 hrs.
o >120 mg/dL: Administrar insulina cristalina (5 U/500 cc de
SF).
o Cesrea electiva:
o Ciruga a primera hora.
o Administrar Insulina cristalina 1U/hr
Post parto:
Evaluar glicemia de ayuno.
Si glicemia de ayuno est elevada al 3 da, se cataloga como Diabetes
Mellitus.
Realizar PTGO 6-8 semanas postparto para reclasificar.

13. Consecuencias para la madre y feto:
Fetales: aumenta la morbimortalidad
8-40 semanas: PHA, Macrosoma, Retardo en el crecimiento, Muerte
fetan en tero, Parto prematuro.

105
Recin nacido: Hipoglicemia, Hipocalcemia, Policitemia,
Hiperbilirrubinemia, Luxacin hombro dao plexo braquial, Distress
respiratorio, Asfixia neonatal.
Maternos:

Descompensacin metablica durante el embarazo.


Infecciones.
Trauma obsttrico, por aumento de parto instrumental (por
macrosoma)
Ms de la mitad de la embarazadas desarrollan diabetes o sndrome
metablico a largo plazo.
Bibliografia
3. Araya F., Dra. Rosa. (2009). Diabetes y Embarazo. REV. MED. CLIN. CONDES, 20(5),
614-629.
4. CEDIP. (2013). Diabetes y Embarazo. Gua Perinatal. 173-188

22. EMBARAZO PATOLGICO : ENFERMEDAD


HEMOLTICA PERINATAL.

Mujer de 32 aos cursando embarazo de 32 semanas, derivada por matrona por
presentar test de Coombs indirecto 1:32 anti d. Inicialmente el titulo fue de 1:2 al
comienzo del embarazo. Su grupo sanguneo es AII Rh(-) y su pareja AII Rh(+). El resto de
los exmenes son normales .
Antecedentes familiares : Madre diabtica padre hipertensa
Antecedentes personales : Hernia inguinal operada .
Antecedentes gineco obsttricos : menarquia 13 aos , RM IV/28 G2P1aA0
Hbitos : Sana

1. Diagnsticos:
-Multpara de 2
-Embarazo de 32 semanas
-Enfermedad hemoltica perinatal por incompatibilidad de grupo Rh
2. Definicin:
La incompatibilidad sangunea materno fetal es la presencia de uno o ms Ag del
glbulo rojo fetal que no posee la madre, provocando una respuesta mediada por
inmunoglobulinas maternas.
3. Etiologa:

106
El riesgo de aloinmunizacin en una mujer Rh-D negativa susceptible que tiene
contacto con sangre Rh + depende de 3 factores:
1.Volumen de la hemorragia feto-materna
2.Grado de respuesta inmune materna
3.Incompatibilidad ABO

4. Fisiopatologa:
Sistema Rh presenta distintos antgenos
C,c
D,d*
E,e

* d: ausencia de alelo D
El antgeno D del sistema Rh es el de mayor capacidad antignica
Los Ac anti-D del sistema Rh son los responsables de la mayora de los casos de
EHP detectables clnicamente
Rh(+): posee el antgeno D en la membrana del GR
Rh(-): no tiene antgeno D
El glbulo rojo fetal (sangre Rh (+)) tiene contacto con la circulacin materna Rh(-),
los antgenos de superficie del glbulo rojo fetal estimulan el sistema inmune
materno y se desarrollan anticuerpos especficos ant-RH, los anticuerpos maternos
(Ig G) que pasan a travs de la placenta hacia la circulacin fetal se unen al
antgeno en la superficie del glbulo rojo fetal, estos aglutinan y hemolizan los
glbulos rojos fetales desarrollndose hemlisis y anemia fetal.
Ante una primera exposicin, la respuesta inmunitaria primaria al antgeno es dbil
y los anticuerpos iniciales son IgM, que, por su alto PM (900.000) , no pasan a
travs de la placenta y no producen hemlisis de los glbulos rojos fetales.
La respuesta inmune 2 puede producirse despus de la exposicin de muy
pequeas cantidades del antgenos, es intensa y los anticuerpos son de predominio
Ig G de bajo peso molecular (160.000).

1. Hemlisis anemia fetal
2. sntesis EPO fetal
3. Se estimula eritropoyesis en MO y eritropoyesis extra medular (hgado y bazo)
hepatoesplenomegalia
4. Se liberan formas inmaduras de GR a la circulacin fetal (eritroblastos)
5. Parnquima heptico es reemplazado por focos de eritropoyesis alteracin
sntesis proteica (hipoalbuminemia) e HT portal edema y ascitis hidrops
6. Anemia insuficiencia cardiaca congestiva
7. In tero: hemlisis bilirrubina no conjugada (indirecta) en feto
placenta la conjuga y excreta
8. Post-parto: el hgado del RN es incapaz de conjugar bilirrubina eficazmente
(inmadurez enzimtica) ictericia neonatal

107

http://escuela.med.puc.cl/paginas/departamentos/obstetricia/altoriesgo/enf_hemol.html
#Figura 1


5. Factores de riesgo:
a. Antecedente de transfusin
b. Incompatible madre rh (-) y cnyuge rh (+)
c. Multiparidad
d. Trauma abdominal
e. Placenta previa y DPPNI
f. Aborto
g. Embarazo ectpico
h. Procedimientos obsttricos: amniocentesis, versin externa, alumbramiento
manual, cordocentesis, extraccin manual de la placenta, biopsia de vellosidades
coriales
i. Drogadiccin endovenosa
j. Parto
k. Operacin cesrea

6. Epidemiologa:
Se ha demostrado hemorragia feto-materna en 75% embarazos, la
frecuencia y volumen aumentan en la medida que avanza edad gestacional
el 2% de las embarazadas con feto Rh D (+), pueden provocar una respuesta

108
inmune durante el embarazo, habitualmente despus de las 28 semanas de
gestacin, la mayora ocurre durante el parto.


7. Anamnesis:
Se le preguntar a las embarazadas por sus antecedentes:
a. Frmula obsttrica : nmero de partos previos
b. Grupo sanguneo hijos anteriores y del padre.
c. Si recibi profilaxis anti-d
d. Morbimortalidad perinatal atribuible a hemlisis
e. Transfusiones previas

8. Estudio de la paciente:
Manejo de pacientes Rh(-)
1. Grupo sanguneo, Rh, test de Coombs indirecto en sangre materna a toda
embarazada en 1 control prenatal

Si Coombs indirecto negativo = madre no sensibilizada
Si Coombs indirecto positivo = madre sensibilizada

2. Si madre Rh(-), evaluar estado Rh paterno

3. Si padre Rh(-) feto Rh-D (-) sin riesgo de anemia fetal control nivel
1

4. Si padre Rh(+) Se puede evaluar cigocidad:
a. Homocigoto (D,D) feto Rh-D (+) 100%
b. Heterocigoto (D,d) 50% probabilidad feto Rh-D (+)

5. Si padre heterocigoto (D,d), se puede determinar Ag D fetal mediante:
Biopsia Vellosidades Coriales, AMCT: PCR de elementos celulares fetales
Anlisis DNA libre fetal en circulacin materna



Manejo paciente Rh(-) sensibilizada( nuestra paciente)

1. Coombs indirecto mensual hasta las 24 sem, luego cada 2 semanas

2. Si ttulos Ac permanecen < 1/16, seguimiento ultrasonogrfico y parto a


trmino

3. si ttulo Ac 1/32 estimar grado de anemia fetal mediante:


109
1) ) Dopler fetal: peak sistlico Arteria Cerebral Media ( ACM )

En fetos anmicos, aumenta velocidad del flujo sanguneo debido a la
disminucin en la viscosidad sangunea
Por qu se prefiere ACM?
Sensibilidad 88 -99%, FP 12% para anemia fetal moderada / severa
Es posible evaluar desde 18 sem
Control cada 2 semanas (segn tendencia)
Sin riesgos para el feto

2) AMCT para espectrofotometra de LA:

-La hemlisis de GR fetales aumenta niveles de bilirrubina en LA
-Extraccin de LA por AMCT guiada por ecografa



Anlisis espectrofotomtrico del LA:

Se mide densidad ptica (DO) en un rango que va de 300 a 700 mu de longitud de
onda (LO)

Se dibuja curva espectrofotomtrica en un grfico:
Abscisa (eje x): LO (corresponde a bilirrubina)
Ordenada (eje y): DO

A mayor elevacin del peak, mayor gravedad de hemlisis fetal

La diferencia de DO entre la lectura obtenida a 450 m* y la considerada normal
para el embarazo (lnea recta entre puntos obtenidos a 375 y 550 m), da un valor
(delta DO) que se traslada a un grfico

110
(*) La longitud de onda en que bilirrubina absorbe la luz es 420-460 nm

Zona en esquema de Conducta
Liley
2
Zona A AMCT cada 3-4

sem

Zona B baja (B1) AMCT cada 2 sem
Zona B alta (B2)
Cordocentesis o
interrupcin si >34
Zona C
sem




Si Vmx ACM 1,5 Mom cordocentesis

Ventaja: reduce necesidad de procedimientos invasivos en 50%

Despus de las 35 sem, tasa falsos (+) en deteccin anemia fetal
Por lo tanto, en fetos > 35 sem, si Vmx ACM > 1,5 MoM
AMCT para espectrofotometra LA y evaluacin madurez pulmonar fetal
Evaluar si es necesario interrumpir

111
3) Cordocentesis

nico mtodo de diagnstico definitivo de anemia fetal y acidosis
Acceso directo a vena umbilical



9. Tratamiento:
Profilaxis pacientes no sensibilizadas: Ig G anti-D 300 mcg IM a las 28 sem y 72
hrs post-parto.

Paciente sensibilizadas : depende del grado de anemia fetal cuantificada en los
estudios anteriores.

112


10. Consecuencias para la madre y el feto:
a. aborto
b. muerte fetal
c. RN: hidrops fetal, anemia congnita y sd. ictrico
d. 20% kernicterus y el 80% de estos fallece y el resto sordera, espasticidad con
movimientos atetosicos, hipoacusia.

Bibliografa
1) Obstetricia, Prez-Sanchez, cuarta edicin, 2011
2) Gua perinatal CEDIP 2011
3) Erythrocyte Alloimmunization and Pregnancy (Up to Date, june 2006)
4) Management of Rhesus alloinmunization in pregnancy (Obstet Gynecol
2008;112:164-76)

113

23. EMBARAZO PATOLOGICO : EMBARAZO


GEMELAR.

Paciente de 27 aos cursando embarazo gemelar doble de 10 semanas de gestacin
ingresa a control prenatal.
Antecedentes Familiares: madre diabtica padre hipertenso
Antecedentes Personales: tonsilectomia
Antecedentes Gineco obsttricos: menarqua 13 aos , RM IV/28 60 G1P0A0
Hbitos: sana

1) CUAL ES EL DIAGNOSTICO MAS PROBABLE DE ESTA PACIENTE?
- Primigesta
- Embarazo 10 semanas
- Embarazo mltiple (2)
2) CUALES SON LOS DIAGNOSTICOS DIFERENCIALES?
Previo a ecografa: Embarazo de mayor EG, Macrosomia fetal, PHA, Tumor
abdominal.
3) COMO SE DEFINE ESTA PATOLOGIA ?
Gestacin de ms de un feto en la cavidad uterina, independiente del mecanismo
de como resulto la fertilizacin.
4) CUAL ES/SON LA/S ETIOLOGIA Y FISIOPATOLOGIA DE ESTE CUADRO CLINICO ?
Es una divisin temprana del producto de la concepcin. Dependiendo de si
provienen de un cigoto o dos (cigocidad: monocigoto (30%) dicigoto (70%)) y el
momento de la divisin se establecera el diagnostico definitivo del tipo de
embarazo multiple. Si la divisin es en los das: 1-3 bicorial-biamniotico (20-
30%); 4-8 Monocorial-biamniotico (70-80%); 8-13 Monocorial-
monoamniotico (<1%); >13 siameses.

114




6) QUE FACTORES DE RIESGO BUSCARA?
Terapia de fertilizacin asistida (estimulacin ovrica, inseminizacin artificial, etc),
edad de paciente, antecedentes familiares.
Como factores pronsticos, el ms importante en embarazo gemelar monocigotico
es la corionicidad, siendo el mocorial el con mas riesgos.

7) CUAL ES LA EPIDEMIOLOGIA DE ESTE CUADRO ?
1,4% del total de embarazos. 12-14% de muerte perinatal. En USA desde los aos
80 el EM ha aumentado 52% y el embarazo triple en un 400%. 10-15% de los PP
<34s.

8) COMO HACEMOS EL DIAGNOSTICO CLINICO ?
La sospecha es clnica cuando durante el segundo trimestre la altura uterina es
evidentemente ms de la esperada. La auscultacin de dos focos de latidos no es
de utilidad diagnostica pero si la sospecha. En una embarazo sin control ecogrfico
podra hacerse el diagnostico ante la palpacin de dos fetos al examen fsico o
incluso durante el parto.

9) QUE NOS INTERESA PREGUNTAR EN LA ANAMNESIS DE ESTA PACIENTE ?
Historia familiar de embarazos mltiples, antecedentes terapia de fertilizacin
asistida.

10)QU BUSCARA DIRIGIDAMENTE EN EL EXAMEN FISICO DE ESTA PACIENTE ?

115
Altura uterina mayor a la esperada para la EG, auscultacin de dos focos de latidos
cardiofetales en control de embarazo y la palpacin de dos fetos en embarazos
mas adelantados.
Examen cardiopulmonar .

10)COMO ESTUDIAMOS ESTE CUADRO Y QUE RESULTADOS ESPERARIA ENCONTRAR EN
ESTOS EXAMENES ?
El estudio y diagnostico definitivo se realiza a travs del ultrasonido. En este se
observan los dos fetos en gestacin dentro de la cavidad uterina. Con la ecografa
podemos realizar el diagnostico de corionicidad (factor pronostico ms
importante) entre las 10-14 semanas (s 100%) a travs del numero de sacos
gestacionales, signo de lambda (insercin de membrana divisoria de ambas
cavidades en la placenta; bicorial-biamniotico), signo de la T (membrana entre los
dos fetos se inserta de forma recta en la placenta; monocorial-biamniotico).
Eco Doppler seriado.



11)CUAL ES EL TRATAMIENTO DE ESTA PACIENTE ?
El manejo de esta paciente se basa en el control del embarazo en centros de alta
complejidad.
Control materno: el control hasta las 22-24 semanas es similar al de un embarazo
normal. Es en esta semana cuando se debe realizar una ecografa TV para medir
longitud cervical (<25 mm >riesgo PP) y establecer el riesgo de PP. El reposo
domicialiario debe indicarse desde las 36-28 semanas, abstinencia sexual en el 3er
trimestre. Luego los controles deben ser cada 2-3 semanas hasta las 32-34
semanas. Control estricto de contracciones y PA, por el riesgo de PE. Posterior a
esto se recomienda control semanal.
Control fetal: una vez establecido el diagnostico de corionicidad, se debe descartar
RCIU, riesgo de parto prematuro, aparicin de complicaciones. Durante el 3er
trimestre se debe seleccionar el momento de interrupcin del embarazo
detenidamente si es que no ocurre de forma espontanea, esto debe ser, en lo
posible, en nivel terciario y contando con unidad neonatal requerida.
El parto en EG es en promedio: gemelos (36s), Triples (33-34s), cuadruples (30-
31s). El momento ideal de interrupcin del embarazo gemelar es a las 37-38
semanas (tanto para bicorial y monocorial), ya que disminuye la mortalidad
perinatal.

12) QUE CONSECUENCIAS PUEDE TRAER ESTA PATOLOGIA A LA MUJER / MADRE/ Y AL
FETO ?
Complicaciones maternas: Sd edematoso
HG Anemia
PE DPPNI
DG

116
PHA
Cesrea
Metrorragia post parto
Fetales:
PP (50%)
Bicoriales:
o RCF Discordancia
fetal
o Obito fetal (no afecta al
otro feto)
Monocoriales:
o RCIU selectivo
o Sd transfusin feto-
fetal
o Secuencia TRAP gemelo
acardio
Obito fetal


Bibliografa
- Manual Obstetricia y Ginecologa; J.A. Carvajal, C.
Ralph, 4ta edicin (2013), Capitulo 30 "Embarazo
mltiple"; pag. 313-329.
- Gua Perinatal 2014; Programa Nacional Salud de la Mujer, Ministerio de Salud, Chile (2014), Capitulo XI
"Complicaciones del embarazo gemelar"; pag. 114-125
- Manual Obstetricia; U. chile, Departamento de
obstetricia y ginecologa, Dr. Hugo Salinas; 2005.
Capitulo 32: "Embarazo gemelar", pag: 395-401
- Manual Ginecologa y obstetricia CTO; M. Muoz,
D.P. Hernndez, 8va edicin (2012), Capitulo 26
"Gestacin mltiple"; pag. 100-1

117

24. EMBARAZO PATOLGICO : PARTO PREMATURO


Mujer de 37 aos cursando embarazo de 29 semanas que consulta por dolor hipogstrico
irradiado a dorso ocasional , se acompaa de perdida de flujo vaginal sanguinolento
Antecedentes familiares : padre hipertenso
Antecedentes Personales: Sana
Antecedentes Gineco obsttricos : G3P2A0 parto anterior 35 semanas , RM V/30

1. Diagnsticos
Multpara de 2
Embarazo de 29 semanas
Sntomas de parto prematuro ?

2. Diagnostico diferencial
Trabajo de parto prematuro
RPO
ITU

3. Definicin
Parto prematuro (PP) se define como aquel parto que ocurre antes de la
37+6 semanas de gestacin, con un lmite inferior de 22 sem y/o 500 grs
y/o 25 cm de longitud cfalo nalgas. Es una de las principales causas de
morbimortalidad neonatal.

4. Epidemiologia Chile presenta una incidencia del 5-6% de las gestaciones y a
nivel mundial la incidencia oscila entre 8-10%. Se clasifica segn edad
gestacional.
Prematurez extrema (28 semanas): 5% de los PP.
Prematurez grave (28-31 semanas): 15% de los PP.
Prematurez moderada (32-33 semanas): 20% de los PP.
Prematurez tarda (34-36 semanas): 60-70% de los PP.
5. Etiologa del parto prematuro
Actualmente se considera el PP como un sndrome, es decir como una
condicin causada por mltiples etiologas, de las cuales se han

118
identificado: Infeccin intra-amnitica, isquemia tero placentaria,
disfuncin cervical, sobre distencin uterina, factores inmunolgicos
hormonales, stress, alergias, etc.
Las principales vas que conducen a parto prematuro, son:
- PP espontneo con membranas integras (1/3)
- PP asociado a rotura prematura de membranas (1/3)
- PP iatrognico (1/3)
Para el estudio y manejo adecuado, lo podemos clasificar en:


III. Trabajo
I.
Prevencin
de parto
del parto
prematuro
prematuro


II. Amenaza de
trabajo de parto
prematuro

6. Prevencin del parto prematuro
1. PREVENCIN PRIMARIA: Se realiza en ausencia de signos de la enfermedad para
prevenir su ocurrencia.
Control preconcepcional: Optimizar condiciones de salud en general, como
hbitos alimenticios, controlar y prevenir sobrepeso u obesidad, control adiccin
de drogas, tabaco o alcohol, y promover control de enfermedad periodontal.
Adems de fortificacin preconcepcional con cido flico, a los menos 3 meses
previos al embarazo (Evidencias demuestran que la ingesta de cido flico un ao
previo al embarazo disminuye la posibilidad de parto prematuro en un 50-70%).

7. Bsqueda de factores de riesgo
Caractersticas maternas Complicaciones del embarazo Historia Obsttrica
Raza Gestacin mltiple (60% Cuello uterino corto
gemelares nacen
IMC bajo Ciruga cervical
prematuros)
Presentar sobre 40 aos o Dos prematuro previo a
Infecciones genito-urinarias
menores 18 aos. trabajo de parto 13 - 21% de
(25-40% de los PP)
riesgo
Uso de tabaco, drogas.
Metrorragia 2da mitad del
Aborto involuntario en

119
Pobre nutricin embarazo. segundo trimestre
Nivel socioeconmico bajo Historia previa de repeticin
TOP
Presentar parto prematuro anterior es el factor ms importante. El riesgo de
recurrencia en mujeres con PP previo vara entre 15-50%. El riesgo esta
inversamente proporcional a la EG, ya que el trastorno subyacente a un PP
indicado, por ejemplo diabetes, hipertensin, obesidad, etc. frecuentemente
persiste entre embarazos.
Bsqueda de infeccin del tracto urinario y tratamiento adecuado.

Medicin del crvix dentro de la semana 22 y 24 en la poblacin general.
La medicin del crvix, se realiza con ultrasonido transvaginal en la semana 22 y
24, estudios demuestran que es una prueba segura, confiable y reproducible para
la deteccin del trabajo de parto prematuro. El lmite para mujeres sin factores de
riesgo es de 15 mm y con factores de riesgo es de 25 mm. Algunos autores indican
20 mm para ambos.
En los embarazos normales la longitud cervical se mantiene relativamente
constante hasta el tercer trimestre. Existe una relacin inversa entre la longitud
cervical y la incidencia de parto prematuro. En la poblacin general, slo el 1,7% de
las mujeres tienen una longitud cervical menor a 15 mm y estas mujeres presentan
100% de los nacimientos antes de las 26 semanas, el 80% de los nacimientos antes
de 30 semanas de gestacin y 60% de los nacimientos antes de las 32 semanas de
gestacin. Por lo tanto, una longitud cervical menor de 15 mm es un predictor
sensible de la prematuridad severa, ya que se asocia con un riesgo del 50% de la
ejecucin antes de 32 semanas de gestacin.
La evidencia indica que cualquier intervencin ayuda a prevenir el parto prematuro
tras la identificacin de acortamiento cervical o canalizacin. Por lo tanto, el
principal beneficio de la deteccin por ultrasonido transvaginal es su alto valor
predictivo negativo del 90% para una longitud cervical longitud de ms de 3 cm a
las 24 semanas, donde estas mujeres pueden estar tranquilas, evitando nuevas
visitas mdicas y/o la intervencin.
Marcadores Bioqumicos: Fibronectina fetal
Fibronectina
fetal (FFN) es
una
glicoprotena
de la matriz
extracelular
localizada en la
interfaz de la

120
membrana amnitica materna fetal, entre corion y decidua. Se encuentra
en secreciones cervico-vaginal antes del parto, como un marcador de
disrupcin de esta membrana. En un embarazo normal, FFN no debe estar
presente entre 20 y 36 semanas de gestacin. Niveles de FFN mayores o
iguales a 50 ng/ml a las 22 semanas se han asociado con un 40% ms de
riesgo de parto prematuro espontneo. Para una toma adecuada, el hisopo
debe ser tomado desde el fondo de saco posterior o exocrvix. Los
resultados falsos positivos pueden ocurrir con el uso de gel lubricante, el
acto sexual reciente, sangrado vaginal reciente y ruptura de las membranas.
Actualidad, los kits de FFN disponibles son cualitativos, dando resultados
positivo o negativos, por lo que resultados negativos no tendran mucha
utilidad. En Chile tienen poca disponibilidad, por ende poco uso.
2. PREVENCIN SECUNDARIA: Corresponden a medidas preventivas que se adoptan
en pacientes con factores de riesgo o que presenten una cervicometra alterada en el
control de la semana 22 y 24. Estas pacientes debiesen controlarse en policlnico
de alto riesgo obsttrico.
Medidas generales: Modificar factores de riesgo, como cese de hbito tabquico,
adecuada alimentacin, disminucin de carga laboral, educacin sobre signos y
sntomas de alarma.
Bsqueda y tratamiento de infecciones cervicovaginales.
Uso de progesterona: Toda paciente que se encuentre un crvix menor de 15 mm
o el antecedente de uno o ms parto prematuro menores de 35 semanas, se
benefician con la administracin de progesterona. Actualmente se puede usar:

Progesterona natural micronizada 200 mg/da por va vaginal (vulos o gel), desde el
diagnstico de cuello corto hasta la semana 36, rotura de membrana o inicio de
trabajo de parto. Reduce en un 34,4% el riesgo de parto prematuro antes de la semana
34, sin reduccin significativa en la morbilidad neonatal
17 hidroxiprogestrona (im) 250 mg/sem desde el diagnstico de cuello corto hasta la
semana 36, RPM o inicio de trabajo de parto. Un anlisis de la base Cochrane indica
que reduce la frecuencia de parto prematuro total (RR 0,65 95% IC 0,54-0,79).


Cerclaje:

121
Sutura cervical o cerclaje es utilizado en
los embarazos de alto riesgo de parto
prematuro. Fue introducido como
tratamiento para la incompetencia
cervical en el que el crvix se crea que
tena algn tipo de debilidad
inherente. Generalmente se realiza de
manera profilctica entre la semana 12
y 14, o como una emergencia, en
presencia de membranas expuestas
antes de la 24 semanas.
El mtodo del cerclaje ms utilizado es la
sutura McDonald, que es una sutura en bolsa de tabaco alrededor del cuello
uterino. Esto se eliminado en la gestacin de aproximadamente 37 semanas, para
permitir un parto vaginal normal.
Todos los tipos de cerclaje cervical tienen factores de riesgo asociados, como la
ruptura de membranas, sangrado, prdida del embarazo, lesin de la vejiga y los
riesgos anestsicos.
Estudio realizado en 253 mujeres con longitud cervical menor de 15 mm, mostr
una reduccin no significativa de PP al utilizar sutura cervical electiva versus
conducta expectante (22 vs 26%). Por lo que en general se usa en pacientes con
antecedentes de parto prematuro espontaneo menor de 34 semanas y cuello
acortado antes de la semana 23, solo as disminuye significativamente la incidencia
de parto prematuro.

8. Diagnostico
En pacientes con sntomas de parto prematuro, lo primero es diferenciar si la
paciente se encuentra en amenaza de trabajo de parto o en trabajo de parto, pues
el estudio y manejo presentan distinto enfoque. El diagnstico es clnico, y el
manejo representa las intervenciones para la prevencin terciaria, sea
estrategias que se aplican en pacientes que presentan sntomas de parto
prematuro o rotura prematura de membrana.

Amenaza de trabajo de parto prematuro Trabajo de parto prematuro

- Entre 22 y 37 semanas - Entre 22 y 37 semanas
-
1 a 2 contracciones x10 min x30 - 3 contracciones x10 min x 30
min. min.
- Borramiento 50% - Borramiento mayor 80%
- Dilatacin 1 cm - Dilatacin 3 cm

9. Exmenes iniciales:

122
- Hemograma y VHS
- Sedimento de orina y urocultivo
- Cultivos cervicovaginales: cultivo corriente, cultivo para estreptococo grupo B,
cultivo para mycoplasma y uroplasma, IF para clamidia, cultivo gonococo,
microscopio directo al fresco para Trichomona. Frotis vaginal para test de
Nugent.
- US: Medicin feto (biometra y anatoma), evaluar LA y placenta.
10. Tratamiento y conducta:
Reposo en cama en rea de preparto
Evaluacin materna
- Control clnico y de signos vitales
- Control dinmica uterina
- Cervicometra ecogrfica
> 30 mm el riesgo de parto prematuro es muy bajo y el
manejo debe ser conservador: Reposo domiciliario,
antiespasmdicos e instrucciones de re consultar. Pues
presenta un 10% de riesgo de presentar un PP antes de la
semana 35 y 0% a las 48 hrs.
< 15 mm el riesgo de prematurez es alto ( 25 en pacientes
con factores de riesgo) Presenta un 50% de riesgo de
presentar un PP antes de la semana 35 y un 20% a las 48 hrs,
por lo que se hospitaliza y se maneja como trabajo de parto
prematuro.
Evaluacin del bienestar fetal: Control clnico (Maniobras de Leopold y
auscultacin de LCF), monitorizacin electrnica continua.
Hidratacin: SF 250 ml/hrs Ev. + Pargeverina clorhidrato 10 mg Ev.
Observacin por 2 horas: Si luego de estas dos horas de observacin la DU
persiste, o las modificaciones cervicales progresan, se inicia manejo de
trabajo de PP.
III. Trabajo de parto prematuro
1. Determinar edad gestacional y confirmar diagnstico






123
a. Identificar la causa y tratarla (Con exmenes previos, ya sea infeccin o
sobredistencin)
2. Tocolisis
Grupo de frmacos que inhiben las contracciones uterinas y se administra en
pacientes en trabajo de parto.
Reduce significativamente el riesgo de parto dentro de las 24 hrs, 48 hrs y 7
das desde el inicio de tratamiento. Sin embargo no permite reducir el riesgo
de PP o muerte perinatal, por lo tanto ofrece el beneficio de implementar
medidas como administracin de corticoides o traslado de la madre.

Criterios decisin de Tocolisis
Edad gestacional 35 semanas
Contracciones uterinas 3 en 30 min
Cambios cervicales: 50% de
Borramiento, 1 cm de dilatacin
Contraindicaciones al uso de Tocolticos
Persistencia de dinmica uterina.
Absolutas:
- Corioamnionitis clnica
- Metrorragia severa
- Malformacin fetal incompatible con la vida
- bito fetal
- Patologa materna grave
- Deterioro grave del bienestar fetal
- Trabajo de parto avanzado (6 cm de
dilatacin)
Relativas:
- Dilatacin 3 y 6 cm
- Edad gestacional 34 semanas
- Madurez pulmonar fetal
- Metrorragia moderada
- Restriccin del crecimiento fetal
- Rotura prematura de membrana
- Sndrome hipertensivo del embarazo

Droga Protocolo de uso Efectos adversos
Nifedipino Carga: 20 mg (vo) c/20 min. (Mx. 60 mg)
Fetal: (-) Disminuye el
(Bloquea canales Mantencin:
10 mg c/6 hrs. (vo). Hasta riesgo de SDR.
de Ca) completar 12 hrs sin contracciones o completar Materno: Hipotensin,
madurez pulmonar (48 hrs). cefalea (38%), rubicundez
No usar en enf. Cardiovascular o HDI. facial (98%).
Fenoterol (Activa Ampolla 10 ml (0,5mg): 4 amp en 500 ml SG 5%. Fetal (-)
la va del AMPc) Carga: 30 ml/hr (2 g/min) Aumentar 0,5 g/min Materno: Taquicardia
c/30 min hasta cesar contracciones. (Mx: 3- (20%), EPA, hiperglicemia,
4g/min). hipokalemia. Arritmias (1-
Mantencin: Mantener 0,5-1g/min hasta 48 2%)

124
hrs, o disminuir progresivamente luego de 12 hrs
sin contracciones. Monitorizar pulso y PA.
Atosiban Carga: Bolo Ev de 6,75 mg/min. (Amp 0,9 mg) Leves, nauseas, cefalea y
(Bloquea Mantencin: 1eras 3 hrs infusin de 300 g/min discreta taquicardia.
receptores de (18 mg/h), siguientes 45 hrs infusin de 100 Elevado costo.
oxitocina) g/min (6 mg/h)
Indometacina 50 a 150 mg vo o uso rectal. No usar ms de 48 Fetal: HIV, ECN,
(Antiinflamatorio hrs. (No usar 32 sem) oligoamnios, cierre precoz
que inhibe la ductus ( 32 sem).
produccin de Materno: Intolerancia
PG) digestiva.
Nitroglicerina Ineficaz Fetal (-)
(Activa la va del Materno: Cefalea.
GMPc: ON)
Sulfato de Ineficaz Fetal: Hipotona
magnesio Materno: Depresin
(Bloquea canales respiratoria, nuseas y
de calcio) vmitos, cefalea.
Los Tocolticos se debe mantener por 48 hrs, y la paciente se debe
mantener en reposo por al menos 7 das. Una vez iniciada la terapia y
persiste la dinmica uterina, se debe efectuar una AMCT, si se confirma
infeccin en la cavidad amnitico, se suspende la Tocolisis. Si se descarta la
infeccin, entonces se inicia Tocolisis de segunda lnea, si persiste dinmica
uterina, entonces se suspende tratamiento y se deja a la paciente en
evolucin espontanea. Nunca se usa dos Tocolticos de manera
concomitante.

4. Corticoides
En todo paciente entre 24 y 34 semanas, se debe administrar corticoides im
para induccin de madurez pulmonar fetal, al momento que inicie la terapia
tocoltica. Su beneficio mximo se logra si el parto se produce entre 48 hrs y
7 das. El efecto se pierde luego de 7 das.

Corticoides
1era lnea: Betametasona 12 mg c/24 hrs por 2 dosis
Reduce aprox. 50% de riesgo de ddodosis (im)
2do lnea: Dexametasona 6 mg c/12 hrs por 4 dosis (im)
SDR, y entre un 10-80% los
riesgos de HIC y ECN. No se
recomienda repetir su
administracin, pues no reduce
el SDR, adems dosis repetidas
(3) se asocian a restriccin de
crecimiento,menor
circunferencia craneana, menor
talla neonatal y posibles
alteraciones del desarrollo
neurolgico. Excepto si habiendo

125
pasado ms de 15 das desde el primer curso e indique paro ocurrir antes
de la semana 34.
El uso oportuno de corticoides antenatales, son la base para que las
terapias neonatales tengan un buen rendimiento.







5.

Proteccin neurolgica del RN Sulfato de magnesio


Se indica en pacientes con riesgo inminente de PP, entre la semana 28 y 32.
En la base Cochrane, se observa reduccin en la incidencia de parlisis
cerebral y disfuncin motora severa, sin cambios en la mortalidad perinatal.

Sulfato de magnesio
4 gr (iv) en 30 min e infusin de 1 g/h por hasta 24 hrs. Diluir 4 amp
(1.25 gr en 5ml al 25%) en 230 ml de SF. Administrar 250 ml en 30 min
(0,02 g/ml).

Siempre evaluar diuresis (Sonda Foley), ideal buscar diuresis 30 ml/hrs y
reflejos osteotendineos (Arreflexia Mg 10-12 mEq/L. Mg teraputico 5-8
mEq/L)
En caso de intoxicacin, se debe indicar Gluconato de calcio 1 gr iv en 3
min.


Atencin de parto prematuro
Si las medidas descritas anteriormente fallan y el trabajo de PP contina, es
necesario evaluar viabilidad neonatal en un centro donde se cuente con los
medios necesarios para la atencin. En los PP se eleva significativamente las
presentaciones distcicas, por lo que cualquier presentacin que no sea
ceflica/vrtice debe ser indicacin de cesrea.

126
Debe coordinarse con residencia de neonatologa, pues es la causa ms
importante de morbilidad y mortalidad perinatal, dependiendo del peso de
nacimiento y de la EG al nacer.

La prematurez < 32 semanas, conlleva a la mayor morbimortalidad neonatal
y la mayor tasa de secuelas, como distrs respiratorio, hemorragia
interventricular, enterocolitis necrotizante, sepsis neonatal, retraso del
desarrollo, dficit visuales y auditivos, enfermedad pulmonar crnica
(hipoplasia pulmonar) y parlisis cerebral. Excluidas las malformaciones
congnitas, el 75% de las muertes perinatales son atribuibles a la
prematurez.

Bibliografa
- Gua Clnica Prevencin del parto prematuro. Santiago MINSAL 2010.
- Management of preterm labour, Sarah A Hamilton Clare L Tower. Review.
Onstetrics, gynaecology and reproductive medicine 20:8 2010.

25. EMBARAZO PATOLGICO : ROTURA PREMATURA


DE MEMBRANAS.

Mujer de 28 aos cursando embarazo de 32 semanas quien consulta por eliminacin de
flujo como agua por gavina desde esta maana
Antec Familiar : madre hipertensa
Antec Pesonales : sana
Antec gineco obsttricos : Fur 28/04 RM V/30 G2P1A0

1. Diagnsticos :
Multpara de 1
Gestacin de 32 semanas
Obs rotura prematura de membranas

2. Diagnostico diferencial

Perdida de orina
Leucorrea
Hidrorrea decidual
Tapn mucoso

127

3. Definicin :
Se define rotura prematura de membrana (RPM) como la rotura espontanea de la
membrana corioamniotica antes de la aparicin de las contracciones o del inicio de
trabajo de parto. Presenta una incidencia del 8 a 10% de los embarazos de trmino
y 7-20% en embarazo gemelar, es causante del 30-40% de los partos prematuros.
Se clasifica en:
- PPROM (Rotura prematura de membranas de pre trmino): Antes de las 37
semanas de gestacin.
- PROM: Despus de las 37 semanas de gestacin.
4. Etiologa
Es multifactorial, excepto en casos de agresin mecnica, como procedimiento
invasivo. Pero en la mayora de casos la etiologa es idioptica y se asocia a
diferentes situaciones, como:

1. Condiciones clnicas asociadas:
- Tabaco
- RPM previa (Recurrencia 16-32%)
- Cuello uterino corto
- PP anterior
- Polihidroamnios- Oligohidroamnios
- Gestaciones mltiples
- Hemorragia comienzo del embarazo (Amenaza de aborto)

2. Infeccin intraamniotica: Es una de las principales causas, por ende siempre
hay que buscarla.
3. Alteracin de las propiedades fsicas de la membrana: Normalmente hay un
equilibrio entre la actividad de las enzimas proteasas y anti proteasas (colgeno
y elastina) que juega un rol importante en la mantencin de la integridad de la
membrana. En los embarazos con RPM son ms delgadas y presentan menos
elasticidad y resistencia, por lo tanto hay un aumento de proteasas y
disminucin de 1 anti tripsina en el lquido amnitico, por lo que favorece la
rotura.
5 Fisiopatologa
Se unen diferentes factores, en primer lugar las condiciones clnicas expuestas
favorecen este desequilibrio de enzimas ya explicado.
En las infecciones ovulares, principal factor asociado, los microorganismos son
reconocidos por toll-like receptors, que permiten la liberacin de citokinas pro-

128
inflamatorias (IL 8, IL 1 y TNF), que estimulas la produccin de PG E2 y el
aumento de la actividad metaloproteinasa (MMP9, MMP2 y MMP3), estas
estimulan la contractibilidad uterina, degradan la matriz extracelular de
membranas fetales. Los MO ms frecuentes: Micoplasma spp, especficamente
Ureaplasma urealyticum.
Las vas de acceso a la cavidad amnitica son:
- Ascenso por vagina y crvix
- Diseminacin hematgena a travs de la placenta
- Introduccin accidental durante procedimientos invasivos
- Diseminacin retrgrada a travs de trompas de Falopio.
La invasin microbiana a la cavidad se asocia a inflamacin intraamniotica y
respuesta inflamatoria fetal, definido como sndrome de respuesta
inflamatoria fetal (FIRS), donde se necesita concentraciones de IL-6 en plasma
fetal 11 pg/mL.
La evolucin natural, se inicia con la entrada de MO, posterior hay una
infeccin subclnica, para terminar en una infeccin clnica, conocida como
corioamnionitis clnica. Se define por la presencia de fiebre > 37,8C y dos o
ms de los siguientes criterios menores:
o Irritabilidad uterina
o Leucorrea maloliente
o Taquicardia materna > 100 latidos/minuto
o Taquicardia fetal > 160 latidos/minuto
o Leucocitosis > 15.000 cels/mm
6 Clnica
Es el escurrimiento de una cantidad variable de LA (Olor a cloro, semen )
por la vagina. Al examen fsico podemos observar salida de lquido por el
OCE (Es importante realizar valsalva para poder observar la salida) y altura
uterina menor que controles previos y las partes fetales se palpan con
facilidad.
El diagnstico es clnico, al objetivar hidrorrea. Pero en 10-20% de los casos
existen dudas por los diferentes diagnsticos diferenciales expuestos en la
siguiente tabla:
Diagnstico diferencial Caracterstica

Leucorrea Flujo genital blanco amarillento, infeccioso,


asociado a prurito.

Incontinencia urinaria Frecuente en segunda mitad del embarazo,


especialmente en multparas (por relajacin
perineal y rectocistocele). Descartar ITU.

129
Eliminacin tapn mucoso Fluido mucoso, a veces algo sanguinolento.

Rotura de quiste vaginal Prdida brusca de lquido cristalino por


vagina. Al examen se observa cavidad
pequea en paredes vaginales (raro).

Hidrorrea decidual Primera mitad del embarazo. Lquido claro


con tinte amarillo, a veces sanguinolento.
Desde espacio entre decidua parietal y
refleja, que se fusiona entre las 16 y 18
semanas.

Rotura de bolsa amniocorial Raro; de espacio virtual entre corion y


amnios; se produce por de laminacin de
este ltimo.


7. Estudio
En caso de dudas, se puede realizar el siguiente estudio:
- Cristalizacin en hojas de helecho: La presencia de LA en el contenido del
fondo de saco vaginal permite identificar al microscopio de luz, la
cristalizacin caracterstica en forma de plumas, que resulta de la
deshidratacin de las sales (NaCl) contenidas en el LA.
- Prueba de Nitracina: El pH del fornix vaginal posterior flucta
normalmente entre 4,5 y 5,5, mientras que el del LA es generalmente de 7.
Las membranas ovulares estn probablemente rotas si el papel nitrazina
seala un pH mayor o igual a 7 (viraje al color azul).
- Deteccin de clulas de descamacin: la piel fetal descama clulas
superficiales que pueden ser detectadas con azul de Nilo al 1%. Dada la
evolucin de la concentracin de clulas naranjas en el LA, el valor
diagnstico de esta prueba aumenta junto con la edad gestacional,
variando desde una sensibilidad de alrededor de 20% a las 32 semanas
hasta un 90% en embarazos de trmino.
- Evaluacin US: La evaluacin ecogrfica permite estimar la cantidad de LA
en la cavidad uterina. En pacientes con historia sugerente, pero sin
evidencias de RPM en la valoracin complementaria, la presencia de
oligoamnios debe asumirse como consistente con el diagnstico. Adems
permite valorar el bienestar fetal, aproximar o certificar una edad
gestacional, descartar la presencia de malformaciones fetales y predecir o
apoyar el diagnstico de una corioamnionitis.
- Inyeccin intraamnitica de colorantes (azul de Evans o ndigo carmn),
cuya deteccin en la vagina confirma el diagnstico. No debe usarse azul de
metileno.

130
- Nuevos Test (AmnioSure: ROM test, PROM test) Deteccin rpida de -
micro globulina 1 placentaria.
8. Conducta

- RPM en embarazos 34 semanas
Interrupcin del embarazo en un plazo no mayor a 3 das. Ella se justifica
considerando que sobre esta edad gestacional normalmente existe
madurez pulmonar fetal; adems, la infeccin es la causa ms frecuente de
mortalidad perinatal y, por otra parte, existe una relacin directa entre
perodo de latencia e infeccin ovular, as el manejo expectante se
relaciona con aumento significativo de la morbilidad infecciosa neonatal.
Va de parto: Ideal parto vaginal (Excepto en contraindicaciones) 70% estar
en trabajo de parto espontneo dentro de las 24 hrs de RPM y el 90% a las
72 hrs.


- RPM en embarazos 34 semanas
Manejo Expectante. Procurando alcanzar madurez pulmonar fetal y
haciendo nfasis en la deteccin precoz de signos de infeccin ovular.
1. Realizar espculoscopia, la que permite verificar el diagnstico, obtener
cultivos cervicales y de fondo de saco vaginal, aproximar las condiciones
cervicales y obtener muestra de LA para deteccin de fosfatidilglicerol.
Se omite el tacto vaginal y se procede a aseptizar el canal.
2. Hospitalizar
- Reposo relativo
o Control de signos vitales y obsttrico cada 6 horas
o Hemograma semanal o bisemanal. Leucocitos cada 48 horas
o Perfil biofsico bisemanal. Fotometra cada 2 semanas
o Apsito genital estril. Evitar tacto vaginal
o Cultivos crvico-vaginales semanales
o AMCT ante la sospecha de infeccin ovular
o Administracin de corticoides: Betametasona 12 mg c/24 hrs x 2 dosis
im.
o ATB: Administracin endovenosa de Ampicilina 1-2 gr c/6 hrs +
Eritromicina 250-500 mg c/6 hrs x 48 hrs Ev. Luego seguir con rgimen
oral de Ampicilina 500 mg c/6 hrs y Eritromicina 250.500 mg c/8 hrs y
completar por 7 das. (Antibioterapia debe incluir agente activo contra

131
estreptococo del grupo B y micoplasmaOtros esquemas: Clindamicina
+Gentamicina, Clindamicina + Cefalosporina).
o Evitar uso de Tocolticos.
3. Interrupcin del embarazo en:
- Muerte fetal
- EG segura 35 semanas
- Corioamnionitis clnica
- Confirmacin bioqumica de madurez pulmonar fetal, con feto 32
semanas y peso fetal 2.000 gramos
- Infeccin intraamniotica demostrada con feto 31-32 semanas
- Deterioro de la unidad feto-placentaria
- Malformacin fetal incompatible con la vida
- Metrorragia que sugiera la presencia de un DPPNI
- Enfermedad materna o fetal que se beneficie de la interrupcin del
embrazo.
- RPM en embarazos 24 semanas
Manejo Expectante. Esperar semana 24 para inicio de induccin de
madurez pulmonar. Se indica ATB y reposo en hogar.

132

Consecuencias Fetales pueden ser mltiples, tales como:
- Sndrome de distrs respiratorio Responsable del 40-70% de muertes
neonatales.
- Infeccin de la cavidad amnitica Causante de 3-20% de muertes
neonatales.
- Hipoplasia pulmonar
- Deformidad esqueltica: orejas de implantacin baja, hipertelorismo,
nariz aplanada, piel arrugada, posiciones anormales de las
extremidades, pies y manos planas.
- Prolapso del cordn
- Discapacidad Parlisis cerebral
- Morbilidad perinatal: SDR, sepsis perinatal, neumona, displasia
broncopulmonar, HIV, leuco malacia peri ventricular, ECN.
La morbilidad materna se ha descrito mayor riesgo de corioamnionitis
clnica tarda (13-60%), infeccin postparto (2-13%) y desprendimiento
de placenta (4-12%), siendo la sepsis materna una complicacin rara
(1%).

133
Bibliografa
- Gua Minsal Parto Prematuro 2010
- T. Cobo, M. Palacio. Rotura prematura de membranas. CURSO
INTENSIVO EN MEDICINA MATERNOFETAL. Cursos CLNIC de Formacin
Continuada en Obstetricia y Ginecologa (Barcelona), del 4 al 9 de marzo
de 2012.
- Practice Bulletin Premature Rupture of Membranes. The American
College of Obstetricians and Gynecologists. OCTOBER 2013.
- Obstetricia y ginecologa, Beckmann, 6ta edicin, editorial Lippincott
William y Wilkins 2010.
- Obstetricia, Prez Snchez, 4ta edicin, Editorial Mediterraneo 2011.

26. EMBARAZO PATOLOGICO : CORIOAMNIONITIS.



Multpara de 3 de 35 aos cursando embarazo de 38 semanas de gestacin quien
presenta fiebre de 38.5 C y escalofros a las 4 hrs de haber sido ingresada por trabajo
de parto y 12 hras despus de la Rotura de membranas. Pa 120/80 y pulso 100 x min.
Embarazo de evolucin fisiolgica a excepcin de anemia en semana 34 (Hb 8.5)
Antecedentes Familiares: madre diabtica padre hipertensa
Antecedentes Personales: vaginosis a repeticin
Antecedentes Gineco obsttricos: menarqua 13 aos, RM IV/28 G2P1A0
Hbitos: sana

1) CUAL ES EL DIAGNOSTICO MAS PROBABLE DE ESTA PACIENTE?
- Multpara de 3
- Embarazo de 38 semanas
- Corioamnionitis

2) CUALES SON LOS DIAGNOSTICOS DIFERENCIALES?
- PNA
- IIA (infeccin intraamniotica)
- ITU
- Sepsis materna
- Endometritis
- Parametritis
- Metrorragia de 2da mitad de embarazo
- Neumona

134
- Estado gripal
- Frmacos

3) COMO SE DEFINE ESTA PATOLOGIA?
La corioamnionitis ( 20 % )es el cuadro clnico originado por la inflamacion aguda
de las embranas placentarias por la presencia de grmenes en la cavidad amnitica
que oinfectan al feto , cordon y liquido amniotico , la que es diagnosticada con
criterios definidos. Se diferencia de la infeccin intraamniotica (IIA)(80%) ya que
esta es una invasin microbiana de la cavidad amnitica ASINTOMATICA,
confirmndose por cultivo

4) CUAL ES/SON LA/S ETIOLOGIA DE ESTE CUADRO CLINICO?
La mayora de los pacientes presenta un cuadro poli microbiano, habitualmente
combinacin de aerobios y anaerobios. Los agentes ms comunes encontrados en
lquido amnitico en pacientes con PP y RPMP son micoplasmas, ureaplasmas, E.
Coli, Listeria, EBH, Chlamydia.
El origen de la infeccion es casi siempore por via ascendente desde nla flora
vaginal.

5) CUAL ES LA FISIOPATOLOGIA DE ESTE CUADRO?
Generalmente los grmenes llegan a la cavidad amnitica por va ascendente,
desde la mucosa vaginal, pasando por el canal cervical. Existen otras formas que
permiten la entrada de bacterias al LA, va Hematogena (desde otro foco infeccioso
o sepsis materna), va canalicular tubaria (por contigidad, como peritonitis
apendicular), iatrognica (amniocentesis, cordocentesis, fetoscopia).
Por lo general la invasin se divide en: alteracin de microbiologa vaginal
invasin de interfase coriodesidual Infeccion intraamniotica Invasin y
bacteremia fetal.

6) QUE FACTORES DE RIESGO/PREDISPONENTES BUSCARA?
Malnutricin materna, infecciones genitales durante el embarazo, RPM e inicio de
trabajo de parto >24 hrs, politacto, dieta (baja actividad antimicrobiana por dficit
de zinc), Aumento del pH vaginal, ausencia de moco cervical, actividad sexual cerca
del trmino, curso de corticoides (no en dosis nica)

7) CUAL ES LA EPIDEMIOLOGIA DE ESTE CUADRO?
2-11% del total de embarazos, dentro de estos el 5% el feto est infectado. La
corioamnionitis subclinica es mucho ms comn en embarazos de pretermino: 24-
28s 40%; 28-32s 30%; 30-36s 20%; >36s 10%. Cabe destacar que se
presenta en un 5-10% de pacientes con RPM y en un 44% aprox precede la RPM.

8) COMO HACEMOS EL DIAGNOSTICO CLINICO?

135
El diagnostico es clnico, se debe buscar en toda embarazada con fiebre y sobre
todo si existe RPM. En el caso de las subclinicas el diagnostico se har ante la
presencia de una complicacin de esta. Muchas veces no se diagnostica hasta
despus del parto, por fiebre puerperal e dolor uterino, adems de infecciones del
feto. La amenaza de parto prematuro sin respuesta a manejo adecuado tambin
hace sospechar una corioamnionitis subclinica.
El diagnostico se realiza con los siguientes criterios de GIBS : Fiebre mayor o igual a
38C (37,8C) y dos o mas de los siguientes: Sensibilidad uterina, secrecin
purulenta por OCE, Taquicardia materna >100 o fetal >160 lpm, leucocitosis
materna >15.000.

9) QUE NOS INTERESA PREGUNTAR EN LA ANAMNESIS DE ESTA PACIENTE?
Hbitos: Malnutricin materna, dieta (dficit zinc) actividad sexual, promiscuidad.
Antecedentes: RPM, RPMP, infecciones genitales a repeticin, PP, infecciones
genitales durante el embarazo. Actuales: RPM, politacto, corticoides.

10)QU BUSCARA DIRIGIDAMENTE EN EL EXAMEN FISICO DE ESTA PACIENTE?
General: fiebre, CEG, sudoracin, taquicardia materno/fetal. TV/Especuloscopia:
dilatacion cervical , secrecin purulenta por OCE, sensibilidad cervical/uterina,
aumento del pH vaginal, ausencia de moco cervical.Dinamica uterina , LCF
Ex fisico generalpara descartar otro foco de la infeccion .

10)COMO ESTUDIAMOS ESTE CUADRO Y QUE RESULTADOS ESPERARIA ENCONTRAR EN
ESTOS EXAMENES?
Al ser el diagnostico clnico los exmenes complementarios se reservan solo si son
necesarios. Entre estos podemos solicitar hemograma donde se observara una
leucocitosis materna sobre 15.000 (s 70%). Es posible, en las subclinicas, encontrar
presencia de leucocitos polimorfonucleares maternos en membranas ovulares y
placa corial. Cultivo de placenta, ecografa (<mov resp, <tono fetal). Adems esta la
opcin de la amniocentesis no se realiza de rutina pero ante dudas diagnosticas y
refractariedad a tratamiento inicial se debe realizar, en la cual se buscaran entre
otras cosas: leucocitos > 50 leucocitos/mm, cultivo, PCR, Gram, gluc <14, LDH>400.
Estudio bienestar fetal : Ecografia, perfil biofisico, doppler , MFE .

11)CUAL ES EL TRATAMIENTO DE ESTA PACIENTE?

En presencia de corioamnionitis clnica:
Interrupcin del embarazo por va mas expedita
Antibioterapia:
o Clindamicina 600 mg c/8 hrs + Gentamicina 3-5 mg/kg/da en una dosis.
o Alternativas:
Penicilina cristalina 5 M c/ 6 hrs + Gentamicina 3-5 mg/kg/dia
Ampicilina 2 gr c/6 hrs + Gentamicina 3-5 mg/kg/dia
Ceftriaxona 1 gr c/12 hrs + Clindamicina 600-900 mg c/8hrs

136
o Cesrea: Ampicilina sulbactam 2gr c8 hrs + Clindamicina 600-900 mg c/8
hrs
o Presencia de DIU o cerclaje: + Fluconazol 400 mg EV o 150 mg VO.
Ante infeccin subclinica:
>30 semanas:
o Antibioterapia
o Corticoterapia para induccin pulmonar, si corresponde.
o Interrupcin del embarazo
<30 semanas:
o Cobertura antibitica
o Observacin y vigilancia clnica y de laboratorio
o AMCT seriada es controversial
o Por cada da extra en tero, la morbimortalidad disminuye
considerablemente.


12)QUE CONSECUENCIAS PUEDE TRAER ESTA PATOLOGIA A LA MUJER / MADRE/ Y AL
FETO?
Fetales:

Infecciosas: sepsis neonatal.
Leucomalacia periventricular
(lesiones qusticas, secuelas)
Apgar bajo (<3 a los 5m)
pH cordn <7.0
Distres respiratorio
Convulsiones
Hemorragia periventricular
Prematurez
Parlisis cerebral
Muerte neonatal

Maternas:

Sepsis
Endometritis
Acidosis
Extensin de foco infeccioso

137

- Manual Obstetricia y Ginecologa; J.A. Carvajal, C. Ralph,
4ta edicin (2013), Capitulo 17 "Corioamnionitis clnica -
Infeccin intraamniotica"; pag. 254-260.
- Gua Perinatal 2014; Programa Nacional Salud de
la Mujer, Ministerio de Salud, Chile (2014),
Capitulo XX "Parto prematuro/RPO"; pag. 229-247
- Manual Obstetricia; U. chile, Departamento de
obstetricia y ginecologa, Dr. Hugo Salinas; 2005.
Capitulo 19: "Parto prematuro y rotura prematura
de membranas", pag

138
27. EMBARAZO PATOLGICO :OLIGOHIDROAMNIOS.

Primigesta 33 aos, cursa embarazo de 34 semanas y consulta porque la matrona le pidi
ecografa que mostr oligoamnios severo. La AU est 6 cm bajo el promedio para la EG y se
palpa fcilmente las partes fetales. Refiere disminucin de movimientos fetales.
Antecedentes Familiares: madre diabtica, padre hipertenso.
Antecedentes personales: anorexia 18 aos.
Antecedentes ginecobsttricos: Menarquia 13 aos, RM IV/28, G2P1A0.
Hbitos: cigarros 10 al da.

Desarrollo:
1. Diagnstico: Multpara, embarazo 34 semanas, Oligohidroamnios, tabaquismo.
2. Diagnstico diferencial: RCIU, menor edad gestacional, RPM.
3. Definicin: Cantidad de lquido amnitico menor a la que corresponde para la edad
gestacional (menor a percentil 5 para edad gestacional, se utiliza como valor de corte ILA
< de 5 cm o bolsillo nico <2cm).
4. Etiologa: Es importante considerar los procesos fisiolgicos del lquido amnitico. El
lquido se produce principalmente por la orina fetal, lquido pulmonar, ingesta. Si se altera
cualquiera de estos procesos podemos tener Oligohidroamnios. Existe otro mecanismo
homeosttico regulado por las membranas ovulares (amnios bsicamente) que interfiere
en la absorcin de lquido amnitico ms que en la secrecin manteniendo el equilibrio del
lquido amnitico en casos de Polihidroamnios.
Primer trimestre: Pensar en aneuploidas, tiene peor pronstico.
Segundo trimestre: Ac el feto comienza a orinar por lo cual puede manifestar
enfermedad fetal o alteraciones de la placenta.
Tercer trimestre: Puede ser idioptica, RPM, enfermedad fetal, placentaria o materna.
Causas maternas: insuficiencia uteroplacentaria: pre eclampsia, hipertensin crnica,
enfermedades del colgeno, nefropatas, trombofilias.
Causas placentarias: DPPNI, Transfusin feto-fetal en embarazo gemelar, trombosis o
infarto placentario.
Causas fetales: Las anomalas cromosmicas, anomalas congnitas, especialmente
aquellos asociados con la produccin de orina alterada, restriccin del crecimiento,
embarazo post-trmino, ruptura de las membranas fetales.
5. Epidemiologa: 3,5-5,5% (en Uptodate sealan que hasta el 11%).
6. Diagnstico clnico: Hallazgos al examen fsico, se confirma con ecografa e ILA<5cm o
bolsillo nico <2cm.
7. Anamnesis y examen fsico: Altura uterina disminuida, se palpan fcilmente las
partes fetales, indagar en rotura de membranas (prdida de lquido como agua
abundante con olor a cloro), investigar condiciones maternas como hipertensin,
alteraciones del colgeno, trombofilias, deshidratacin, que pudieran interferir. Uso de
medicamentos como IECA, Inhibidores de prostaglandinas pueden producir
Oligohidroamnios. Especuloscopa para descartar trabajo de parto o RPM.

8. Estudio del paciente:

139
1. Se debe realizar ecografa con biometra fetal, medir bolsillos de lquido amnitico
y bolsillo nico, alteraciones fetales como alteraciones renales, del sistema
nervioso central, cardiacas, esquelticas. Evaluar placenta en bsqueda de
desprendimiento o isquemia placentaria, evaluar crecimiento fetal en bsqueda
de restriccin del crecimiento fetal, marcadores de aneuploidas.
2. Solicitar test de nitrazina o Amnisure en bsqueda de rotura prematura de
membranas.
3. Amniocentesis en caso de alteraciones fetales que pueden ser compatibles con
Trisoma 13 o triploidas.
4. Solicitar MSAFP como marcador pronstico de rotura prematura de membranas.
9. Tratamiento y conducta:
1. Amnioinfusin: En casos donde no es posible estudiar adecuadamente la anatoma
fetal bajo ecografa, o para realizar versin del feto en ceflica o para disminuir los
riesgos asociados al Oligohidroamnios en situaciones idiopticas.
2. Resonancia nuclear magntica asociada a amnioinfusin: Se realiza amnioinfusin
y luego se toma RNM para lograr identificar mejor alteraciones presentes.
3. Hidratacin materna: Puede ser oral o endovenosa. Lo importante es
administracin slo de agua va oral entre 1 y 2 Litros o bien de administracin de
fluidos hipotnicos endovenosos para disminuir la osmolaridad materna y permitir
el paso de agua desde sangre materna al feto. Tambin se ha reportado el uso de
hidratacin ms vasopresina pero slo de modo experimental, logrando disminuir
la osmolaridad materna y aumentando la cantidad de lquido amnitico
rpidamente.
4. Selladores de membranas fetales: parches de fibrina, han demostrado un xito
relativo que an no ha sido validado.
5. Manejo segn edad gestacional.
1. Durante el primer trimestre el pronstico es ominoso, la mayora abortan.
Debe informarse a la madre respecto de los riesgos y realizar seguimiento
ecogrfico del proceso.
2. Durante el segundo trimestre el pronstico depender de la edad
gestacional, severidad del Oligohidroamnios, y patologa subyacente. El
50% de ellos termina en parto prematuro con alta mortalidad fetal o
perinatal. Se realiza de norma, ecografa, si requiere amnioinfusin, o
hidratacin materna y se observa.
3. Durante el tercer trimestre los resultados negativos estn dados por
compresin del cordn umbilical, la insuficiencia tero-placentaria, y la
aspiracin de meconio. La madre se hospitaliza para estudio, hidratacin y
amnioinfusin segn requiera y se solicita perfil biofsico y RBNE
diariamente. En casos de manejo ambulatorio, se solicita RBNE y perfil
biofsico 2 veces por semana hasta el parto. En Oligohidroamnios
idioptico la mayora llegan a trmino sin complicaciones.
6. Interrupcin del embarazo: Estudios muy contradictorios. Si se demuestra
bienestar fetal con RBNE y perfil biofsico, se puede observar 2 veces por semana
hasta el parto. Si se demuestra compromiso materno y/o fetal se debe
interrumpir. En casos de edad gestacional menor a 36 semanas (segn Uptodate)
se debe realizar maduracin pulmonar y luego interrupcin. No hay consenso en
la va de interrupcin del embarazo, aunque las inducciones en condiciones no

140
favorables para un trabajo de parto ha demostrado aumentar el ndice de
cesreas, se sigue manteniendo la va vaginal como eleccin.
10. Complicaciones para la madre y el feto: El Oligohidroamnios produce
deformaciones fetales, compresin del cordn y muerte. Otras
complicaciones se atribuyen a la causa especfica de
Oligohidroamnios.


28. COMPLICACIONES DEL EMBARAZO :


POLIHIDROAMNIOS.

Paciente de 30 aos cursando embarazo de 30 semanas que consulta por disnea
de reposo, edema de extremidades inferiores y aumento importante del
volumen abdominal en ltimos dos meses .El control del embarazo ha sido
normal y los exmenes al inicio del embarazo fueron normales. La AU esta 8 cm
por sobre el promedio para la EG y es difcil palpar partes fetales por sobre
distensin uterina.

Antecedentes Familiares: madre diabtica padre SANO
Antecedentes Personales: no refiere.
Antecedentes Gineco-obsttricos: menarquia 13 aos, RM IV/28 G2P1A0
4300 grs (36 sem)
Hbitos: sana

1. Diagnsticos apropiado:
Multpara de 1
Embarazo de 30 semanas
Obs Polihidroamnios
2. Diagnsticos diferenciales:
Macrosomia fetal
Embarazo mltiple
tero miomatoso con gestacin.
EGD

3. Definicin: Polihidroamnios se define como la medida de ILA >25 cm o una
columna mxima >10 cm. Se definen tres niveles de severidad:
- ILA entre 20 y 25: se considera el lmite alto dela normalidad.

141
- ILA entre 25 y 32: Polihidroamnios moderado.
- ILA >32: Polihidroamnios severo.

4. Etiologa:
Hasta el 60% de los Polihidroamnios son idiopticos y de rango moderado. En
el restante 40% se puede identificar una causa, de las cuales se clasifican:
- Causas fetales: Las malformaciones representan el 20% de las causas de
Polihidroamnios y sobre todo ante casos de aparicin ms precoz,
evolucin rpida y severa. La causa malformativa ms frecuente es la
obstruccin alta del tubo digestivo. Por otra parte, los fetos macrosmicos,
an en ausencia de diabetes materna, presentan frecuentemente un LA en
el lmite alto de la normalidad sin que se acompae de malformaciones.
-
Causas fetales de Polihidroamnios
- Alteraciones gastrointestinales: onfalocele, atresia (esfago, leon, yeyuno), gastrosquisis
- Alteraciones del SNC: anencefalia, defectos del tubo neural
- Causas infecciosas: parvovirus, CMV, toxoplasma, les
- Hidrops fetal
- Cardiopatas congnitas: arritmias, truncus, Co Ao
- - Causas maternas: Mal control metablico por diabetes (principalmente),
Alteraciones torcicas: MAQ, secuestro, HDC, quilotrax
- responsable de hasta el 20% de Polihidroamnios con causa conocida, o la
Alteraciones renales: tubulopatas renales (como el sndrome de Bartter)
- Alteraciones esquelticas: acondroplasia, displasia tanatofrica
isoinmunizacin Rh.
- Alteraciones neuromusculares: distrofia miotnica o enfermedad de Steiner, artrogriposis
- Causas placentarias: algunas anomalas placentarias (Ej. Corangioma:
- Alteraciones metablicas: gangliosidosis, enfermedad de Gaucher
- Tumor benigno ms frecuente de la placenta).
Alteraciones cromosmicas: T 18, T 21, sndrome de Turner
-
Tumores fetales: teratoma sacrococxgeo
5. Epidemiologa: Se estima que afecta cerca del 0,2% de las gestaciones. La
mayora son moderados, apareciendo a partir del tercer trimestre.
6. Fisiopatologa:
El lquido amnitico desempea un papel fundamental en el desarrollo
normal del tracto respiratorio y gastrointestinal y del sistema musculo
esqueltico, a la vez que amortigua traumas fsicos y proporciona una
barrera contra la infeccin en un ambiente termorregulador.
La cantidad total del LA es el resultado del balance entre su produccin y su
eliminacin. Durante el primer trimestre existe un transporte de agua y de
pequeas molculas a travs del amnios y de la piel fetal, por lo que la
composicin del LA es muy semejante a la del lquido extracelular. A partir
del segundo trimestre (>17 semanas) el volumen del LA est regulado por
diferentes vas de entrada y salida a travs del feto y de la placenta. El
principal mecanismo que participa en la formacin es el tracto urinario,
seguidamente de las secreciones traqueales y de las membranas fetales.
Los elementos que participan en la eliminacin del LA son: tracto
gastrointestinal (a travs de la deglucin fetal), respiratorio (mediante

142
movimientos respiratorios fetales) y, el ms importante, la resopcin
intramembranosa. Cualquier alteracin anatmica, estructural o funcional
de las estructuras anteriormente descritas, altera su produccin,
aumentndola o disminuyndola.

7. Diagnstico clnico-Al examen fsico: (Criterios clnicos Sospecha)
1.- Altura uterina mayor que la que corresponde para la edad gestacional.
2.- Dificultad para delimitar al feto con las maniobras de Leopoldo o
incapacidad para palpar las partes fetales.
3.- Dificultad en auscultar los latidos cardiacos fetales y a la palpacin se
aprecie exceso de lquido amnitico y/o exista peloteo fetal.
8. Anamnesis:
- Nuseas y vmitos
- Disnea
- Edema de extremidades inferiores
- Vrices
- Piel abdominal tensa y brillante
- Paciente relata aumento excesivo del abdomen.
9. Estudio de la paciente:

1. Sospecha clnica
2. Se debe solicitar US, para evaluar LA. Las dos tcnicas semicuantitativas ms
utilizadas son:
- La mxima columna vertical (MCV): Se realiza midiendo la mxima columna
vertical de lquido libre de partes fetales y de cordn de manera vertical.
Se considera normal cuando es 8 cm por debajo de la semana 20 y cuando es
10 cm a partir de la semana 21. El lmite bajo de la normalidad es 2 cm a
cualquier edad gestacional.
- ndice de lquido amnitico (ILA): Es el valor obtenido a partir de la suma de
las mximas columnas verticales de lquido, libre de partes fetales o cordn
umbilical, en cada uno de los cuatro cuadrantes que se delimitan por la
interseccin de dos lneas perpendiculares en el abdomen materno
(Ombligo): la lnea media longitudinal con la lnea transversal media entre la
snfisis pbica y el fondo uterino (mtodo de Phelan). til a partir de las 24
semanas, cuando el tero se encuentra por encima del ombligo. El
transductor se coloca en posicin sagital y lo ms perpendicular posible al
suelo. Se considera normal valores entre 5 y 25 cm.
3. Una vez realizado el diagnostico ecogrfico de PHA, es necesario SIEMPRE:
- Descartar diabetes: Solicitar PTGO.

143
- Descartar isoinmunizacin: Solicitar Grupo Rh ,Coombs y anticuerpos
irregulares.
- Descartar patologa infecciosa: Solicitar serologas IgG e IgM de Parvovirus
B19, IgG de Toxoplasma y ELISA IgG Treponmica (TORCH).
4. Estudio ecogrfico fetal: Descartar patologa mal formativa.
- Ecografa anatmica detallada
- Ecocardiografa
- Actitud y movimientos fetales
- ACM
5. Estudio de lquido amnitico:
- Cariotipo
- PCR de toxoplasma y CMV
- Estudio de metabolopatas y enfermedad de Steinert (estudio de
distrofia miotnica)
- Estudio del Cloro para descartar tubulopata (Sndrome de Bartter). Si
ste se encuentra aumentado, se solicitar Aldosterona, Na y K.

10. Tratamiento y conducta:
El tratamiento del Polihidroamnios puede ser etiolgico o sintomtico:
1. Etiolgico: cuando se identifica la causa potencialmente tratable entra tero; por
ejemplo, control metablico ptimo de la diabetes materna, tratamiento anti
arrtmico en fetos hidrpicos con arritmias, colocacin de shunt torcico en
derrames pleurales, etc.
2. Sintomtico: cuando no exista una causa definida o esta no sea tratable intra tero.
Amniodrenaje: Tratamiento sintomtico con dos objetivos, disminuir la
sintomatologa materna y la amenaza de parto prematuro. Por ese motivo no se
realizarn amniodrenajes ms all de la semana 35.
Las complicaciones relacionadas con esta tcnica son el desprendimiento de
placenta, la rotura prematura de las membranas y la bradicardia fetal (hasta en un
3,1%) los casos.
Indicaciones:
- Polihidroamnios severo y longitud cervical inferior a 15 mm o dinmica
uterina materna clnica.
- Discomfort materno importante (sensacin de disnea o dinmica uterina
clnica) independientemente de la severidad del Polihidroamnios.
Inhibidores de las prostaglandinas: Disminuyen el filtrado glomerular fetal
disminuyendo secundariamente la cantidad de orina fetal, tambin favorecen la
reabsorcin pulmonar y el paso de LA a travs de las membranas, as se consigue
una disminucin de la cantidad de LA.

144
Aunque tienen importantes efectos secundarios fetales, el ms importante de ellos
es el cierre prematuro del ductus arterioso, enterocolitis necrotizante e
insuficiencia renal. Por este motivo, es un tratamiento de segunda lnea, cuando el
Polihidroamnios siga siendo sintomtico a pesar de los amniodrenajes y por debajo
de las 32 semanas.
Indometacina 50 mg/8-12 horas durante un periodo mximo de 5-7 das. Se
realizar el tratamiento bajo estricto control eco cardiogrfico (a las primeras 24
horas y a los 4 das de iniciar el tratamiento; si normal, control semanal) para
detectar precozmente una posible restriccin del ductus arterioso. En caso de que
aparezca se debe suspender.
Seguimiento (Depende de la severidad)
ILA 18-24 (lmite alto de la normalidad)
Descartar anomala estructural
Control en dos semanas para evaluar evolucin. Si estable, control
obsttrico habitual
ILA 25-29
Realizar el estudio descrito previamente. Evaluacin de longitud cervical
Control en una semana para valoracin de los resultados del estudio y
evolucin del Polihidroamnios. Si estable, control cada 2 semanas
ILA 30
Realizar estudio descrito previamente. Evaluacin de longitud cervical
Valorar amniodrenaje
Control semanal
Finalizacin (Se decidir en funcin de la etiologa)
Polihidroamnios idiopticos no se ha asociado a un riesgo aumentado de
resultados perinatales adversos, la finalizacin se decidir en funcin de la
clnica materna:
- Sintomtico: A partir de las 37 semanas
- Asintomtico: Se considerar como gestacin de alto riesgo y se
recomendar la finalizacin de la gestacin a las 41.0 2 das.
11. Consecuencias para la paciente y feto:
i. Mortalidad fetal (10-30%): casos severos y precoces
ii. Parto prematuro (22%)
iii. Incremento en la morbimortalidad perinatal
iv. Aumento del riesgo de cesreas por macrosomia, mal posicin fetal y
DPPNI
v. Bajo peso fetal


Bibliografa:

145
Alfredo Prez Snchez, obstetricia, cuarta edicin, pagina 530 539, editorial
mediterrneo
Protocolo medicina fetal, Polihidroamnios, hospital clinic Barcelona

29. EMBARAZO PATOLGICO : PLACENTA PREVIA.


Mujer de 41 aos cursando embarazo de 32+4 semanas que consulta por sangrado
vaginal abundante de color rojo fresco. No refiere dinmica uterina y refiere que hace
10 das tambin presento un sangrado escaso
Antecedentes Familiares: no tiene
Antecedentes Personales: sana
Antecedentes Gineco obsttricos: menarquia 13 aos, RM IV/28 G2P0A1
miomectomia hace 5 aos, legrado por aborto hace 2 aos
Hbitos: sana

1. Cul es el diagnstico ms probable de esta paciente?
Embarazo de 32 semanas
Metrorragia de tercer trimestre: Obs Placenta Previa

2. Cules son los diagnsticos diferenciales?
Otras metrorragias de tercer trimestre como DPPNI, Rotura uterina, Vasa previa.


Tabla extrada Gua Perinatal 2014 Minsal

3. Cmo se define esta patologa?

Placenta previa es la implantacin placentaria despus de la semana 26 en el
segmento uterino a menos de 2 cm o sobre el OCI.

Clasificacin :

146

1. Placenta previa oclusiva total : la placenta cubre completamente el orificio cervical
interno (OCI)
2. Placenta previa oclusiva parcial : la placenta cubre parcialmente el orificio cervical
interno
3. Placenta previa no oclusiva:
Placenta marginal: la insercin de la placenta es adyacente al margen del OCI
pero sin sobrepasarlo
Placenta de insercin baja: borde inferior placentario a mas de 2 cm del OCI


4. Cul es/son la/s etiologa de este cuadro clnico?

No se ha encontrado una causa especfica para la placenta previa, pero la principal
teora est relacionada con una vascularizacin anormal del endometrio, ya sea
por cicatriz o atrofia por traumas previos. Tambin ocurre en placentas de gran
tamao ( embarazos mltiples )

5. Cul es la fisiopatologa de este cuadro?

En esta patologa la placenta se inserta en la parte inferior del tero, la que
durante el tercer trimestre formar el segmento inferior, que es una zona delgada
por menor contenido muscular y poco irrigada. Es por esto que durante la
formacin del segmento inferior la placenta puede migrar separndose del OCI, o
puede desgarrarse produciendo desgarros placentarios. Cuando esto ocurre el
sangrado en el sitio de implantacin se incrementa y las contracciones musculares
no son suficientes para detener el sangrado. Debido a la formacin del segmento
inferior es que cada vez el sangrado es ms frecuente y de mayor volumen. La
placenta al estar bajo la presentacin , impide la expulsin del feto .

6. Qu factores de riesgo buscara?

Los principales factores de riesgo son:

placenta previa en embarazo previo (8-9%)
cicatriz uterina previa (cesrea previa o ciruga uterina, como por ejemplo
miomectomia)
multiparidad
edad materna >35 aos
uso de cocana
tabaquismo
legrado uterino

147
embarazo gemelar
miomas uterinos

7. Cul es la epidemiologia de este cuadro?

La prevalencia es del 0,25-0,5% en las gestaciones nicas. El riesgo es superior en
caso de cesrea previa y aumenta proporcionalmente al nmero de cesreas (2)
8. Cmo hacemos el diagnstico clnico?
La placenta previa clnicamente se presenta como sangrado rojo, fresco , indoloro,
suele aparecer en el tercer trimestre, no es de gran cantidad , guarda relacin con
el estado hemodinmico de la paciente, pero suele recurrir y cada vez con mayor
volumen de sangrado.
El diagnstico ante parto de placenta previa se realiza mediante ecotomografia
transabdominal y/o transvaginal, confirmando la relacin del borde placentario
con el cuello uterino.

9. Qu nos interesa preguntar en la anamnesis de esta paciente?
Antecedente de trauma, actividad sexual , color, cantidad y forma de presentacin
del sangrado, presencia o ausencia de dolor, episodios previos de genitorragia,
HTA concomitante, cirugas uterinas, uso de drogas. Confirmacin de edad
gestacional.

10. Qu buscara dirigida mente en el examen fsico de esta paciente?

Evaluacin del estado general de la paciente y repercusin hemodinmica del
sangrado, signos vitales y latidos cardio-fetales, examen fsico abdominal para
evaluacin de tono uterino, realizar espculoscopia para evaluar sangrado y origen
de este. NO realizar tacto vaginal hasta tener clara la ubicacin placentaria, por
riesgo de producir mayor sangrado o ruptura placentaria en caso de ser placenta
previa.

11. Cmo estudiamos este cuadro y que resultados esperara encontrar en estos
exmenes?
El estudio de este cuadro se realiza mediante imgenes, la ecografa abdominal es
diagnstica en el 96% de los casos, en donde se ve una placenta de insercin baja o
menos de 2 cm del OCI, pero no siempre es capaz de determinar la relacin con el
OCI.( Dopler ayuda ) La ecografa transvaginal es segura de realizar y tiene un
mayor rendimiento que la ecografa abdominal. La RNM es una alternativa no
daina que entrega informacin certera de la localizacin de la placenta y permite
evaluar presencia de placenta acreta.
Adems es necesario solicitar Hemograma, pruebas de coagulacin y
monitorizacin fetal (> 24 semanas).

12. Cul es el tratamiento de esta paciente?

148

El tratamiento depende de edad gestacional, madurez fetal, cantidad sangrado y
severidad del mismo.
Se debe hospitalizar, y dar corticoides a toda embarazada entre las 24 y 34
semanas (Betametasona 12mg cada 24 horas por 2 veces).
Monitorizacin fetal electrnica permanente
Ecografa obsttrica con Dopler.
En gestantes Rh D negativas, administrar gammaglobulina anti-D

En caso de cese del sangrado por ms de 48 horas y con bienestar materno fetal,
se puede optar por manejo ambulatorio, siempre y cuando la paciente tenga
rpido acceso al hospital. Con reposo fsico y sexual. Cesrea electiva a las 36 - 37
semanas.
En caso de sangrado activo sin compromiso materno-fetal se puede tener manejo
expectante, con soporte materno, monitorizacin fetal, corticoides, y cesrea
electiva a las 36 semanas.
Si la edad Gestacional es menor a 36 semanas se debe tomar conducta expectante
con la paciente hospitalizada con vigilancia materna-fetal, con induccin de
madurez pulmonar con corticoides si Edad Gestacional es menor a 35 semanas,
tocolisis si hay actividad uterina concomitante, vigilancia de disminucin del
hematocrito y ferroterapia.
En caso de sangrado activo con compromiso materno-fetal se debe realizar manejo
hemodinmico intensivo con interrupcin inmediata del embarazo, independiente
de la edad Gestacional o del tipo de placenta previa.
La cesrea se realiza con incisin que evite seccionar la placenta, puede haber
sangrado importante post-alumbramiento por disminucin de capacidad contrctil
del segmento inferior o presencia de algn grado de acretismo placentario .
En caso de placenta previa asintomtica se debe realizar cesrea electiva a las 36 -
37 semanas si la placenta es oclusiva ( para evitar el sangrado que ocurrir si hay
contracciones ) , si la placenta es de insercin baja se puede esperar el inicio del
trabajo de parto y decidir va de parto segn evolucin.

13. Qu consecuencias puede traer esta patologa a la mujer / madre/ y al feto?
Las principales consecuencias que conlleva la placenta previa son:
Riesgo de hemorragia ante y post-parto
Hemorragia grave asociada a shock hipovolmico
Presencia de placenta acreta,
Histerectoma urgente
Embolia de lquido amnitico
Morbimortalidad perinatal asociada al parto prematuro
Muerte fetal
Prematurez

149

Referencias
1. Gua Perinatal 2014 Minsal
2 Guas Servicio de Medicina Materno fetal. Instituto Clnico de Ginecologa,
Obstetricia y Neonatologa, Hospital Clnic de Barcelona.

30. EMBARAZO PATOLGICO : DPPNI



Mujer de 38 aos cursando embarazo de 31 semanas de gestacin que acude a urgencia por
metrorragia escasa de sangre oscura con cogulos, dolor abdominal epigstrico, disminucin de
mov fetales .Hasta la semana anterior el embarazo fue de evolucin normal.
Antecedentes Familiares: madre con hemofilia
Antecedentes Personales: HTA crnica en tratamiento
Antecedentes Gineco obsttricos : menarquia 13 aos , RM IV/28 G5P4A0
Hbitos: Antecedentes de consumo de cocana.

1. Diagnstico apropiado:
Multpara de 4
Embarazo de 31 semanas
Metrorragia del 31 trimestre, obs Desprendimiento prematuro placenta normo
inserta
Adiccin a cocana

2. Diagnstico diferencial:
Placenta previa
Rotura uterina
Rotura de vasa previa
Rotura del seno marginal

150
Tabla extrada de Gua Perinatal 2014


3. Definicin:
Desprendimiento total o parcial de la placenta no previa desde su sitio normal
de implantacin del cuerpo uterino, a partir de las 22 semanas y antes de la
expulsin completa del feto.

Clasificacin:
Parcial: Parte de la placenta se separa de la pared uterina
Total : Separacin completa de la placenta desde la decidua
Marginal: Separacin est limitada por el borde de la placenta

4. Etiologa
Se desconoce su causa

5.Fisiopatologa
Se inicia por sangrado en la decidua basal por dilaceracin de vasos sanguneos.
Se forma un hematoma retro placentario y este por tensin infiltra fibras
musculares uterinas, lo que genera estimulacin de la actividad uterina de

manera exagerada, acabando en hipotona uterina. La sangre alcanza los
mrgenes de la placenta y diseca las membranas ovulares
La acumulacin de sangre en el hematoma retro placentario produce un rpido
consumo de factores de la coagulacin, lo que puede desencadenar una CID.
La intensidad del sangrado externo no se correlaciona con la severidad del
cuadro clnico

6. Factores de riesgo
HTA
Hipertensin arterial-Pre eclampsia
Edad materna > 35 aos
Tabaquismo
Cocana
Antecedente de DPPNI (15% probabilidad de repetirlo)
Trauma abdominal
Descompresin uterina brusca en el parto del primer gemelar
RPM

7. Epidemiologa
Causa ms frecuente de metrorragia del segundo trimestre

151
Frecuencia 1-2% de los embarazos
Alto riesgo de morbimortalidad materna y perinatal

8. Diagnstico clnico

El diagnstico es clnico
Dolor abdominal, generalmente epigstrico ( puede se lumbar )
Metrorragia segunda mitad del embarazo, cuanta variable, rojo oscuro que no
se relaciona con el compromiso hemodinmico de la paciente.
Hipertona uterina (aumento del tono uterino), contracciones frecuentes sin
relajacin completa
Polisistola
Bradicardia fetal, desaceleraciones, ausencia LCF

9. Anamnesis
Cuantificar el sangrado, tiempo de evolucin, coloracin.
Frmacos de consumo habitual
Cocana: tiempo de uso, ltimo consumo.
Buscar factores de riesgo

10. Examen fsico
General: Evaluar compromiso hemodinmico. Puede ser discordante con el
sangrado
Obsttrico: tero sensible, Hipertona, Polistolia
Compromiso fetal: Bradicardia fetal o ausencia LCF
Especuloscopia

11. Estudio de la paciente
Diagnstico es clnico
Sospecha: Metrorragia segunda mitad de gestacin + Dolor abdominal +
Hipertona uterina
Eco TV: til en diagnstico diferencial de placenta previa.
Se puede observar a la ecografa obsttrica la presencia de hematoma retro
placentario (imagen retro placentaria eco refringente, heterognea, que puede
aumentar el grosor placentario, con o sin diseccin de las membranas ovulares),
sin embargo, su ausencia no descarta DPPNI
Laboratorio: Hemograma, pruebas de coagulacin, Fibringeno , Grupo y RH

12. Tratamiento

152
Hospitalizar a la paciente: Cuanta hemorragia, Control PA, FC, debito urinario
(1)
Monitorizacin fetal continua
2 Vas venosas
Hematocrito, clasificacin de grupo y Rh en toda paciente. Pruebas cruzadas
ante la necesidad de transfusin.
Solicitar recuento plaquetario, tiempo de protrombina (TP, INR), tiempo de
tromboplastina parcial activada (TTPK) y, en los casos con coagulopata
evidente solicitar fibringeno, dmero-D y productos de degradacin del
fibringeno (PDF) si estn disponibles

1. Severo
Metrorragia masiva, descompensacin hemodinmica materna, CID, hipertona
uterina y/o sufrimiento fetal.
Interrumpir el embarazo de inmediato por la va ms expedita
En casos de bito fetal, debe favorecerse un parto vaginal, pero es importante
recordar que la muerte del feto traduce un desprendimiento masivo, con alto
riesgo de descompensacin hemodinmica
Administrar cristaloides y valorar la necesidad de transfundir glbulos rojos.
Acompaar 1 unidad de plasma fresco congelado por cada 4 U de glbulos
rojos para evitar coagulopata por dilucin. Mantener un hematocrito > 28% y
una diuresis > 30 ml/h.

2. Moderado

No se presentan condiciones graves de DPPNI Severo. Depende de Edad
Gestacional:

EG > 36 sem: interrupcin del embarazo mediante induccin oxitcica o
cesrea segn condiciones obsttricas.
EG < 36 sem: manejo expectante con vigilancia estricta, el que debe transcurrir
en un centro de atencin terciaria:
o Induccin de madurez pulmonar fetal.
o Interrumpir el embarazo segn evolucin clnica.
o Monitorizacin de la FCF y PBF cada 48-72 horas.
o Evaluar cuidadosamente el volumen de lquido amnitico porque existe
un porcentaje de pacientes con DPPNI crnico que desarrollan
oligoamnios.
o Dopler umbilical cada 48-72 horas, intercalado con monitorizacin y PBF

153
o Amniocentesis si se sospecha infeccin intrauterina subclnica.
o Si hay contractilidad uterina asociada, valorar individualmente la
posibilidad de administrar tocolisis endovenosa en edades gestacionales
menores a 32 semanas. De lo contrario, permitir la evolucin
espontnea.




13. Consecuencias para la paciente y el feto
Maternas:
Prdida excesiva de sangre y CID, que puede llevar a un Shock hipovolmico
con insuficiencia renal, sndrome de distres respiratorio del adulto, falla
multiorgnica y muerte materna.
Cesrea

Fetales:
Parto prematuro
RCIU
Bajo peso al nacer
Asfixia fetal


Referencias:
1. Ministerio de Salud. Gua Perinatal 2014

31. EMBARAZO PATOLOGICO : RCIU



Paciente de 42 aos cursando embarazo de 31 semanas de gestacin que presenta AU
menor a la esperada por EG, refiere disminucin de movimientos fetales.
Talla 1,68 mts, pesa 55 kg y ha aumentado 2,5 kg en el embarazo. A la semana 20 la ECO
mostro feto nico en percentil 50 y la eco actual muestra feto de 31 semanas en
percentil 5 y oligoamnios moderado.
Antecedentes Familiares : madre diabtica padre Ca colon
Antecedentes Personales : no refiere
Antecedentes Gineco obsttricos : menarquia 13 aos , RM IV/28 G1P0A0
Hbitos : cigarro 12 al da

154

1. Diagnstico apropiado:

Primpara
Embarazo de 31 semanas
Restriccin del crecimiento intrauterino.
Oligoamnios moderado
Tabquica

2. Diagnstico diferencial:
En sospecha por AU menor a la esperada por EG.
- Feto con EG menor a la indicada
- Fetos pequeos constitucionalmente
- Oligohidroamnios
- Malformaciones
- Posicin transversa
3. Definicin: Condicin en la que el feto posee una estimacin de peso al
percentil 10 para la Edad Gestacional de una poblacin determinada para una
edad gestacional. EG p3Todos son RCIU.
Clasificacin:
- RCIU inicio precoz: 28 semanas
- RCIU inicio tardo: 28 semanas
4. Etiologa:
Grupos etiolgicos
- Fetos con un crecimiento intrauterino restringido (CIR). Son fetos pequeos
con insuficiencia placentaria.
- Fetos pequeos para la edad gestacional normales (PEG). Son fetos
constitucionalmente pequeos, representando el espectro inferior de peso
de fetos normales.
- Fetos pequeos para la edad gestacional anormales. Son fetos pequeos
por una condicin patolgica extrnseca a la placenta Sospecho con EG
percentil 3 o alguna malformacin ecogrfica.

PEG
constucional
Simtrico (50-70%)
(Proporcionado)
PEG anormal
Pequeo edad (10-20%)
gestacional

Asimtrico RCIU placentaria
(Masa
abdominal) (20-30%)

155

Fetales Placentarias Maternas
Alteraciones cromosmicas Insuficiencia placentaria Bajo peso materno
Malformaciones congnitas Infartos placentarios Tabaco, OH, drogas
Infecciones (TORCH) Patologa del cordn umbilical Mala historia obsttrica
Gestacin mltiple Factores ambientales
Sexo masculino Malformacin uterina

5. Epidemiologa: Aproximadamente 10%.
6. Fisiopatologa:
CIR: (crecimiento intrauterino retardado) La ms aceptada en la actualidad es la
vasoconstriccin crnica de los stem villi terciarios debida a la invasin
trofoblastica inadecuada de las arterias espirales maternas, produciendo infartos
en el rea placentaria, obliteracin y fibrosis vascular, llevando a un aumento en la
resistencia vascular en la placenta, produciendo una reduccin de la masa
placentaria.
En las fases iniciales de esta situacin patolgica, el feto se adapta disminuyendo
su velocidad de crecimiento modificando sus patrones de conducta y capturando
de manera ms eficiente el
oxgeno, mediante la
policitemia Si esta situacin se
mantiene, el feto entra en una
fase de hipoxemia que se puede
prolongar durante semanas.
Por debajo de cierto umbral de
saturacin de oxgeno se
activan una serie de
quimiorreceptores que ponen
en marcha todo un conjunto de
cambios hemodinmicos que
tienen como principal funcin
preservar el aporte de oxgeno a los rganos nobles, llamado centralizacin. Los
cambios son:
- La vasodilatacin cerebral o brain sparing.
- Oligohidroamnios que se observa en los fetos con insuficiencia placentaria.
- Redistribucin cardiaca donde el corazn fetal se encuentra en una situacin que
no es fisiolgica ya que el ventrculo izquierdo irriga un territorio de baja
resistencia, el cerebro, mientras que el derecho irriga un territorio de
progresivamente alta resistencia, la placenta.

156
Si la situacin persiste, el feto pone en marcha mecanismos de obtencin de
energa va anaerbica a partir de la glucosa que conllevan la produccin de
hidrogeniones que acidifican el medio, produciendo acidosis. Esta afecta al corazn
fetal, pues las fibras miocrdicas sufren fenmenos de necrosis y son substituidas
por tejido fibroso. Estos cambios histolgicos a su vez conllevan una alteracin de
la funcin diastlica del corazn. Si esta situacin persiste, la muerte fetal acontece
en horas.
PEG anormales: Es muy complejo y tiene poco inters clnico. En general, la
mayora de causantes combinan una accin sobre el desarrollo de la placenta
(mediante lesin endotelial) as como un efecto cito ltico que limita el potencial de
crecimiento intrnseco del feto.
7. Diagnstico clnico:
Es importante determinar la EG al comienzo del embarazo, ya que a medida que
avanza el embarazo es cada vez mas imprecisa.
La exploracin fsica es de utilidad reducida a la hora de identificar un CIR, pero es
una prueba importante de deteccin de crecimiento anmalo.
El tamao de la madre, tambin es de utilidad reducida, pero un leve aumento de
peso de la madre durante el embarazo me hace sospechar de un CIR.
La determinacin seriada de alturas uterinas se utiliza comnmente para detectar
un CIR.
Por lo tanto si se sospecha un CIR basndose en los factores de riesgo y evaluacin
clnica, debe realizarse una ecografa para determinar el tamao y el crecimiento
fetal.
En resumen:

Sospecha Clnica (Menor altura uterina para su edad gestacional, menor tamao
de la madre, determinacin de la altura uterina) ecografa para determinar
tamao y crecimiento fetal

8. Anamnesis:
Debe ir dirigida a descartar los factores de riesgo antenatal que se han asociado
con las alteraciones del crecimiento, en bsqueda de la posible etiologa:
Historia previa de alteracin de crecimiento o muerte perinatal
Factores ambientales: alcohol, tabaco y otras drogas
Infecciones
Enfermedades maternas:
Trombofilias
Enfermedades renales y vasculares
Obesidad
Diabetes
Hipertensin arterial

157

9. Estudio de la paciente:
Siempre evaluar correcta EG.
Ecotomografia (Gold estndar); debe considerar un estudio anatmico
detallado
- Estimacin del peso fetal
- Valoracin del crecimiento fetal
- Velocidad de crecimiento
- Estudio morfolgico fetal
Estudio hemodinmico (Dopler: Diferencia de un CIR-PEG normal)
1. Ingreso por RCIU
- Dopler de arteria uterina
Permite evaluar el flujo de sangre materno hacia la placenta.
Normalmente es un territorio de baja resistencia, y por lo tanto debiese
haber poca diferencia entre la velocidad de flujo en sstole y distole.
sea cuan tapada esta la arterial espiralada que llega a la placenta.
Se ve alterada con resistencia aumentada, por lo tanto un IP elevado.
- Dopler de arteria umbilical
Evala el flujo de la arteria umbilical,
bombeada por el feto a la placenta, por lo
tanto estima la resistencia placentaria.
Lo normal es que sea un territorio de baja
resistencia, por lo tanto la diferencia entre
sstole y distole es poco (Imagen 1).
Si se observa enfermedad placentaria, y se
tapa en aprox. 30%, entonces aumenta la
resistencia, IP elevado (Imagen 2).
Si se tapa aprox. el 50% de la arteria, con
resistencia muy aumentada, entonces se
produce un flujo diastlico ausente o cero
(Imagen 3).
Si se tapa el 80%, entonces se produce un
flujo diastlico reverso (Imagen 4).
- Evaluar peso fetal
2. Control por RCIU
- Dopler de arteria umbilical c/sem
- Evaluar peso fetal c/2 sem


158

Importante: En fetos con percentil 3, realizar SIEMPRE:
- Cariotipo y cardiograma
- Estudio de TORCH
- Estudio de trombofilia (Sd. Anti fosfolpidos)
- Estudio de PE.

10. Tratamiento y conducta:
Una vez realizadas las pruebas de ingreso, podemos aclarar nuestros diagnsticos
diferenciales:
- Alteracin Dopler: CIR.
- Infeccin o malformacin: PEG anormal.
- Todas las pruebas son normales: PEG normal.
A modo de resumen, se inicia con dopler de arteria uterina y arteria umbilical, se
observa alterado, evalo con dopler de arteria cerebral media (Mejor marcador de
hipoxia fetal del feto: Normalmente se observa una gran diferencia entre la sstole
y la distole, y en situaciones de hipoxia, la arteria se vasodilata para llevar ms
sangre a tejidos blandos y nobles, que es el cerebro, por lo tanto disminuye la
resistencia hacia distal, aumentando el flujo diastlico).
En caso que se ve alterado la ACM, se debe realizar dopler del ductus venoso, que
indica la entrada de sangre a la aurcula derecha. Es necesario evaluarla, ya que la
persistencia de la hipoxia de manera crnica, desencadena la acidosis metablica
que provoca la destruccin de fibras miocrdicas. Esta situacin condiciona una
disminucin de la distensibilidad del miocardio con aumento de las presiones tele
diastlicas, por tanto, una mayor dificultad de llenado del corazn derecho
(Corazn ms rgido). Esta situacin se manifiesta en las ondas de velocidad de
flujo venosas como una disminucin de las velocidades durante la contraccin
atrial y en un aumento general de los ndices de pulsatilidad.
Adems como marcador agudo, es necesario realizar evaluacin de la unidad feto
placentaria, en el siguiente orden de efectividad para evaluar hipoxia fetal.
1. TTC (Excepto si est en periodo expulsivo)
2. Dopler (Cambios ms precoces)
3. PBF (Cambios tardos): La hipoxia persiste se produce una disminucin
progresiva de la cantidad de lquido amnitico, siendo ste el nico parmetro
del PBF que refleja cronicidad. En estadios avanzados de hipoxia se produce la
desaparicin de los movimientos respiratorios. Finalmente, cuando aparece la
acidosis se afectan el tono y los movimientos groseros, y el RCTG muestra
patrones manifiestamente patolgicos.

159
4. RBNE: En situaciones severas de hipoxemia, estmulos procedentes de quimio y
barorreceptores perifricos generan estmulos parasimpticos que dan lugar a
la aparicin de desaceleraciones. Adems, en estadios avanzados de
compromiso fetal, el efecto de la acidosis sobre el sistema nervioso y el efecto
directo sobre la actividad intrnseca cardiaca favorecen la aparicin de
desaceleraciones y de la prdida de la variabilidad.
Seguimiento:
- PEG anormales: Manejo especfico de cada patologa.
- PEG normales: La evidencia recomienda un control no intensivo. Existe un
estudio randomizado que observa como el control intensivo aumenta las
inducciones sin mejorar los resultados.
- CIR: Existe un buen consenso en considerar que existe un deterioro progresivo
fetal. As que una clasificacin en estadios fisiopatolgicos parece adecuada, a
pesar de que no se acepta de forma universal. El control del bienestar fetal est
condicionado al grado de severidad del CIR, por lo que tambin depender del
estadio de severidad en el que nos encontremos. Lo importante es seguir al
feto, con control de dopler semanales y control de peso c/2 semanas. En caso
de cualquier alteracin, se debe modificar su clasificacin y actuar
dependiendo de ella.



160
11. Consecuencias para la paciente y el feto:
Fetales:
- Corto plazo: Hipoglicemia, hipocalcemia, hipotermia, policitemia, enterocolitis
necrotizante, hipertensin pulmonar, HIV y SRD.
- Largo plazo: Enfermedades cardiovasculares, AVE, DM no insulino dependiente,
HTA


32. PARTO PATOLGICO : CESREA



Paciente de 36 aos cursando embarazo de 39 semanas que es derivada para
interrumpir embarazo por presentar registro fetal ominoso y no presentar condiciones
cervicales para la induccin.

Antecedentes personales:
- HTA crnica en tratamiento con metildopa
- anorxica a los 18 aos
- Talla actual: 1,65 mts
- Peso actual: 54 kg.
Hbitos: 10 cigarrillos al da
Antecedentes familiares:
- madre DM2
- Padre HTA
Antecedentes Gineco-obsttrico
- menarquia: 13 aos
- RM: IV/28
- G2 P1 A0

1. Diagnostico mas probable:
Multpara de 1
Embarazo de 39 semanas
MFE tipo I
Parto patolgico CESAREA
HTA crnica en tratamiento
Tabaquismo

2. Definicin: la operacin por cesrea se define como la extraccin del feto, placenta y
membranas a travs de una incisin en la pared abdominal y uterina.

161
3. Etiologa del cuadro clnico: la cesrea es un procedimiento quirrgico que puede ser
utilizado en cualquier momento durante la gestacin si es necesario. Hay
situaciones en que es mandatorio realizarla por ser la forma mas expedita y rpida
de interrumpir el embarazo y extraer el o los productos de la concepcin.
En el caso presentado, el registro fetal ominoso hace inminente extraer el feto lo
antes posible, y la ausencia de modificaciones cervicales hace que la cesrea sea la
mejor opcin.

4. Factores de riesgo:
Indicaciones de cesrea
Causas maternas:
o Doble cesrea anterior.
o Cesrea corporal anterior
o Enfermedad materna con contraindicacin de parto vaginal con informe de
la especialidad respectiva.
o Prueba de parto vaginal fracasada.
o Embarazo de 42 semanas sin condiciones obsttricas.
o Cicatriz uterina no obsttrica
o Induccin fallida segn protocolo respectivo.
o Cese secundario de la dilatacin segn protocolo respectivo.
o Estados hipertensivos y/o pre-eclampsia severos, eclampsia sin condiciones
obsttricas va baja.
o Cncer crvico uterino invasor.
o Herpes activo.
o Cese del descenso sobre tercer plano.
o VIH materno con carga viral alta.
o Cesrea anterior, feto igual o mayor a 4000grs.

Causas fetales:

o Presentacin distcica.
o Peso fetal igual o mayor a 4300grs.
o Peso fetal menor de 1kg.
o Test de Tolerancia a las contracciones uterinas positivo.
o Sufrimiento fetal agudo con respaldo documentado segn protocolo.
o Deterioro de la unidad feto placentaria que requiera extraccin fetal
inmediata, sin condiciones obsttricas.
o Embarazo gemelar en Primigestas o multparas y/o al menos con un feto en
presentacin distcica.
o Muerte materna con feto vivo.
o Malformaciones fetales con riesgo de lesin con parto vaginal.
- Gastrosquisis
- Higroma qustico
- Onfalocele

162
- Espina bfida
- Meningoencefalocele

Causas ovulares:

o Placenta previa oclusiva total o parcial( nica indicacion absoluta ).
o Desprendimiento prematuro de placenta normo inserta.
o Prolapso de cordn.
o Infeccin ovular sin condiciones obsttricas para Parto

5. Epidemiologa:
El promedio nacional (sector pblico y
privado) de cesrea es alrededor del 45
%.Hay clnicas privadas en que llegan a
ser un 80% del total de los partos.
Estos altos nmeros se deben
principalmente a dos razones: en primer
lugar la presencia de una cesrea
anterior segmentaria es
contraindicacin relativa a una nueva
cesrea, y una cesrea corporal o dos cesreas previas independiente del lugar son
contraindicacin absoluta de este procedimiento quirrgico.
En segundo lugar, en Chile existe la tendencia de programar el da del parto, y
asociado a lo anterior que el obstetra que ha hecho el seguimiento del embarazo
sea el encargado de atender el parto.

6. Cmo hacer el diagnstico clnico?: cuando existe indicacin absoluta de cesrea o
relativa, siendo en esta ltima situacin juicio del obstetra si se realiza o no.

7. Que es importante preguntar en la anamnesis?:
sensacin de movimientos fetales
cesrea previa
patologas durante el embarazo
antecedentes de partos previos y peso fetal en caso de haberlo
antecedente de cirugas
patologas maternas crnicas

8. Qu buscar dirigida mente al examen fsico?: el presencia de un registro detal
ominoso hay que interrumpir el parto por la forma mas expedita. Bajo esta
premisa el tacto vaginal es mandatorio para tomar una decisin cual ser la va de
interrupcin. En este caso no se apreciarn modificaciones cervicales por lo que se
decide parto cesrea.

163
9. Cmo estudiar el cuadro? qu resultados espera encontrar en los exmenes?: en
el caso presentado se informa que el hay un registro fetal ominoso y que las
condiciones para el parto vaginal no son optimas para la interrupcin inmediata.

10. Tratamiento: la resolucin en este caso es la
interrupcin inmediata del embarazo por
la va mas expedita, en ese caso es la
cesrea por las condiciones cervicales
antes descritas.
Existen 2 tcnicas para realizar la
laparotoma:
- Pfannenstiel: entrada abdominal
transversal baja, ventaja esttica, pero la
desventaja es que se asocia a mayor dolor e infeccin post-quirrgica
- Media infra- umbilical (vertical baja): mejor desde el punto de vista anatmico.
Permite un abordaje mas rapido y expedito, muy til en emergencias en que se
debe hacer una interrupcin inmediata. La desventaja es desde el punto de vista
esttico.

11. Consecuencias de esta patologa para la madre y feto:
Feto:
o Muerte fetal en caso de no interrumpirse el parto en el momento
adecuado.
o Taquipnea transitoria
o Traumatismo fetal
Maternos:
o Hemorragia post parto
o Endometritis puerperal
o Infeccin de herida operatoria

Bibliografa
- Gua Perinatal CEDIP 2014
- Perez A, Donoso E. Prez Snchez Obstetricia. Cuarta edicin (2011). Operacin
cesrea. Capitulo 24.
- Beckmann C. Obstetricia y Ginecologa. Sexta edicin (2010). Parto anmalo y
vigilancia fetal durante el parto. Capitulo 9.
- Carvajal J, Ralph C. Manual obstetricia y ginecologa. Cuarta edicin (2013).
Operacin cesrea. Capitulo 15.

164
33. PARTO PATOLGICO : INDUCCIN DE PARTO.

Multpara de 1 de 27 aos cursando embarazo de 42 semanas que es ingresa por
matrona para interrupcin del embarazo por embarazo en vas de prolongacin.
Embarazo de evolucin fisiolgica con exmenes normales.
Antecedentes Familiares : no refiere
Antecedentes Personales: no refiere.
Antecedentes Gineco obsttricos : menarquia 13 aos , RM IV/28 G2P1A0
Hbitos: alcohol ocasional

1. Diagnsticos:
M1
Emb 42 semanas
Embarazo prolongado

2. Diagnsticos diferenciales:
Diferenciar entre un verdadero embarazo prolongado o una edad gestacional mal
calculada.

3. Definicin:
Embarazo prolongado: >42 semanas (>294 das)
Embarazo en vas de prolongacin: entre 41- 41+6 semanas
Induccin: Inicio provocado o artificial de las contracciones uterinas con el
objetivo de obtener un parto vaginal.

4. Etiologas:
En los verdaderos embarazos prolongados la causa es desconocida. En general es
una falla en el determinismo del parto que se puede dar por:
Quiescencia prolongada
Falla activacin miometrial
Falla en la estimulacin miometrial
La principal causa es el error en el clculo de la EG
Se cree que hay factores genticos involucrados ya que se ha visto relacin entre
madres e hijas con embarazo prolongado. Incluso se cree que hay gentica paterna
involucrada.

5. Fisiopatologa: (Fisiologa normal del embarazo y parto)

Mecanismos regulatorios tanto maternos como fetales determinan la duracin de
la gestacin y el inicio del trabajo de parto. Estos mecanismos no se tienen claros,
pero hay diferentes hiptesis.

165
Teoras actuales del inicio de parto
1. Supresin de la mantencin del embarazo: algo produce que se deje de
mantener el embarazo y por consecuencia se producir el parto.
2. Induccin espontnea por uterotoninas
3. Teora de la seal fetal: feto maduro enva seal para inicio de trabajo de parto
(no tiene base cientfica).
La teora de la seal fetal se plantea debido a que se ha visto que en fetos
que tienen: Agenesia de hipfisis, Anencefalia, Hipoplasia suprarrenal o
Dficit de Sulfatasa Placentaria. Se asocian a embarazo prolongado, pero no
siempre. Esto sera por un dficit en la formacin de Estrgenos
placentarios ms que a la secrecin defectuosa de cortisol por las
suprarrenales.
Fases uterinas del embarazo


0 Quiescencia miometrial
1 Activacin del miometrio.
2 Estimulacin del miometrio.
3 Involucin del miometrio
(retraccin uterina).




Fase 0
Periodo de relajacin del
miometrio (activa)
Es el 95% del embarazo
La fibra es refractaria a la oxitocina (no tiene receptores para sustancias pro-
contrctiles)
Cuello firme y rgido
Acortamiento cervical 24 - 26 semanas del embarazo hay mayor riesgo de Parto
prematuro.
Fase 1
Despertar uterino.
Se reestablece la capacidad de las clulas miometriales para regular la
concentracin de calcio citoplasmtico.
Aumento de sensibilidad a las Uterotoninas (aumento receptor).
Inicio contracciones
Hay reblandecimiento y maduracin del cuello y aumento Gap junction.
Formacin del segmento inferior (espacio entre cuello y fondo que se distiende al
embarazo-aqu est el anillo de Bandl).
Fase 2

166
Sinnimo de trabajo de parto
El tero es estimulado al contraerse
Dividido en 3 estadios del Trabajo de Parto (contracciones uterinas,
dilatacin cervical y expulsin del feto y la placenta).
Fase 3
Involucin uterina
Periodo fisiolgico de recuperacin post-parto
Restablecimiento de la anatoma y fisiologa normal de la mujer
Duracin aproximada: 42 das.
Mecanismos de control del parto
1. Hay factores que disminuyen la relajacin uterina, o sea, que quitan la
quiescencia.
La Progesterona (P) favorece la quiescencia (relaja el tero- fase 0 a 1) vs
Estrgeno (E) que se encargan de la activacin, por lo tanto la quiescencia
miometrial est dada por la relacin E/P. Adems de la accin de la P, est dada
por NO, pptidos natriurticos, activadores de canales de K, AMPc y GMPc.
2. Hay factores que aumentan la contractilidad del tero (capacidad de
contraccin fase 0 a 1).
La activacin est dada por la accin de uterotropinas (favorecen receptores para
la contraccin, en general hormonas), o sea, de: Estrgenos, GAP junctions,
receptores a uterotoninas.
3. Hay factores que aumentan las contracciones (contracciones en s, fase de 1 a
2).
Lo que va a producir la estimulacin son las Uterotoninas (sustancias que inducen
las contracciones) y estas son: oxitocina (neurohipfisis), prostaglandinas,
endotelina y factor activador plaquetario.

Recordar que trabajo de parto es cuando hay contracciones 2 en 10 y
borramiento cuello mantenido y regular; pero previo a esto se comienza poco a
poco a tener contracciones (paso de quiescencia a activacin inicio parto).
No se puede pasar de fase 0 a 2 porque son necesarias las uterotropinas para
la accin de uterotoninas.

6. Fisiologa del parto
Caractersticas de las contracciones uterinas y de las modificaciones cervicales
durante el trabajo de parto.

Caractersticas de las contracciones uterinas

Estn definidas por:
TONO BASAL: la presin ms baja registrada entre las contracciones, entre 8-12
mmHg.

167
INTENSIDAD (AMPLITUD): aumento en la P IU con cada CU, entre 10-50
mmHg.
FRECUENCIA: n de contracciones producidas en 10min, entre 4-5 en 10 min (si
es regular > a 2 ya se considera trabajo de parto).
DURACIN: entre 30-50 seg.

Control clnico de las contracciones
Manual: es la mejor, detecta contracciones sobre 15 mmHg.
Electrnica: registro no relacionado con intensidad, su ventaja es poder la
relacin con la Frecuencia cardiaca fetal (FCF).
Presin Intraamniotica: electrodo a travs del lquido hasta el msculo, muy
raro.

Fisiologa de la dinmica uterina
La onda contrctil uterina es un marcapaso funcional, no anatmico. Es la zona
donde nace la contraccin y es en el fondo del tero.
Triple gradiente descendente (TGD):

Contraccin se inicia en fondo y tiene propagacin descendente.
Es ms intensa en el fondo (intensidad es mayor en partes altas del tero).
La duracin de la fase sistlica es mayor en las partes altas del tero (dura
ms en el fondo)
Por lo tanto, la dinmica uterina se mide en el fondo uterino.

Maduracin del cuello uterino
En la quiescencia miometrial el cuello se encontrar rgido y duro (consistencia
como la nariz)
En la activacin miometrial se ablanda (consistencia como la pera)
En la estimulacin se comienza a acortar y es blando (como los labios).
El ablandamiento del cuello depende de las prostaglandinas, estrgeno,
disminucin progesterona, xido ntrico (NO) y prolactina (uterotoninas y
uterotropinas).

6. Factores de riesgo:
Maternos
Primigesta
Edad materna mayor a 30 aos
Nivel socioeconmico, mujeres de Amrica latina, Asitica y Afroamericanas.
Antecedente partos anteriores o de historia obsttrica familiar
Obesidad
Hipotiroidismo
Desproporcin cfalo plvica
Raza blanca

168
Turnos nocturnos

Fetales
Anencefalia
Ausencia de glndula hipofisaria,
Hipoplasia adrenal bilateral
Macrosomia
Sexo masculino

7. Epidemiologia:
Incidencia de un 5-10%
USA: 18% embarazos nicos continan sobre 41 semanas, solo 10% pasan 42
semanas
Chile: 5-8%
Mujeres que han presentado un embarazo prolongado, presentan un 20% de
recurrencia de parto prolongado.

8. Diagnostico:
Conocimiento exacto de la edad gestacional.
Posibilidades:
o Amenorrea segura y confiable
o Examen ultrasonogrfico:
LCN (7-12 sem)
Fmur o DBP
9. Anamnesis:
En este caso lo principal es diferenciar si estamos frente a un diagnstico de
edad gestacional correcto o no. Evaluar ecografas
Antecedentes obsttricos personales y familiares dando nfasis al
antecedente de embarazo prolongado
Evaluar bienestar fetal preguntando por contracciones, perdidas de fluido por
vagina, movimientos fetales
Uso de frmacos
10. Examen fsico:
Evaluacin bienestar fetal, RBNE, Evaluacin ecogrfica del LA,
Condiciones cervicales: Bishop

169

11. Estudio de la paciente:
Ecografa abdominal: evaluar lquido amnitico, peso fetal, presentacin.
RBNE
ndice Bishop: predice la probabilidad de xito de parto vaginal, es mejor que la
evaluacin de la ecografa.
o Buenas condiciones obsttricas: > 6. Probabilidad cercana al 100% de
parto vaginal si induzco.
o Mala condiciones obsttricas: < 6.

12. Tratamiento:
Induccin:
1.- Indicacin: Cuando el riesgo de continuar el embarazo excede el riesgo
materno y/o fetal de una induccin
2.- requisitos de induccin:
Exista una indicacin.
Certeza de edad gestacional.
Cuello favorable.
Presentacin y tamao fetal.
Conocer estado de las membranas.
Bienestar fetal.
Discusin previa con la paciente.
3.- si cuello favorable (Bishop 6)
Amniotomia y/o despegue de membranas: en conjunto con
oxitocina
Oxitocina: produce contracciones miometriales, usar con
monitorizacin fetal

Bomba de Dosis inicial aumento intervalo Dosis mxima


infusin
5 U en 500 ml 1 mU/min (6 Doblar dosis 20 min. 30 mU/min
10 U en 1000 ml/h) hasta
ml de SF conseguir DU
eficaz o
mximo 48
ml/h


170
Prostaglandinas vaginal: si cuello desfavorable, produce
contracciones miometrial, maduracin cervical, vasodilatador,
broncodilatador.
VAGINAL (Prostin E2 gel vaginal): Prostaglandina E2 1 2 mg
fondo saco posterior.
VAGINAL (Cervidil cnula vaginal): Prostaglandina E2 10 mg fondo
saco posterior.
ENDOCERVICAL: Prostaglandina E1 0,5 mg / 6 hrs canal cervical
Misoprostol: 25 mcg cada 4 hrs en fdo de saco posterior usar con
monitorizacin continua, no usar en cesrea anterior

13. Consecuencias madre y feto:

Materna:

- Trauma perineal, vaginal y cervical
- Cesreas
- Corioamnionitis
- Hemorragia postparto
- Endometritis
- Ansiedad


Fetal:
Mortalidad fetal aumenta despus de las 42 semanas y se duplica a las 43
semanas. Un tercio de las muertes se deben a asfixia intrauterina
Insuficiencia placentaria: RCF + OHA
Compresin del cordn umbilical
Sndrome de aspiracin meconial.
Morbilidad:
o Restriccin de crecimiento fetal
o Macrosomia fetal (Traumatismo obsttrico: distocia de hombres y trauma
neurolgico)
o Dificultad para regular la temperatura
o Hipoglicemia
o Policitemia
o Convulsiones
o Encefalopata
o Insuficiencia respiratoria
Bibliografa
- Simpson, P. Stanley, K. (2011). Prolonged pregnancy. Obstetrics, Gynaecology and
Reproductive medicine 21 (9) 257-262.

171
- Timofeev, T. (2011). Postterm Pregnancy. Postgraduate Obstetrics and Gynecology
31 (18) 1-8.
- McCarthy, F. Kenny, L. (2013). Induction of labour. Obstetrics, Gynaecology and
Reproductive medicine 24 (1) 9-1

34. PARTO DISTOCICO : RETENCION DE HOMBROS.



Paciente de 33 aos cursando embarazo de 40+5 semanas que se despus de una fase
activa del parto de 12 horas entra a sala de partos. Despus de la extraccin de la
cabeza fetal, al intentar extraer los hombros no se logra desprender en hombro anterior
con maniobras habituales que usamos para este fin.

Antecedentes familiares: no refiere
Antecedentes personales: tonsilectoma a los 18 aos
Hbitos: 1 cigarrillo al da.
Antecedentes gineco-obsttricos:
- menarquia a los 13 aos
- RM IV/28
- G4 P1 A1
o Ectpico operado
o RN peso de 3800 grs.

1. Diagnsticos:

Multpara de 4
Embarazo de 40+5 semanas
Distocia de hombro
Embarazo ectpico operado
Tabaquismo

2. Diagnsticos diferenciales:


3. Definicin:

Fallo en la salida del tronco fetal, en
que es necesario maniobras
obsttricas adicionales para la
extraccin de los hombros del feto
durante el parto vaginal, una vez que
la traccin moderada de la cabeza

172
hacia abajo ha fallado, situacin en la que el tiempo entre la salida de la cabeza y el
resto del cuerpo es mayor a 60 segundos y/o precisa de maniobras auxiliares para
resolverse.

4. Etiologa del cuadro clnico:

La distocia de hombros es causada por la incapacidad del feto de sortear los
reparos anatmicos de la pelvis materna en el proceso de salida. Este problema
presenta generalmente por un tamao aumentado del feto en relacin al espacio
anatmico materno, siendo la macrosomia fetal el principal factor de riesgo para
presentar esta complicacin durante le parto , ya que el dimetro bi acromial es
mayor al dimetro AP de la pelvis .

5. Fisiopatologa:

La distocia de hombro se presenta cuando el hombro anterior fetal o menos
comnmente el posterior se impactan contra la snfisis pbica o el promontorio del
sacro maternos. El dimetro biacromial normalmente entra en la pelvis en un
ngulo oblicuo con el hombro posterior por delante del anterior y se produce la
rotacin al eje antero-posterior en la salida de la pelvis ,con la rotacin externa de
la cabeza fetal. El hombro anterior entonces puede deslizarse bajo la snfisis del
pubis.

Si los hombros fetales permanecen en posicin antero-posterior durante el
descenso o descienden simultneamente, se puede producir la impactacin de
alguno de ellos con la snfisis pbica o el promontorio. Si la cabeza continua
descendiendo con algn hombro impactado se puede producir lesin del plexo
braquial por estiramiento. Tambin la compresin del cordn umbilical, o de los
vasos del cuello fetal pueden producir daos hipxicos al cerebro fetal e incluso la
muerte por asfixia.
La Distocia de Hombros puede asociarse a una alta morbilidad y mortalidad
perinatales incluso cuando se maneja apropiadamente

6. Factores de riesgo:

Macrosomia fetal (principal factor de riesgo)
Diabetes mellitus
Parto vaginal instrumentalizado
Antecedente de distocia de hombro
Embarazo prolongado
Ganancia de peso elevada durante el embarazo
Obesidad materna (IMC>30)
Pelvis estrecha.

173
7. Epidemiologa:

Se estima que ocurre en el 0,2-3% de todos los partos vaginales
Representa una emergencia obsttrica.

8. Cmo hacer el diagnstico clnico? :

Cuando no es posible extraer el feto con las maniobras habituales (traccin de
cabeza) y/o la duracin entre la salida de la cabeza fetal y el resto del cuerpo es
mayor de 60 segundos.

9. Que es importante preguntar en la anamnesis?:

Diabetes materna
Antecedente de parto anterior con distocia de hombro
Antecedente de macrosomia fetal
Conocer bien la edad gestacional ( embarazo prolongado )
Estado nutricional durante el embarazo

10. Qu buscar dirigida mente al examen fsico?:

La incapacidad de continuar el descenso fetal con las maniobras habituales una vez
salido de cabeza. Hay que realizar traccin continua y adecuada de la cabeza fetal,
si pese a ello no es posible extraer al feto, se hace el diagnostico de distocia de
hombro y la necesidad de aplicar maniobras especiales para finalizar el parto.

11. Cmo estudiar el cuadro? qu resultados espera encontrar en los exmenes?:

La distocia de hombro no se puede predecir o prevenir porque no existen mtodos
exactos para identificar en que parto va a ocurrir. Solo se puede tener en cuanta
los factores de riesgo antes mencionados y estar atentos en caso de que se
presente para actuar en forma oportuna y adecuada.

12. Tratamiento:

Se recomienda en primer lugar un intento suave de traccin hacia abajo de la
cabeza fetal, asistido por el esfuerzo expulsivo materno. Si esta medida inicial no
da resultado, debemos indicar a la madre que cese los pujos por el riesgo de
agravar el impacto del hombro anterior sobre la snfisis materna y colocarla con las
nalgas al borde de la cama.

Pedir ayuda inmediatamente tras el diagnstico de distocia de hombro. El personal
presente en la sala de partos debe incluir: matrona, obstetra, neonatlogo y
anestesista.

174

Se recomienda comenzar con la maniobra de McRoberts (ayudada por presin
supra pbica) por ser la maniobra eficaz ms sencilla.
Con dos ayudantes se realiza flexin de las piernas y abduccin de las caderas
maternas, colocando los muslos sobre el abdomen. Endereza la lordosis
lumbosacra lo que aplana el promontorio, y rota la snfisis del pubis cranealmente
disminuyendo el ngulo de inclinacin de la pelvis, permitiendo la liberacin del
hombro anterior. Hace disminuir las fuerzas necesarias para liberar el hombro fetal

al aumentar la eficiencia del pujo.





Se puede combinar con la presin suprapbica.El operador se debe colocar por el
lado del dorso fetal y la presin adecuada se aplica con la palma o el puo y tiene
una direccin caudal y lateral (empuja la cara posterior del hombro anterior hacia
el esternn del feto) y una duracin de 30 segundos; con ello se reduce el dimetro
bi-acromial y el hombro anterior rota desde el dimetro antro posterior hacia el
dimetro plvico oblicuo facilitando su desprendimiento. No hay ninguna
diferencia clara en cuanto a eficacia entre la presin continua o el movimiento con
balanceo.

Si no es posible la extraccin del hombro
anterior con las maniobras descritas, puede
intentarse la extraccin del hombro
posterior (maniobra de Woods). El obstetra
tracciona suavemente la cabeza fetal hacia
arriba, logrando que el hombro posterior
descienda en la concavidad sacra. Si se logra
el desprendimiento del hombro posterior,
luego ser posible el desprendimiento del
hombro anterior.

Maniobra de Rubins

175
Aplicar presin sobre la escapula fetal para rotar el hombro hacia el dimetro
oblicuo reduciendo el dimetro bi-acromial

Maniobra de Zavanelli

Consiste en reposicionar la cabeza fetal en el canal vaginal para intentar extraer el
feto mediante operacin cesrea


13. Consecuencias de esta patologa para la madre y feto:

La morbilidad fetal tiene dos orgenes:
Traumatismos mecnicos, responsables de
o Fractura de clavcula
o Fractura de hmero
o Dislocacin cervical
o Lesiones del plexo braquial (parlisis braquial)
o Endometritis puerperal.

Hipoxia fetal, con sus secuelas neurolgicas y de otros rganos, relacionada
con la condicin previa del feto, y el tiempo en que trata de resolverse la
distocia.

La morbilidad maternas por DH incluye: hemorragia postparto, desgarros
perineales, endometritis, Lesiones vesicales, rectales, hematomas y rara vez
ruptura uterina (6,7).

Bibliografa
- Gua perinatal CEDIP 2014
- Rodis JF, Lockwood CJ, Barss VA. Intrapartum management and outcome of
shoulder dysto- cia. In: UpToDate, Rose BD (Ed), UpToDate; Waltham,MA,2010.
- Spong CY, Beall M, Rodrigues D, Ross MG. An objective definition of shoulder
dystocia: Pro- longed head-to-body delivery intervals and/or the use of ancillary
obstetric maneuvers. Obstet Gyne- col 1995;86:433.
- Ilustraciones: Carvajal J, Ralph C. Manual de Obstetricia y Ginecologa. Cuarta
edicin 2013. Capitulo 11 Emergencias del parto vaginal en presentacin ceflica

176
35. PUERPERIO FISIOLGICO.
Paciente de 32 aos , cursando embarazo de 39 semanas de gestacin que acaba de
parir un RN termino por parto vaginal de 3600 grs , alumbramiento dirgido y retraccin
uterina adecuada

Antec Familiar : no tiene

Antec Personal : sana

Antec Gineco obsttrico : Multipara de 1 parto vaginal

1. Definicion : Es el perodo donde se producen transformaciones progresivas de orden


anatmico y funcional, que hacen regresar todas las modificaciones gravdicas,
para lograr la recuperacin anatmica y fisiolgica de la mujer, a su estado pre
grvido. Se extiende desde la finalizacin del perodo de post alumbramiento
hasta 45-60 das post parto


2. Clasificacion :Se clasifica segn periodo post parto:
- Inmediato (24 horas postparto)
- Temprano (primeros 7 das)
- Tardo (retorno a condicin pre-embarazo, hasta 6 semanas post parto)

3. Fisiologia El puerperio inmediato, est destinado a lograr la hemostasia


uterina. En el alumbramiento, se produce el desprendimiento, expulsin de la
placenta y membranas ovulares, ste se produce a nivel del lmite entre
decidua compacta y esponjosa; y queda en el tero un lecho esponjoso con
vasos sangrantes. Para una real hemostasia se produce retractilidad y
contractilidad uterina. Estas contracciones son intensas y frecuentes
para disminuir tamao del tero, situacin conocida comnmente como los
entuertos, para que al 7mo da post parto el tero este a nivel hipogstrico.
Aun as la hemostasia lograda no es total, y se pierden pequeas cantidades de
sangre rojo rutilante, acompaada de restos de decidua esponjosa, que se
elimina por genitales, llamados loquios", donde las primeras 24 horas son
hemticos.
Posterior a esto la cavidad uterina comienza a secretar un material que forma una
capa sobre l. Aparece un infiltrado leucocitario, peri vascular, y se produce
hialinizacin de las paredes de los vasos y trombos endoluminales, produciendo un
cambio en el color de los loquios. Todos estos mecanismos contribuyen a
disminuir las prdidas sanguneas.

177
En el miometrio las fibras musculares hipertrofiadas vuelven a su tamao normal,
las fibras neo formadas sufren una degeneracin hialina. Se manifiesta
clnicamente con una disminucin de la altura uterina a una velocidad de 1 2
cm/da.
El segmento uterino se adelgazado y se pliega, donde sufre un proceso de
involucin que lo lleva a desaparecer al final de este periodo. Las contracciones
disminuyen en intensidad y frecuencia. El cuello uterino reduce su dilatacin
rpidamente a 2-3 cm despus del parto. Al tercer da, recupera su forma, longitud
y consistencia, se restablece el canal cervical donde al quinto da no es permeable
a la palpacin. En la vagina, se reaparecen los pliegues vaginales y recupera su
tonicidad, puede sufrir laceraciones, principalmente en el tercio inferior, que se
recuperan a la tercera semana.
A nivel humoral, se presenta un aumento del volumen vascular renal que provoca
un aumento en la diuresis, lo que se denomina poliuria fisiolgica del puerperio.
Adems se aumenta el catabolismo proteico, por ende un aumento de la uremia.
La Creatinina se mantiene elevada por unos das, pero luego se normaliza. El
hematocrito y la hemoglobina disminuyen por dilucin. La leucocitosis se
normaliza.
La temperatura al tercer o cuarto da experimenta una elevacin por aumento del
metabolismo mamario, lo que comnmente se designa "bajada de la leche" este
aumento no debe durar ms de 24 hrs y se debe descartar algn foco infeccioso.
A nivel de otros rganos, ocurren diferentes cambios:
- Piel y mucosas: Desaparece la hiperpigmentacin cutnea, las estras pasan de un
color rojizo a un tinte nacarado definitivo, desaparecen los hematomas
suconjutivales, las petequias y las hemorragias capilares originadas por los pujos.
- La pared abdominal presenta un estado de flacidez, los msculos rectos del
abdomen experimentan distintos grados de distasis.
- Aparato urinario: En la vejiga y uretra, existe una cierta desinflamacin, que
facilitando la miccin, pero en ocasiones la hiperdistensin de la musculatura
pelviana, las lesiones vesicales y la relajacin vesical pos parto, pueden inhibir el
reflejo miccional favoreciendo la retencin urinaria, estos hechos desaparecen al
final de este perodo.
- Aparato digestivo: los rganos digestivos vuelven a su posicin normal y retoman
su normal fisiologa, durante los primeros das suele haber una hiperdistensin
intestinal con disminucin del peristaltismo, apareciendo cuadros de constipacin
que deben ceder espontneamente.
- Aparato respiratorio: la respiracin se normaliza, vuelve a ser abdominal y
desaparece la alcalosis respiratoria.

178
Durante el puerperio tempano, ocurre la re epitelizacin endometrial. Los loquios
de sero hemticos pasan a ser serosos y tienden a desaparecer hacia la tercer
semana pos parto. Hacia el da 12, el orificio cervical interno se debe encontrar
totalmente cerrado. Entre los das 10 y 14, el tero se hace intra plvico mediante
el proceso involutivo miometrial. El da 21 por un mecanismo no muy claro, se
produce una perdida hemtica denominada pequeo retorno. Entre los 30 y 45
das puede reaparecer la ovulacin y en caso de no haber fecundacin reaparece la
menstruacin, excepto si hay lactancia activa, ya que evita el retorno al ciclo sexual
normal.
El puerperio tardo, se extiende hasta la 6ta semana post parto. Donde vuelve las
condiciones pre gestacin.
4. Estudio, conducta y tratamiento, en postparto inmediato:
Sala de recuperacin 2-4 hrs.
Vigilancia de complicaciones precoces: infecciosas, urinarias,
psicolgicas, etc.
Apego desde que nace el nio, inicio de la lactancia
Hospitalizacin por 2-3 das en parto vaginal y 3-4 das en
cesrea.
Lactancia: fundamental es el rol de las matronas. Madres se
debe sentir cmoda y tranquila. Realizar adecuada educacin.
- Control de miccin: Retencin urinaria por altas dosis de anestesia
peridural u oliguria como complicacin de cesrea.
- Laboratorio. Se solicitar hemograma, VDRL y grupo y factor
sanguneo.
- Indicaciones teraputicas:
a) Utero rretractores:
Ergotnicos: se utilizara Ergonovina 0.6 mg./da, va oral o intramuscular, metil
ergometrina 0.6 mg./da intramuscular, metil ergonovina 0.4 mg./da va oral.
El uso de ergotnicos est contraindicado en pacientes hipertensas, se indicar
por 24 hs. o se podr utilizar ms das segn evolucin obsttrica.
Oxitcicos: est indicado en pacientes hipertensas o con factores de riesgo ,
para hemorragias, se usar asociado a derivados del Ergot
b) Antibioticoterapia: se indicar el uso de antibiticos en pacientes con
episiorrafia, cesrea abdominal y factores de riesgo para infeccin. Si durante la
cesrea abdominal o la episiorrafia se mantuvieron normas de asepsia y no
existen factores de riesgo para infecciones se realizar antibioticoprofilaxis con
cefalotina 1 gr. por va endovenosa o intramuscular durante el acto quirrgico y
otro gramo a las 6 hs. si no se dan las condiciones enunciadas anteriormente se

179
realizar tratamiento antibitico completo durante 7 10 das, las drogas de
eleccin son: ampicilina, amoxicilina, cefazolina, eritromicina, gentamicina, etc.
c) Analgesia: en caso de cesrea abdominal o episiorrafia se indicar analgesia
reglada las primeras 24 hs.
5. Indicaciones al Alta:
- Reposo relativo: contrarresta el alto riesgo trombtico del perodo puerperal
- Rgimen comn
- Abstinencia sexual por 1 mes: reduce riesgo de endometritis. Es preciso confirmar
el cierre del cuello uterino.
- Analgesia con AINES: Ketorolaco c/8 hrs
- Aseo genital frecuente: agua corriente.
- Apoyo de lactancia: principalmente en primparas
- Retirar puntos a la semana post alta, en caso de cesrea
- Control a las 3-4 semanas
- Vigilar fiebre, evolucin de loquios, lactancia y herida operatoria, evaluar
precozmente en sospecha de infeccin (mastitis y endometritis)
- Licencia post natal

6. Complicaciones

Se debe siempre buscar dirigida mente, complicaciones del puerperio, las cuales son:
1. Infecciosas
o Tacto vaginal: Endometritis, Para metritis, Pelvi peritonitis,
Infeccin de episiorrafia o herida operatoria
o Va urinaria: Cistitis, Pielonefritis aguda
o Mastitis: Linfangtica, Abscedada
o Otras infecciones: Tromboflebitis pelviana sptica, Shock
sptico, Fascetis necrotizante.
2. Hemorragia post parto
3. Depresin post parto
Bibliografa
- Denis Walsh . A review of evidence around postnatal care and breastfeeding.
OBSTETRICS, GYNAECOLOGY AND REPRODUCTIVE MEDICINE 2011.
- Obstetricia, Prez Snchez, 4ta edicin, Editorial Mediterraneo 2011.
- M. Lpez. Hemorragia posparto: prevencin y tratamiento, Pg. 269. CURSO
INTENSIVO EN MEDICINA MATERNOFETAL Instituto Clnic de Ginecologa,
Obstetricia y Neonatologa (ICGON). Hospital Clnic. Universidad de Barcelona.
(Director: Prof. Juan Balasch) Cardona (Barcelona), del 4 al 9 de marzo de 2012.

180
36. HEMORRAGIA POST PARTO : INERCIA UTERINA

Mujer de 37 aos , multpara de 2 partos vaginales que acaba de tener un parto vaginal
eutcico a las 40 semanas de gestacin con RN de 3600 grs , APGAR 7 10 ,
alumbramiento espontaneo quien comienza 5 minutos despus con hemorragia
importante por vagina .
- Antecedentes familiares : madre diabtica y padre diabtico
- Antecedentes personales : apendicetoma
- Antecedentes Gineco - Obsttricos : MENARQUIA 13 AOS , RM IV/30
- Hbitos : 10 cigarrillos diarios .
Diagnsticos : Multpara de 2
Purpera Parto vaginal eutcico de 40 semanas
Hemorragia Post Parto inmediata
Tabaquismo

Diagnostico Mas probable : Inercia Uterina

Diagnstico diferencial: 4T de sangrado post parto:
Tejido (retencin de productos de la concepcin)
Trauma lesin canal vaginal, tero o cuello)
Trombina (alteraciones de la coagulacin)


1. Definicin : Se define hemorragia post parto a la prdida sangunea despus del parto
mayor a 500 ml en parto vaginal o mayor de 1.000 ml en cesrea, o aquella que
produce un descenso del hematocrito 10% o cuando hay compromiso
hemodinmico de la paciente.
2. Clasificacin : Se clasifican en
- Inmediata o precoz, cuando sucede en las primeras 24horas posparto, como
por ejemplo inercia uterina, desgarros del canal delparto, restos placentarios o
rotura uterina;
- secundaria o tarda, cuando sucede entre las 24 horas y las 12 semanas posparto(
42 das ), como restos ovulares , endometritis , enfermedad trofoblastica
gestacional.
3. Etiologa : Son variadas, dependen de mltiples factores de riesgo expuestos en la
tabla 1, pero se pueden agrupar en cuatro grandes grupos: dficit del tono

181
uterino, trauma obsttrico del canal del parto , Retencin de tejido
placentario ( cotiledones o membranas ) y alteraciones de la coagulacin.
4. Fisiopatologa : Durante el embarazo el volumen sanguneo aumenta en un 48% en
relacincon los niveles pre gestacionales para cubrir las necesidades metablicas
del feto y de la madre, asegurar el retorno venoso adecuado al corazn con los
cambios de posicin materna, y protege a la mujer de las prdidas hemticas
durante el parto.
El parto vaginal espontneo supone una prdida promedio de 500 ml de
sangre, y una operacin cesrea aproximadamente de 930 ml. En ambas
circunstancias, la prdida hemtica se produce fundamentalmente durante el
expulsivo y en las primeras horas post-parto. Independientemente de la va de
parto, 7-8% de las mujeres excedern la prdida promedio.
Cuando la hipovolemia es intensa, aparece insuficiencia circulatoria y riego
tisular insuficiente. El menor volumen de sangre ocasiona disminucin
del llenado y del gasto cardaco, lo que disminuye la presin arterial. En ese
momento se ponen en marcha mecanismos compensadores que intentan
revertir los cambios mencionados. La mayor actividad simptico
suprarrenal produce taquicardia, incremento de la contractilidad
miocrdica, aumento dela resistencia vascularperifrica, y contraccin de las
arteriolas pre capilares y vnulas pos capilares. Los cambios vasculares
generan aumento de la presin arterial, absorcin intravascular del tejido
intersticial, por disminucin de la presin hidrosttica capilar y aumento del
retorno venoso al corazn por movilizacin de la sangre almacenada en los vasos
de capacitancia. Por las razones expuestas es que los signos clsicos de

182
hipovolemia incluyen taquicardia, signos de vasoconstriccin
perifrica, hipotensin y oliguria.
La inercia uterina es la causa ms frecuente cuente de hemorragia
pospartosiendo responsable de un 80% de ellas. Es la principal causa de
mortalidad materna a nivel mundial, producindose en un 1-5% de los
partos. La incidencia de hemorragia severa es de alrededor del 0,2-0,4% de
los partos (3,7/1.000).
Diagnstico : es clnico, caracterizado por un sangrado Inmediato que descompensa
hemodinamicamente a la madre .
Factores de riesgo:
- Distencin uterina, ya sea gestacin mltiple, macrostomia fetal, malformaciones
fetales, hidrocefalia, etc.
- Agotamiento de la musculatura uterina, como parto prolongado o multiparidad.
- Parto intempestivo.
- Infeccin uterina o Corioamnionitis, sospechada en RPM prolongada o fiebre.
- Anomalas uterinas, como miomas uterinos o placenta previa.
- Frmacos tero relajantes como Nidefipino, anestsicos, beta mimticos, So4Mg,
etc.
- Antecedente de Inercia en parto anterior , es principal factor de riesgo
Examen Fsico : Al examen fsico podemos observar falta de retraccin del tero que se
manifiesta con tero sobre nivel umbilical. Blando que tiende a retraer
con masaje uterino
Conducta : En presencia de esta hemorragia, debemos tomar una conducta rpida, la
cual se debe iniciar con solicitar ayuda, pues es una patologa grave, de rpida
evolucin que puede desencadenar un shock hipovolmico y la muerte de la
paciente. Se debe avisar a obstetra, matronas , anestesistas, banco de sangre,
UTI, cirujanos etc.
La valoracin basal y el manejo inicial debe incluir:

1) Identificar causa de la hemorragia: Trasladar la paciente a camilla de parto en
pabelln si ya esta en sala , Palpar altura uterina y ver retraccin , Revisin del
canal del parto, revisin intrauterina y revisin de la integridad de la placenta. El
objetivo del correcto diagnstico etiolgico ser realizar tambin un
tratamiento etiolgico desde principio.


183





Inspeccin de la placenta post parto , debe ser conducta de rutina


2) Valorar prdida real de sangre: Solicitar hemograma, coagulacin y lactato
3) Buscar signos de hipovolemia: Monitorizacin constante de signos vitales,
diuresis (Sonda vesical) y mantener temperatura corporal (evitar hipotermia).
4) Restaurar volumen sanguneo y capacidad de transporte de oxgeno:
5) 2 Accesos venoso de buen calibre, manejo de volumen (Iniciar con
cristaloides, tener reserva de sangre), oxigenoterapia.
Si la causa es inercia uterina (80%), adems del masaje uterino bimanual
permanente se inicia tratamiento mdico con frmacos uterotnicos, los ms
frecuentes son:
- La oxitocina es el uterotnico inicial de eleccin, estimula las clulas
musculares del segmento superior del tero para que se contraigan
rtmicamente, produciendo constriccin de los vasos sanguneos y disminucin
de la sangre que fluye a travs del tero. Es un seguro y eficaz. Se prefiere una
infusin IV para proporcionar un flujo constante de la droga.
- Alcaloides (Ergometrina, Metilergometrina y Sintometrina): Causan contraccin
tetnica del msculo liso del segmento superior e inferior del tero. raiga
tetnicamente. La dosis recomendada de ergometrina o metilergometrina es
0.2 mg IM, que se puede repetir cada 2-4 horas durante un mximo de 5 dosis
(1 mg) en un perodo de 24 horas. Contraindicados en mujeres con
hipertensin, enfermedad cardaca, o pre eclampsia debido que alza la presin
arterial.
- Misoprostol: Se ha demostrado que una dosis nica de 800 mg Misoprostol
administrado por va sublingual es un seguro y eficaz en mujeres que han
recibido oxitocina profilaxis y a las que no. reduce significativamente la
necesidad de intervenciones adicionales.
- Carbetocina 100 mcg Im o Ev a pasar en 1 minuto

184

Los frmacos uterotnicos deben ir acompaados de
masaje uterino bimanual para detener la prdida
excesiva de sangre. Se debe utilizar guantes estriles,
colocar una mano en la vagina y apretar la mano en un
puo, y la otra mano en el fondo del tero. Llevar las 2
manos para exprimir el tero entre ellos
Si el tratamiento mdico no es suficiente para tratar
una atona uterina, ser necesario aplicar otras
tcnicas. La histerectoma es la ltima opcin
teraputica, dada su elevada morbilidad. Se debe
valorar el riesgo vital de la paciente, el contexto clnico, el equipo de profesionales
y el deseo de fertilidad posterior de la paciente. La aplicacin de una u otra
tcnica deber individualizarse en funcin de otros factores como son la va
del parto, la posibilidad de traslado, el riesgo vital de la paciente y la experiencia
del equipo profesional responsable.

185








- El baln intrauterino (Sos Bakri) Actualmente ya no se usa. Puede ser usado
mientras se traslada la paciente a pabelln u otro hospital ,no como
tratamiento de la inercia (Imagen 1)

- Suturas de compresin uterina, la sutura de B-Lynch y la sutura de
Hayman. Ambas se consideran tcnicas conservadoras de fcil aplicacin cuyo
objetivo es conseguir la disminucin de la llegada de sangre desde los
vasosuterinos a la zona sangrante y el contacto y compresin de las paredes
anterior y
posterior y art
uterinas con
suturas
alrededor del
tero con
material
grueso
absorbible.

La sutura de
B-Lynch
requiere
histerotoma,
incluso en
caso de parto
vaginal.
Consiste en
una sutura
alrededor del
tero.La sutura de Hayman es una modificacin de la sutura de B-Lynch que NO
requiere histerotoma.

186
Es tcnicamente ms sencilla y rpida. Requiere realizar habitualmente 2
puntos, pero se pueden realizar ms.
Se recomienda realizar un punto de unin a nivel de fondo uterino entre los
dos puntos longitudinales, para evitar desplazamientos de los mismos. Su
eficacia global se sita entre el 81 % y el 91,7%. Complicaciones destaca la
isquemia y la infeccin uterina, sobretodo en casos en que se apliquen
suturas transversas que pueden dificultar el drenaje del contenido uterino.
Existe tambin un riesgo terico de estrangulacin intestinal en el espacio
que queda entre la sutura y el tero cuando este involuciona. Por este
motivo deben utilizarse suturas reabsorbibles.

o Ligadura de vasos plvicos, ya sea de arterias
uterinas o de arterias hipogstricas, ambas siempre
de forma bilateral, que puede incluir la parte terminal
de la rama ascendente (arteria uteroovrica), una
segunda sutura ms baja para incluir las ramas
cervicales, o bien ligar en masa arterias y venas
uterinas, incluyendo parte del miometrio (sutura de
OLeary). Tiene una eficacia de entre 40-100%, puede
conservar el tero y la fertilidad posterior y es ms
sencilla que la ligadura de arterias hipogstricas,
aunque se asocia un mayor riesgo de lesin ureteral.
Por otro lado, la ligadura de ambas arterias ilacas
internas resulta tcnicamente ms compleja, requiere
la apertura del peritoneo desde la bifurcacin de los
vasos ilacos, identificar y separar medialmente el
urter y posteriormente individualizar la arteria ilaca
interna procediendo a la doble ligadura de su rama
anterior (a unos 2-3 cm de la bifurcacin) sin
seccionar el vaso. Entre sus complicaciones, destaca
la necrosis uterina o de otros territorios vecinos, la
perforacin vascular (vena ilaca principalmente) y la
lesin ureteral.




o La histerectoma subtotal o total es la opcin
teraputica ms radical y compromete de forma
definitiva la fertilidad posterior, por lo que no debe

187
considerarse una tcnica de primera eleccin sino de
rescate en caso de fracaso de las tcnicas
conservadoras.
o Se ha descrito el uso de factor VIIa recombinante.
Sus indicaciones se limitan a casos de hemofilia, CID ,
y en casos de hemorragias severas. Eficacia en la
reduccin o cese del sangrado del 85%, pero no es
despreciable riesgo de complicaciones,
principalmente trombo embolismos (descritos hasta
en un 5%).
Lo fundamental en la hemorragia post parto, es la prevencin, valorar factores
de riesgo antenatales predice nicamente el 40% de casos de sangrados. Por
tanto, adems de evitar factores predisponentes, la medida preventiva de
mayor eficacia consiste en el manejo activo del alumbramiento, con una
reduccin eficaz de un 60%, en la disminucin significativa de la anemia
posparto y de la necesidad de transfusin. Consiste:
- Uso profilctico de agentes uterotnicos una vez que ha salido el hombro
anterior (Cuadro 2)
- Pinzamiento del cordn y traccin controlada hasta extraer la placenta 2
minutos despus del
parto . Solo debe ser
realizado por
obstetras .

- Masaje uterino.






188
Bibliografa
- FIGO Safe Motherhood and Newborn Health (SMNH) Committee, Prevention
and treatment of postpartum hemorrhage in low-resource settings.
International Journal of Gynecology and Obstetrics (2012). Disponible:
www.elsevier.com/locate/ijgo
- Lin Lin Su, Massive obstetric haemorrhage with disseminated intravascular
coagulopathy. Best Practice & Research Clinical Obstetrics and Gynaecology
(2012). Disponible: www.elsevier.com/locate/bpobgyn
- M. Lpez. Hemorragia posparto: prevencin y tratamiento, Pg. 269. CURSO
INTENSIVO EN MEDICINA MATERNOFETAL Instituto Clnic de Ginecologa,
Obstetricia y Neonatologa (ICGON). Hospital Clnic. Universidad de Barcelona.
(Director: Prof. Juan Balasch) Cardona (Barcelona), del 4 al 9 de marzo de 2012.
- Obstetricia y ginecologa, Beckmann, 6ta edicin, editorial Lippincott William y
Wilkins 2010.

37. PUERPERIO PATOLGICO :INFECCIN PUERPERAL



Paciente de 28 aos , purpera de 4 das que ha presentado fiebre hasta 39C en 2
ocasiones en ultimas 24 hrs , cefalea , mal estar general. Refiere flujo mal olor por
vagina. Parto inducido por RPO de 24 hrs que termino en cesrea 12 hrs despus por
dilatacin estacionaria. Lactancia mixta.
Antecedentes Familiares: no refiere
Antecedentes Personales: no relata.
Antecedentes Gineco obsttricos: menarqua 13 aos, RM IV/28 G3P2A1
Hbitos : cigarro 6 al da

1) CUAL ES EL DIAGNOSTICO MAS PROBABLE DE ESTA PACIENTE ?
- Multpara de 2
- Purpera de 4 das
- Fiebre puerperal / infeccin puerperal / endometritis
- Tabaquismo activo

2) CUALES SON LOS DIAGNOSTICOS DIFERENCIALES ?
Genitales
o Endometritis
o Endomioometritis
o Parametritis
o Pelviperitonitis
o Tromboflebitis septica pelviana

189
o Infeccin episiotoma/herida
Urinarias:
o ITU baja
o PNA
Mastitis:
o Linfangitica
o Abscedada
Otras:
o Pared abdominal
o Tromboflebitis pelviana sptica
o Shock sptico
o Estado gripal

3) COMO SE DEFINE ESTA PATOLOGIA ?
Fiebre puerperal: se define como temperatura posterior a las 24 hrs post parto >
37.8 en dos tomas separadas por 12 hrs.
Infeccin puerperal es fiebre con un origen infeccioso identificado.
Endometritis puerperal: contaminacin de la cavidad uterina, secundaria a la
invasin de microorganismos de la flora vaginal o enteral, con invasin de
endometrio y miometrio.

4) CUAL ES LA ETIOLOGIA Y FISIOPATOLOGIA DE ESTE CUADRO CLINICO ?
Infeccin polimicrobiana, por microorganismos de la flora habitual de la vagina,
que por va ascendente alcanzan el tero. Los agentes causales ms frecuentes
son: aerobios (SGA-B, S. faecalis, E. Coli) anaerobios (bacteroides fragilis,
peptococcus, peptostreptococcus).
La endometritis suele aparecer en el 2do a 3er dia postparto. Siendo la cesarea el
factor mas importante, donde el riesgo de endometritis llega a ser hasta 30 veces
mayor que en el parto vaginal.

6) QUE FACTORES DE RIESGO BUSCARA ?
Rotura de membranas prolongada, corioamnionitis, PP, bajo nivel socioeconmico,
anemia, trabajo de parto prolongado, politacto, extraccin manual de placenta,
cesrea, monitorizacin fetal interna, parto instrumental, preeclampsia,
desnutricin, obesidad.

Factores de riesgo asociados al hospedero Factores de riesgo asociados a la atencin en salud
Bajo nivel socioeconmico Trabajo de parto prolongado
Anemia Nmero de tactos vaginales desde el inicio del trabajo de
Rotura prolongada de membranas antes del parto parto
Vaginosis bacteriana Extraccin manual de placenta
Corioamnionitis Cesrea con trabajo de parto
Edad gestacional pretrmino Cesrea
Menor edad materna Monitoreo fetal interno

190
Instrumentacin uterina
Atencin de cesrea sin antimicrobianos

7) CUAL ES LA EPIDEMIOLOGIA DE ESTE CUADRO ?
1-3 % de partos vaginales, 5-10% de cesreas electivas y 15-20 % de cesreas de
urgencia, como nmeros globales. En chile se observo entre 2000-2004 que la tasa
de endometritis en partos por cesrea programadas fue 1,24 veces ms que en
partos vaginales, mientras que en cesreas con trabajo de parto fue de 2,02 veces
ms que en los partos vaginales. Aunque la tendencia en la incidencia de la
endometritis puerperal a ido en disminucin con los aos.

8) COMO HACEMOS EL DIAGNOSTICO CLINICO ?
El diagnostico es clnico, presentando fiebre (>37.8C) + uno de los siguientes
signos: Loquios turbios, hemopurulentos o purulentos con o sin mal olor; dolor
hipogstrico; tero subinvolucionado; sensibilidad uterina. Adems se puede
presentar con mal estar general, hipotensin, leo y shock.

10) QU BUSCARA DIRIGIDAMENTE EN EL EXAMEN FISICO DE ESTA PACIENTE ?
CSV
Examen general: fiebre (>37.8C), CEG, sudoracin, taquicardia.
Examen ginecolgico: Masas parauterinas con dolor e induracin de los fondos de
saco de Douglas y dolor en parametritos.

10) COMO ESTUDIAMOS ESTE CUADRO Y QUE RESULTADOS ESPERARIA ENCONTRAR
EN ESTOS EXAMENES ?
El diagnostico es clnico. Se pueden realizar exmenes complementarios ante la
duda diagnostica, cultivo y antibiograma de secrecin genital son de utilidad pero
no se usan de rutina, hemograma (leucocitosis, VHS elevada), PCR, hemocultivo.
Adems es importante realizar exmenes de orina para descartar ITU, ya que se
presenta de manera similar.
Imagenologia : ECO Tv, TAC , RNM

11) CUAL ES EL TRATAMIENTO DE ESTA PACIENTE ?
La prevencin de este cuadro es importante, reducen el riesgo de endometritis: <5
TV, TV con guantes estriles + lavado de manos, tcnica asptica de atencin de
parto, profilaxis atb para cesreas (<10-30% endometritis, 25% herida operatoria.
La profilaxis se realiza con Cefazolina 1 gr EV antes de la incisin).

Tratamiento:

Endometritis leve (<38 sin CEG): ambulatorio con amoxicilina + acido clavulanico.
Endometritis severa:
o Hospitalizacin
o Rgimen 0 (24 hrs)

191
o ATB Ceftriaxona 1-2 gr c/12-24 hrs + MTZ 500 mg c/8 hrs EV
o Gentamicina 1,5 mg/kg cada 8 hrs EV + Clindamicina 900 mg c/8 hrs EV (*)
o Penicilina sdica 5 M UI c/6 hrs + gentamicina 3-5 mg/kg/dia en una dosis
IM
o Gentamicina 1,5 mg/kg cada 8 hrs EV + Clindamicina 900 mg c/8 hrs EV
o Completar 14 das de ATB.
*Esquema ms eficaz en ltimas revisiones de Cochrane del tema.

12) QUE CONSECUENCIAS PUEDE TRAER ESTA PATOLOGIA A LA MUJER / MADRE/ Y AL
FETO ?
Las complicaciones ms importantes son:
Pelviperitonitis: por invasin de cavidad pelviana, serosa peritoneal y rganos
adyacentes.
PIP / ATO
Sepsis


Bibliografa
- Norma para la prevencin de la endometritis puerperal; Ministerio de Salud Chile; Departamento de calidad y seguridad del
paciente 2008.

- Manual Obstetricia y Ginecologa; J.A. Carvajal, C. Ralph, 4ta edicin (2013), Capitulo 17 "Puerperio normal y patolgico";
pag. 170-173.

- Manual Ginecologa y obstetricia CTO; M. Muoz, D.P. Hernndez, 8va edicin (2012), Capitulo 30 "Posparto y puerperio";
pag. 114-116
- Manual Obstetricia; U. chile, Departamento de obstetricia y ginecologia, Dr. Hugo Salinas; 2005. Capitulo 9: "Puerperio
normal y patologico", pag: 121-122.

38. PUERPERIO PATOLGICO :MASTITIS.



Mujer de 29 aos purpera de 15 dias PTVE 3400 Grs que ha presentado fiebre hasta
39C en 2 ocasiones en ultimas 24 hrs, cefalea, mialgias, calofros. Adems dolor en
mama izqda desde hace 2 das. Lactancia exclusiva.
Antecedentes Familiares: madre sana padre hipertenso
Antecedentes Personales: alergia tetraciclina.
Antecedentes Gineco obsttricos: menarquia 13 aos, RM IV/28 G2P2A0
Hbitos: nada

1) CUAL ES EL DIAGNOSTICO MAS PROBABLE DE ESTA PACIENTE ?
- Multpara de 2
- Purpera de 15 das
- Fiebre puerperal / infeccin puerperal / Mastitis

192

2) CUALES SON LOS DIAGNOSTICOS DIFERENCIALES ?
Genitales
o Endometritis
o Parametritis
o Pelviperitonitis
o Infeccin episiotoma/herida
Urinarias:
o ITU baja
o PNA
Mastitis:
o Linfangitica
o Abscedada
Otras:
o Pared abdominal
o Tromboflebitis pelviana sptica
o Shock sptico
o Estado gripal

3) COMO SE DEFINE ESTA PATOLOGIA ?
Inflamacin e Infeccin del parnquima mamario, mayoritariamente asociado a
lactancia materna.
Mastitis puerperal: mastitis de origen infeccioso, originada por la proliferacin
bacteriana al interior del tbulo galactforo ocluido asociada a lactancia materna.

4) CUAL ES LA ETIOLOGIA Y FISIOPATOLOGIA DE ESTE CUADRO CLINICO ?
La fuente principal de microorganismos causales son los de la boca y faringe del
recin nacido (estreptococos y anaerobios) y de la piel (Staphylococcus aureus),
por lo que es necesario un tratamiento antibitico de amplio espectro.
El agente causal ms frecuente en la mastitis de cualquier causa, es el S. Aureus.
Otros agentes comunes son S. epidermidis, S. saprophyticus, Strepto viridans y E.
coli.
La mastitis se puede clasificar en linfangitica (en la cual no existe absceso) y
Abscedada. Las grietas en el pezn producidas por las lactancia (en general mala
tcnica) y la estasis de la leche en las mamas son los factores predisponentes. Las
bacterias penetran a la mama por va canalicular y se multiplican en los conductos
galactoforos en forma progresiva, as se forma pus el que tiene una densidad
mayor que la leche y produce obstruccin y estasis provocando la lesin
abscedada.
La mastitis se puede clasificar en linfangitica (en la cual no existe absceso) y
Abscedada.

193
6) QUE FACTORES DE RIESGO BUSCARA ?
El principal factor de riesgo para la mastitis es la mala tcnica de lactancia y un mal
acople bucal pezon .
Adems existen otros factores de riesgo como: historia de mastitis previa, grietas
en pezon, uso de cremas anti fngicas u otras, extraccin de leche por medios
manuales, fatiga, estrs o cansancio materno, y alimentacin materna deficiente.
Tambin existe una asociacin con la discusin del riesgo de mastitis cuanto menor
es el numero de mamadas en el da.

7) CUAL ES LA EPIDEMIOLOGIA DE ESTE CUADRO ?
Tiene una incidencia del 2-33% de las mujeres que estn lactando. La mayora de
las mastitis son linfangitica 90% y abscedada el 10%. Mientras que el 3,6% de las
linfangiticas se complican a abscesos.

8) COMO HACEMOS EL DIAGNOSTICO CLINICO ?
Por lo general se presenta en semanas o meses posteriores al alta (principalmente
las primeras semanas), asociado a fiebre muy alta (39-40C), CEG, mialgias y signos
inflamatorios de la mama afectada, zonas eritematosas. En la mastitis abscedada
dolor e induracin de la zona mamaria, aumento del tamao y signos inflamatorios
notorios en la piel.
Como estudios complementarios se puede utilizar ecografa mamaria,
especialmente en pacientes con mamas de gran tamao, donde ayuda al
diagnostico de abscesos retro mamarios. La leucocitosis es un hallazgo frecuente,
encontrndose una elevacin discreta.

9) QUE NOS INTERESA PREGUNTAR EN LA ANAMNESIS DE ESTA PACIENTE ?
Principalmente hay que evaluar y preguntar por la tcnica de lactancia. Adems de
preguntar por los sntomas clsicos, descritos en el punto anterior. La historia
previa de mastitis e infecciones puerperales, e indagar en los factores de riesgo
descritos.

10)QU BUSCARA DIRIGIDAMENTE EN EL EXAMEN FISICO DE ESTA PACIENTE ?
Examen general: fiebre elevada, CEG, mialgia, taquicardia, sudoracin.
Descartar otro foco de infeccion .
EFM: Dolor a la palpacin local, con signos claros de inflamacin en cuadrante
afectado (calor, eritema, dolor), en el caso de ser abscedada se palapa una masa
local, aumento del tamao de la mama afectada y dolor intenso en la zona.

10)COMO ESTUDIAMOS ESTE CUADRO Y QUE RESULTADOS ESPERARIA ENCONTRAR EN
ESTOS EXAMENES ?
El diagnostico es clnico, el cultivo de secrecin por el pezn no es de utilidad. Se
pueden realizar exmenes generales para ver el estado general de la paciente,
como hemograma, donde se observara una leucocitosis discreta, adems se puede

194
estudiar parmetros spticos sistmicos (hemocultivo). Otras pruebas
complementarias son la imagenologia, en este caso la ecografa puede ser de
ayuda para el diagnostico como se explico en el punto anterior.


11)CUAL ES EL TRATAMIENTO DE ESTA PACIENTE ?
La prevencin es un punto clave en la mastitis. En un estudio (Peters & Flicks,
1991) se logro reducir la incidencia de mastitis puerperal de 2,9% a 0,65% con solo
el aseo de manos con desinfectante antes y despus de atender al recin nacido.

En los casos en que la mastitis ya se presenta el tratamiento se basa en:

Evaluar y corregir tcnica de lactancia y presencia de grietas.
Se debe asegurar un buen y efectivo vaciamiento.
Sintomtico: antipirticos, analgsicos, antiinflamatorios (paracetamol / AINES)
En una primera etapa compresas frias y una vez abscedada calor local.
ATB: Cloxacilina 500 mg c/6 hrs VO x 7-10 das Flucloxacilina 500 mg c/8 hrs VO x
10 das; otra alternativa es: Lincomicina 30-50 mg/kg VO cada 6-8 hrs x 7 das
Se espera que a las 48 hrs baje la fiebre.
NO suspender lactancia
En mastitis abscedada: Hospitalizar, ATB EV (antibioticos utilizados en no
abscedadas), drenaje de absceso (bajo anestesia general, incisin periareolar). La
lactancia NO est contraindicada, se debe alimentar por la mama contralateral y
de la leche extrada de la mama abscedada. Otras alternativas ATB EV:
Clindamicina, ciprofloxacino, ampicilina/sulbactam, amoxi/clavulanico.


Bibliografa
- Obstetricia; Perez Sanchez, 4ta edicion (2011), Capitulo "Puerperio" (S. Silva, A. Perez); pag. 454-457

- Protocolos de medicina fetal y perinatal Hospital Clinic-Hospital Sant Joan De Du - Universidad de Barcelona - Protocolo:
"Fiebre Puerperal"; T. Cobo, S. Ferrero, M. Lpez, M. Palacio, J. Bosch, J. Mensa - Ultima actualizacin y revisin:
18/02/2014.

39. ANTICONCEPCIN POST PARTO



Purpera de 30 das de un parto vaginal que acude a control de rutina y deseo de
anticoncepcin.

Antecedentes Familiares : madre diabtica padre hipertensa
Antecedentes Personales : alergia penicilina.
Antecedentes Gineco obsttricos: menarquia 13 aos , RM IV/28 G1P1A0 Ha
utilizado como anticonceptivos inyectables y ACO oral
Hbitos : cigarro 10 al da

195

1. Diagnostico

Multpara de 1
Purpera 30 das



El intervalo entre los embarazos es importante para la salud de las madres, ya que
les permite recuperarse del embarazo y parto y del esfuerzo que requiere la
atencin de nios y nias.
Las recomendaciones para el uso de mtodos anticonceptivos que se describen
son distintos para las mujeres que amamantan y las mujeres que no amamantan.

Anticoncepcin para la mujer que amamanta:

Existen consideraciones especiales para el uso de mtodos anticonceptivos, ya que
no deben interferir con la lactancia ni el crecimiento de los nios

Mtodos no hormonales: Se consideran la mejor opcin ya que no interfieren con
la lactancia ni el crecimiento de los lactantes y tampoco implican transferencia de
esteroides a travs de la leche

o Amenorrea de lactancia: ofrece una proteccin anticonceptiva muy alta en
mujeres que estn con lactancia exclusiva, con tasas de embarazos < 4 % en
los primeros 6 meses postparto. Es importante tener en claro que: (1) la
eficacia anticonceptiva disminuye rpidamente despus del primer
sangrado postparto, con la alimentacin suplementaria y cuando se
cumplen 6 meses postparto, por lo que es necesario utilizar otro mtodo
anticonceptivo (2) la lactancia se debe apoyar con una elevada frecuencia
de succin para que sea exitosa (3)Adems de la proteccin anticonceptiva,
la amenorrea de lactancia proporciona un periodo de tiempo de varios
meses, durante los cuales la mujer puede comenzar otro mtodo
anticonceptivo sin temor a que quede embarazada
o DIU: son uno de los mtodos reversibles de mayor eficacia anticonceptiva
con tazas <1% al final del ao, lo que representa una gran ventaja en este
periodo.
o Esterilizacin quirrgica
o Mtodos de barrera: su eficacia puede ser mas alta durante la lactancia, ya
que hay una reduccin parcial de la fertilidad en los primeros ciclos
postparto
o Abstinencia peridica: El reconocimiento de los signos y sntomas de la
fertilidad, como en el mtodo de la ovulacin, se puede aprender durante

196
la amenorrea de lactancia para aplicarlo cuando se reinicien los ciclos
menstruales

Mtodos hormonales

o Anillos vaginales de progesterona: La progesterona es la hormona natural
producida por el ovario en la fase ltea del ciclo menstrual y durante el
embarazo y no es activa por la va oral. La progesterona se administra a
travs de anillos vaginales, iniciando su uso a las seis semanas postparto. Se
ha demostrado que son muy efectivas como anticonceptivo, sin alterar la
lactancia ni el crecimiento de los lactantes. La progesterona prolonga el
periodo de anovulacin y amenorrea postparto, por lo que actuara
reforzando los mecanismos naturales de la infertilidad asociada a la
lactancia.

o Progestgeno solo: pueden ser administrados despus de las primeras 6
semanas postparto y pueden usarse pastillas, inyecciones e implantes.
-Este mtodo no tiene efectos negativos sobre la lactancia, ni sobre el
crecimiento y el desarrollo temprano de los lactantes
-Taza elevada de eficacia anticonceptiva <1%
-Cuando se inician durante la lactancia, prolongan el periodo de amenorrea
y las mujeres presentan ciclos mas largos y escasos das de sangrado
cuando reanudan los ciclos menstruales, lo que tiene un efecto positivo
sobre la hemoglobina
-Esta hormona pasa a la leche y se desconoce su efecto sobre el RN, es por
esto que se da despus de los seis meses para evitar la ingestin en un
lactante en la etapa en que los sistemas metablicos y la barrera
hematoencefalica son ms inmaduros.

Las progestinas, son progestgenos sintticos

Generaciones de progestinas :

1.Noretindrona, noretinodrel, acetato de noretindrona, diacetato de
etinodiol
2.Levonorgestrel, noretisterona, norgestrel
3.Desogestrel, gestodeno, norgestimato, drospirenona
4.Dienogest, drospirenona, nestorona, acetato de nomegestrol y
trimegestona13

197


1. Mecanismo de accin

- Cambios en el moco cervical: Disminuyen la cantidad de moco cervical,
aumentando su viscosidad, hacindolo grueso, denso, opaco e incrementndola
densidad celular, lo cual es un obstculo para la migracin espermtica,
impidindose el ascenso de los espermatozoides al canal cervical y a la cavidad
uterina.
- Efectos sobre el endometrio: Causan decidualizacin de las capas funcionales del
endometrio, impidiendo su crecimiento y maduracin, provocando su principal
efecto colateral, sangrados uterino. Pero a la vez reducen el sustrato de
produccin de prostaglandinas, disminuyendo la dismenorrea.
- Cambios en la funcin ovrica: Logran inhibir la ovulacin, en el 50-60% de los
ciclos, y defectos en la funcin del cuerpo lteo en los ciclos ovulatorios

2. Indicacin (Criterios de elegibilidad de la OMS)

- Categora 4: Riesgo inaceptable. Cncer de mama
- Categora 3: Riesgo mayores que los beneficios
. Para todos los mtodos: Lactancia en las 1eras 6 semanas post parto, TVP, TEP,
sangrados vaginales inexplicables, ant. de cncer de mama, hepatitis viral,
cirrosis descompensada, tumores hepticos, migraa con aura.
. Solo para inyectables (Entrega mayor dosis de esteroide) Factores de riesgo
cardiovascular, cardiopata isqumica, AC, cardiopata valvular complicada,
diabetes con nefropata o retinopata.
. Solo para pldoras: Uso de anti convulsionante y ATB (Rifampicina y griseofulvina)
- Categora 1 y 2: Mayor beneficio.

3. Tipo de mtodos

198
Pldoras de progestinas puras (PPP)
- Linestrenol 0,5 ug
- Levonorgestrel 30 ug
- Noretindrona 300 ug
- Desogestrel 75 ug

Progestinas Inyectables
- Acetato de depomedroxiprogesterona (DMPA o depoprovera) 150 mg
c/3meses im
- Enantato de noretisterona (NET-EN o Norigest) 200 mg c/2 meses im

Implantes

- Norplant y Jadelle: Levonorgestrel c/5 aos
- Implanon: Etonogestrel c/3 aos.

Anillo de progestina (AVP)
- Progering: Progesterona c/4 meses.



Los anticonceptivos con estrgenos estn contraindicados en el puerperio
por aumentar notoriamente el riesgo trombotico de la madre, adems de
disminuir la lactancia .










199
40. VAGINOSIS BACTERIANA
Paciente 23 aos, consulta por secrecin vaginal griscea, cremosa y olor a pescado que
aumenta antes de cada regla y despus de la relacin sexual, escaso prurito e irritacin vaginal.
Antecedentes familiares: no
Antecedentes personales: HTA
Antecedentes ginecobsttricos: menarquia 13 aos, RM IV/28, G1P1A0, mltiples
parejas sexuales en 2 aos.
Hbitos: 10 cigarros al da.

1. Diagnstico apropiado:

Multpara de 1,
Vaginosis bacteriana,
Tabaquismo
2. Diagnstico diferencial:
ETS,
Vaginitis descamativa
inflamatoria,
Tricomoniasis,
Candidiasis vulvovaginal.

3. Definicin: alteracin de flujos blancos ms comn producida por un desequilibrio en la
flora comensal vaginal.

4.Etiologa: Es producida por un desequilibrio de la flora comensal vaginal inducida por una
disminucin de la produccin de perxido de hidrgeno de los
lactobacilos que altera el pH vaginal y produce un aumento de microorganismos
de flora comensal. Adems, los microorganismos anaerobios producen enzimas
proteolticas que generan detritus y exfoliacin de clulas del epitelio y exudado,
lo que resulta en el flujo vaginal patolgico. El agente etiolgico principal
corresponde a la Gardnerella vaginalis.

Son factores de riesgo para esta patologa: actividad sexual con mltiples
parejassexuales,duchas vaginales, uso de jabones y qumicos, tabaco, uso de
preservativos. Los anticonceptivos orales se han mostrado como protectores.

5.Epidemiologa: 29% poblacin general entre 14-49 aos.

6.Diagnstico clnico: El diagnstico es clnico y se establece con 3 Criterios de Amsel positivos:

200
Imagen de flujo
patolgico por vaginosis bacteriana.
i. PH aumentado, mayor a 4,5
ii. Clue cells ms del 20% en microscopio ptico (clulas epiteliales
escamosas rodeadas de abundantes cocobacilos adheridos)
iii. Descarga vaginal homognea, griscea

El diagrama muestra
clue cells

Fotografa de microscopio ptico que


muestra clue cells de epitelio vaginal.

201

iv. Olor a pescado con KOH (test de aminas +).

b. Anamnesis y examen fsico: 75% son asintomticos, el resto presenta descarga
vaginal griscea de mal olor (pescado). No produce disuria, ni dispareunia,
ni prurito, ni inflamacin vaginal, si estos ltimos estn presentes se
sospecha que es un compromiso mixto, ya que implica la presencia de otro
microorganismo.
c. Estudio de la paciente: Cabe destacar que el diagnstico es clnico, por lo tanto,
no requiere de estudio complementario. Sin embargo, existen otros
mtodos para apoyar el diagnstico, como son: el test de gardnerella
vaginalis, testcard, OSOM, test de Nuget (tincin de Gram del flujo
vaginal). Este ltimo (Test de Nuget) es el ms recomendado, con valores
entre 0-3 es normal, con valores entre 4-6 es dudoso, con valores entre 7-
12 es diagnstico de vaginosis bacteriana. El PAP no sirve, los cultivos no
tienen rendimiento ya que es una infeccin poli microbiana, la
espculoscopia debe realizarse siempre para evaluar el flujo y
compromiso de mucosa vaginal y descartar la presencia de otras
alteraciones. Existe PCR para gardnerella, mycoplasma y ureaplasma pero
son de alto costo.
d. Tratamiento y conducta:
1. Metronidazol 500 mg cada 12 horas va oral x 7 das.
2. Metronidazol, vulos 500mg cada noche por 5-7 das.
3. El tratamiento con 2 gr de metronidazol va oral x 1 vez no ha
mostrado ser tan eficaz.
4. Clindamicina 300 mg cada 12 horas va oral x 7 das.
5. Clindamicina crema 2% x 7 das.
6. Clindamicina vulos de 100 mg al da x 3 das.
7. Tinidazol 1 gr va oral cada 24 horas x 5 das, o 2 gr cada 24 horas x
2 das.
8. Lactobacilos vaginales por 7 das o lactobacilos orales x 30 das.
9. En mujeres embarazadas:
10. Metronidazol 500 mg cada 12 horas va oral x 7 das.
11. Metronidazol 250 mg cada 8 horas va oral x 7 das.
12. Clindamicina 300 mg va oral cada 12 horas x 7 das.
13. En embarazadas con riesgo de parto prematuro, asintomticas se
realiza tratamiento emprico con Clindamicina.
Se define Recurrencia como: 3 episodios o ms en 1 ao. El tratamiento
recomendado es:
14. Metronidazol crema (gel) al 0,75% x 7-10 das, luego 2 veces por
semana x 4-6 meses.
15. Metronidazol o Tinidazol oral por 7 das + vulos de cido brico
600 mg cada noche x 21 das.

ii. Consecuencias para la paciente y el feto: si hay embarazo existe riesgo de
parto prematuro. Puede haber colonizacin del endometrio,
endometritis, fiebre postparto, celulitis de cpula vaginal post
histerectoma, infeccin postaborto, factores de riesgo de adquisicin y

202
transmisin de VIH, VHS, gonorrea, Chlamydia, VPH y NIE asociado a
mltiples parejas sexuales.


Bibliografa:
1. Jack D. Sobel. Bacterial vaginosis. UptoDate. Nov 2013.
2. Amy Boardman. A practical approach to vulvar pruritus and the vulvar dermatoses:
Part I. Pstgraduate Obstetrics & gynecology. Vol 33. No 6. March 31, 2013.
3. Natalie Grant, David Nunns. Vulval pruritus and vaginal discharge. Obstetrics,
Gynaecology and Reproductive Medicine 22:11. Elsevier 2012.
4. Chapter 14. Bacterial vaginosis. Diagnosis and management of STDs (including HIV
infection). Sixth Edition 2005. Page 52-53.

41. PATOLOGIA VULVAR :PRURITO POR CANDIDA.



Mujer de 29 aos consulta por prurito vaginal hace 2 das. Refiere prurito interno vulvo
vaginal , flujo blanco espeso como quesillo , ardor vulvar , irritacin vaginal y disuria.
Esta tomando antibiticos que le indico el dentista.

Antecedentes Familiares : asmtica
Antecedentes Personales : anorexia a los 18 aos.
Antecedentes Gineco obsttricos : menarquia 13 aos , RM IV/28 G1P1A0 usa
ACO.
Hbitos : cigarro 5 al da

1. Diagnostico Apropiado:

Multpara de 1
Vulvovaginitis Obs- micosis
Infeccin urinaria ?

2. Diagnsticos diferenciales:

Caracterstica Normal Vaginosis Candidiasis Tricomoniasis
bacteriana
Sntomas -Leucorrea -Prurito -Flujo espumosos
frecuentes -Olor que -Escozor -Mal olor
empeora tras el -Irritacin -Disuria
coito; puede ser -Flujo blanco y -Dispareunia
asintomtica espeso -Prurito y escozor
vulvar
Cantidad de flujo Pequea Con frecuencia A veces mayor Mayor

203
mayor
Aspecto del flujo -Blanco -Poco espeso, -Blanco -Gris verdoso
-Transparente homogneo -Parecido a la -Espumoso
-Floculado -Gris verdoso leche cuajada -Adherente
-Blanco -Parecido al
-Adherente quesillo
pH vaginal 3.8 4.2 >4.5 normal >4.5
Prueba del olor Ausente (+) olor a pescado ausente Posiblemente
con KOH presente
Aspecto -Clulas -Aumento de Hifas y brotes -Clulas epiteliales
microscpico epiteliales leucocitos normales
escamosas -Reduccin de -Aumento de
normales lactobacilos leucocitos
-Numerosos -Muchas clulas -tricomoniasis
lactobacilos clave
Tratamiento No disponible -Metronidazol Imidazoles Metronidazol o
(oral o tpico) tpicos sintticos Tinidazol oral
-Clindamicina(oral o fluconazol oral
tpica)

Vulvitis por contacto
Cuerpo extrao
Vulvitis inespecfica


3. Definicin:

Infeccin vaginal generada por un hongo, principalmente la especie Cndida
Albicans. Se puede clasificar como complicada o no complicada.

No Complicada: Episodios espordicos o infrecuentes
Signos y sntomas de leves a moderados
Presunta infeccin por C. Albicans
Mujer no embarazada sin complicaciones medicas

Complicadas: Episodios recurrentes (cuatro o mas al ao)
Sntomas y signos graves
Infeccin presunta o demostrada por cndida no albicans
Mujer diabtica, enfermedad grave, inmunosupresin u otras
Vulvovaginitis
Embarazo

Recurrente : 4 o mas episodios en el ao

204
a. Etiologa: Causada por hongos . Un 90% causada por Cndida Albicans, el resto
y menos frecuente dados por Cndida Glabrata, Cndida tropicalis o
Torulupsis glabrata.

4. Fisiopatologa:


Todas la especies de cndida ( flora saprofita vaginal ) producen una enzima
llamada aspartil proteinasa cuya capacidad de produccin est asociada
directamente con su virulencia y la capacidad de adherencia al epitelio vaginal.
Adems de esta enzima necesita un pH acido para una mejor expresin
Tambin se describe una mico toxina que tambin perece tener un papel
importante en la virulencia de este hongo, tiene propiedades inmunosupresoras
dirigida a macrfagos y neutrfilos, alterando la quimio taxis.

5. Factores de riesgo:

La infeccin por cndida se da principalmente en mujeres embarazadas, diabtica,
obesas, inmunocomprometidas, diabticas, que toman anticonceptivos orales,
cortico esteroides o hayan recibido algn tratamiento antibitico previo,
alteraciones locales de la piel
La prctica de mantener la zona vaginal caliente y hmeda, como llevar ropa
ajustada o el uso habitual de protectores ntimos, tambin aumentan el riesgo de
infeccin por cndida.

6. Epidemiologia:
Cndida puede encontrarse entre un 5 40% de la poblacin general en forma
asintomtica. La cual produce sintomatologa al exponerse a algunos de los
factores de riesgo mencionados anteriormente

7. Diagnstico:
El diagnostico exige la visualizacin de blastosporas o pseudohifas en la
microscopia con solucin salina o KOH al 10% o un cultivo positivo en una mujer
asintomtica. El test de KOH solo muestra la especie albicans.
Es posible medir el pH vaginal el cual debera estar en rango de normal o acido. Un
pH sobre 4.5 es indicativo de Vaginosis bacteriana o Tricomoniasis.
El pH es pilar fundamental en el diagnstico, el cultivo se usa ocasionalmente
(Prez Snchez) y sobre todo cuando se sospechan cepas resistentes y en casos de
candidiasis recurrente.

8. Anamnesis:
El sntoma inicial ms frecuente es el prurito, aunque hasta un 20% de las mujeres
pueden ser asintomticas. El escozor, la disuria externa y la dispareunia tambin
son habituales.

205

9. Examen fsico:
Con frecuencia los tejidos vulvar y vaginal tienen un color rojo intenso y en los
casos graves la escoriacin no es infrecuente. Generalmente se observa una
secrecin espesa y adherente con aspecto de quesillo con un pH de 4.5. Esta
secrecin es inodora.

10. Estudio de la paciente:
Observar al microscopio blastosporas o pseudohifas
Solicito cultivo para candica Albicans

11. Tratamiento:

El tratamiento de la candidiasis son los anti fngicos.
Nistatina (aumenta la permeabilidad) o derivados de los imidazolicos (inhiben el
crecimiento del hongo) forma tpica , es de eleccin en embarazo.
Clotrimazol 100 mg vulos. 1 ov. c/noche por 7 10 das
Clotrimazol 599 mg cada 7 das
Betametasona crema asociado a clotrimazol pata vulvitis.
Fluconazol 150 mg por 1 vez v.o a pareja solo si es sintomtica o recurrente
Fluconazol 150 mg da por 3 das y luego 1 a la semana por 3 meses en candidiasis
recurrente. Tambin se indica capsulas de acido brico en resistencia a fluconazol


42. FLUJO VAGINAL TRICHOMONAS


Paciente de 35 aos, consulta por flujo desde hace 3 meses que le irrita la vulva
y la vagina. Hace 2 meses se coloc vulos vaginales cuyo nombre no recuerda y
algo mejor, pero ahora empeor incluso present un pequeo sangrado post
coital y disuria ocasional.
Antecedentes Familiares: no refiere
Antecedentes Personales: colesterol alto.
Antecedentes ginecobsttricos: menarqua 13 aos, RM IV/28, G1P1A0, cambio
pareja sexual hace 4 meses.
Hbitos: 7 cigarros al da.

Desarrollo:
1. Diagnstico apropiado:
1) Multpara de 1 ,
2) vulvovaginitis por Trichomona,
3) tabaquismo

206

2. Diagnstico diferencial: Vaginosis bacteriana, candidiasis vulvovaginal.

3. Definicin: Corresponde a una infeccin de transmisin sexual producida por un
protozoo flagelado intracelular (Trichomona vaginalis)
4. Etiologa: Trichomona vaginales. Son factores de riesgo: cambio de pareja sexual,
coito 2 o ms veces por semana, ms de 3 parejas sexuales el ltimo
mes, otra ITS concomitante.
5. Epidemiologa: ETS ms frecuente en Estados Unidos, causa de 5-50% de casos
de vaginitis aguda.
6. Diagnstico clnico: clnico, pH vaginal mayor a 4,5, observacin de Trichomonas
en el microscopio ptico con aumento de plimorfonucleares.
7. Anamnesis y examen fsico: Flujo verde amarillento, abundante, espumoso y de
mal olor, asociado a extensa inflamacin de mucosa vaginal, eritema,
ardor, dispareunia, sin prurito. En especuloscopa se puede observar
crvix en fresa (ppulas eritematosas en el crvix) 2-5% de los casos.
8. Estudio de la paciente: slo microscopio ptico y pH.
9. Tratamiento y conducta:
1) Metronidazol 2 gr va oral en dosis nica. (resistencia de Trichomona
vaginalis a metronidazol es 2 a 5%).
2) Tinidazol 2 gr va oral en dosis nica. (Tasa de cura del 90-95%)
3) Metronidazol 500 mg va oral cada 12 horas por 7 das.
4) Siempre tratar a la pareja.

10. Consecuencias para la paciente y el feto: si hay embarazo existe riesgo de parto
prematuro. Puede haber colonizacin del endometrio, endometritis,
fiebre postparto, celulitis de cpula vaginal post histerectoma, infeccin
postaborto, factores de riesgo de adquisicin y transmisin de VIH, VHS,
gonorrea, Chlamydia, VPH y NIE.

43. INFECCIONES GINECOLGICAS : HERPES.


Paciente de 39 aos consulta por malestar general, fiebre y lesiones maculopapulosas
pruriginosas en la vagina y vulva que aparecieron hace 4 semanas y ahora son como
ampollas dolorosas y adenopatas inguinales.
- Antecedentes Familiares : no
- Antecedentes Personales: diabetes mellitus.
- Antecedentes Gineco -obsttricos : menarquia 13 aos , RM IV/28 G1P1A0 1,
- Habito: no.

1. Diagnstico
Multpara de 1
Infeccin ginecolgica por virus herpes
DM

207

2. Diagnsticos diferenciales (Tabla 2 y 3)
o Sfilis
o Chancroide
o Linfogranuloma venreo
o Pnfigo
o Imptigo buloso
o Dermatitis de contacto
3. Definicin
El herpes simple genital es una de las infecciones de transmisin sexual ms
frecuente causada por virus, se caracteriza por episodios repetidos que se
desarrollan con una erupcin de pequeas ampollas, generalmente dolorosas,
sobre los genitales. Perodo de incubacin de 2-20 das. En el hombre se ubica en
prepucio, glande, cuerpo del pene y en las mujeres en vulva y cuello uterino.
4. Etiologa
Es ocasionado por dos virus que pertenecen al grupo herpes virus hominus,
conocidos, respectivamente, como herpes simple tipo 1 (HSV-1) y como herpes
simple tipo 2 (HSV-2). El HSV-l es responsable, aproximadamente, del 5 al 10% de
los herpes genitales. Ambos virus son transmitidos por contacto sexual. Es normal
que ocurra un cruce de infecciones de tipo 1 y 2 durante el contacto sexual oral -
genital.
El virus se transmite por medio de:
- El contacto sexual, incluidos el coito, el sexo oral y anal
- El lquido de las ampollas del herpes que se pone en contacto con otras partes
del cuerpo
- Una madre infectada que se lo transmite a sus hijos durante el embarazo o el
parto
5. Fisiopatologa
El virus ingresa al cuerpo a travs de una ruptura en la piel o la exposicin de las
membranas mucosas al HSV permite su entrada y replicacin en clulas epiteliales.
Posteriormente hay infeccin de terminaciones nerviosas sensitivas o
autonmicas. La nucleocpside viral se transmite va axonal hasta el soma neuronal
en el asta dorsal de la mdula espinal, desde donde descender al rea
comprometida al momento de la reactivacin. Despus de la resolucin de la
primo infeccin no se encuentra virus infectante en el ganglio. Sin embargo se
puede encontrar ADN viral en el 10 a 50% de las clulas ganglionares. Desde su
ubicacin original en la mdula, puede migrar y comprometer otras races
nerviosas, con el consiguiente crecimiento del rea cutnea comprometida.
No se conocen claramente los mecanismos que participan en la reactivacin del
virus. Las recurrencias sintomticas y asintomticas pueden ser espontneas o
responder a un estmulo, que puede ser estrs emocional, perodo premenstrual,
fiebre, luz UV, dao tisular o neural, inmunosupresin, calor, congelamiento, y
otras infecciones locales.

208
Puede ocurrir reinfeccin por otra cepa de VHS, as como autoinfeccin por
inoculacin cuando los ttulos de anticuerpos an no se elevan.
El virus es ms contagioso cuando las ampollas estn presentes. Adems, es
contagioso durante la etapa de incubacin. sta es la etapa antes de que las
ampollas o las llagas sean visibles. El virus tambin puede diseminarse cuando no
hay lceras cutneas visibles. Si bien el herpes tipo 1 suele aparecer con mayor
frecuencia alrededor de la boca, tambin puede causar una infeccin genital, al
igual que el herpes tipo 2.
6. Factores de riesgos
- *Conductas/estilo de vida: Tener relaciones sexuales sin proteccin, Ser
sexualmente activo a temprana edad, sexualmente activo durante muchos aos,
varias parejas sexuales, tener una pareja que est infectada con herpes genital,
participar en actividad homosexual entre hombres.
- Sexo: Las mujeres (una da cada cuatro mujeres) son ms propensas que los
hombres (uno de cada cinco) a ser infectadas con herpes genital.
- Factores Socioeconmicos: Adultos jvenes, La mayora de nuevas infecciones
ocurren en adolescentes y adultos jvenes. En adultos jvenes, la infeccin por
virus de herpes simple 1 (HSV-1) se est volviendo una causa ms comn de
herpes genital. Edad: 20-40
- Factores de Salud: Infeccin por VIH, Historial de otras infecciones de transmisin
sexual, Sistema inmune debilitado
- Factores Externos (Una vez que usted tiene herpes genital, algunos estudios
sugieren que existen varios factores que pueden desencadenar el virus y hacerlo
activo): Estrs, Luz solar excesiva, Menstruacin, Actividad sexual vigorosa.

7. Epidemiologa
El herpes genital es comn en los Estados Unidos.
En Chile son el 3.8% de todas las ITS: VHS 1 (55%) VHS 2 (90%). La infeccin por HSV 1
ocurre en forma ms precoz que la infeccin por HVS 2. La infeccin por VHS 2 se
encuentra ampliamente distribuida a nivel mundial, especialmente elevada en frica y
Amrica central.
El 90% de los adultos de 50 aos tienen anticuerpos contras VHS 1. La mayora de las
personas de nivel socioeconmico bajo adquieren la infeccin antes de los 30 aos.
En los Estados Unidos, aproximadamente una de cada seis personas entre 14 y 49 aos
tiene herpes genital. 22% mayores de 12 aos VHS 2.
Tasas mayor en mujeres (26%) v/s hombres (18%).
Adolescentes con mayores tasas.
8. Diagnstico clnico
Preguntar acerca de los sntomas y antecedentes clnicos. Se le realizar un examen
fsico. Si tiene ampollas y lceras visibles, el mdico las examinar. Es posible que las
lesiones dentro de las vas urinarias, la vagina o el cuello uterino no se puedan
observar fcilmente. Para ayudar con el diagnstico, el mdico puede:
- Abrir una ampolla para tomar una muestra

209
- Realizar anlisis de sangre: su mdico realizar anlisis para averiguar si tiene el
virus del herpes simple tipo 1 o virus del herpes simple tipo 2.
- Si se le diagnostica herpes genital, es posible que se le realicen anlisis para
detectar otras enfermedades de transmisin sexual , incluido VIH.

9. Anamnesis
- Preguntar por conducta sexual (nmero de parejas sexuales, proteccin).
- Inicio de cuadro, si haba pasado antes (primo infeccin: carece de anticuerpos; o
recurrentes)
- Sntomas generales: compromiso del estado general, fiebre, cefalea sntomas
asociados.
- Sntomas locales: dolor, disuria, prurito, secrecin uretral y vaginal, linfadenopata
inguinal dolorosa.
- Lesiones cutneas y mucosas (distintos estadios): vesculas, pstulas y lceras
eritematosas dolorosas.

10. Examen fsico
Inspeccin de vulva y cuello uterino: en busca de lesiones cutneas y mucosas como
vesiculares/ampollares (1-3 mm), mltiples lceras de bordes lisos, de base
eritematosa, dolor a la palpacin, indurada, secrecin serosa moderada, prurito,
adenopatas inguinales bilaterales, fiebre.
11. Estudio de la paciente
- Cultivo viral: es el gold standard; muestra a travs de vesculas, LCR, heces, orina,
nariz, faringe, conjuntiva, crvix. Con anticuerpos monoclonales en el cultivo
puede diferenciarse entre VHS 1 y 2. Los resultados se obtienen entre 24 y 48
horas, pero debe esperarse hasta 7 das para confirmar un resultado negativo.
Puede hacerse estudio de sensibilidad en pacientes con mala respuesta clnica.
- Test Tzank: observacin bajo microscopio del raspado de la base de la lesin
(frotis), se colorea con tincin Giemsa, muestra clulas gigantes multinucleadas
donde identifica al VHS (no hace distincin entre 1 y23) o Varicela Zoster. Demora
slo 30 minutos.
- PCR: es muy sensible, no se afecta con el tratamiento, puede identificar virus en
muestras de herpes recurrente de ms de 72 horas de evolucin y es el mtodo de
eleccin cuando se quiere pesquisar excrecin asintomtica del virus (como en el
momento del parto). Su resultado demora 4 horas.
- Inmunofluorescencia: de un frotis de la lesin tiene menor sensibilidad que el
cultivo, pero arroja un resultado en 1 hora.
- Serologa anticuerpos: diagnstico rpido, se hace para averiguar si una persona ha
estado alguna vez infectada con herpes genital u oral. El examen busca anticuerpos
contra el virus del herpes simple 1 (VHS-1) y el virus del herpes simple 2 (VHS-2),
pero no detecta el virus en s.
- Serologa por Elisa: para detectar AC contrae el VHS 1 y 2, con sensibilidad y
especificidad del 95% despus de 2 semanas de contagio.

210
- Citologa crvico-vaginal: sensibilidad del 50% para detectar infeccin del epitelio
por VHS.
- Biopsia: sensibilidad y especificidad similar a Tzanck.

12. Tratamiento
Es importante recibir tratamiento lo antes posible, ya que as disminuye el riesgo de
que infecte a otras personas y lo ayuda a recuperarse ms rpido de un brote.
Es importante recordar que el virus se mantiene en su organismo. No existen
tratamientos que puedan matar el virus. Sin embargo, existen medicamentos para
reducir el riesgo de tener un brote.
- Primoinfeccin:
Aciclovir 400 mg vo c/8 hrs por 7 das o Valaciclovir 1 gr vo c/12 hrs por 7 das
- Recurrencia:
Aciclovir 400 mg vo c/8 hrs por 5 das o Valaciclovir 500 mg vo c/12 hrs por 5 das
Si tiene una infeccin recurrente, su mdico puede indicarle que tome un
medicamento antiviral todos los das para prevenir un brote. Esto se llama terapia
inhibidora.
Otros:
- Analgesia
- Paos fros sobre ampollas o lceras
- Baos tibios
- Ropa de algodn
- Mantener ampollas o lceras secas cuando no se est tomando el bao
- Tratamiento ATB si tiene una infeccin bacteriana de las ampollas
- Tratamiento para parejas sexuales
Es importante que su pareja sexual se realice un anlisis para detectar herpes genital y
que reciba asesoramiento profesional. Si su pareja tiene el virus, tambin debe recibir
tratamiento.
Las estrategias de prevencin incluyen:
- Usar condones de ltex
- Evitar tener sexo oral si su pareja tiene ampollas de herpes en la boca o el rea
genital
- Evitar tocar las ampollas para prevenir el contagio en otras partes del cuerpo
- Si est embarazada y tiene herpes, infrmele al mdico. Los medicamentos
suministrados a los recin nacidos inmediatamente despus del parto pueden
reducir la probabilidad de infeccin

13. Consecuencias para paciente/feto
- Para la mujer existe mayor riesgo de padecer otras ITS (especialmente VIH), ya
que hay dao de la piel y mucosa. Tambin superinfeccin bacteriana de las
lceras herpticas: sfilis, cancroide.
- Para madre y RN puede ocurrir infeccin visceral diseminada y muerte
- Mujeres que han contrado el VHS hacia el final del embarazo tiene mayor riesgo el
RN de nacer con bajo peso y parto prematuro.

211

212
44. INFECCIN POR VPH
Mujer de 23 aos consulta por presentar mltiples lesiones verrugosas en regin vulvar
y cerca del ano desde hace 2 semanas.
Antecedentes Familiares: no refiere.
Antecedentes Personales: sana
Antecedentes Gineco obsttricos: menarquia 11 aos, RM V/25 G0P0A0, inicio
act sexual 14 aos, MAC condn ocasional, 3 parejas sexuales anteriores.
Hbitos : no refiere
Desarrollo:
1. Diagnsticos apropiado:
- Nuligesta
- Verrugas vulvares obs Condiloma en genitales Infeccin por VPH
2. Diagnsticos diferenciales:
- Micropapilomatosis labial
- Acrocordones
- Plipos fibroepiteliales
- Condilomas planos (sfilis secundaria)
- Molusco contagioso
- Quistes sebceos
- Foliculitis vulvar .

3. Definicin:
Infeccin de transmisin sexual, provocada por el virus papiloma humano
(VPH). El HPV se encuentra ampliamente distribuido en todo el mundo y
provoca un amplio espectro de enfermedades epiteliales, desde verrugas a
papilomas en los epitelios de distintas mucosas.

4. Etiologa:
El HPV es in virus DNA que pertenece a la familia Papovaviridae, del gnero
papiloma virus. Se ha descrito ms de 160 genotipos diferentes.
- Bajo riesgo: Papilomas ano genitales, cervicales, oro faringe y tracto
respiratorio 6, 11, 30, 34, 40, 42, 44, 55, 57, 59.
- Alto riesgo: Papilomas ano genitales y cervicales, papulosis Bowenoide,
displasia cervical, oro faringe, cncer cervical y ano genital 16, 18, 31, 33,
35, 39, 45, 51, 52, 56.

5. Epidemiologa:

213
El HPV se encuentra ampliamente distribuido en todo el mundo. En Chile el
ao 1998 la tasa fue de 2.2 casos por 100000 habitantes (3).
El HPV est fuertemente asociado al desarrollo de displasia, neoplasia
intraepitelial, y cncer del cuello uterino. Ms del 95% de los cnceres de
cuello uterino poseen DNA de HPV de alto riesgo

6. Fisiopatologa:
Slo un 10% de las infecciones por HPV tiene manifestacin clnica, ya sea
en la forma de verrugas, papilomas o displasias. El virus se replica en el
estrato granuloso y es detectado en el estrato crneo, no as en el estrato
basal. La infeccin por HPV no tiene rol oncognico per se, sino que juega
un rol que es potenciado por factores fsicos y qumicos. En los carcinomas
no se encuentran partculas virales activas, pero s su DNA y sus genes
tempranos. Los productos de los genes E5, E6 y E7 tienen actividad
oncognica, ya que sus protenas estn involucradas en el control del ciclo
celular y estimulan la proliferacin o interfieren con la diferenciacin de
clulas infectadas. El blanco de estas protenas virales est representado
por las protenas retinoblastoma (Rb), y p53. Normalmente retinoblastoma
acta inhibiendo la transcripcin de genes como c-myc, ras, entre otros que
ests encargados de la proliferacin celular, de manera que la inactivacin
de Rb provoca una replicacin celular descontrolada. La p53 se encarga de
promover la transcripcin de genes para reparar el DNA daado o inducir
apoptosis, de modo que su inactivacin provoca la prdida de la capacidad
de bloquear la proliferacin celular como respuesta al dao del DNA. El
resultado es la inestabilidad gentica y el desarrollo de mutaciones crticas
que favorecen el desarrollo de tumores.

7. Diagnstico clnico:

La principal manifestacin clnica son las verrugas del rea genital externa y
el condiloma acuminado, que se presenta como lesiones papulares con
superficie lobulada e irregular, de color rosado oscuro, con prolongaciones
digitiformes con aspecto de coliflor. El nmero de lesiones es variable, su
tamao va de 2mm a 1 cm, pero si son numerosas pueden confluir
comprometiendo grandes reas genitales que se traumatizan durante el
coito. En la mujer se ubican con mayor frecuencia en los labios mayores y
menores, parte posterior del introito, cltoris, monte de Venus, paredes
vaginales y en el cuello uterino.

8. Estudio de la paciente:

- El diagnstico es clnico.

214
- Las lesiones subclnicas deben ser visualizadas con colposcopa y con la
aplicacin de cido actico al 3-5% que ayuda a delimitar la lesin mediante
la reaccin blanco actica. Sin embargo, es un examen de baja
especificidad, ya que se altera en otras enfermedades, como por ejemplo el
lquen plano, la candidiasis, etc.
- En todas las mujeres con condiloma acuminado debe realizarse citologa
con tcnica de Papanicolaou anual.
- Debe biopsiarse verrugas resistentes al tratamiento, atpicas o
pigmentadas.
- Se debe solicitar exmenes, en bsqueda de otras ITS (VIH y Sfilis)

9. Tratamiento y conducta:

Tratamientos Qumicos
- Podofilino al 10%-30%-45% en solucin alcohlica

Aplicacin por profesional mdico en lesiones de genitales externos y
perianales. La aplicacin debe repetirse semanalmente por 3 a 4 semanas
hasta la desaparicin de las lesiones. Se debe lavar la zona 1 a 4 horas post
aplicacin.
Si la lesin no mejora considerar otra posibilidad diagnstica o la presencia
de una cepa ms agresiva. La aplicacin de grandes cantidades de
podofilino puede provocar toxicidad sistmica como por ejemplo: vmitos,
coma, depresin respiratoria, hematuria, falla renal, y muerte por frenacin
medular. Por lo que se debe utilizar menos de 0,5 ml de podofilino en cada
aplicacin o en un rea menor a 10 cm2. Su uso est contraindicado en el
embarazo y la lactancia.

- cido Tricloroactico al 80-90%

Aplicacin local por el mdico 2 a 3 veces por semana por un mximo de
tres semanas. Se forma una erosin que sana en unas 3 semanas sin
cicatriz. Puede usarse como terapia combinada con podofilino al 40% en
pacientes inmunodeprimidos. Es el tratamiento de eleccin en el
embarazo.

Tratamientos Fsicos

- Crioterapia (Nitrgeno Lquido): El mecanismo de accin es la produccin
de una necrosis epidrmica y drmica, junto a una trombosis de la
microvasculatura drmica (2).

215
Las complicaciones son infrecuentes. Cura aproximadamente el 90% de las
lesiones, aunque a veces se requiere varias aplicaciones.
- Electrociruga
Eventualmente desaparecen todas las lesiones, aunque el 20 a 30%
desarrolla nuevas lesiones en los bordes quirrgicos o en sitios alejados.
- Extirpacin Quirrgica
- Lser

Inmunomoduladores

- Imiquimod (Aldara): Factor inmunomodulador inductor de la sntesis de
interfern gamma, TNF alfa, IL 1,6,8,10 y factor estimulante de colonias
granulocticas. Estimula la inmunidad celular y carece de actividad antiviral
directa in vitro.
- La presentacin en crema al 5% se utiliza en el tratamiento de los
condilomas y verrugas planas, se aplica cada 48 horas y se deja actuar por 2
a 4 horas. La respuesta se observa a las seis semanas. Es bien tolerado
incluso en pacientes inmunodeprimidos, aunque dos tercios de los
pacientes presentan eritema y ardor. Produce curacin en el 60% de los
casos y las mujeres responden mejor que los hombres.
Seguimiento Cualquier modalidad de tratamiento tiene aproximadamente
30% de recurrencia o aparicin de nuevas lesiones en sitios alejados. stas
aparecen generalmente entre los 3 y 6 primeros meses de terminado el
tratamiento.
Se debe citar a los pacientes a las 3 semanas de concluido el tratamiento y a los
3 meses para el alta definitiva. La curacin definitiva se considera despus de 6
meses sin lesiones.

Referencias
1. Human papillomavirus update. Mahalakshmi Gurumurthy .Obstetrics
gynecology and reproductive medicine 23:8
2. Diagnstico y terapia del virus papiloma humano. Marcela Concha. Rev. chil.
infectol. 2007, vol.24, n.3
3. Enfermedades de Transmisin Sexual. Dr. Gonzalo Eymin. Publicaciones
medicina UC. Disponible:
http://publicacionesmedicina.uc.cl/TemasMedicinaInterna/pdf/EnfTransmision
Sexual.pdf

216
45. SNDROME PREMENSTRUAL

Mujer de 32 aos consulta por sensacin de hinchazn generalizada y mastodinea en
periodo pre menstrual. La sensacin es ms intensa a nivel abdominal y mamario,
nerviosismo, irritabilidad, cambios de humor y mal estar general. Muy irritable a veces,
le pasa generalmente antes de la regla y se le pasa tras la regla y cuando tomo ACO .Su
examen ginecolgico es normal.
- Antecedentes Familiares: no refiere.
- Antecedentes Personales: no refiere.
- Antecedentes Gineco obsttricos : menarquia 13 aos , RM V/35 G2P2A0 ACO
ocasional
- Hbitos: cigarro 10 al da.

1. Diagnstico
Multpara de 2
Sndrome premenstrual (SPM)
Tabquica crnica

2. Diagnsticos diferenciales

- Enfermedades psiquitricas: Sndrome depresivo
- Endometriosis
- Tu ginecolgico

3. Definicin

Desorden cclico caracterizado por sntomas emocionales y fsicos de severidad
variable, que se manifiestan durante la fase ltea del ciclo menstrual, es decir, 2
semanas antes o durante la menstruacin. El concepto definido como Sndrome
dismrfico pre-menstrual es un SPM severo. Cabe mencionar que ambos son
diagnsticos de descarte. Afecta a cualquier rango etario y es diferente para cada
mujer. Si la mujer no ovula, claramente no presentara SPM.

4. Etiologa

Actualmente Desconocida, se plantea:


- Hereditaria

217
- Se cree que podra ser un mal funcionamiento en la regulacin neurohormonal
y neurotransmisores: desregulacin de serotonina
- Gran carga emocional: estrs


5. Fisiopatologa
Probablemente es el resultado de una compleja interaccin entre los
esteroides ovricos y los neurotransmisores centrales.
La serotonina participa aumentando los sntomas del SPM. Incrementa la sntesis
de allopregnenolona (neuroesteroide biolgicamente ms activo y la sensibilidad a
los esteroides)-
Susceptibilidad Gentica: variacin en receptor a los estrgenos de tipo familiar
incrementando SPM. Induciendo la desregulacin de los niveles normales de
esteroides.

6. Factores de riesgos

- Menores de 20 aos
- Mayor de 40 aos
- Multparas
- Antecedentes familiares de depresin post parto o trastorno del nimo
- Antecedentes familiares o personales de depresin grave

7. Epidemiologa

Prevalencia variable, siendo de un 3-24%.
El 85% de las mujeres menstruantes tienen al menos 1 sntoma del SPM. De stas
10-25% presentan SPM y 3-8% mujeres menstruantes tienen una forma ms
severa de SPM (SDPM).

8. Diagnstico clnico

Diagnstico de exclusin. Basado generalmente en sus sntomas y como afectan en
su vida.
Presentacin por al menos 3 ciclos consecutivos y al menos 5 das antes de la
menstruacin de sintomatologa:
- Afectiva: depresin, ira, irritabilidad, ansiedad, confusin o aislamiento social.
- Somtica: sensibilidad mamaria, cefalea, distensin abdominal, edema de
extremidades.

9. Anamnesis

218
Desde cundo presenta los sntomas, duracin y en qu momento aparecen.
Antecedentes personales y familiares de depresin.
- nimo depresivo*
- Labilidad emocional*
* DEBE ESTAR PRESENTE
Los sntomas fsicos ms comunes abarcan:
- Distensin abdominal, sensacin de gases
- Sensibilidad en las mamas
- Estreimiento o diarrea
- Deseos vehementes por alimentos
- Cefalea
- Menor tolerancia al ruido y las luces
- Confusin
- Dificultad para concentrarse
- Fatiga
- Sentimientos de tristeza y desesperanza
- Sentimientos de tensin, ansiedad o nerviosismo
- Tendencia al olvido
- Comportamiento irritable, hostil o agresivo, con arranques de ira hacia s
mismo o hacia los dems
- Prdida del impulso sexual (puede incrementarse en algunas mujeres)
- Altibajos en el estado de nimo
- Baja autoestima, sentimientos de culpa o aumento de los miedos
- Problemas para dormir (dormir demasiado o muy poco)
- Movimientos letrgicos, lentos y perezosos





10. Examen fsico

No hay hallazgos en el examen fsico: TV normal, no se palpan masas,
especuloscopa no se aprecia flujo patolgico ni alteracin vaginal ni cervical. PAP
si es necesario. Examen mamario: puede haber sensibilidad mamaria. Examen
neurolgico: normal.
11. Estudio de la paciente

No existen signos o pruebas de laboratorio especficas que puedan diagnosticar el
sndrome premenstrual. Para descartar otras posibles causas de los sntomas es
importante tener:
- Una historia clnica completa

219
- Un examen fsico (que incluya un examen plvico)
- Un calendario de sntomas puede ayudar a las mujeres a identificar los
sntomas ms problemticos y a confirmar el diagnstico del SPM.
- PAP: normal
- Ecografa: normal
- Rxamenes biquimicos si se sospecha otra patologia concomitante

12. Tratamiento

- Cambio en el estilo de vida y alimentacin: restriccin de sal, cafena, OH y
dieta sana. Los dias previos a la regla El triptfano que se encuentra en los
lcteos tambin ayuda. Abundante lquido para reducir la distensin,. Vitamina
B6 (100 mg da, no hay muchos estudios sobre riesgo-beneficio), vitamina E
(400 UI/da, no tiene efectos secundarios y buen antioxidante), calcio (1200 mg
da, reduce en un 49% sntomas, despus de 3 ciclos). Magnesio asociado a
Vitamina B6
- Dormir mnimo 8 hrs diarias, Ejercicio fsico y reduccin de estrs
- Diario de sntomas: se les pide que expliciten sus sntomas, esto permite darles
frmacos segn los sntomas registrados
- AINES: para dolores significativos (sensibilidad mamaria, dolor menstrual,
cefalea)
- Hormonal:
a. ACO: han sido utilizados muy a menudo en el tratamiento, depende del tipo de
mujer segn sus sntomas, su eficacia es bastante limitada. Algunas mujeres
responden de forma positiva, pero en cambio otras no responden e incluso
empeoran. Drosperinora 3 mg/ Etinilestradiol 0.03 mg mejoran molestias y
sintomatologa, Tambien la clormadinona.
b. Los posibles regmenes incluyen: anovulatorios; progesterona en supositorios
vaginales (200-400 mg/d) o inyeccin (5-10 mg i.m., forma oleosa) durante 10-
12 das previos a la menstruacin; progestgenos de accin prolongada (p. ej.,
acetato de medroxiprogesterona 200mg i.m. cada 2-3 meses), o un agonista
GnRH (p.ej., leuprolida 3,75 mg i.m. o goserelina 3,6 mg i.m. al mes) con
estrgenos y progestgenos a dosis bajas para eliminar los cambios cclicos.
(Progesterona oral y vaginal: solo disminuye sensibilidad mamaria y algunos
sntomas psicolgicos)
- Los inhibidores selectivos de la recaptacin de serotonina (p. ej., fluoxetina 20
mg/d v.o. o sertralina 50 mg/d v.o. Escitalopram , Paroxetina ) son los frmacos
ms efectivos en el tratamiento de los sntomas psicolgicos y fsicos del sndrome
premenstrual.
- Se puede usar como complemento la fototerapia y terapia conductual cognitiva
para reducir necesidad de medicamentos.

220
- Se pueden utilizar tranquilizantes (p.ej. una benzodiacepina) en pacientes con
irritabilidad, nerviosismo y falta de control, especialmente cuando no pueden
variar el ambiente determinante del estrs.
- La terapia conductual cognitiva puede ser una alternativa a los antidepresivos.
- La fototerapia puede disminuir la necesidad de antidepresivos.
- Los diurticos pueden ayudar a las mujeres con retencin severa de lquidos que
causa distensin, sensibilidad en las mamas y aumento de peso: espironolactona
100 mg/ da
- La bromocriptina, el danazol y el tamoxifeno son frmacos que se emplean
ocasionalmente para aliviar el dolor de mamas.
- Sntomas severos: Danazol, anlogos de GnRh, ooforectoma bilateral
(extra)
- Si el sntoma ms importante es el trastorno del nimo es de gran utilidad el uso
de inhibidores selectivos de la recaptura de serotonina (fluoxetina y sertralina).


(Panay, Nick. (2011). Management of premenstrual sndrome: evidence-based guidelines. Obstetrics, Gynaecology and Reproductive Medicine, 21(8), 211-228)

13. Consecuencias para la paciente/feto


- El SPM no desaparece con las medidas de cuidados personales.
- Lo sntomas ms intensos limitan su capacidad de desempeo y mayor ausentismo
laboral.
- Las tasas de suicidio en mujeres con depresin es mucho ms alta durante la
segunda mitad del ciclo menstrual.

Bibliografa
5. Panay, Nick. (2011). Management of premenstrual sndrome: evidence-based
guidelines. Obstetrics, Gynaecology and Reproductive Medicine, 21(8), 211-228.
6. Carvajal, J. Ralph, C. (Eds.) (2013). Dismenorrea y Sndrome pre-menstrual
(SPM). Manual Obstetricia y Ginecologa (pp 507-509). Chile: Ediciones PUC

221
46. QUISTE OVRICO (FUNCIONAL)
Mujer de 32 aos acude a consulta por presentar dolor hipogstrico ocasional y
meteorismo, que comenz hace 2 meses. Al examen clnico a Ud. le parece palpar una
masa en anexo izdo. y le solicita una eco tv que muestra una imagen qustica de 6 cm de
probable origen ovrico, de aspecto simple, unilocular y sin excrecencias .
Antecedentes Familiares : madre miomas padre HTA
Antecedentes Personales : ITU ultimo ao
Antecedentes Gineco obsttricos : menarquia 13 aos , RM VII/25 GoPoA0 FUR
25 das atrs
Hbitos: cigarro 10 al da.

Desarrollo:
1. Diagnsticos mas apropiado: Quiste ovrico

2. Diagnsticos diferenciales:
Ovario : Funcional
Tumor benigno , borderline o maligno
Endometrioma
Tumor metastasico
Trompa : Hidrosalpinx
Absceso tubo ovrico
Embarazo ectpico
Intestinal : Ca colon
Enf Diverticular
Constipacin
Otros : Vejiga
Rin Plvico
Quiste mesentrico




222
3. Definicin:
Las tumoraciones funcionales del ovario son transformaciones qusticas de
sus glndulas cclicas (folculo y cuerpo amarillo) que se caracterizan por un
crecimiento anatmico exagerado, anormal (hasta 6 a 10 cms)
Se clasifican en:
- Quistes foliculares: Falla la ovulacin y se acumula lquido folicular.
- Quistes del cuerpo lteo (lutenicos de la granulosa): Desarrollo
excesivo del cuerpo lteo con acumulacin de sangre.
- Quistes de la teca lutenica: Mltiples folculos con clulas teca
luteinizadas. Se produce por hiperestimulacin de HCG.

4. Etiologa:
Trastorno hormonal que repercute sobre el ovario o que parte del mismo
ovario en su calidad de glndula de secrecin interna. El tabaquismo es uno
de los pocos factores de riesgo que han sido identificados para los quistes
funcionales ovricos (Holt, 2005, OR de 2.48 con un 95 % de IC)

a. Epidemiologa: No existen estadsticas precisas disponibles sobre su
incidencia. Aparecen sobre todo en edad reproductiva (20-45 aos),
siendo la causas ms frecuentes de origen funcional. En peri
menopusicas y postmenopusicas deben ser objeto de estudio
inmediato, por el aumento del riesgo de malignidad.

5. Fisiopatologa:
Quiste folicular: Son las estructuras qusticas ms comunes halladas en los
ovarios normales. Estos quistes provienen de variaciones patolgicas
temporarias de un
proceso fisiolgico y no son neoplsicos. Son el resultado de la falta de
evolucin de un folculo maduro dominante o de la insuficiencia de un
folculo inmaduro para sufrir el proceso normal de atresia, sin reabsorcin
del lquido folicular. Como factor etiolgico se ha postulado la liberacin
anormal de gonadotrofinas.

Quiste del cuerpo lteo: Es el resultado de la hemorragia intraqustica que
se produce como una parte normal del perodo de vascularizacin, 2 a 4
das despus de la ovulacin. El sangrado rpido y excesivo puede provocar
la ruptura y el hemoperitoneo. El sangrado gradual y profuso provoca el
agrandamiento del quiste y si persiste, la sangre es reemplazada por un
lquido claro. Es comn que estos quistes no tengan actividad hormonal y
posean un dimetro promedio de 4 cm. Cuando se produce la hemorragia
predomina en el lado derecho, posiblemente porque la presin intraluminal

223
es ms elevada sobre el lado derecho, por las diferencias en la arquitectura
de las venas ovricas.

6. Anamnesis:
- Asintomticos
- Atraso menstrual variable
- Meteorismo
- Dolor-dispareunia
- Sndrome doloroso agudo del abdomen, el dolor es intenso en todo el
abdomen pero principalmente en hipogastrio y fosas iliacas; se asocian
vmitos, constipacin, aceleracin del pulso y resistencia muscular.
Quiste de cuerpo lteo: Lo caracterstico de la ruptura es que se produce
durante los das 20 a 26 del ciclo menstrual. Muchas pacientes presentan
dolor agudo de menos de 24 horas de duracin, aunque en 23 % de ellas el
dolor puede tener una duracin de 1 a 7 das. Cerca de 17 % de las
pacientes relatan el comienzo del dolor durante el coito.

7. Examen fsico:
- Sin alteraciones (Hallazgo a la Ecotomografia TV)
- Masa plvica
- Dolor anexial
- Taquicardia En caso de torsin o rotura

8. Estudio de la paciente:
1. Ecotomografia
Quiste folicular: Se pueden encontrar en 68 % de los ovarios
premenrquicos a travs de US (usualmente tienen menos de 10 mm de
dimetro). Los quistes tienen paredes delgadas, son uniloculares y su
tamao va desde algunos milmetros hasta 8 cm de dimetro, con un
promedio de 2 cm.
Quiste Cuerpo lteo: Generalmente menor a 5 cm.
Quiste teca lutenicos: Generalmente bilaterales y mayor a 20 cm.
Recordar Tranquilizar a la paciente al encontrar quistes simples sin reas
slidas, septos papilas.
2. HCG (Descartar embarazo ectpico: principalmente en pacientes
jvenes, sin uso de MAC y que presenten dolor agudo)
3. Marcador tumoral (CA 125: En casos de duda de presentar quiste
simple)

9. Tratamiento y conducta:

224
Descartar con anamnesis de factores de riesgo de malignidad y
Ecotomografia compatible con quiste simple:
- Quiste folicular y del cuerpo lteo, su manejo es conservador. Muchos
de ellos se resuelven dentro de las 8 a 12 semanas de observacin. El
tratamiento de la paciente mediante ACO podra acelerar la resolucin
del quiste.
- Quiste tecaluteinizantes: Realizar reposo, abstinencia sexual y control
con Eco.
En caso de torsin o rotura Tratamiento quirrgico de urgencia.

10. Consecuencias para la paciente:
- Torsin del quiste ovrico (2): es la de mayor frecuencia y se origina en
tumores de mediano tamao y pedculo largo. El dolor suele ser el
sntoma principal y es de carcter agudo o subagudo. Si origina
compromiso circulatorio puede llegar a producir necrosis tumoral
siendo necesario el tratamiento quirrgico urgente.
- Rotura: suele ser secundaria a traumatismo, torsin o exploracin
previa. Es poco frecuente y la clnica depender del tipo de tumor y de
su contenido.

Referencias
1. Management of a pelvic Mass. Hilary L Turnbull. (Figura 1)
2. Conducta ante una masa anexial. Antonio Rodrguez Oliver. Servicio de
Obstetricia y Ginecologa. Hospital Universitario Virgen de las Nieves (Granada).

47. DISMENORREA PRIMARIA



Paciente de 21 aos que consulta por dolor tipo clico en hipogastrio que se acompaa
de sudoracin, nuseas y diarrea ocasional. Estos dolores le ocurren desde hace 2 aos y
se relacionan con las reglas ya que aparecen con ellas y terminan con ellas.
- Antecedentes Familiares: No refiere.
- Antecedentes Personales: alergia a AAS.
- Antecedentes Gineco-obsttricos: menarquia 13 aos, RM IV/28 G0P0A0 reglas
dolorosas moderada a severa . usa condn
- Hbitos : cigarro 3 al da

225

1. Diagnstico
Nulpara
Dolor plvico crnico cclico : dismenorrea
Tabquica crnica

2. Diagnstico diferenciales
- Dismenorrea secundaria: inicio inmediatamente con la menarquia, inicio despus
de los 25 aos, el dolor se inicia con la fase ltea, con el mximo al inicio de la
menstruacin, exmen plvico anormal, sin respuesta a tratamiento usual
(AINES/ACHs), dispareunia. Endometriosis, estenosis cervical, enfermedad
inflamatoria plvica, plipos, miomas, adenomioma, endometritis por DIU, quistes
ovricos funcionales, tumores ovricos malformaciones uterinas.
Otras:
- Urolgicas: infecciones del tracto urinario, litiasis renal.
- Enfermedades gastrointestinales: sndrome de colon irritable, constipacin.
- Alteraciones msculo-esquelticas.
- Psicosomticos sin que exista una patologa subyacente: la percepcin del dolor es
la combinacin del dolor real, del estado emocional y de los aspectos sociales
concomitantes del momento.

3. Definicin
Podemos definir dismenorrea como una condicin mdica de algia plvica cclica que
aparece con la menstruacin o coincidiendo con sta y dura ano mas que la regla .Hay
que considerarla un trastorno de cierta relevancia, ya que es una causa importante de
incapacitacin laboral, de automedicacin y de consulta.
La dismenorrea primaria es cuando no hay patologa estructural que la explique y se
presenta como un dolor agudo o espasmdico en la zona baja del abdomen de tipo
clico, de cualquier intensidad, sin patologa plvica asociada. Puede comenzar con la
menarquia (6-12 meses posterior), su patrn es durante la menstruacin (primeros 2-3
das), similar entre las menstruaciones, situado en zona hipogstrica. Sin sntomas
asociados, salvo en los casos de sndrome premenstrual. Suele producirse en mujeres
de entre 17 y 25 aos y es poco habitual en edades posteriores o tras haber tenido
hijos,
Cuando hay condiciones asociadas que explique en dolor, se denomina dismenorrea
secundaria

4. Etiologa
La causa de la dismenorrea depender de si sta es primaria o secundaria. En general,
las mujeres con dismenorrea primaria experimentan contracciones uterinas anormales
como resultado de un desequilibrio qumico en el cuerpo uterino (particularmente la
prostaglandina y el cido araquidnico - ambas sustancias qumicas controlan la
contraccin del tero) lo que provocara contracciones uterinas muy intensas , las

226
cuales causaran isquemia miometrial y no existe una patologa ginecolgica como
causa del dolor.

5. Fisiopatologa
- Producido por contracciones del miometrio inducido por prostaglandinas
(progesterona estimular la sntesis de Fosflipasa A-2 en los lisosomas de las
clulas endometriales, e incluso la sntesis de prostaglandinas de vida media muy
corta y de accin fugaz. Principalmente prostaglandina F2 y E2) en el lquido
menstrual, las cuales producen contraccin miometrial importante,
vasoconstriccin de vasos endometriales y miometriales, isquemia del tero y
sensibilizacin de las terminales nerviosas, que en conjunto se va a traducir en
dolor:
o Cambios en motilidad uterina: aumento en la frecuencia de contracciones
uterinas, arritmias en la peristalsis uterina, aumento del tono uterino en
reposo (mayor a 10), Aumento de la fuerza de contraccin uterina (mayor a
120).
o Cambios vasculares locales: vasoconstriccin, disminucin del flujo del
miometrio, aumento del consumo local, isquemia uterina.
o Cambios neurosensiorales: hipersensibilidad alterada de las fibras
nociceptivas a los productos de la muerte celular.
- La vasopresina tambin puede jugar un papel cada vez mayor en la contractibilidad
uterina y que causa el dolor isqumico como resultado de la vasoconstriccin.
- Aumento de tromboxanos, leucotriones

Cuerpo lteo Produccin de progesterona Secrecin anormal y aumentada de PG
PG inducen contracciones uterinas Dolor



6. Factores de riesgos
Si bien todas las mujeres corren el riesgo de sufrir dismenorrea, se cree que las
mujeres ms propensas a padecer este trastorno son

227
- Ciclos menstruales ovulatorios (condicin normal)
- Tabaco
- Alcohol durante el perodo menstrual: el OH prolonga la duracin del dolor
menstrual
- Obesidad y Sobrepeso
- Bajar de peso
- Menarquia temprana: menor de 12 aos
- Edad menor de 20 aos
- Depresin o Ansiedad
- Trastornos de la personalidad
- Hipermenorrea
- Nuliparidad
- Historia familiar de dismenorrea

7. Epidemiologa
La dismenorrea es frecuente entre las adolescentes y las mujeres en edad
reproductiva y afecta aproximadamente a un 25%-50% de las mismas en algn
momento durante este periodo de la vida. Es la principal causa de consulta
ginecolgica y tambin principal causa de ausentismo escolar y laboral
La prevalencia de mujeres menstruantes que presentan dismenorrea primaria, oscila
desde un 20 hasta un 90%.
En general :
- 15% severa e invalidante
- 30% leve a moderada


8. Diagnstico clnico
Se considera una patologa de descarte. En general la paciente consulta porque sus
menstruaciones son muy dolorosas. Inicio varios meses despus de la menarquia,
inicio con la menstruacin o poco antes, duracin menos de 72 horas, dolor clico
supra pbico, examen fsico en su mayora normal.

9. Anamnesis
Antecedentes: historia menstrual, uso de anticonceptivos (y su efecto con el dolor),
antecedentes de familiares de dismenorrea primaria y secundaria. Uso de DIU, ETS,
dispareunia, infertilidad, ciruga plvica o abdominal previa
Antecedentes previa del mismo cuadro, duracin, frecuencia, localizacin,
circunstancias en que aumenta, asociacin con ciclo menstrual, grado de discapacidad,
sntomas asociados, tratamientos pasados y actuales. Cirugas previas .
Hay sntomas comunes, sin embargo, cada persona puede experimentarlos de una
forma diferente. Los sntomas pueden incluir:
- clicos en la parte inferior del abdomen
- dolor en la parte inferior del abdomen
- dolor lumbar

228
- dolor irradiado hacia las piernas
- nuseas y/o vmitos
- diarrea
- fatiga y debilidad
- sncope
- cefalea

10. Examen fsico
Examen fsico general, plvico y ginecolgico (examen mamario, tacto vaginal,
espculoscopia) en la mayora es normal en la dismenorrea primaria . El dolor plvico
y/o abdominal puede ser de caractersticas muy variables y asociarse a otros sntomas.
Siempre ir a descartar dismenorrea secundaria: masa plvica, flujo vaginal anormal.

11. Estudio de la paciente
En general no hay estudios especficos para dismenorrea primaria, ya que es una
patologa de descarte.
Habra que descartar dismenorrea secundaria
- Cultivo y serologa para ETS: normal
- Leucocitos y VHS: normal
- Orina completa: normal
- Ecografa abdominal o transvaginal: normal
- Laparoscopia o laparotoma: normal
- PAP: normal

12. Tratamiento
El tratamiento especfico para la dismenorrea ser variable dependiendo de una serie
de factores como la edad, el estado de salud, los antecedentes, la severidad del dolor,
la tolerancia a medicamentos y si la dismenorrea es primaria o secundaria.
Si existe una patologa ginecolgica se debe diagnosticar y tratar. Si la dismenorrea es
primaria, el gineclogo y en funcin de las preferencias, optar por cambios higinico-
dietticos, ejercicio fsico, analgsicos y/o tratamiento hormonal entre otros.

No farmacolgico:

- Dieta y ejercicios: disminuir consumo de OH y cafena, abundante lquido.
- Cese tabaco
- Calor local: parche trmico por 12 horas diarias
- Neuroestimulacin elctrica tras cutnea (TENS):

o Eficacia parcial del 42-60%
o Requieren menor dosis de AINES
o Mediantes impulsos aferentes aumentara el umbral al dolor y aumentara
la secrecin de endorfinas en nervios perifricos
- Acupuntura:

229
o Eficacia parcial hasta un 91%

Farmacolgico
- AINES:
o Exitoso en el 80-85% de las pacientes
o cido mefenmico 250 mg vo c/6 hrs hasta 500 mg c/ 8hrs
o Ibuprofeno 400-600 mg vo c/6 hrs.
o Naproxeno: 250 mg vo c/6 hrs o 500 mg vo c/12 hrs
o Inhiben la secrecin endometrial de prostaglandinas
o Efecto analgsico central directo
o Por 3 das y para ser efectiva su uso se debe iniciar antes del inicio de la
regla.
o Evitar uso de AAS: la dosis que requiere es muy alta
o Si la paciente no responde bien en 2 ciclos, intentar con otro AINES o
agregar ACO.
- ACO combinados
o Mayor del 90% de xito, se puede asociar a AINES .
o Reducen volumen menstrual
o Disminuyen secrecin de prostaglandinas
o Efecto cede al suspender su uso
o Si no hay respuesta en 3 meses realizar laparoscopia exploratoria para
descartar endometriosis.

13. Consecuencias para la paciente/feto

Para la mujer tiene un impacto social en trminos de estrs y prdidas individuales no
cuantificadas.
Abstencin escolar o laboral : 35-50%
Mortalidad: nula

Bibliografa
1. Kolhe, S. Deb, Shilpa. (2011). Dysmenorrhea. Obstetrics, Gynaecology and
Reproductive medicine, 21 (11), 3113-316.
2. Monterrosa C, A. (2001). Dismenorrea primaria: visin actual. Revista
Colombiana de Obstetricia y Ginecologa, 52 (4), Art 028.
3. Castro, Magdalena. (2009). Dismenorrea primaria en adolescente: Revisin de
la literatura. SOGIA, 12 (2), 24-36.

230
48. DOLOR PLVICO :DISMENORREA POR DIU.

Mujer de 38 aos acude por dismenorrea desde hace 4 meses y reglas abundantes desde
hace apx 1 ao .
Antecedentes Familiares: madre ca mama padre diabtico.
Antecedentes Personales: gastritis crnica
Antecedentes Gineco obsttricos : menarquia 13 aos , RM IX /25 FUR hace 15
das G3P3A3 usa DIU de cobre hace 1 ao
Hbitos: no tiene.

1. Diagnostico:
G3P3A3
Dismenorrea
Gastritis crnica
SUA I ( sangrado uterino anormal Iatrogenico ? )

2. Diagnsticos diferenciales:
.
Plipos.
Miomas.
Adenomiosis.
DIU.
Causas extrauterinas.
Endometriosis.
Quistes ovricos funcionales.

3. Definicin:
Dismenorrea es el Dolor abdominal o plvico durante la menstruacin.
Dolor plvico crnico cclico, mayor de 6 meses de duracin.
Dismenorrea secundaria se define como un dolor menstrual asociado a
patologasubyacente. Por lo general se inicia aos despus de la menarqua ya que
estas patologas demoran aos en aparecer. El hecho de ser secundario se refiere a
que hay una causa no en relacin a la edad.

4. Etiologia:
Efecto inflamatoriod de el Dispositivo Intrauterino

5. Fisiopatologa:
Dispositivo que se inserta en la cavidad uterina, est constituido por un vstago de
plstico radiopaco, que puede tener diversas formas, se les agrega a algunos cobre
en forma de un alambre enrollado en su vstago, en otros una progestinas de
liberacin lenta, son los llamado medicados.

231
Actan produciendo una alteracin del endometrio como reaccin a cuerpo
extrao, lo cual es paciente dependiente y se acenta con las infecciones
intrauterinas

6. Epidemiologa:
Es variable, no hay cifras claras.

7. Diagnostico:
En pacientes con DIU y con dolor hipogstrico de origen uterino , ya sea cclico o
permanente ,

8. Anamnesis:
Derivar a nivel secundario si:
El dolor inicia con la fase ltea, con el mximo al inicio de la menstruacin.
Examen plvico anormal.
Sin respuesta al tratamiento usual (AINEs/ACHs).
Dispareunia

9. Examen fsico:

En la espculoscopia ver las guas si estn bien o la ausencia de ellas. Si no estn
realizar una eco tv y buscar si el dispositivo est presente , esta incrustado o fue
expulsado con lo cual debera buscar otro diagnostico diferencial.


http://www.polgalvan.sld.cu/contenido/galdiu.htm

232

http://consultafamiliar.net/articulos-profesionales-medicina/herramientas-y-
procedimientos/diu-insercion-de-dispositivo-intrauterino/

10. Estudio de la paciente:

Anamnesis
Examen fsico
Especuloscopia
Ecografa
TAC- RNM .

11. Tratamiento:
Conducta a nivel primario:
Si no est desplazado, tratar con AINE y antibiticos ) DOXICILCINA (
AMOXICILINA + Ac CLAVULANICO ), si se sospecha un PIP leve, si no cede, retirar el
DIU y observar si el dolor cede con esta conducta. Si el dolor cede al retirar el DIU y
la ecografa es normal, no es necesario enviar a nivel secundario.
Si se retira el dispositivo intrauterino darle una alternativa a nuestra paciente para
planificacin familiar.

Derivar a nivel secundario y terciario: Si la paciente presenta algia plvica crnica no
cclica, en que puede haber :
- Antecedente de PIP severo
- Antecedente de uso de DIU de larga data.
- Retroversin uterina fija
- Para metritis crnica
- tero fijo
- tero sensible
- Dispareunia de mantencin y/o post coital

233

Extraccin de un DIU
1. Informar a la paciente del procedimiento que se le realizara. Si es por patologa
explicar qu patologa y por qu sacarlo..
2. Aseo sol fisiolgica en el cuello uterino
3. Traccionar guas con pinza de Bosseman.
4. Si se cortan guas: histermetro para ver longitud de cavidad, ver si hay
permeabilidad del canal, corroborar posicin de tero.
5. Advertir a la paciente que sentir dolor y si es soportable, se contina, con la
extraccin con crochet, de lo contrario, se extraer con anestesia
6. Tomar cuello con pinza Pozzi
7. Introducir crochet hacia el fondo con gancho horizontal, en el fondo girar a
vertical y luego extraer.
8. Delicadamente intentar varias veces enganchar el DIU
9. Si hay xito: MOSTRARLO A LA PACIENTE.


http://consultafamiliar.net/articulos-profesionales-medicina/herramientas-y-
procedimientos/diu-insercion-de-dispositivo-intrauterino/

Recambio de DIU
Usuarias de DIU con cobre: cada 10 aos
NO retirar un DIU e insertar otro inmediatamente
En lo posible ACO u otro MAC por 1 a 3 meses y luego otro DIU
DIU de plstico: tiempo indeterminado de uso, no se recambia
En DIU de plstico (tipo Lippes) slo vigilar clnica y ecogrficamente anualmente la
presencia de tumores anexiales (actinomicosis), especialmente sobre los 10 aos
de uso.
El PAP es til para diagnosticar actinomicosis.
En caso de PAP con actinomices, slo retirar DIU, esperar 2- 3 meses utilizando
ACO o barrera y luego reinsertar en lo posible uno con cobre.

234
Previamente repetir PAP para determinar si no hay actinomices. Si se repite PAP
con actinomices despus de 2 a 3 meses con ACO y sin DIU, aplicar vulos de
clindamicina o metronidazol por cinco das. Si en el control el PAP persiste con
actinomices, derivar a Ginecologa.
En toda paciente con actinomices en el PAP no olvidar examen pelviano acucioso
y/o ecografa para descartar actinomicosis clnica

11. Consecuencias para la madre y el feto:
Disconfort de la paciente e incluso infeccin.

Bibliografa
Guas de Ginecologa / Edicin Cedip Internet 2009 Dr. Sergio Silva 8

49. DOLOR PLVICO CRNICO ENDOMETRIOSIS



Paciente de 32 aos que consulta por dismenorrea desde hace 8 meses que ha ido
aumentando cada vez ms, incluso debe faltar al trabajo.
Antecedentes Familiares: padre hipertensa
Antecedentes Personales: no refiere
Antecedentes Gineco obsttricos : menarqua 13 aos , RM IV/28 G0P0A0
Hbitos : cigarro 10 al da

1) Diagnsticos
1. Nulpara
2. Dismenorrea
3. Tabquica crnica
4. Endometriosis ?

2) Diagnstico diferencial
1. Miomas
2. Distopia uterina ( retroversion uterina )
3. Quiste ovarico .
4. DIU.

Causas extrauterinas.
1. Endometriosis.
2. Quistes ovricos funcionales.
3. Tumores.
4. Sndrome de colon irritable.

235
3) Definicin

Presencia de glndulas endometriales y estroma fuera de la cavidad uterina.

4) Etiologa: existen diferentes teoras

1. Metaplasia Celmica: Maatsura 1999: in vitro clulas peritoneales con 17OH-


estradiol a clulas endometriales glandulares.

Sugiere que el peritoneo parietal es un tejdo pluripotencial que puede sufrir


transformacin metaplsica hasta convertirse en un tejido que es idntico al
endometrio normal desde el punto de vista histolgico.

2. Diseminacin endometrial
Implantes peritoneales por Menstruacin retrograda: Sampson1927

o Gran n de mujeres presenta menstruacin retrgrada sin endometriosis
o Adems deberan tener Disfuncin inmune interferira con lisis de lesiones
o Clulas in Vitro con proliferacin e implantacin mas rpida
o Enfermedad en sitios distantes por diseminacin hematgena o linftica

3. Implante directo (cesrea)

3. Otras teoras

Diseminacin linftica o vascular (Ueki, 1991). Esta teora se basa en la presencia de


endometriosis en sitios raros como el perin o la regin inguinal. La regin
retroperitoneal posee una circulacin linftica abundante. De esta manera, cuando
no se identifican implantes peritoneales sino nicamente algunas lesiones
retroperitoneales, se supone que la diseminacin es linftica.

Adems, la tendencia del adenocarcinoma endometrial a diseminarse por va


linftica demuestra la facilidad con la que el endometrio puede ser transportado
por esta ruta (McMeekin, 2003). muy pocos estudios que valoren esta
diseminacin de la endometriosis.

FACTORES ASOCIADOS
Mutaciones genticas y polimorfismos: Los ndices de agrupacin familiar
mencionados sugieren una evidencia poli gnica y se han investigado diversos genes
posibles.Aunque no se ha observado un patrn mendeliano, la frecuencia elevada en
los familiares de primer grado sugiere un patrn poli gnico/multifactorial. 34 veces

236
mayor en parientes primer grado.

Defectos anatmicos : La obstruccin del cuello uterino predispone a la


endometriosis,probablemente al exacerbar una menstruacin retrgrada
(Breech,1999).Ej. retroeversoflexin del tero, tabique cervical, mioma, malf
uterina, plipos endometriales

* Los DIU tambin se han visto asociados

Toxinas ambientales : Se han realizado diversos estudios cuyos resultados


sugieren que el contacto con ciertas toxinas ambientales contribuye a la
endometriosis. Las toxinas que se mencionan con ms frecuencia son 2,3,7,8-
tetraclorodibenzo-p-dioxina (TCDD) y otros compuestos similares a la dioxina

Inmunolgico: El sistema normal remueve material extrao al alcance de la


cavidad peritoneal, si esta disminuido puede dar endometriosis.

5. Ubicacin

Generalmente en las zonas declive de la pelvis:

Ovario : 44%
Fondo de saco posterior 34%
Fondo de saco anterior 34%
Saco Douglas 15%
Rara vez en la vejiga, pericardio, cicatrices quirrgicas y pleura.

Tipos :

Peritoneal/superficial:

Lesiones negras despigmentadas: por la hemosiderina

Lesiones rojas

Lesiones despigmentadas y atpicas: son las que ms pasan inadvertidas


Ovrica

237
Lesiones superficiales: se coagulan con electro bistur

Quistes achocolatados

Se ven grandes ovarios (kissing ovaries)



Del tabique recto vaginal/profunda
Lesiones infiltrantes y profundas que obliteran el fondo de saco posterior
(pacientes describen disquexia: dolor defecatorio).


Endometriosis NEJM 2010
5) Fisiopatologa

Para que una clula se pueda implantar en un lugar ectpico debe adherirse a un
tejido diferente, infiltrar la matriz extracelular degradndola, interactuar con otras
clulas, proliferar en forma controlada, producir neo angiognesis y adaptarse a
un nuevo medio ambiente hostil. Todo esto ocurre mientras las clulas son
estimuladas por cambios cclicos de las hormonas esteroidales y dentro de una

238
produccin de factores autocrinos y paracrinos que afectan este proceso.

Factores genticos: se conoce que existe una tendencia familiar tanto en humanos
como primates. Varios estudios muestran que la incidencia de endometriosis es
mayor en parientes de primer grado

Medio ambiente: algunos componentes del ambiente interactan como que en las
mujeres con endometriosis hay niveles ms elevados de dioxina.

Apoptosis: se ha demostrado que en endometriosis eutopico como ectpico
tienen disminuida su apoptosis

Invasin endometrial: para que una clula pueda implantarse en un tejido extrao
debe invadir su matriz extracelular y para que pueda desarmar la matriz
extracelular que acta como sustento de la arquitectura epitelial. Para ello
existen las enzimas llamadas metaloproteinasas que que estn involucradas en la
remodelacin de la matriz extracelular
Alteraciones inmunolgicas y endometriosis: la reabsorcin del tejido endometrial
por el flujo retrogrado esta a cargo del sistema inmune muy activo en la pelvis
femenina.
Citoquinas y factores de crecimiento: en mujeres con endometriosis se encuentran
elevados los niveles de citoquinas pro inflamatorias como las GM-CSF, interlukina 6
y el TNF.



6) Factores de riesgo
1. Edad
La endometriosis es una patologa que afecta a las mujeres en edad de
procrear.
Las mujeres ms afectadas son las que tienen entre 30 y 50 aos de edad.
Las adolescentes tambin pueden sufrir una endometriosis.
Las mujeres pueden presentar muy raramente una endometriosis despus
de la menopausia.

2. Los antecedentes familiares y la herencia
Una predisposicin familiar de primer grado puede representar un factor de
riesgo. La presencia de dolores pelvianos en una mujer cuya madre presenta una
endometriosis debe alertar.

3. Los antecedentes ginecolgicos
Malformaciones obstructoras congnitas.
Raspados uterinos.
Electrocoagulacin del cuello.

239
El uso de un dispositivo uterino puede representar un factor de riesgo.

4. Anomalas de las menstruaciones :Las reglas precoces, abundantes y dolorosas,
as como ciclos cortos pueden representar factores de riesgo.
5. Hipofertilidad:La asociacin de hipo fertilidad y endometriosis son frecuentes.
6. Tabaquismo
7. Factores asociados
- Anatmico
- Gentico
- Hormonal
- Inmunolgico
- Peritoneal

8. Factor anatmico
Obstruccin al tracto de salida
a. Sinequia uterina cervical, mioma, malformacin uterina, retroversin
uterina marcada. Otros como DIU, plipos
9. Factor Gentico
Pacientes de primer grado 34 veces mas.
75% de gemelas monocigoticas afectadas (estudio noruego en 515
mujeres) cuando la hermana tiene endometriosis.
2% de familiares femeninos en segundo grado vs 0,7% de poblacin
general.

10. Factor hormonal
Enfermedad estrgeno dependiente (vida frtil)
Menstruacin prolongada mayor a 7 das, se asocia a flujo retrogrado
Ciclos regulares, espontneos y prolongados
Embarazo tiene efecto protector (involutivo)
Interrupcin artificial de clico puede provocar regresin

12. Factor inmunolgico
Sistema inmune normal contempla mecanismo de limpieza
b. Remocin de material extrao que alcance la cavidad peritoneal
(endometrios bacterias)
c. Mujeres con endometriosis exhiben
i. Disminuye inmunidad humoral
ii. Activacin de macrfagos que no suprimen proliferacin celular
endometrial en cavidad peritoneal
iii. Igg y complemento: mayor estimulacin
Presencia de factor de crecimiento epidrmico y de trasformacin derivado de
macrfagos

240
7) Epidemiologa
0,7% poblacin general, incidencia real difcil de precisar
Afecta:
6-10% edad reproductiva
50-60% adolescentes y mujeres con dismenorrea
50% de las pacientes con infertilidad

8) Diagnstico clnico
Se manifiesta con dismenorrea, dispareunia, dolor plvico crnico e infertilidad.
Clnica: sus sntomas son sugerentes, pero su ausencia no la descarta. No existe
relacin entre la extensin de las lesiones y la clnica. Al examen fsico: tero
puede estar fijo, doloroso a la movilizacin, presencia de ndulos en vagina ,
perin , tero sacro o tabique recto-vaginal .La dismenorrea tpica es una mujer
con aos de ciclos sin dolor que comienza con dolor progresivo, en todos sus ciclos,
y que dura durante todo el periodo
Puede tener reglas dolorosas recurrentes lo cual debe hacer el diagnostico
diferencial con adenomiosis.
Dolor durante el coito se debe descartar problemas psicosexuales y atrofia genital.
Miccin dolorosa, se debe descartar cistitis.
Disquexia durante las reglas, se debe descartar constipacin por fisura anal.
Dolor abdominal bajo crnico se debe diferencar con SII, dolor neuropatico,
adhesiones.
Dolor crnico dorsal bajo se debe descartar contractura muscular
Infertilidad.

9) Anamnesis
a. Por menarqua temprana.
b. Menopausia tarda
c. Antecedentes familiares
d. Uso de ACO el cual es factor protector
e. Preguntar por historia de fertilidad
f. Reglas
g. Dolor a la actividad sexual
h. Por hbitos intestinales
i. Hbitos urinarios

10) Examen fsico
Al examen fsico: tero fijo, pegado a uterosacros, doloroso a la movilizacin,
presencia de ndulos en vagina o perin. Implantes en episiotoma. La
dismenorrea tpica es una mujer con aos de ciclos sin dolor que comienza con
dolor progresivo, en todos sus ciclos, y que dura durante todo el periodo.

11) Estudio del cuadro

241
El gold estndar es la laparoscopa, la ecografa ayuda en endometriomas, puede
ser un hallazgo quirrgico, ayuda el CA125 y la sospecha clnica.
Ubicacin de las lesiones: trompas, ovarios, cara anterior y posterior del tero,
ligamento ancho, terosacros, tabique rectovaginal, saco de douglas, ligamento
ancho, apndice, colon y vejiga.
Infertilidad: ya sea por adherencias, o por un ambiente inflamatorio citotxico que
altera la inmunidad, el transporte ovular y la implantacin.
Clasificacin ASRM: grado I: mnima (se trata igual, ya que mejora la fertilidad y
aumenta la tasa de embarazos), II: leve, III: moderada, IV: severa (endometrioma).

El estudio de imgenes es fundamental en el diagnstico de la endometriosis,
siendo la ecografa ginecolgica (transvaginal y pelviana) un pilar importante
cuando se sospecha esta patologa. Hallazgos ecogrficos, como un tero en
retroversin fija, ovarios adheridos a la pared pelviana o a la cara posterior del
tero o entre s, son indicadores indirectos de endometriosis. Adems, la presencia
de quistes ovricos nicos o mltiples de aspecto granular, bien delimitados, con
cpsula gruesa sugieren fuertemente el diagnstico de endometriosis. En el caso
de la endometriosis recto vaginal y de infiltracin profunda en el recto sigmoides
es de utilidad la endosonografa ano rectal y la resonancia nuclear magntica
(RNM) de pelvis. En ciertos casos particulares, cuando existe sospecha de
compromiso de la va urinaria o intestinal, es necesaria la realizacin de estudios
complementarios como la cistoscopia, la pielo Tomogafa Axial Computarizada
(TAC), la rectoscopia o colonoscopa.
En la actualidad se ha avanzado significativamente en el diagnstico a travs de
imgenes, particularmente con la endosonografa ano rectal de alta resolucin, sin
embargo, an se considera que el diagnstico definitivo de la endometriosis se
establece a partir de la visualizacin directa de las lesiones mediante laparoscopa
diagnstica.
En los casos de pacientes en estudio de infertilidad, en las cuales se sospecha
endometriosis, es fundamental incluir dentro del estudio una evaluacin completa
del estado reproductivo de la pareja, que contemple el estudio de la reserva
ovrica (FSH, estradiol y recuento de folculos antrales), seguimiento folicular
ecogrfico, histerosalpingografa y espermiograma.

12) Tratamiento y conducta
Tratamiento: depende de: edad, sntomas (dolor o infertilidad o ambos), paridad y
efectos adversos de la terapia. Alternativas: nada, anti prostaglandnicos (cada 8-
12 horas por 5 das, ojal antes del inicio de la menstruacin), ACO y quirrgicos.

1. Algia plvica crnica
a. AINES
b. Pseuoembarazo con ACO , ciclo extendido cada 3 6 meses
c. Dienogest : produce amenorrea prolongada. Se administran 2 mgs por 3
4 meses .

242
d. Danazol: se una a la SHBG y desplaza la testosterona, al ser un andrgeno
produce un doble efecto andrognico. Se da 600mgs cada 6 horas, y su
efecto adverso son los derivados de los andrgenos.
e. Agonistas GnRH: infusin continua de GnRH exgeno con supresin de la
liberacin de gonadotrofinas por unin firme al receptor que impide su
estimulacin, por lo que primero producen una estimulacin con
vaciamiento de la hipfisis y luego una menopausia qumica reversible.

2. Infertilidad: ciruga ablativa resectiva y reparadora, inseminacin intrauterina e
IVF.
3. Endometrioma: ciruga resectiva , hay que extraer capsula del quiste .No basta con
fulgurar o puncionar.

Datos laparoscpicos: polvo de carbn o escopetazo. Ndulos negros, pardos
oscuros o azulados, de quistes pequeos con hemorragia vieja rodeada de fibrosis.
Petequias vesiculares, polipoides, hemorragias. Vesculas serosas o claras, placas
blancas o retraccin cicatricial, coloracin amarillenta del peritoneo y adherencias su
ovricas. Pelvis congelada lo da la endometriosis, Cncer y actinomicosis.

13) Consecuencias: La infertilidad, dolor plvico crnico , cirugas mltiples, secuelas
sicolgicas , histerectoma,

Bibliografa
Endometriosis NEJM 2010
Endocrine-disrupting chemicals: an Endocrine Society scientific statement. Endocr
Rev 2009
Williams, ginecologa 1era edicin

50. QUISTE OVARICO TORCIDO.



Paciente de 23 aos consulta por dolor importante desde hace ayer en FID , dolor clico
de inicio sbito que se irradia a dorso .Se acompaa de vmitos, nauseas y mal estar
general. No cede a AINES .Su ultima regla fue hace 25 das .
Antecedentes personales :NO
Antecedentes familiares : madre mioma padre hipertenso
Antecedentes Gineco obsttricos: menarquia 13 aos. RM IV/28 G0P0A0 FUR hace 25
das MAC: nada

243

1. Diagnsticos : Nuligesta
Dolor plvico agudo
Quiste ovrico complicado

2. Diagnsticos Diferenciales : Apendicitis
Clico Renal
Gastroenteritis
Colon Irritable
Embarazo ectpico
ITU
PIP
PATOLIGIA ANEXIAL : anexitis, quiste hemorrgico,
folculo roto
3. Fisiopatologa

El quiste ovrico torcido es una condicin quirrgica poco frecuente, causada por
la rotacin parcial o completa del pedculo ovrico sobre su eje, produciendo
congestin y necrosis del ovario. Es la quinta emergencia ginecolgica en
frecuencia (2,7%), afecta a mujeres de todas las edades, pero el 80% est en edad
reproductiva, hay una mayor tendencia en el ovario derecho, el riesgo aumenta en
embarazo y TRH y presenta una recurrencia aproximadamente del 10%.
En las embarazadas se presenta asociado a un cuerpo lteo grande, sucede en un
10 a un 20 % de todas las torsiones del ovario con una incidencia de 5 por cada 10.
000 embarazos, ms frecuente en el primer trimestre, es raro que ocurra en el 2do
y 3er trimestre.
Los tratamientos con inductores de la ovulacin favorecen la formacin de los
quistes teca lutenicos que incrementan el peso del ovario y por tanto la torsin.
La ligadura de la trompa aumenta el riesgo de torsin del ovario.
El ejercicio puede desencadenar la torsin del ovario en presencia de los factores
anteriores.
Las variedades histolgicas ms encontradas son los quistes funcionales, el
cistoadenoma seroso, el cistoadenoma mucinoso, el quiste dermoides, el fibroma
del ovario y cistoadenofibroma seroso. En los tumores malignos ocurre con una
incidencia menor de un 6 % porque las adherencias cancerosas fijan el ovario.

4. Factores de riesgo :
Una trompa larga y ausencia mesosalpinx, situacin ms frecuente en la adolescencia.
Cuadro clnico que puede llevar a diferentes diagnsticos diferenciales.

244

El estudio consta hemograma, VHS y PCR para evaluar parmetros inflamatorios,
Bhcg para descartar los principales diagnsticos diferenciales ginecolgicos, como
embarazo ectpico. Y Ecotomografia transvaginal, los hallazgos puedes ser mltiples,
como los siguientes:
5. Diagnostico
La sospecha es clnica en una paciente que presenta un dolor plvico agudo en alguna
fosa iliaca en segunda mitad del ciclo menstrual en ausencia de uso de MAC

6. Anamnesis : Se presenta con un dolor abdominal bajo de inicio brusco, acompaado
de nuseas y vmitos en el 60 a 70% de los casos. Dentro del examen fsico se
puede palpar en una masa unilateral (90%), sensacin febril, taquicardia y
resistencia muscular.

Caractersticas del dolor ( Intensidad, ubicacin, irradiacin, tipo )
Fur y uso de ACO
Antecedentes quirrgicos ginecolgicos
Antecedentes de quistes ovrico.
Sntomas asociados : fiebre , diarrea, vmitos, meteorismo

7. Ex fsico : CSV
Examen General : conciencia, lucidez .
Ex abdominal : Blumberg , Masas .
Espculoscopia : Flujos , presencia de DIU
Examen ginecolgico : dolor anexial , presencia de masas , fondo de saco
ocupado

8. Ecografa : Presencia de una masa anexial

- El hallazgo ms caracterstico es el aumento de tamao de un ovario,
hipoecognico por el edema con imgenes eco lcidas de aspecto qustico,
localizados en la periferia del ovario que se corresponden con folculos, son el
resultado de la congestin del ovario que determina una trasudacin hacia ellos.
- Masa con un aspecto ultrasonogrfico variable que puede ser eco lcido,
ecognico o complejo en dependencia de la naturaleza del tumor.

245
- En ocasiones se visualizan pequeas imgenes tubulares eco lcidas en la periferia
del ovario correspondiente a vasos ingurgitados por la congestin.
- Algunas masas muestran un grupo de ecos curvos similares a un pico o una
estructura parecida a un caracol, ecognico o hipoecognico que representa el
sitio de torsin.
- Lquido libre en el saco de Douglas y rodeando el ovario torcido.
- Cuando la necrosis ovrica es severa se puede visualizar un ovario muy aumentado
de tamao con una textura hipoecognica por la presencia de edema y necrosis, da
apariencia similar a un tumor.

Podemos utilizar dopler en la ecotomografia, donde hay que evaluar:

- Si el flujo arterial y venoso son normales, indica que el ovario es viable.
- Disminucin o ausencia de flujo venoso con conservacin del flujo arterial, en este
caso el ovario no es viable.
- Ausencia de flujo arterial y venoso, indica ausencia total de circulacin sangunea
en el ovario, lo que produce un infarto hemorrgico y si no se opera a tiempo lleva
a la peritonitis y a la muerte.
El objetivo del estudio es confirmar el diagnstico y destorcer el rgano
comprometido, de la manera ms precoz posible.
Hemograma
PCR
Bhcg
Sed orina , urocultivo
TAC RNM
9. Tratamiento El tratamiento es siempre quirrgico, puede ser conservador, donde se
realiza destorsin y quistectoma,o anexectoma en caso de necrosis
irreversible, peritonitis, paridad completa o tumor maligno.
El manejo conservador evita la anexectoma y es importante en mujeres en edad
reproductiva. Si se produce perfusin y el aspecto sugiere dao transitorio, se deja
rgano in situ y se procede a destorsin. Si no se produce perfusin adecuada, es
necesaria la reseccin de la estructura.


Bibliografa

246
- Aida de la Caridad lvarez Snchez, Ovarian torsin, Rev Cubana Obstet
Ginecol v.37 n.1 Ciudad de la Habana ene.-mar. 2011. Disponible:
http://scielo.sld.cu/scielo.php?pid=S0138600X2011000100013&script=sci_arttext.
- Freddy Maita Q, Erwin Hochstatter Arduz, Diagnstico en Torsin de Pedculo de
Quiste Anexial en Ginecologa, Gac Med Bol v.34 n.1 Cochabamba 2011. Disponible:
http://www.scielo.org.bo/scielo.php?pid=S1012-
29662011000100007&script=sci_arttext.
- Nikolaos Burbos Timothy J Duncan, Management of a pelvic mass
OBSTETRICS, GYNAECOLOGY AND REPRODUCTIVE MEDICINE.
- Obstetricia, Prez Snchez, 4ta edicin, Editorial Mediterraneo 2011.
- Obstetricia y ginecologa, Beckmann, 6ta edicin, editorial Lippincott William
y Wilkins 2010.

58. SINDRME OVARIO POLIQUISTICO.


Mujer 18 aos, acude a control ginecolgico, presenta reglas ocasionales, la ltima hace
3 meses y principalmente por un crecimiento exagerado del vello en hemi abdomen
inferior y barbilla. La ecografa que se practica muestra un tero pequeo con la
presencia de mltiples folculos en disposicin perifrica.
88 Kg, 1,60 mts, IMC 31, lesiones hiperpigmentadas en axila y cuello. Vello grueso en
hipogastrio y mentn. Genitales externos e internos normales.

Antecedentes familiares: madre diabtica
Antecedentes personales: no refiere
Antecedentes ginecobsttricos: menarqua 16 aos, RM V/60-90 das, G0P0A0,
condn.
Hbitos: 10 cigarros diarios.

Desarrollo:
1. Diagnstico apropiado: Nulpara, hiperandrogenismo, SOP.

2. Diagnstico diferencial: hipotiroidismo, hiperplasia suprarrenal congnita,
Hiperprolactinemia, tumor secretor de andrgenos, sndrome de Cushing.

3. Definicin: Corresponde a un trastorno endocrinolgico que engloba
hiperandrogenismo, anovulacin crnica y disfuncin endocrino-metablica y
conforma un conjunto de signos y sntomas caracterizado por irregularidades del ciclo
menstrual, hirsutismo y acn asociado a trastornos metablicos como
insulinoresistencia, diabetes mellitus tipo 2, sndrome metablico enfermedad
cardiovascular, obesidad y alteraciones psicolgicas.

247
4. Etiologa:
a. Factor gentico: Esto explica el inicio precoz de la patologa y el carcter
crnico. Herencia autosmica dominante con penetrancia superior al 90%. Se
han detectado 39 mutaciones dentro de ellas, polimorfismos para el citocromo
P450 que es el encargado de la esteroidogenesis, mutaciones del receptor de la
insulina.
b. Adrenarquia exacerbada: Hiptesis de la Adrenarquia, consiste en que el
hiperandrogenismo se origina a partir del desarrollo anmalo de la glndula
suprarrenal, esta enva estmulos no CT dependientes al ovario e inducen
hiperplasia de la zona reticular y estas aumentan la produccin de andrgenos
y aparece el hiperandrogenismo.
c. Alteraciones hipotlamo-hipofisarias: top Down. Alteraciones
neuroendocrinas secundarias en el hipotlamo estn interfiriendo. La
exposicin precoz a andrgenos, podra cambiar el patrn de secrecin
hormonal por uno ms masculino. Pueden haber defectos congnitos de
dopamina.
d. Teora de la desregulacin enzimtica: Desregulacin de la produccin de
andrgenos por las clulas de la teca. Puede haber hiperactividad de enzimas
esteroidognicas que dependen del citocromo p 450.
e. Teora de la insulinoresistencia e hiperinsulinismo secundario: 60-70%
pacientes con SOP tienen IR. Tanto la insulina como el IGF 1 tiene receptores
en el ovario y la actividad enzimtica de este se ve estimulada con el aumento
de insulina, adems la insulina deprime la accin de las SHBG aumentando los
andrgenos libres. Tambin se reportan alteraciones genticas del receptor de
la insulina.

5. Epidemiologa: 6-8% de la poblacin (5 a 10% dependiendo de la fuente bibliogrfica)
en mujeres en edad frtil.

6. Fisiopatologa: La esteroidognesis de la va del colesterol a androstenediona es
amplificada por las clulas de la teca, lo normal es que el colesterol se transforme
a progesterona y luego a andrgenos que por accin de la aromatasa son
transformados a estrgenos. Pero en este caso, hay proliferacin y diferenciacin
de componentes de la teca y de la granulosa, y la expresin de receptores para LH
y protenas asociadas con la esteroidognesis es mayor en la teca. Por lo tanto,
existe un aumento de clulas productoras de andrgenos y aumento de la
capacidad de produccin de andrgenos por cada clula de la teca.

Adems, la actividad 5a-reductasa est aumentada en las clulas de la granulosa
del ovario, lo que significa un aumento de la androstenediona a 5-androstano-
3,17-diona que inhibe de la actividad de la aromatasa y disminuye la conversin de
andrgenos a estrgenos aumentando los andrgenos libres, esto mismo ocurre
con el aumento de la insulina.

248
7. Diagnstico clnico: Requiere de la exclusin de otras causas de hiperandrogenismo.

Criterios National Institute of Criterios de Rotterdam 2003 (2 de 3)
Health (NIH) 1990 (1 de 2)
1. Anovulacin crnica 1. Oligomenorrea o anovulacin crnica.
2. Clnica o bioqumica 2. Signos clnicos o bioqumicos de hiperandrogenismo
de hiperandrogenismo 3. Ovario poli qusticos ecogrfico:
y exclusin de otras a. Ms de 12 folculos de menos de 10 mm en
causas ambos ovarios.
b. Volumen ovrico mayor a 10cc, en fase folicular
temprana, sin ACO y sin estimulantes de
concepcin.

Clasificacin de SOP:
1) Fenotipo 1: clsico, hiperandrogenismo clnico y/o laboratorio +
oligoanovulacin + con o sin ovarios poli qusticos + resistencia a la insulina.
2) Fenotipo 2: ovulatoria, hiperandrogenismo clnico y/o laboratorio + ciclos
menstruales normales + ovarios poli qusticos + resistencia a la insulina.
3) Fenotipo 3: sin hiperandrogenismo + anovulacin crnica + ovarios poli
qusticos + sin resistencia a insulina.

8. Anamnesis y examen fsico: alteraciones menstruales 60-70% con oligomenorrea y
amenorrea, 25% con ciclos irregulares, hirsutismo en el 65-75%, acn 12-14%, piel
grasa, alopecia andrognica en menos del 5%, seborrea, subfertilidad, 75% mujeres
con ciclos anovulatorios, hipersecrecin de h en el 40% aumenta riesgo infertilidad
y aborto. Obesidad en 35-60%, aumento de grasa abdominal, acantosis nigricans,
insulinoresistencia 50%, diabetes tipo 2 en un 15%, dislipidemia.

Los trastornos menstruales slo deben cobrar importancia luego de 2 aos de
transcurrida la menarqua.

9. Estudio de la paciente:
a. Insulina: aumentada en un 50%
b. Glicemia: intolerancia a la glucosa 40%
c. Perfil lipdico: hipertrigliceridemia, disminucin del HDL.
d. Hipertensin.
e. Androstenediona, sulfato de dihidroepiandrostenediona, LH, prolactina:
aumentados o normales.
f. LH: su aumento indica una secrecin anormal de gonadotrofinas.
g. FSH: normal o disminuida.
h. Relacin LH/FSH: aumentada mayor a 2. Lo normal es que se encuentre en
relacin 1:1.
i. Globulina transportadora de hormonas sexuales SHBG: disminuida.
j. ndice de andrgenos libres: valor normal es menor a 4,5, pero el SOP se
encuentra aumentado. Este valor se calcula dividiendo la testosterona total por

249
la globulina transportadora de hormonas sexuales SHBG y el resultado se
multiplica por 100. (Testosterona libre aumentada y SHBG disminuida).
k. Ecografa: ms del 30% de la poblacin tiene imagen compatible con ovario
poliqustico, pero por s solo no es criterio suficiente.

10. Tratamiento y conducta: El objetivo es corregir la anovulacin, el hiperandrogenismo
y alteraciones secundarias a la hiperinsulinemia.
a. Corregir anovulacin:
i. Ejercicio peridico, dieta hipocalrica baja en ndice glicmico, comidas
fraccionadas, apoyo psicolgico.
ii. Progestgenos cclicos por 7-10 das cada 6 semanas para favorecer la
menstruacin regular o anticonceptivos anti andrognicos.
iii. Si hay deseo de embarazo: citrato de clomifeno o FSH inyectable como
inductores de ovulacin. Slo se intentan 3-6 ciclos antes de
fertilizacin in vitro. 50% se embaraza dentro de los primeros 6 meses.
iv. Metformina 1000-2000 mg al da, para pacientes con resistencia la
insulina. Beneficios de esta son: mejora la resistencia a la insulina,
reduce el hiperandrogenismo, aumenta niveles de SHBG, mejora el
hirsutismo, disminuye el metabolismo hormonal ovrico, disminuye el
peso, mejora la sensibilidad de receptores de insulina, mejora
alteraciones del perfil lipdico, disminuye los niveles de insulina en
ayunas y la incidencia de diabetes.
v. Biopsia en cua o drilling ovrico, corresponde a la ltima eleccin.
Genera baja transitoria de andrgenos y ovulacin espontnea, puede
durar hasta 3 a 12 meses.
b. Hiperinsulinemia:
i. Dieta, ejercicio, apoyo psicolgico, metformina.
c. Hiperandrogenismo:
i. Anticonceptivos orales anti andrognicos: Ciproterona 2mg+35mcg de
etinilestradiol (Dixi 35, Diane 35, Anuar) eficaz despus del tercer ciclo
con mejora clnica en 6 meses. Dinogest, Desogestrel, Drosperinona
(yasmin, femelle) posee accin diurtica leve por ser anlogo de
espironolactona, Clormadinona (Belara, lovinda). Estos anticonceptivos
contienen etinilestradiol que bajan la LH y aumentan las SHBG con lo
que aumentan la unin de testosterona a la protena transportadora
(SHBG) y disminuye la testosterona total (libre). Tanto ciproterona
como desogestrel aumentan LDL y TGL por lo que es importante
controlar el perfil lipdico de la paciente.
ii. Otros antiandrgenos: Espironolactona 200 mg/da, Finasteride (bloque
alfa reductasa), Flutamida (Etaconil) 125 mg/da, bloquea el receptor de
OH-Testosterona.
iii. Tratamiento cosmtico.

250
iv. Anlogos de GnRH deben evitarse ya que disminuyen la sntesis de
gonadotrofinas y producen Hipogonadismo hipogonadotrofico con
consecuente hipoestrogenismo.
11. Consecuencias para la paciente y el feto:
a. El hiperestrogenismo relativo por el aumento de grasa y conversin perifrica
de andrgenos a estrgenos condiciona aumento del riesgo de cncer de
mama y endometrio.
b. La hiperinsulinemia crnica condiciona mayor riesgo de diabetes mellitus tipo
2, sndrome metablico 30%, cardiopata coronaria, dislipidemia, HTA.



57. HIPERPROLACTINEMIA
Mujer 36 aos consulta por amenorrea secundaria de 10 meses de evolucin, al
comienzo regla irregular y hace 10 meses no le llega.
Antecedentes familiares: no refiere
Antecedentes personales: no refiere
Antecedentes ginecobsttricos: menarqua 13 aos, RM V/28 G2P2A0, MAC
preservativos.
Decir al interno que en el estudio apareci una PRL de 60 ng/ml.

Desarrollo:
1. Diagnstico apropiado: Multpara, Amenorrea secundaria, Hiperprolactinemia
2. Diagnstico diferencial: embarazo, anorexia, distrofia ovrica, sinequias traumticas,
falla ovrica precoz.
3. Definicin: Ausencia de menstruacin por 3 meses en mujeres que previamente
menstruaban normalmente o tenan oligomenorrea por ms de 9 meses.
4. Etiologa:
i. De amenorrea secundaria: embarazo, psicgenas, deporte excesivo, bajo peso,
Hiperprolactinemia, distrofia ovrica, sinequias traumticas, insuficiencia ovrica
precoz.
ii. De Hiperprolactinemia: adenoma pituitario, sndrome de silla turca,
hipotiroidismo, medicamentos como anticonceptivos orales, antipsicticos,
antidepresivos, antihipertensivos, histamina, opiceos, cocana, alteraciones
hepticas, alteraciones renales que disminuyen el clearence, carcinoma bronco
gnico es productor de prolactina, gonadotroblastoma, quiste ovrico dermoide,
carcinoma de clulas renales, teratoma, estimulacin cerebral.
5. Epidemiologa: Hiperprolactinemia corresponde al 1% de las mujeres, los micro
adenomas hipofisarios constituyen un 9-27% en mujeres y un 60% corresponde
prolactinomas.

251
6. Fisiopatologa: La prolactina es sintetizada y secretada normalmente en la
adenohipfisis. La secrecin es controlada fundamentalmente por inhibicin desde
el hipotlamo por dopamina y en menor grado por GABA y somatostatina. La
secrecin de dopamina por el hipotlamo es afectada por estrs, emociones y
frmacos.
La Hiperprolactinemia interfiere con la liberacin pulstil de GnRH. Esto a su vez
hace que se pierda la retroalimentacin positiva y negativa de los estrgenos,
aunque la FSH se encuentre en niveles normales y se altera la pulsatilidad de LH
produciendo anovulacin (Hipogonadismo/hipogonadotropo) y amenorrea.
7. Diagnstico clnico: amenorrea secundaria, galactorrea, cefalea, trastornos visuales y
prolactinemia aumentada (valores normales corresponden a menores a 25ng/ml).
8. Anamnesis y examen fsico: Ejercicio fsico, bajo peso, antecedentes de patologas,
crnicas, uso de cocana, menarqua, historia menstrual, radiacin o quimioterapia
del sistema nervioso central, radiacin de pelvis, estrs social, nutricin, actividad
sexual. En la historia familiar defectos genticos, patrn de vello pbico,
infertilidad, historia menstrual de la madre o hermanas, historia pubertad, peso,
etc.
En el examen fsico: IMC, tero rudimentario o ausente, joroba de bfalo, obesidad
central, hipertensin, debilidad muscular proximal, examen de tiroides, himen
imperforado o septo vaginal transverso, hipertrofia de cltoris, virilizacin,
galactorrea, cefalea, alteraciones visuales.
9. Estudio de la paciente:
1. Solicitar sub unidad b-HCG para descartar embarazo.
2. TSH, Prolactina.
3. Prolactina elevada:
4. Si resulta mayor a 100 ng/ml asociada a galactorrea, cefalea, y trastornos
visuales, se realizar estudio con RNM y TAC para determinar la presencia
de tumor hipofisario. Los adenomas hipofisarios (prolactinoma) son la
causa frecuente.
5. Si se descarta un tumor, 30% se presentan sin galactorrea, y los niveles de
prolactina son inferiores a 100, la mayora de los casos corresponder a
uso de medicamentos antidopaminrgicos que al inhibir la dopamina esta
deja de actuar inhibiendo la prolactina y esta aumenta (ACOs,
antipsicticos, antidepresivos, antihipertensivos, bloqueadores de
histamina, opiceos, haloperidol, fenotiazidas, metildopa, antidepresivos,
estrgenos, metoclopramida y reserpina).
6. Si los valores de prolactina son inferiores a 100 ng/ml se debe solicitar TSH
y cociente LH/FSH, la existencia de estrs y repetir la analtica plasmtica
de prolactina.
7. TSH elevada obliga a completar estudio en busca de hipotiroidismo.
8. Si gonadotrofinas FSH mayor a 40 y LH mayor a 25 estn elevadas, se debe
solicitar estrgenos (17b estradiol) si estos estn disminuidos (menor a 50)
se habla de insuficiencia ovrica precoz (realizar por lo menos 2 a 4 tomas).
10. Tratamiento y conducta:

252
1. Si se detectan micro adenomas (<10mm) estos se derivan a endocrinologa
para seguimiento, la mayora son benignos.
2. Si se detectan macro adenomas, la mayora de ellos es de resolucin
quirrgica transesfenoidal.
3. Agonistas dopaminergicos: Bromocriptina 2,5 mg dos o 3 veces al da, la
dosis se incrementa hasta que desaparezcan sntomas, Cabergolina 0,5 mg
a la semana, se puede incrementar la dosis semanalmente luego de 1 mes
para evaluar sntomas, se sugiere seguimiento de prolactina 1 vez al mes
para realizar ajuste de dosis teraputica.
11. Consecuencias para la paciente y el feto: Infertilidad.

58. MIOMA

Paciente de 42 aos que consulta por dolor abdominal y distensin abdominal,
menstruaciones abundantes y dolorosas de 10 das de evolucin, sensacin de peso
plvico, dispareunia, cansancio y aumento de la frecuencia miccional los ltimos meses.
Antecedentes Familiares: no refiere.
Antecedentes Personales: no refiere.
Antecedentes Gineco obsttricos: menarqua 13 aos, RM VII/25 G2P2A0
Hbitos: cigarro 10 al da.

1) CUAL ES EL DIAGNOSTICO MAS PROBABLE DE ESTA PACIENTE ?
Multipara de 2
Sangrado menstrual abundante y prolongado obs Miomatosis uterina
Tabaquismo activo

2) CUALES SON LOS DIAGNOSTICOS DIFERENCIALES?
Plipos
Coagulopatias
Hiperplasia endometrial
Prolapso Pelvico

3) COMO SE DEFINE ESTA PATOLOGIA?
Tambin llamados leiomioma o fibroma, como su nombre lo indica es un tumor
benigno de musculo liso de la pared uterina. Adems de de fibra muscular lisa est
constituido por tejido conectivo y estroma vascular. Siendo la cantidad de tejido
conectivo lo que le da la consistencia.

253
4) CUAL ES LA ETIOLOGIA DE ESTE CUADRO CLINICO?

La etiologa de los miomas es aun desconcida, se postula un rol gentico-
ambiental, sin existir resultado concluyentes hasta el momento. Sin embargo se
sabe el rol de los estrgenos principalmente en el crecimiento de estos tumores,
sin claridad del mecanismo de accin, por este motivo durante la premenarquia no
se desarrollan miomas, y durante el embarazo y la
post menopausia sufren cambios importantes.Es de
origen monoclonal.

5) CUAL ES LA FISIOPATOLOGIA DE ESTE CUADRO
CLINICO?
Como se describi anteriormente no es clara la
fisiopatologa ni la etiologa de los miomas, si se
conoce el rol de los niveles elevados de estrgenos
que favorecen el crecimiento de estos.
En cuanto a las manifestaciones que se observan en
la miomatosis uterina va a depender de la ubicacin donde se desarrolle este, por
lo que se distinguen principalmente 7 tipos: Subseroso (10-15%), submucoso (15-
25%), Intramural (60-70%), Cervical (8%), Pediculado, parasito (pediculado con
nutricin de tejido cercano), intraligamentoso (ligamento ancho). Dependiendo de
la localizacin es la clnica que van a presentar, 1. Mioma subseroso. 2. Mioma submucoso.
principalmente la sangrado menstrual abundante 3. Mioma intramural. 4. Mioma pediculado.
Ginecologa; Prez Snchez, 3ra edicin
(por aumento de la superficie endometrial), anemia, (2007)
Ginecologa; Prez Snchez, 3ra edicin
fatiga, sntomas compresivos (pujo, poliaquiuria), dolor plvico (por necrosis
(2007

central), Alteraciones reproductivas (infertilidad, poco frecuente, aborto


espontaneo).



En Ultimo concenso FIGO se determino usar la siguiente clasificacion de los
miomas :

254



Tambin se producen cambios degenerativos en los miomas siendo principalmente:
Degeneracin hialina (65%) se cambia material miomatoso por hialino acelular,
ms frecuente en miomas subserosos.
Degeneracin qustica (4%): el contenido hialino se licua y forma cavidades
qusticas.
Degeneracin por calcificacin (4-10%): ms comn en menopusicas, al igual que
la atrofia miomatosa.
Degeneracin roja: es por necrosis del centro, y se produce por un crecimiento
importante en poco tiempo, producindose un infarto muscular agudo provocando
dolor e irritacin peritoneal. Frecuente durante el embarazo.
Degeneracin maligna o sarcomatosa: muy poco frecuente (0.5%)

6) QUE FACTORES DE RIESGO BUSCARA?
Raza (negra), Edad (35-54), obesidad, historia familiar, menarqua temprana.
Mientras que la multiparidad, tabaquismo, mayor edad en ultimo embarazo y
menopausia se postulan como factores protectores.

7) CUAL ES LA EPIDEMIOLOGIA DE ESTE CUADRO?
Tumor benigno ms frecuente del aparato genital femenino. Tiene una incidencia
de 20-30% en mujeres mayores de 35 aos en la raza blanca, llegando a 50% en la
raza negra. Es la causa mas frecuente de ciruga mayor ginecolgica. La edad peak
es entre los 35-54 aos (90% de los casos) y suelen se mltiples en la mayora de
los casos.

8) COMO HACEMOS EL DIAGNOSTICO CLINICO?
La sospecha diagnostica se puede realizar de forma clnica, basado en una buena
anamnesis, y examen fsico. Cuando el mioma provoca sangrado este respeta el

255
ciclo, producindose una hipermenorrea, no una metrorragia. Pero cabe recordar
que alrededor del 50-80% de los miomas son asintomticos y son hallazgos en
exploracin ginecolgica o en ecografa.
En los casos sintomticos el cuadro clnico depende de la localizacin, tamao y el
numero de miomas.
Hipermenorrea: ms frecuente. Principalmente en miomas submucosos,
provocan menstruaciones ms abundantes en cantidad y das, rara vez
producen sangrados intermenstruales irregulares.
Dolor: crnico y persistente, sensacin de pesadez. Puede presentarse de
forma aguda en torsin de miomas pediculados. En los miomas pediculados
pueden haber dolores por contracciones y dilatacin del cuello (mioma parido)
Sntomas de compresin: el aumento del tamao del mioma puede comprimir
estructuras cercanas: vejiga (poliaquiuria), recto (pujo), urteres e intestino.
Anemia: esta secundaria a la hipermenorrea.

9) QUE NOS INTERESA PREGUNTAR EN LA ANAMNESIS DE ESTA PACIENTE?
Principalmente hay que indagar en los factores de riesgo, morbilidades, uso de
terapias hormonales, historia familiar de miomatosis, menarqua, menopausia,
paridad, gestaciones, etc.

10)QU BUSCARA DIRIGIDAMENTE EN EL EXAMEN FISICO DE ESTA PACIENTE?
El examen fsico en la miomatosis tiene poca manifestaciones, dentro de estas
podemos buscar: masa palpable abdominal, tero aumentado de tamao de
bordes irregulares que se mueven junto con el tero a diferencia de la masa
anexial (la mayora de los miomas > 4 cms son palpables) observacin de mioma
pediculado protruyendo por OCE, hipermenorrea, clnica de sndrome anmico,
dolor a la palpacin abdominal.

10)COMO ESTUDIAMOS ESTE CUADRO Y QUE RESULTADOS ESPERARIA ENCONTRAR EN
ESTOS EXAMENES?
Ante la sospecha por historia y examen fsico, el estudio se puede continuar a
travs de imgenes, idealmente un ecoTV, donde se pueden ver los miomas bien
definidos. Otras imgenes empleadas son la Rx, TAC, y RNM.

11)CUAL ES EL TRATAMIENTO DE ESTA PACIENTE?
La ciruga es el tratamiento definitivo de la miomatosis uterina. Existen otras
alternativas. La conducta y tratamiento a seguir va a depender principalmente de
la sintomatologa del cuadro.
Expectante: en pequeos, asintomticos y durante embarazo, con revisin
peridica.
Tratamiento quirrgico: miomectomia (laparotoma, laparoscopia,
histeroscopia), histerectoma.

256
Embolizacin: se realiza desde la arteria femoral a travs de la arteria uterina
embolizar la irrigacin del mioma reduciendo su tamao.
Tratamiento mdico: anlogos de GnRH, hipoestrogenicos (depo-provera,
lupron, synarel), agentes antiprogestagenos (mifepristona).
Ulipristal , estimulador receptoires de progesterona , reduce sangrado en 70 %
Sintomticos: AINES y antifibrinoliticos (acido tranexamico)

12) QUE CONSECUENCIAS PUEDE TRAER ESTA PATOLOGIA A LA MUJER / MADRE/ Y AL
FETO?
Las consecuencias principales y ms frecuentes de la miomatosis sintomtica es la
anemia secundaria a la hipermenorrea. Adems se observan rara vez alteraciones
reproductivas: aborto, parto prematuro, distocias de presentacin, oclusin
tubaria, infertilidad.
Adems durante el embarazo: crecimiento (20% crecen), degeneracin roja (por el
rpido crecimiento), parto prematuro, anomalas de presentacin, dolor, distocias
dinmicas, retencin de placenta, cesrea, etc.

Bibliografa
- Ginecologa; Prez Snchez, 3ra edicin (2007), Capitulo 22 "Mioma uterino" (H. Odd, P. Villaseca, A. Prez); pag. 396-406

- Manual Obstetricia y Ginecologa; J.A. Carvajal, C. Ralph, 4ta edicin (2013), Capitulo 42 "Miomas uterinos"; pag. 423-435

- Manual Ginecologa y obstetricia CTO; M. Muoz, D.P. Hernndez, 8va edicin (2012), Capitulo 15 "Patologa del cuerpo
uterino y endometrial"; pag. 46-49

- OBSTETRICS, GYNAECOLOGY AND REPRODUCTIVE MEDICINE 23:3; 2013, Review : Modern management of fibroids; M.
Lumsden; pag. 65-70

59. Dolor plvico agudo : EITGS .



Paciente de 30 aos consulta por dolor plvico, ms intenso en FII de 2-3 das de
evolucin y ahora aparece fiebre 38.5C, sensacin de mal estar, nauseas. Refiere
episodios de leucorrea a repeticin. Al examen paciente decada m fiebre 38.5 C, dolor
intenso en pelvis, sudoracin, PA 105/50, BLUMBERG ++, espculoscopia leucorrea
amarillenta, cervicitis con secrecin amarilla, cuello tero doloroso a la movilizacin.
FUR hace 7 das , 17.000 blancos y PCR 200
Antecedentes Familiares: no refiere
Antecedentes Personales: no refiere.
Antecedentes Gineco obsttricos: menarqua 13 aos, RM IV/28 G0P0A0
Hbitos: cigarro 10 al da

1) CUAL ES EL DIAGNOSTICO MAS PROBABLE DE ESTA PACIENTE ?
- Nuligesta
- Dolor pelvico agudo Obs Proceso inflamatorio plvico

257
2) CUALES SON LOS DIAGNOSTICOS DIFERENCIALES?
Embarazo ectopico
Ruptura de quiste ovrico
Apendicitis
SII
ITU
PNA
Colico Renal
Gastroenteritis
Dismenorrea
Endometriosis

3) COMO SE DEFINE ESTA PATOLOGIA?
El nombre correcto de esta patologia es Enfermedad Inflamatoria del Tracto
Genital Superior y corresponde al Compromiso inflamatorio de los rganos
genitales internos femeninos, incluyendo endometrio, trompas, ovarios y
estructuras contiguas, producto de la infeccin por un microorganismo que
asciende desde el crvix o vagina. Dependiendo del la ubicacin de la infeccin el
tipo de PIP, estos pueden ser, en orden ascendente: endometritis, salpingitis,
pelviperitonitis, peritonitis difusa y absceso tubo-ovrico (ATO).

4) CUAL ES LA ETIOLOGIA DE ESTE CUADRO CLINICO?
Es causada por grmenes que generalmente se transmiten a travs de las
relaciones sexuales, las mas frecuentes son: Chlamydia trachomatis seguido de
Neisseria gonorrhoeae. En usuarias de DIU se asocia tambin Actynomices israelii.
Sin embargo en los cultivos del pus del PIP aparecen polimicrobianos,
principalmente aerobios y gran negativos (presentas en las vaginosis bacterianas).

5) CUAL ES LA FISIOPATOLOGIA DE ESTE CUADRO CLINICO?
Normalmente la cavidad uterina es estril, por ciertos factores y situaciones puede
ser colonizada por microorganismos, ingresando desde la vagina y crvix, avanza
rpidamente hasta endometrio y trompas. Por lo general los PIP, son infecciones
polimicrobianas, que son favorecidas en un comienzo por la infeccin de los
agentes etiolgicos ms frecuentes (chalmydia y gonorrea), esto se explica porque
estas infecciones producen un edema, inflamacin, y trombosis de los vasos
sanguneos produciendo un cambio importante en las trompas y cavidad uterina,
que favorece la invasin y proliferacin de los agentes anaerobios y gran negativos
principalmente.
Todo esto provoca un dao inicial por estos agentes tpico de transmisin sexual,
que provoca un cambio en la flora que favorece la entrada de la polimicrobia.

258
Siendo esta la forma ms frecuente de PIP, menos frecuente, pero de relevancia,
son las causa iatrognica, por instrumentalizacin uterina (dilatacin, legrados,
abortos provocados) o el uso de DIUs.

6) QUE FACTORES DE RIESGO BUSCARA?
Edad (joven)
Conducta sexual (cambios de pareja)
*Antecedentes de PIP o infecciones por agentes ms frecuentes
Uso de DIU (4 semanas)
Vaginosis bacteriana
NO uso de MAC barrera
Bajo nivel socioeconmico
Tabaquismo
Coito durante menstruacin o inmediatamente despus
*Ms importante

7) CUAL ES LA EPIDEMIOLOGIA DE ESTE CUADRO?
Enfermedad de prevalencia elevada, siendo el peak entre los 20-24 aos. En USA se
estiman 850.000 nuevos casos por ao, mientras que en reino Unido la prevalencia
es de 1.7% en mujeres entre 16-46 aos. La incidencia del PIP es de difcil
determinacin ya que muchos casos no son identificados y pueden pasar
desapercibidos con manejo ambulatorio sin ser notificados.
La mortalidad es de 1% y su recurrencia es de 25%.

8) COMO HACEMOS EL DIAGNOSTICO CLINICO?
El diagnostico es fundamentalmente clnico, con la presencia de : dolor abdominal
bajo, fiebre, leucorrea, dispareunia, sangrado vaginal anormal, nauseas, vmitos se
puede tener una sospecha diagnostica. Existen criterios diagnsticos generales
para PIP, sin especificar de qu tipo estamos hablando, estos son:
Mayores (al menos 1):
o Dolor hipogstrico
o Dolor anexial
o Dolor a la movilizacin cervical
Menores (aumentan E al estar presentes):
o Secrecin mucopurulenta vaginal o cervical
o T oral >38.3C
o Leucocitosis en frotis de secrecin
o VHS / PCR elevadas
o Infeccin demostrada por agentes ms frecuente

259
9) QUE NOS INTERESA PREGUNTAR EN LA ANAMNESIS DE ESTA PACIENTE?
Habra que hacer una historia detallada para confirmar presencia de factores de
riesgo nombrados anteriormente, principalmente por actividad sexual,
promiscuidad, historias previas de PIP e ITS, vaginosis, MAC (DIU?), tabaquismo,
etc.

10)QU BUSCARA DIRIGIDAMENTE EN EL EXAMEN FISICO DE ESTA PACIENTE?
CSV
El examen fsico puede ayudar a excluir diagnsticos diferenciales, y a definir el
tipo de PIP a la que nos estamos enfrentando.
Examen general: Fiebre, taquicardia, sudoracin, CEG, sintomatologa sptica.
Examen abdominal: Dolor a la palpacin de hipogastrio, signos de irritacin
peritoneales.
Examen ginecolgico: en la especuloscopia podramos observar secrecin
purulenta en vagina y OCE, adems de sangrado. TV: dolor a la movilizacin del
cuello uterino, y a la palpacin de anexos y tero.

10)COMO ESTUDIAMOS ESTE CUADRO Y QUE RESULTADOS ESPERARIA ENCONTRAR EN
ESTOS EXAMENES?
Existen variado exmenes complementarios para el diagnostico de PIP, los cuales
se usaran caso a caso.
Laparoscopia: El gold estndar en el estudio y diagnostico de los PIP, sin embargo
por su costo, complejidad e invasin, por lo que se reserva solo para un
diagnostico incierto, o falla en el tratamiento.
Biopsia endometrial: donde se evidencia histolgicamente la endometritis, tb se
reserva para algunas pacientes por su complejidad e invasin.
EcoTV : muy til en ATO. Se puede observar: trompas engrosadas, fluidos, liquido
libre, ATO, incluso doppler con hiperemia tubaria.
TAC: til para descartar diagnostico diferenciales.
RNM: poca utilidad.
Muy importantes son: Test de embarazo, orina y ecografa abdominal
(diagnsticos diferenciales)
Otros exmenes: estudio de ITS, VHS (>15), PCR, hemograma (leucocitosis). PCR
para chlamydia, cultivo para gonorrea ( PCR), cultivo de muestra endocervical, ph
vaginal, test de aminas.
11)CUAL ES EL TRATAMIENTO DE ESTA PACIENTE?
El tratamiento precoz y adecuada disminuye la prevalencia de secuelas por PIP. El
tratamiento puede ser ambulatorio para las infecciones leves u hospitalizado
.Dentro de las medidas de tratamiento estn:
Soporte vital (hidratacin, monitoreo)
Antibioterapia
o Parenteral (48-72 hrs)
Cefoxitin 2 gr EV c/12 hrs + Doxiciclina 100 mg c/12 hrs VO/EV

260
Clindamicina 900 mg c/8 hrs EV + Gentamicina 3-5 mg/kg/dia +
Doxiciclina
o Oral:
Ceftriaxona 250 mg IM dosis nica + Doxiciclina 100 mg c/ 12 hrs
(+) MTZ 500 mg c/12 hrs
Levofloxacino 500 mg c/24 hrs + MTZ 500 mg c/12 hrs
Amoxi/clavulanico 875/125 mg c/12 hrs + Doxiciclina 100 mg
c/12 hrs
o Ciruga: separar estructuras, liberar adherencias y abundante lavado
con suero. Si hay presencia de un ATO el drenaje es principal
tratamiento y se realiza cuando no responde a tratamiento medico en
48 hras, si es mayor a 8 cm o si la paciente esta embarazada.Todo ATO
roto debe ser oprado

12) QUE CONSECUENCIAS PUEDE TRAER ESTA PATOLOGIA A LA MUJER / MADRE/ Y AL
FETO?
Las secuelas ms importantes del PIP son: dao ciliar tubario, oclusin
tubaria, adherencias, abscesos plvicos, necesidad de reparacin tubaria.
Todo esto provoca a largo plazo: infertilidad, algia plvica crnica,
embarazos ectpicos. Por cada recurrencia de PIP el riesgo de estas
secuelas aumenta 4-6 veces.
Histerectomia y anexectomia biilateral.
Bibliografa
- Ginecologa; Prez Snchez, 3ra edicin (2007), Capitulo 27 "Infecciones del aparato genital femenino" (S. Silva); pag. 482-
505

- Manual Obstetricia y Ginecologa; J.A. Carvajal, C. Ralph, 4ta edicin (2013), Capitulo 52 "Proceso inflamatorio pelviano";
pag. 531-541
- Manual Ginecologa y obstetricia CTO; M. Muoz, D.P. Hernndez, 8va edicin (2012), Capitulo 09 "Infecciones pelvicas";
pag. 30-32
- POSTGRADUATE OBSTETRICS & GYNAECOLOGY 33:15; August 15, 2013; Pelvic Inflammatory Disease; Nathan Webb, MD,
MS; pag. 65-70

60. AMENORREA : PRIMARIA.



Mujer de 17 aos quien consulta por no haber tenido nunca una menstruacin.
Antecedentes Familiares: madre HTA, Padre dislipidemia
Antecedentes Personales: Naci por PTVE, hernia inguinal operada a los 3 aos,
sin enfermedades.
Antecedentes Gineco obsttricos: G0P0A0N menarquia no, No refiere dolor
plvico
Hbitos: no
1. Cul es el diagnstico ms probable de esta paciente?

261
Nuligesta
Amenorrea primaria

2. Cules son los diagnsticos diferenciales?
Los principales diagnsticos diferenciales son amenorrea fisiolgica, como la de
embarazo ya que es la principal causa de amenorrea en mujeres en edad frtil; o
una amenorrea secundaria,

3. Cmo se define esta patologa?
Amenorrea primaria se define como la ausencia de menstruacin en una mujer que
nunca ha presentado menarquia:
- En mayores de 16 aos con la presencia de caracteres sexuales secundarios.
- En mayores de 14 aos sin desarrollo de caracteres sexuales secundarios.

4. Cul es/son la/s etiologa de este cuadro clnico?
Tiene mltiples etiologas:
- Con desarrollo de caracteres sexuales secundarios (30%)
o sndrome de insensibilidad a los andrgenos (9%), agenesia Mlleriana
(10%), obstruccin al tracto de salida (1-2%), constitucional (8%).
- Sin desarrollo de caracteres sexuales secundarios:
o Hipogonadotrofico (30%): constitucional (10%), prolactinoma (5%),
Sndrome de Kallman (2%), otras causas de SNC (3%), baja de peso,
ejercicio (3%), otros (4%)
o Hipergonadotrfico (40%): Falla ovrica prematura (15%), Sndrome de
Turner (20%), 46XY (5%)

5. Cul es la fisiopatologa de este cuadro?
- Agenesia Mlleriana: se presenta en 1/5.000 mujeres, es causada por mutaciones
genticas espontnea que altera la diferenciacin de los conductos de Mller
durante la embriognesis. Estas pacientes presentan ausencia de tracto vaginal
superior y un tero hipo o aplasico. La funcin ovrica es normal, por lo que
desarrollan caracteres secundarios normales. Este sndrome puede presentarse de
manera aislada (tipo 1) o asociado a defectos cardiacos, auditivos, renales o
vertebrales (tipo 2). El tratamiento consiste en intentar restaurar la funcin sexual
normal con dilatacin de la vagina (xito en 88%) o reconstruccin quirrgica.
- Sndrome de insensibilidad a los andrgenos: incidencia 1/60.000 y es causado
por una mutacin en el gen del receptor de andrgenos, los afectados tienen
genotipo 46XY y fenotipo femenino. Las gnadas son testculos por lo que secretan
factor inhibidor Mlleriano in tero que lleva a la regresin de estructuras
femeninas, estos testculos son incapaces de producir espermatognesis, pero son
capaces de producir testosterona que al estar alterado el receptor de andrgenos
se produce aromatizacin (produccin de estrgeno) por lo que se desarrollan
caracteres sexuales secundarios. El tratamiento va a depender del grado de

262
insensibilidad a andrgenos, las gnadas se pueden extirpar luego del desarrollo
puberal y usar TRH. Esta mujer es infrtil y requiere apoyo psicolgico.
- Alteracin del tracto de salida: estas pacientes presentan amenorrea primaria y
dolor abdominal cclico debido al descame endometrial mensual que, al estar
bloqueada la salida produce hematocolpos y hematmetra. Pueden presentar
retencin urinaria. El tratamiento es la reseccin quirrgica del defecto, ya sea
himen imperforado o septo vaginal transverso. Y no representa ninguna
consecuencia para la mujer una vez solucionado.
- Retardo constitucional: se presenta con un crecimiento lineal retardado durante
los primeros 3 aos de vida el patrn de crecimiento se adquiere de los padres, por
lo que la historia familiar es importante. El retraso de la pubertad se asocia con
baja estatura, pero el desarrollo suele ser adecuado para la edad sea, que se logra
estimar con radiografa de mano. Se deben descartar otras causas de amenorrea.
No requiere tratamiento.
- Sndrome de Turner: se diagnostica con cariotipo 45XO o mosasmo. La incidencia
vara de 1/2.000 a 1/5.000 y es causado por errores en la meiosis paterna que lleva
a la ausencia de cromosoma X. Junto con este cromosoma se pierde el gen 5HOX
que es el encargado del crecimiento de huesos largos, por lo que estas mujeres son
de baja estatura. Hay poco desarrollo ovrico, por lo que no suelen desarrollar
caracteres sexuales secundarios. Las caractersticas de estas pacientes son talla
baja, alteracin anatmica de genitales internos como externos, malformaciones
cardiacas y renales, cuello alado, alteracin en la alineacin de huesos de
antebrazos, pezones separados, implantacin baja del cabello. El tratamiento
consiste en TRH para el desarrollo normal de caracteres secundarios y Hormona
del crecimiento para alcanzar una estatura adecuada. La mayor repercusin de
estas pacientes es a nivel de fertilidad, ya que por el desarrollo rudimentario de
ovarios, no presentan ovocitos.
- Falla ovrica prematura: son pacientes 46XX con alteraciones genticas
localizadas, ms del 60% de los ovarios no son detectables, sin crecimiento
folicular, con niveles de FSH en rangos menopasicos. Se cree que hay alteraciones
a nivel de receptor de FSH. El tratamiento es TRH para desarrollo de caracteres
sexuales secundarios y menstruacin. La fertilidad es un gran tema, pero por lo
general no se logra.
- Hipotalmico funcional: el estrs, ejercicio, alteraciones alimentarias, dficit
nutricional pueden causar amenorrea por alteracin de la secrecin de GnRH. Estas
pacientes cursan con disminucin de flujos menstruales, por un hipogonadismo
hipogonadotrfico, sin alteracin orgnica. Que lleva a hipo-estrogenemia, que si
es prolongado puede llevar a dficit de masas sea. El tratamiento es revertir la
causa, subiendo de peso, nuevos hbitos alimentarios, disminuir el estrs, requiere
apoyo psicolgico/psiquitrico.
- Hipotalmico estructural: trauma craneano o lesin ocupante de espacio de
hipotlamo como como craneofaringioma, glioma, etc. Alteran el patn de
secrecin de GnRH. Estas pacientes suelen presentar sntomas de lesiones

263
ocupantes de espacio como cefalea, alteraciones visuales. El diagnstico se logra
con imgenes y el tratamiento suele ser quirrgico.

De acuerdo a la OMS, el origen de la amenorrea primaria reside en :

264

6. Qu factores de riesgo buscara?
No hay factores de riesgo claros para esta patologa. Si hay cierta historia familiar
que conlleva a un retraso constitucional de la menarquia.

7. Cul es la epidemiologia de este cuadro?
La amenorrea primaria se presenta hasta en el 1-5% de las mujeres.

8. Cmo hacemos el diagnstico clnico?


Figura 1.

As, en la primera visita sera necesario realizar (requerimientos mnimos): -
Anamnesis y exploracin. - Test de embarazo, FSH, LH, E2, PRL, TSH - Test de
gestagenos (opcional)
.
En la segunda visita:
- Obtener, en funcin de pruebas anteriores, una orientacin diagnstica.
- Solicitar estudios complementarios (requerimientos opcionales).
Para en la tercera visita:
- Obtener un diagnstico final.

265
- Plantear un plan teraputico, en funcin de si la paciente tiene o no
deseo gensico.








9. Qu nos interesa preguntar en la anamnesis de esta paciente?
Determinar si es sexualmente activa , si practica ejercicio extenuante, hbitos
alimenticios, aumento o disminucin de peso, eventos estresantes e historia
anticonceptiva. Desarrollo de caracteres sexuales secundarios, como desarrollo
mamario y vello pbico y axilar. Otros sntomas orientadores como dolor
abdominal, cefalea, alteraciones visuales, galactorrea, hirsutismo y sntomas
vasomotores. Antecedentes de enfermedades sistmicas, como hipotiroidismo y
uso de frmacos como antipsicticos, antiepilpticos. Uso de drogas como
herona, cocana. Y antecedentes familiares de menarquia o menopausia precoz.

10. Qu buscara dirigida mente en el examen fsico de esta paciente?

266
Evaluacin de caracteres sexuales secundarios, segn escala de Tanner, evaluar
peso, talla, IMC, tiroides, galactorrea, hiperaldosteronismo, evaluar cltoris por
signos de virilizacin. Evaluacin de diferentes sistemas y tracto urinario segn
historia obtenida. Examen ginecolgico solo debe realizarse inspeccin ya que
estas pacientes suelen ser jvenes sin actividad sexual, por posible obstruccin del
tracto de salida. Puede realizarse tacto rectal para descartar embarazo o lesin
tumoral.

11. Cmo estudiamos este cuadro y que resultados esperara encontrar en estos
exmenes?

Test de orientacin diagnstica

Actualmente se pone en duda la sistematizacin de estos tests diagnsticos.
Pueden ser de ayuda, pero no se consideran imprescindibles. Probablemente
deberan formar parte de los requerimientos opcionales, y no de los mnimos.

o Test de progesterona

Valora si el nivel de estradiol endgeno es suficiente para proliferar el
endometrio y producir una hemorragia por deprivacin si el tracto genital est
intacto. Se administra un ciclo de progestgeno carente de actividad
estrognica (progesterona natural micronizada 200-400 mg/da o acetato de
medroxiprogesterona 10 mg/da, 5 das). En los 2-7 das siguientes a la
interrupcin del progestgeno, la paciente sangra (test positivo) o no (test
negativo).

- Test negativo: Se puede establecer con seguridad el diagnstico de un
defecto de los sistemas de la va de salida (amenorrea genital).
La realizacin de los tests clnicos no excluye la necesidad de solicitar
anlisis basal de FSH, LH, PRL y E2 que son imprescindibles para el estudio
de las amenorreas, por lo que es recomendable efectuar en primer lugar estas
determinaciones para evitar interferencias en sus resultados

Test positivo: Se establece con fiabilidad y seguridad un diagnstico de
anovulacin. Se confirma la presencia de una va de salida funcional y un
endometrio reactivo suficientemente preparado por los estrgenos endgenos.
Con ello se establece una funcin mnima del ovario, la hipfisis y el sistema
nervioso central. Con determinaciones hormonales normales, no sera necesario
continuar con la evaluacin.

Test de estrgeno -gestagenos:

Est indicado cuando el test de gestagenos es negativo. El objetivo es

267
determinar si la falta de menstruacin es debida a alteracin en el tracto de
salida o a falta de estrgenos por alteracin gonadal o central. Se administran
preparados secuenciales que contengan estrgenos y progesterona (ACHO) o
estrgeno en cantidad y duracin suficientes para estimular la proliferacin del
endometrio (3 semanas) seguidos de progesterona que permita la hemorragia
por deprivacin (5-10 das); por ejemplo 2,5 mg/da de estrgenos conjugados
+ 10 mg/da de medroxiprogesterona, o 2 mg/da de valerato de estradiol + 200
mg/da de progesterona natural micronizada.


- Test positivo: Confirma un tracto genital intacto, y explicara la
existencia de un estado hipoestrognico como consecuencia de
anomalas a nivel gonadal o a nivel hipotlamo-hipofisario. En funcin de
los valores de gonadotropinas, se establecer el diagnstico de una
amenorrea gonadal (Hipogonadismo hipergonadotropo) o amenorrea
central (Hipogonadismo hipogonadotropo).
- Test negativo: Se puede establecer con seguridad el diagnstico de un
defecto de los sistemas de la va de salida (amenorrea genital).


2. Requerimientos opcionales

Test de estimulacin de GnRh
Tiene como objetivo descartar una lesin hipofisaria como causa del
Hipogonadismo hipogonadotropo. Se administran 100g de GnRh en bolus
endovenoso determinando la respuesta de FSH y LH a los 30 y 60 minutos.
- Test positivo: Se doblan los niveles de gonadotropinas tras el estmulo.
Se confirma con ello la integridad hipofisaria.
- Test negativo: Se confirma que la causa del Hipogonadismo radica en la
hipfisis.
La utilidad del test es controvertida, ya que la alteracin pura de la hipfisis
es poco frecuente y existe una tasa considerable de falsos negativos, debido a
que la hipfisis precisa para su correcto funcionamiento la impregnacin previa
de GnRh y estrgenos. Por tanto un test positivo confirma la integridad
hipofisaria, pero un test negativo obliga a proseguir estudio de la hipfisis,
probablemente con neuroimagen 5
.

12. Cul es el tratamiento de esta paciente?
El tratamiento depende de la causa especfica de la amenorrea, y se detallan en
cada una.

13. Qu consecuencias puede traer esta patologa a la mujer / madre/ y al feto?

268
Las consecuencias van a depender de la causa especfica de la amenorrea. Algunas
como las de obstruccin del tracto de salida, o hipogonadismo hipogonadotrofico
por estrs o baja de peso, una vez resultas no representan complicaciones. Pero
Sndrome de Turner, Sndrome de insensibilidad a los andrgenos agenesia
Mlleriana y falla ovrica presentan grandes problemas de fertilidad en el futuro.
Referencias
1. Investigation and treatment of primary amenorrhoea. Obstetrics, gynecology
and reproductive medicine. Tim Child. Figura 1
2. Investigation and treatment of primary amenorrhoea. Obstetrics, gynecology
and reproductive medicine. Sonela Basak

61. AMENORREA :SINDROME OVARIO POLIQUISTICO



Mujer de 30 aos que consulta por esterilidad primaria de 1 ao y medio de evolucin.
Estudio de factor masculino es normal y la Histerosalpingofrafia es normal. Peso 100 kg ,
1,50 mts IMC 44 , Hirsutismo facial , tambin en areolas mamarias , linea media
umbilical y muslos . Acn facial y en la espalda.
Antecedentes Familiares : Madre diabtica
Antecedentes Personales : Refiere exceso de vellos siempre , se trato con ACO
Antecedentes Gineco obsttricos : menarquia 11 aos , RM VII/60 90 G0P0A0
debe tomar progesterona o ACO para menstruar.
Hbitos : cigarro 10 al da.

1. Diagnstico:

Nuligesta
Infertilidad primaria
Sangrado menstrual infrecuente irregular m probable Sndrome Ovario
Poliquistico.
Hirsutismo
Obesidad
Tabquica

2. Diagnsticos diferenciales:

El diagostico diferencial debe establecerse con otros cuadros clinicos que suelen
presentar la asociacion de hirsutismo y trastornos menstruales como:

Hiperplasia supraadranal congenita
Tumores virilizantes

269
Hiperprolactinemia
Sindrome de cushing
Acromegalia
Uso de drogas como esteroides anabolicos y acido valproico

3. Definicin:

El SOP se define como el sndrome que presenta un hper androgenismo asociado a
anovulacin crnica sin otra causa especifica de enfermedad adrenal o hipofisaria
que cursa con irregularidades menstruales o exceso de andrgenos, adems de la
presencia de ovarios poliquistico en la ultrasonografa.

Criterios diagnsticos

Los criterios mas recientes para diagnostico de SOP se basan en
combinaciones de hiperandrogenismo clnico o bioqumico, evidencia de
anovulacin y ovarios poliquistico. Estas combinaciones determinan 4 tipos de
fenotipos presentes en estas pacientes :

Fenotipo 1 : Hiperandrogenismo Oligo-anovulacin -- Ovario Poliquistico
Fenotipo 2 : Hiperandrogenismo con anovulacin
Fenotipo 3 : Hiperandrogenismo con Ovario poliquistico ( sin disfuncin ovrica )
Fenotipo 4 : Oligoanovulacion con Ovarios poliquistico.


En adolescentes se sugiere diagnostico con :

- Hiperandrogenismo , preferentemente confirmado por laboratorio
- Sangrado menstrual irregular para la edad de la paciente .



4. Etiologa:

El SOP tiene una base gentica y puede comprometer varios miembros de la
familia, por lo que es considerada una enfermedad familiar multigenica compleja.
Entre los factores ambientales destacan la obesidad y los eventos que ocurren en

270
la vida intrauterina (hiperandrogenismo, diabetes gestacional, y sobrepeso de la
madre durante el embarazo). Por lo tanto es muy importante el manejo adecuado
de la embarazada ya que se sugiere que existe una relacin entre el ambiente
prenatal y el riesgo de desarrollar enfermedades metablicas durante la edad
adulta.

5. Fisiopatologa:

La fisiopatologa destaca al menos tres tipos de alteraciones interrelacionadas
entre si: (1) disfuncin neuroendocrina (hipersecrecin de LH con perdida de su
pulsatilidad ), (2) un trastorno metablico (resistencia insulinica e hiperinsulinemia)
y (3) una disfuncin de la Esteroidognesis y de la foliculogenesis ovrica.
a. Disfuncin endocrina: Se caracteriza por un aumento de la secrecin de LH y
una secrecin de FSH normal o disminuida. En estas pacientes se ha observado
un aumento de la amplitud y frecuencia de los pulsos de LH, lo que reflejara un
aumento de los pulsos de factor liberador de gonadotrofinas (GnRH). No se ha
identificado alteraciones en neurotransmisores especficos que expliquen este
trastorno y las evidencia actuales sugieren que se tratara probablemente de
una disfuncin hipotalmica secundaria a los niveles elevados de andrgenos e
insulina.
b. Disfuncin metablica: Est representada principalmente por una RI perifrica
que se expresa por una hipersecrecin de insulina. Esta a su vez, promueve una
mayor secrecin de andrgenos por el ovario y las suprarrenales; estimula la
secrecin de LH y adems disminuye la sntesis heptica de SHBG con lo cual
aumenta la fraccin libre y actividad biolgica de los andrgenos. Segn
estudios, la hipersecrecin de insulina se manifiesta desde la pubertad
temprana y precede al hiperandrogenismo bioqumico.
En la minora de los casos (20 30%), el SOP puede manifestarse sin RI, lo que
se debera por ser una enfermedad multigenetica compleja no siempre se
heredan conjuntamente genes asociados a RI con genes asociados a disfuncin
reproductiva.
c. Disfuncin de la esteroidogenesis ovrica/suprarrenal: Es el pilar fundamental
y se caracteriza por una alteracin de la biosntesis de los andrgenos, la cual
tanto el ovario como en la suprarrenal esta determinada por la actividad de
una enzima denominada citocromo P450c17. En pacientes con SOP la actividad
de esta enzima esta aumentada, lo que lleva una mayor produccin de
andrgenos ovricos y adrenales. El aumento de los andrgenos intraovaricos,
alteran el desarrollo de los folculos y la ovulacin. Se ha propuesto que la
disfuncin de esta enzima (P450c17) seria exclusiva del SOP pudiendo ser un
evento primario o secundario al exceso de LH y/o insulina. Adems, cabe
destacar que el tejido adiposo juega un papel preponderante en la
fisiopatologa del SOP ya que tiene una funcin esteroidogenica intrnseca y es
un tejido blanco para los andrgenos.

271
d. Disfuncin de la foliculogenesis: Se ha podido establecer gracias a la
ultrasonografa y biopsias ovricas que pacientes con SOP presentan un pool de
folculos en crecimiento 2 a 3 veces superior que las mujeres sanas. Por lo que
en el SOP habra mayor reclutamiento y una menor seleccin, lo que mantiene
un pool de folculos en crecimiento productores de andrgenos.

6. Epidemiologia:

Prevalencia del 7 10%. Existen regiones ms afectadas, siendo infrecuente en
pases nrdicos y ms frecuentes en mediterrneo. En estados unidos hay una
prevalencia del 4% y en Europa alrededor del 6 7%. Se observa una mayor
incidencia en poblaciones con mayor prevalencia de RI .Es la causa mas frecuente
de infertilidad en la mujer .

7. Diagnstico:

El diagnstico se basa en la combinacin de sus caractersticas clnicas, bioqumicas
y ultrasonografas. El diagnostico debe ser planteado clnicamente y confirmado
bioqumicamente. La eco sugerente de SOP por si sola no permite establecer el
diagnstico y su normalidad no lo descarta.
EL diagnstico clnico se hace con los criterios mencionado anteriormente. Pero el
cuadro clnico es muy polimorfo. Por lo general las manifestaciones clnicas se
inician en el periodo perimenarquico con la aparicin de alteraciones menstruales
en 2/3 de las adolescentes principalmente sangrado menstrual infrecuente
(sangrados > 45 das o menos de 9 sangrados al ao) alternados con periodos de
amenorrea secundaria, las que cursan con buen nivel de sangrado y responden a la
administracin de progesterona. Ocasionalmente se presentan episodios de
metrorragia disfuncional por hiperplasia endometrial.
Los trastornos menstruales se asocian frecuentemente a la obesidad por lo
general de tipo androide y a manifestaciones del hiperandrogenismo siendo el
hirsutismo la manifestacin ms clsica. El hirsutismo es leve a moderado (entre 8
y 15 puntos en la escala de Ferriman y Gallowey), se presenta en 2/3 de las
pacientes, aparece en la pubertad y progresa lentamente o se detiene alcanzada la
madurez sexual. La virilizacion es rara y su presencia debe hacer sospechar otras
etiologas como tumor secretor de andrgenos o una hiperplasia adrenal
congnita. Entre las manifestaciones cutneas solo el hirsutismo, acn y alopecia
han sido aceptadas en los consensos como criterios diagnsticos de
hiperandrogenismo. La obesidad esta presente en alrededor de la mitad de las
pacientes y es tpicamente androide o forma de manzana (ndice cintura/cadera
> 0.85); por lo general se inicia en la niez y se acenta en la pubertad. En estas
pacientes puede observarse acantosis nigrans como marcador de RI. Adems estas
pacientes adquieren un aspecto cushingoideo (obesidad central, dorso de bfalo y
aumento de la grasa para clavicular) o acromegaloideo (aumento del grosor de los
rasgos faciales por efecto trfico de la insulina, pero sin prognatismo). El cuadro

272
clnico no regresa espontneamente, con el tiempo van cambiando las
manifestaciones fenotpicas. Es decir durante la post menarquia y edad
reproductiva predominan alteraciones reproductivas, mientras que las peri
menopasicas se acentan las alteraciones metablicas.

8. Examen fsico:

Debe ponerse especial atencin en las manifestaciones clnicas del
hiperandrogenismo e hiperinsulinismo. Para consignar la distribucin del vello se
utiliza el score de Ferriman y Gallwey. Debe consignarse el acn, la alopecia
andrognica y la acantosis nigricans. Importante calcular IMC y la distribucin de la
grasa
Es aconsejable completar el examen fsico con medicin de PA, examen de tiroides,
mamas y la inspeccin de genitales externos buscando una clitorimegalia, para
terminar finalmente con un examen ginecolgico en mujeres sexualmente activas.
Recordar que cuadro clnico es polimorfo, por lo tanto no todas la mujeres
presentaran todos los sntomas

273



9. Estudio de la paciente:

Las paciente con SOP tienen andrgenos elevados de leve a moderado, aunque
algunas determinaciones caen en rango normal y ello no significa que no tengan la
patologa.

Testosterona total: andrgeno circulante ms importante en la mujer y es
el principal causante del hirsutismo. No obstante en el SOP la testosterona
total solo se eleva discretamente en el 50% de los casos. Por esta razn el
consenso de Rotterdam sugiere medir el ndice de andrgenos libres (IAL),
el cual consiste en la relacin de testosterona total y su protena
transportadora, de acuerdo a la siguiente formula: testosterona / SHBG x
100 (valor normal < 4.5). Para transformarla testosterona en ng/ml a
nmol/L debe multiplicarse por 3.467.
Dehidroepiandrosterona sulfato (DHEAS): Esta hormona se origina
exclusivamente en las suprarrenales por lo que se utiliza como marcador de
hiperandrogenismo suprarrenal. Aproximadamente entre 25 40% de
estas pacientes pueden presentar un aumento de la concentracin srica
de DHEAS, pero raramente > 600 ng/dl.
Androstenediona: Andrgeno fundamental de origen ovrico y puede ser
el nico andrgenos elevado en una mujer con SOP y se mantiene elevado
hasta etapas tardas de la transicin menopasica. Aunque no es un
andrgeno de primera lnea, puede determinarse en caso de duda
diagnostica.
17 hidroxiprogesterona (17-OHP): Es el mejor metabolito para descartar
dficit de la enzima 21 hidroxilasa; valor normal en ayunas en fase
folicular temprana del ciclo menstrual es < 2 ng/dl. Valores > 6 son
indicadores de bloqueo enzimtico. Concentraciones 2 y 4 hacen necesario
efectuar un test de ACTH. El cual consiste en administras ACTH Ev 0,25 ug

274
(valores > a 10 a los 60 minutos post carga establecen el diagnostico)
Alrededor el 50% de las pacientes con SOP pueden presentar elevaciones
leves de esta hormona.
Relacin LH/FSH: Generalmente pacientes SOP tienen una relacin LH/FSH
aumentada (> a 2). Originalmente se consider un marcador de SOP, pero
debido a que su normalidad no descarta el diagnostico, en la actualidad
solo es un elemento orientador.
ndice de Andrgenos Libres (IAL) : es una formula matemtica que nos
dice cuanto andrgenos circula libre
Perfil Lipdico .

Debe completarse el estudio hormonal con la toma de prolactina y
hormonas tiroideas como alteraciones que cursan con irregularidad
menstrual.

Evaluacin del componente metablico: La mayora de las pacientes con
SOP presentan anormalidades metablicas como RI con hiperinsulinemia
compensatoria, obesidad y dislipidemia, por lo que se aconseja solicitar
exmenes para toda ellas.
Se solicita perfil lipdico, test tolerancia a la glucosa con medicin de
insulina.


Valores sugerentes de RI Test HOMA: modelo matemtico para
Nias hasta 2 aos postmenarquia estudiar RI en una poblacin y no para
establecer RI en un individuo determinado,
Insulinemia basal >18 uU/ml debido a que su normalidad no permite
Insulinemia postcarga >100 descartar RI
uU7ml Pero en la realidad si se usa.
Nias postmenarquicas tardas (>2
aos)
Insulinemia basal >12.5
uU/ml
Insulinemia postcarga >60 uU/ml

Si la paciente presenta un IMC mayor a 35 o signos de acantosis nigricans,
asumimos que tiene la RI y no es necesario solicitar exmenes.

Ultrasonografa: Especialmente la TV es til para detectar morfologa de
ovarios poli qusticos. En mujeres sexualmente maduras el volumen del ovario
alcanza aprox. 6cc y no excede los 8cc cuando contiene un cuerpo lteo o
folculo maduro. En pacientes con SOP el volumen es mayor, > 10cc. No
obstante un 30% de las pacientes pueden presentar ovarios normales.
Criterios ecogrficos para definir SOP: NO APLICA A MUJERES USUARIAS
DE ACO

275
o Presencia de 12 o ms folculos en cada ovario que midan entre 2
9 mm de dimetro.
o y/o volumen ovrico aumentado, >10cc en fase folicular temprana
o Si hay evidencia de un folculo dominante (>10 otros dicen >18cc) o
un cuerpo lteo, debe repetirse el examen el prximo ciclo.













El estroma ovrico no esta considerado en la definicin de SOP. No obstante, cabe
destacar que hasta un 94% de los casos de SOP presentan aumento de la
ecogenicidad ovrica.
El SOP suele confundirse con los ovarios multifoliculares los que se observan como
ovarios aumentados de volumen con varios folculos en desarrollo hasta 9 mm sin
dominancia y que aparecen dispersos en el estroma ovrico. Se presenta durante
le desarrollo puberal y despus de la reanudacin de la ciclicidad ovrica que sigue
a una fase de amenorrea (lactancia, pubertad).

10. Tratamiento:

Est orientado a corregir el hiperandrogenismo, los trastornos menstruales, las
alteraciones metablicas asociadas (RI y obesidad) y la ovulacin, en los casos que
la mujer quiera embarazarse.
Tratamiento de las manifestaciones clnicas del HA y de la hiperandrogenemia.

Los mtodos que se utilizan para el manejo adecuado del hiperandrogenismo
estn dirigidos a: inhibir la esteroidogenesis ovrica, suprarrenal o ambas,
aumentar la concentracin de SHBG, evitar la accin de andrgenos en tejidos
blancos, bloqueando la unin a sus receptores e inhibir la conversin perifrica
de testosterona en dihidrotestosterona y por ultimo actuar directamente sobre
los tejidos, utilizando recursos cosmticos


Dieta hipoglucidica y ejercicio : pilares del tratamiento.

276
Los anticonceptivos orales: Son considerados de primera lnea en la
teraputica para mujeres en edad reproductiva, estos frmacos suprimen la
secrecin de LH y, por lo tanto disminuyen la biosntesis de andrgenos
ovricos, aumentan la concentracin plasmtica de SHBG disminuyendo los
andrgenos libres y adems permite una descamacin regular del
endometrio, con el que se evita el riesgo de hiperplasia endometrial y
cncer de endometrio. El inconveniente de los ACO es que pueden
deteriorar la RI y aumentar la sntesis heptica de triglicridos, lo que
depender el tipo de progestinas que contenga. Las progestinas que tienen
actividad andrognica son el norgestrel y levonorgestrel que no son
recomendadas, porque exacerban las manifestaciones cutneas del
hiperestrogenismo. Otras progestinas como el acetato de ciproterona, la
drospirenona, el acetato de clormadinona y el dienogest tienen efecto anti
andrognico. De estas, el acetato de ciproterona, por su actividad
glucocorticoide, tienen ms efecto anablico y puede producir aumento de
peso corporal y de la RI por lo cual su uso no debe exceder los 6 meses. Las
progestinas ms recomendadas son, la drosperinona, al ser derivado de la
espironolactona, tendra un efecto beneficioso sobre la RI y el dienogest y
acetato de clormadinona tendran la capacidad de reducir la actividad de la
alfa reductasa a nivel de la piel.

Las pacientes que ms se benefician de ACO son aquellas con anovulacin
crnica y niveles elevados de andrgenos y LH.

Los anti andrognicos: Son compuestos de tipo esteroidales, como el
acetato de ciproterona y la espironolactona, o no esteroidales como la
flutamida y el finasteride, que antagonizan el receptor de andrgenos en el
folculo piloso y la glndula sebcea. La eleccin del anti andrognico
depende de cada caso en particular. Los dos primeros son muy efectivos en
suprimir la hiperandrogenemia, mientras que los siguientes son efectivos
como bloqueadores perifricos de la accin andrognica pero no modifican
el nivel de andrgenos.
Manejo de la oligo-ovulacion crnica: En primer lugar debe corregirse la
obesidad, ya que esta demostrado que esta sola medida disminuye los
niveles e insulina, testosterona y LH, permitiendo la reanudacin
espontanea de la ciclicidad ovrica y de la ovulacin. En pacientes que no
menstran espontneamente y que no desean embarazo se puede utilizar
progestinas en dosis bajas en forma cclica o ACO, con el fin de lograr una
proteccin endometrial. En pacientes que deseen embarazo esta indicado
llevar a cabo una induccin de ovulacin
o Se ha descrito que pacientes con SOP tienen hasta 3 veces mas
riesgo que la poblacin general en desarrollar hiperplasia
endometrial y cncer de endometrio, lo que se debe a:
hiperandrogenismo relativo por ausencia de ovulacin, presencia de

277
factores de crecimiento e inflamatorios por obesidad, RI, diabetes y
nuliparidad. (Las mujeres que presentan amenorrea, metrorragia
disfuncionales y engrosamiento endometrial a la eco deben ser
vistas por especialistas)
Metformina: disminuye los niveles de insulina y LH, por lo que aumentan
los niveles de SHBG y tiene beneficios al ayudar a la reduccin de peso.
Recomendado 1500 2000 mg
Deseo de embarazo : Debe asociarse un inductor de ovulacin (
Clomifeno ) con metformina, adems de la dieta y ejercicio, pilares
fundamentales del tratamiento
Cuando la paciente esta recibiendo ACO , no tiene sentido realizar
ecografas TV de control ni medir niveles hormonales .
El tratamiento debe mantenerse hasta que la paciente logre bajar a su peso
ideal o dese embarazarse


11. Consecuencias para la paciente y el feto:

Los riegos a largo plazo del hiperestrogenismo relativo (por falta de ovulacin), se
asocia a cncer endometrial y de la hiperinsulinemia crnica, diabetes mellitus 2 y
sndrome metablico. De la mano con lo anterior aumenta el riesgo de
enfermedad cardiovascular e hipertensin.
Por otro lado, las mujeres con SOP que logran embarazarse tienen mayor riesgo
aborto, de diabetes gestacional, SHE, preclampsia, parto prematuro y recin
nacidos pequeos para la edad gestacional.

Bibliografia:
Polycystic ovarian syndrome, nina madnani et al. departaments of dermatology
and endocrinology P.D. Hinduja National Hospital, Mahim, Mumbai, India
Sndrome ovario poliquistico. Diagnstico y manejo, rev. Med. Clin. Condes 2013;
24(5) 818 826

62. AMENORREA SECUNDARIA : ANOVULACION


HIPOTALAMICA.

Mujer de 16 aos que consulta por amenorrea de 9 meses de evolucin. Actividad
sexual sin proteccin 2 test () embarazo ayer, no ingiere medicamentos.

278
1,68 mts 46 Kgs IMC 16. Ferriman 3 .Genitales externos normales y desarrollo de
caracteres sexuales normales.
Antecedentes Familiares: no refiere.
Antecedentes Personales: apendicetoma a los 9 aos.
Antecedentes Gineco-obsttricos: menarquia 13 aos, ciclos regulares 6 meses
despus con ciclos de VII/25 FUR hace 9 meses.
Hbitos: cigarro 10 al da. Alcohol social, footing 1 hora 3 veces por semana
Desarrollo:
1. Diagnstico apropiado

Amenorrea secundaria
Amenorrea hipotalmica
Nuligesta.

2. Diagnstico diferencial:
Hipotiroidismo
SOP
Hiperprolactinemia
falla ovrica (Causas ms frecuentes)

3. Definicin: Ausencia temporal o permanente del flujo menstrual por un
periodo mayor de 90 das en una mujer que ya ha menstruado por 3 ciclos.
Clasificacin como momento de presentacin:
- Primaria: Falta de flujo menstrual en ausencia de caracteres sexuales
secundarios a los 14 aos. O falta de flujo menstrual a los 16 aos en
presencia de desarrollo de caracteres sexuales secundarios.
- Secundaria: Falta de flujo menstrual luego de la menarqua.
Clasificacin significancia clnica:
- Fisiolgica: Embarazo, premenrquica, postmenopusica.
- Patolgica: Funcional u orgnica.

4. Etiologa:

279


5. Epidemiologa: 3- 5 % de la poblacin general, 80 100 % en atletas de alto
rendimiento.

6. Fisiopatologa: (Amenorrea hipofisaria)
Estrs: secrecin de opioides endgenos y CRH que aumenta el
cortisol, ambos inhiben la secrecin de gonadotrofinas, a travs de una red
de seales.

7. Anamnesis:
- Evaluar inicio y duracin de amenorrea
- Consultar por antecedentes personales, infecciones previas, etc.
- Desarrollo e historia reproductiva
- Antecedentes familiares de anomalas genticas
- Consultar ocupacin y estado emocional
- Consultar por nutricin y actividad fsica
- Sntomas acompaantes: Presencia de galactorrea.

280
- Frmacos y drogas

8. Examen fsico:
- Peso y talla
- Evaluar genitales externo e internos
- Espculoscopia observar cuello uterino
- Tacto vaginal: buscar masas.

9. Estudio de la paciente

Puede clasificarse en : Amenorrea Normogonadotropica
Amenorrea Hipogonadotropica
Amenorrea Hipergonadotropica
Amenorrea normogonadotrfica:

Anovulacin hiperandrognica
SOP Enfermedad tiroidea
Andrgenos exgenos
Obstruccin del tracto de salida:
Sndrome Ashermann
Estenosis cervical


Hipogonadismo hipogonadotrfico:

Amenorrea hipotalmica:
Enfermedad crnica
Anorexia nerviosa
Bulimia
Ejercicio excesivo
Excesiva prdida de peso / malnutricin
Radiacin craneal Tumor del SNC
Destruccin hipotlamo / hipfisis
Sndrome Sheehan


Hipogonadismo hipergonadotrfico:

Falla ovrica prematura:

281
Autoinmune
Gentica
Quimioterapia
Radiacin plvica
Falla ovrica postmenopusica
Disgenesia gonadal
Sndrome Turner
Cariotipo en mujer menor de 30 aos



10. Tratamiento y conducta:

282
Objetivo: Restauracin de los ciclos anovulatorios y prevencin de las
consecuencias a corto y largo plazo (osteoporosis, hiperplasia endometrial,
enfermedad cardiaca, infertilidad).



- Tratamiento integral: psicolgica, psiquitrica, nutricionista
- Intervencin familiar
- Reinicio de alimentacin saludable y cese del deporte extremo.
- El tratamiento es el revertir cualquiera de estas entidades extremas y
una vez que se restablece la salud, los ciclos menstruales se presentan
dentro de los siguientes tres meses.

- Terapia hormonal: Limitados beneficios porque no promueven la
recuperacin del eje HHO. En general pareciera que el uso de ACO
puede prevenir adicional prdida sea, pero no apreciablemente
revertirla . Ninguna terapia ha sido significativamente til para
incrementar la masa sea.

- La densidad mineral sea ha sido (+) correlacionada al peso corporal y
actividad fsica, y (-) con amenorrea , baja de peso y desrdenes de
alimentacin.

- Estrgenos importantes para mantener la masa sea: remodelacin,
resorcin y formacin sea.

Amenorrea y deficiencia estrognica conduce a rpida prdida de masa
sea, particularmente hueso trabecular ( columna). Sobre 50% de pacientes
con anorexia tiene osteoporosis.
Si la causa no es fcilmente tratable se debera iniciar tratamiento
hormonal para prevenir excesiva prdida sea.
Calcio- vitamina D.
Citrato de clomifeno: amenorrea menor de 1 ao. DIU T de C




11. Consecuencias para la paciente:
- Problemas psiquitricos.

283

Bibliografa:
Amenorrea: evaluation and treatment. Taconna Master Hunter, M.D, Fam
physician 2006;73: 1374-82, 1387.
Obstetrician y Ginecologia, Beckmann M.D, 2010, 315-318

63. PAREJA INFRTIL : ESTUDIO BSICO.


Paciente de 28 aos consulta porque lleva 1 ao 4 meses tratando de embarazarse.
Pareja 35 aos, sano, Act sexual 3 veces a la semana.
- Antecedentes Familiares: no refiere.
- Antecedentes Personales: no refiere.
- Antecedentes Gineco obsttricos : menarquia 13 aos , RM V/30 G0P0A0
- Hbitos: no refiere.

1. Diagnstico

Nulpara
Infertilidad primaria

2. Definicines.

Fertilidad es la capacidad de reproducir .
Fecundabilidad es la probabilidad de lograr un embarazo cada mes y la tasa de
fecundabilidad es de apx 0,22 por mes.
Fecundidad es la capacidad de lograr un parto a termino con RN vivo con 1 ciclo
menstrual.La tasa de fecundidad es de 0,15 0,18 por mes, con una tasa
acumulada de 90 % por ao.
Infertilidad es la incapacidad de lograr un embarazo clnico espontneo con la pareja
actual despus de 12 meses de exposicin , teniendo relaciones sexuales
frecuentes (3-4 veces por semanas) y sin utilizar ningn mtodo anticonceptivo.

Esterilidad es la incapacidad de llevar un embarazo a termino.
Podemos diferenciar 2 tipos de infertilidad:
- Primaria: sin embarazos anteriores entre la pareja (no importa si ha tenido
embarazo con otra pareja)
- Secundaria: antecedentes de embarazo previo, aunque haya sido aborto

3. Etiologa :

284
Masculino 35 % : Pretesticular - Testicular Post testicular
Femenino 35 % : Ovulatorio 25%
Tuboperitoneal 20 25 %
Endometriosis 20 %
Uterina 5 - 10 %
Cervical 5 10 % (actualmente no se considera causa por si sola)
Ambos 5%
Inexplicada 10 %

4. Factores de riesgos

- Mayores de 35 aos
- Pacientes con historia de oligo o amenorrea, patologa tubaria, o peritoneal
conocida (endometriosis)
- Parejas que exista el conocimiento de un factor masculino de infertilidad
- Exposicin a teratognicos
- Antecedentes de PIP
- Antecedentes familiares de RN con malformaciones congnitas
- Uniones conyugales tardas
- Cambios en la conducta sexual
- Aumento de las ITS
- Incorporacin de la mujer al mundo laboral

5. Epidemiologa
En los ltimos aos la tasa de infertilidad en la poblacin ha aumentado, pues difieren
el embarazo a edades mayores.
Problema frecuente que afecta a la poblacin mundial en edad reproductiva siendo un
10-15%.
Intervalo necesario para la concepcin:
- 1er mes: 20-25%
- 3 meses: 50%
- 6 meses: 75%
- 1er ao: 85%
- 2 ao: 92%


6. Diagnstico clnico
Escencial una buena historia clnica, examen fsico a la pareja, y exmenes
complementarios segn el factor que queremos evaluar.

7. Anamnesis

285
Es importante tener una aproximacin en la primera consulta tanto a la mujer como al
hombre.
Historia clnica completa:
- Antecedentes de la mujer: debe incluir GPA, resultado de los embarazos y
complicaciones. Hijos en comn con la pareja actual y con otras parejas, edad
de menarquia y caractersticas de los ciclos, presencia o no de dismenorrea y/o
dispareunia. MAC y frecuencia coital, cirugas pelvianas o abdominales previas,
PIP, ETS, sntomas de enfermedad tiroidea, galactorrea, acn e hirsutimos.
Ocupacin, hbitos (alimentacin, OH, tabaco), medicamentos, alergias.
Antecedentes familiares por casos de malformaciones congnitas familiares o
falla reproductiva de la madre o hermanas. Problemas genticos: Exposicin a
teratognicos, edad materna avanzada, antecedente de RN con
malformaciones. Tcnicas sexuales (lubricantes, duchas despus del coito). Si
han tenido estudios y tratamientos previos.

8. Examen fsico

Se debe examinar a la pareja, cuyo enfoque debe incluir la bsqueda de enfermedades
sistmicas, signos de anormalidades genticas y disfuncin andrognica.
Ambos: Peso-talla (IMC), tiroides, examen mamario (galactorrea), signos de
hiperandrogenismo (alopecia, hirsutismo, piel grasa y acn), signos de
hiperinsulinismo (acantosis nigricans y acrocordones), masas abdominales y pelvianas,
cicatrices.
Femnino: Alteraciones vaginales-cervicales-uterinas-anexiales: toma de PAP si es
necesario
o Tacto recto-vaginal: en caso de dismenorrea o dispareunia en busca de
dolor y/o ndulos en ligamento terosacros y fondos de saco vaginales.
Masculino: estigmas de hipogonadismo y desvirilizacin asociada (ginecomastia,
testculos pequeos, ausencia de caractersticas sexuales secundarias), localizacin e
indemnidad meato urinario, escroto (evaluar de pie, contenido consistencia y
sensibilidad (hernia, hidrocele, testculos), epiddimo (quistes, engrosamiento y
sensibilidad a palpacin).

9. Estudio de la paciente

Debemos evaluar a la pareja y de manera simultnea.
Objetivos
- Consignar una efectiva Ovulacin de ovocitos de buena calidad
- Adecuada produccin de espermatozoides (cantidad y calidad)
- Si ambos gametos son capaces de encontrarse e interactuar de una forma
adecuada en el tracto genital femenino (Factor tubo/uterino)

286

FEMENINO

1. Factores ovulatorios
Alteraciones menstruales: amenorrea, sangrado menstrual infrecuente o sangrado
menstrual frecuente y/o alteraciones de la fase ltea (evaluar el contexto de la
paciente)
Enfoque:
- Orientacin estados ovulatorios de la paciente:
o Calendario menstrual
o Temperatura Basal bifasica.
- Si hay alteracin del ritmo menstrual:
o Medicin progesterona plasmtica durante la fase ltea: idealmente
sptimo da post ovulacin. Valores mayores a 3 ng/mL evidencia
presuntiva de ovulacin. Mayores a 10 ng/ml aseguran funcin lutea
adecuada.
o Deteccin LH en orina: identifica el pick preovulatorio. Ayudan para definir
el perodo frtil. 3 -4 das antes de la ovulacin.
o Biopsia endometrial: desarrollo secretor endometrio
o Seguimiento folicular ecogrfico: mayor certeza de ovulacin y
caractersticas del ciclo ovulatorio: determina n y cantidad de folculos en
desarrollo, desarrollo endometrial (Menstrual: endometrio tipo 0; Pre-
ovulatorio: endometrio tipo I con un gran folculo dominante; Fase ltea:
endometrio hiperecognico o tipo III, cuerpo lteo se ve colapsado o
aspecto qustico) , moco cervical, aumento de lquido en el fondo saco
posterior.
o Si hay pobre funcin ovulatoria realizar exmenes complementarios: TSH,
PRL, perfil andrognico (Testosterona total, SHBG, Androstenediona, 17-
OH-Progesterona y DHEAS)

- Reserva ovrica: potencial reproductivo de la mujer en relacin al nmero de
folculos y calidad de ovocitos en los ovarios: Disminuida: Alerta para
problema de fertilidad.
o Medicin Estradiol y FSH en fase folicular temprana del ciclo (da 3): si FSH
>10mUI/mL y estradiol 80 pg/mL: prueba alterada
o Test de citrato de clomifeno: medir FSH y estradiol basal de da 3. Se
administra Citrato de Clomifeno 100 mg vo desde el 5 al 9 da del ciclo,
repitiendo la medicin de FSH el da 10.
Anormal: FSH el da 10 es 10 mUI/mL. Este test permite
identificar a un subgrupo de pacientes con reserva ovrica
disminuida, en que la medicin en el da 3 es normal, pero de todas

287
maneras, tendran una respuesta inadecuada a estimulacin de
ovulacin y menor tasa de xito de los ttos de fertilidad. Importante
informacin sobre pronstico en > 35 aos o con historia de ciruga
ovrica previa y/o una respuesta previa inadecuada a estimulacin
con gonadotrofina exgenas.
o Conteo de folculos antrales: durante la ecografa basal que se realiza el 3
da del ciclo.
Conteo <5 folculos antrales en ecografa basal (de ambos ovarios):
mayor alteracin con exmenes alterados (FSH basal e Inhibina B),
mayores tasas de cancelacin de ciclos de fertilizacin asistida y
menores tasas de embarazo en los mismos.

o Volumen ovarico , Inhibina b , hormona antimulleriana (AMH)


- Si hay amenorrea: (no estara en el contexto de la paciente del caso)
o Prueba de la progesterona: la anovulacin crnica se debe de ir a
descartar. Prueba de progesterona por 7-10 das vo, y ver si se produce
sangrado genital: (+) adecuada produccin de E2 pero que no est
ovulando. (-) adiciono E2. Luego solicito FSH, LH y estradiol.


Fig 4 Foliculo pre ovulatorio


Fig 5 Ovario estimulado para ovulacion con Hmg

2. Factores cervicales

288

En relacin a estenosis cervical o la anormalidad de produccin de moco cervical o
interaccin moco y espermios son raramente identificadas como la nica o principal
causa de infertilidad.
o Test Post Coital: muestra de moco cervical obtenido en perodo frtil
examinado bajo microscopio para determinar presencia de espermios
mviles algunas horas luego de haber tenido relaciones sexuales.. Se
extrae muestra de moco del canal cervical 2-10 hrs posteriores al coito. Es
bastante subjetivo y ya no se usa de rutina
(+) muestra con >10 espermios mviles por cambios
(-) muestra con < 10 espermios por campo, se repite hasta la
ovulacin debido a que el moco va mejoran cerca de la ovulacin
por aumento de E2.
o Algunos lo consideran til sobre todo para la evolucin de la efectividad del
coito en aquellos casos de disfunciones sexuales.
o PAP



Fig 1 Cambios cervicales durante el ciclo menstrual

289



3. Factores uterinos y Tubarios
o Orina completa y cultivo de infecciones sospechadas
o Eco transvaginal: excelente mtodo de estudio de los rganos genitales
internos. Objetiva estado uterino y alteraciones como miomas (tamao y
ubicacin) y adenomiosis y sospechar lesiones endometriales (plipos y
sinequias) que alteran la anatoma normal de la cavidad del tero. Para
distender paredes y evidenciar tamao y ubicacin de lesiones se utiliza la
inyeccin de suero fisiolgico estril (hidrosonografa). Tambin visualiza
ovarios y patologas asociadas como endometriomas o quistes
endometriosicos implicados en la patologa de reproduccin.
o Histerosalpingografa (HSP): examen radiogrfico de los rganos genitales
complementado el uso de medio de contraste. Estudio primario a toda
pareja que consulta por infertilidad. Define el tamao, forma de la cavidad
uterina y revela anormalidades del desarrollo del tero (unicorne, bicorne
o septado) o adquiridas (polimos, miomas submucosas o sinequias).
Determina la permeabilidad de las trompas identificando lesiones distales y
proximales (fimosis o adherencias peritubarias). Alta sensibilidad 81%
o Histeroscopia: mtodo definitivo para la evaluacin de la cavidad uterina y
el diagnstico de sus anormalidades. Costoso e invasivo.
o Laparoscopia: como mtodo diagnstico y teraputico se reserva para
pacientes sin causa identificable. O cuando hay patologa asociada donde
puede ser resuelta. Se realiza en fase ltea: los primeros 10 das de llegada
la ovulacin o bajo efecto de anticonceptivos para mantener a los ovarios
en reposo (predominio progesterona)
o TAC pelvis
o RNM pelvis

290

FIG 2 Endometriosis Pelvica con compromiso tubario,

291

Fig 3 Histerosalpingografia Normal .

MASCULINO

4. Factor masculino
Espermiograma se debe solicitar a todo varn en este contexto. Examen de incio. 3-7
das de abstinencia. Muestra fresca (no mayor a 1 hora de emisin). Evitar frio
extremo. Evitar contaminar recipiente.


Si hay alteracin se debe repetir a las 2-3 semanas para confirmar los hallazgos e
idealmente evaluacin por mdico andrlogo-urlogo para descartar patologas

292
genitales masculinas que requieran tratamiento: Varicocele severo, Tumor testicular y
Epidimitis.

10. Tratamiento
- Cambios en el estilo de vida
- Corregir causa:



- Ciruga Endoscpica Ginecolgica (F. Uterino, F. Tubario, endometriosis)
o Laparoscopa
o Laparotoma
o Histeroscopa
- Induccin de Ovulacin (F. Ovulatorio):
o Citrato Clomifeno : es un estrogeno no esteroidal capaz de unirse al
receptor de estrogeno en forma prolongada, asi se comporta como un anti
estrogeno contrarestando el feed back negativo ejercido por los estrogenos
endogenos . De esta forma normaliza la liberacion de GnRh y se libera FSH y
LH que son capaces de reclutar los foliculos .Su dosis habitual es 50 mg del
dia 3 al 8 del ciclo y el foliculo debe crecer hasta 23 24 mm para que
ocurra el pick de LH .
o Letrozol.

- Tcnicas de Reproduccin Asistida

o Baja complejidad
Inseminacin Intrauterina
o Alta Complejidad
GIFT (transferencia intrafalopiana de gametos)
FIV (F. masculino y F. Tubario)
ICSI (inyeccin del espermatozoide en el vulo)
Donacin Oocitaria / Espermtica
Crio Medicina

293

11. Consecuencias para la paciente/feto
Repercusiones psicolgicas: sentimiento de culpa, rabia

Bibliografia
7. OMS. (2015). Infertility definitions and terminology. Consultado el da 3 de
abril de 2015, en
http://www.who.int/reproductivehealth/topics/infertility/definitions/en/
8. Masoli, Dr. Diego. (2010). Diagnstico de la infertilidad: Estudio de la pareja
infrtil. REV. MED. CLIN. CONDES, 21(3), 363-367.
9. Schorge, J.O - Schaffer, J.I - Halvorson, L.M - Hoffman, B.L - Bradshwa, K.D -
Cunninghan, F. G. (Eds.) (2009). Valoracin de la pareja estril. Williams
Ginecologa. (pp. 426-446). Dallas, Texas: Ediciones McGrawHill.
10. Carvajal, J. Ralph, C. (Eds.) (2013). Infertilidad conyugal. Manual Obstetricia y
Ginecologa (pp 467-490). Chile: Ediciones PUC.
11. Elizabeth E Puscheck ( Nov 2014 )
http://emedicine.medscape.com/article/274143-overview#aw2aab6b2


64. MENOPAUSIA SINTOMTICA

Mujer de 52 aos acude a control ginecolgico. FUR hace 2 aos , refiere bochornos, mal
dormir, irritabilidad, dolor seo , sequedad vaginal
Antecedentes Familiares : padre infarto , madre artritis
Antecedentes Personales : no refiere.
Antecedentes Gineco obsttricos : menarquia 13 aos , RM VII/25 G2P2A0
Hbitos : cigarro 10 al da.

1) Diagnostico
1. Multpara de 2
2. Menopausia.
3. Tabaquismo crnico
4. Sndrome climatrico

2) Diagnsticos diferenciales
Causas endocrinas:
Hipertiroidismo

294
Sndrome carcinoide
Feocromocitoma
Insulino resistencia

Amenorreas secundarias:

Causas neurolgicas
Ansiedad
Depresin
Epilepsia

Frmacos:
Nitroglicerina,nifedipina,niacina,vancomicina,calcitonina,etanol,glutamatosdico,d
isulfiram,hormona liberadora de la corticotropina.

Deprivacion de OH

3) Definicin

La menopausia es una condicin fisiolgica, es un proceso que se produce por la
ausencia de menstruacin de 1 ao de duracin, expesado en la completa o casi
completa deplesin folicular y ausencia de secrecin de estrogenos. Proceso que
empieza desde los 35 aos aproximadamente y se establece definitivamente a una
edad promedio de 49 aos (rango normal entre los 45 a 55) en chile.

4) Etiologa

Puede ser menopausia fisiolgica espontnea o artificial ( antecedente de dao
gonadal)

5) Fisiopatologa
Dependiendo de cada mujer y en promedio apartir de los 40 aos, comienza la
disminucin de la cantidad y calidad de los folculos ovricos que provoca una
disminucin de la inhibina ovrica lo cual provocar un aumento de la FSH,
terminando en ciclos anovulatorios por la disminucin progresiva de la funcin
folicular. Luego la produccin de estrgenos por parte de los folculos del tejido
ovrico remanente tambin disminuye, hasta desaparecer. Durante la presencia de
estrgenos se permite la proliferacin endometrial y a pesar de no haber
progesterona para madurar este endometrio, pueden persistir flujos rojos
irregulares . Cuando los estrgenos disminuyen bajo cierto nivel ya no existe
proliferacin endometrial y cesan las menstruaciones.

6) Factores de riesgo

295

-Genticos: a qu edad le llego a su madre.
-Tabaquismo
-Obesidad (Enflaquecidas con ms riesgo de osteoporosis,obesas ms protegidas)
-Exposicin a quimioterapia o radioterapia.
-Ciruga ovrica o Histerectoma.

7) Epidemiologa

Es un proceso fisiolgico.

8) Diagnstico clnico

- Bochornos 72% el ms frecuente
- Insomnio
- Cambios animo
- Irritabilidad
- Depresin
- Cefalea
- Molestias vaginales (sequedad, atrofia, dispareunia, prurito)
- Incontinencia orina

9) Anamnesis

Fecha de ltima regla
Sntomas de presentacin temprana bochornos, trastornos menstruales, insomnio,
irritabilidad, alteraciones del nimo. Dolor asteo muscular .
Cambios fsicos presentacin en tiempo intermedia: atrofia vaginal, incontinencia
urinaria de esfuerzo, atrofia de piel.
Enfermedades, presentacin tarda: osteoporosis, enfermedad cardiovascular,
demencia tipo Alzheimer.

10) Examen fsico

CSV
Sudoracin, diaforesis, sudoraciones nocturnas, palpitaciones.
Atrofia de la mucosa de la vagina con prdida progresiva de la elasticidad y
humedad, empalidecimiento de la mucosas y eventual debilitamiento del tono
muscular del diafragma urogenital.
Piel y fanereos: adelgazamiento del cabello, alopecia e hipertricosis facial.
Masa muscular: disminucin de la masa muscular.
Osteoporosis y fracturas: aplastamiento vertebral, deformidad de columna dorsal,
reduccin de estatura

296

11) Estudio del cuadro

Diagnstico

- FSH > 40 UI , no es necesario solicitar en paciente sintomtica , a menos que este
sexualmente activa sin mtodo anticonceptivo
- Diagnstico retrospectivo despus de un ao de amenorrea

Previo a toda iniciacin de un tratamiento hormonal de reemplazo, toda mujer en


este periodo debe realizarse un chequeo completo que incluya:

CSV
Examen fsico completo.
Exmenes de laboratorio: Perfil Lipdico, glicemia, P hepticas, TSH
PAP
Mamografa.
Ecotomografia transvaginal.
Densitometra sea. En mayores de 65 aos o antes si hay factores de riesgo.

12) Tratamiento y conducta



- Busca mejorar la calidad de vida y reducir incidencia de fracturas
- Modificar estilo de vida
- Ejercicios
- Calcio 1200 mg da y Vit D 600 UI da
- Agentes antiresortivos (bifosfonatos) si hay osteopenia u osteoporosis
- Terapia hormonal (estrgenos) en menopausia precoz
- Cardiovasculares. Se pueden controlar otros factores de riesgo (tabaco, HTA,
colesterol) ms ejercicios.
TRH
- Indicacin nica, para los bochornos
- Estrgenos puros, solo para pacientes con histerectoma, dosis continua (28ds)
- Combinados, esquema secuencial continuo (28) el nico que mantiene regla. Esta
indicado cuando aun no ha pasado un ao desde la ultima regla
- Combinados continuos (28) o secuenciales discontinuos (21/7) no tiene regla,
esta indicado cuando ya ha pasado mas de un ao sin reglas
- Duracin de tratamiento se debe discutir con la paciente

Contraindicaciones TRH
- Cualquier tumor de mama hasta que no se demuestre sus caractersticas
histolgicas
- Antecedente de trombosis venosa profunda o trombo embolismo pulmonar
- Hipertensin arterial de difcil manejo

297
- Perfil lipdico alterado ) TG muy elevados
- Antecedente de angina de pecho o dolor precordial no definido por los
especialistas
- Alteracin de flujos rojos sin causa definida histolgicamente
- Antecedente de accidente vascular enceflico de tipo isqumico

Paciente que consulta especficamente por molestias vaginales y sexuales, sin
sintomatologa general que amerite TRH, con o sin tero:

- Aplicar supositorio, tableta o crema de estradiol o estriol : 1 vez cada noche por
15 das y luego 2 veces a la semana .No es necesario dar Progesterona
- Tambin se puede usar la misma crema como lubricante durante o previo al
coito.

13) Consecuencias
A la mujer le trae mala calidad de vida especialmente por los sntomas
vasomotores, problemas en la esfera sexual por la atrofia genital. Riesgo de
fracturas y problemas cardiovasculares.
Problemas psicolgicos y mentales.

Bibliografa
Cedip 2009 ginecologa

65. SANGRADO UTERINO ANORMAL



Paciente de 17 aos consulta por que desde hace 15 das presenta regla abundante , Su
ultima regla fue hace 3 meses.
Antecedentes Familiares : no refiere.
Antecedentes Personales : no refiere.
Antecedentes Gineco obsttricos : menarquia 13 aos , RM IV/35 60 das
G0P0A0
Hbitos: cigarro 10 al da.

1) Cul es el diagnostico ms probable de esta paciente?

1. Nuligesta
2. Sangrado uterino anormal: sangrado menstrual prolongado, abundante e
infrecuente
3. Tabquica crnica

298
2) Cules son los diagnsticos diferenciales?

Patologas asociadas al embarazo: embarazo ectpico, aborto , Mola
Patologa no estructural
Trastornos de coagulacin: enf. de Von Willebrand, otras coagulopatas
Anovulacin.
Patologa endometrial : Infecciosas : cervicitis, endometritis, PIP
Endocrinopatas: Trastornos tiroideos, Hiperprolactinemia, Sind. Ovarios
poliqustico,
Tumorales (ovricas)
Asociadas a medicamentos : ACO, anticoagulantes
Asociadas a enfermedades sistmicas
Otros : cuerpo extrao, hemangiomas ,fistulas arterio venosas .

3) Cmo se define esta patologa?

Se define como cualquier alteracin en volumen, regularidad, duracin o
frecuencia del ciclo menstrual normal en ausencia de embarazo


Primero definamos que es normal :

El ciclo menstrual comienza el primer da de sangrado un periodo y termina con el
primer da del prximo periodo.

Volumen 5 80 ml
Duracin 2 7 das
Regularidad +- 2 a 20 das
Frecuencia 24 - 38 das


Se define Sangrado menstrual abundante ( SMA) a la Prdida de sangre menstrual
excesiva que interfiere con la actividad fsica, emocional o social normal de la
mujer, afectando su calidad de vida. Puede ocurrir solo o asociado a otros
sntomas .
Tambin se considera SMA al sangrado que, desde la perspectiva de la mujer,
aument en volumen, independientemente de la regularidad, frecuencia o
duracin

Sangrado anormal agudo : episodio de sangrado en una mujer en edad
reproductiva, no embarazada, de tal cantidad que requiere intervencin inmediata

299
para evitar una mayor prdida de sangre.
Sangrado anormal crnico: Sangrado del cuerpo uterino que es anormal en
duracin, volumen, y / o la frecuencia y ha estado presente en la mayora de los
ltimos 6 meses
Sangrado intermensual : sangrado irregular, habitualmente corto y escaso que
ocurre entre dos reglas normales .Antiguamente llamado metrorragia .
Sangrado Post Coital : sangrado posterior a una relacin sexual (antes sinusorragia
)
Sangrado post menopausia : mas de un ao des pues de la ultima regla
Spotting pre o post menstrual : sangrado que puede ser regular uno o varios das
Menstruacin precoz : regla antes de los 9 aos de edad



Alteracin de la regularidad

Variacin normal : +- 2 a 20 das
Sangrado Menstrual Irregular : variacin del sangrado mayor a 20 das, en
un periodo de 90 das hay menos de 4 episodios de regla
Sangrado menstrual ausente ( amenorrea ) : ausencia de sangrado por mas
de 90 das

Alteracin de la frecuencia

Normal : cada 24 -38 das
Sangrado Menstrual infrecuente : Sangrado con intervalos > a 38 das
( hay 1 a 2 episodios en 90 das )

Sangrado Menstrual Frecuente : Sangrado con intervalos menores a 24 das
( hay mas de 4 episodios en 90 das )

Alteracin de la duracin

Normal : 3 8 das
Sangrado menstrual prolongado : Mas de 8 das sangrado
Sangrado menstrual acortado : menos de 3 das de sangrado

Alteracin del Volumen

Normal : 5 80 ml
Sangrado menstrual escaso : < 5 ml

300
Sangrado menstrual abundante : > 80 ml





Terminologia :
VOLUMEN REGULARIDAD FRECUENCIA DURACION OTRO
ABUNDANTE IRREGULAR FRECUENTE PROLONGADO INTERMENSTRUAL

NORMAL REGULAR NORMAL NORMAL PREMENSTRUAL

ESCASO AUSENTE INFRECUENTE ACORTADO POSTMENOPAUSICO






Es Diagnostico de exclusion :

Es importante enfatizar que en el ciclo ovulatorio de periodicidad normal el
endometrio que se descama es estable, dado que ha respondido a la secuencia
apropiada de estrgenos y progesterona, evitando as la descamacin irregular.
Adems, el fenmeno de descamacin endometrial es universal; es decir, los cambios
isqumicos ocurren simultneamente en todo el endometrio debido a que la
descamacin ocurre por depravacin de estrgenos y progesterona, lo que determina
que el flujo menstrual tenga una magnitud y duracin auto limitada. En concomitancia
con el descenso hormonal brusco que induce la isquemia endometrial, se inicia la
reparacin endometrial favorecida por el aumento de la secrecin estrognica del
nuevo ciclo. La ausencia de ciclos ovulatorios normales altera la ocurrencia de los
eventos descritos, favoreciendo el sangrado uterino anormal.

4) Cual es/son la/s etiologa de este cuadro clnico?

301

Clasificacin FIGO 2011 :


PALM (estructurales) COEIN (no estructurales)
Plipos Coagulopatas
Adenomiosis Ovulatoria
Leiomiosis Endometrial
Malignidad/ hiperplasia Iatrognico
No clasificado

- La comprobacin de una de estas causas no descarta otra(s) concomitantes


- Muchas veces la presencia de una de estas causas es asintomtica.




A modo de ejemplo , si una paciente presenta un sangrado vaginal de origen
uterino por miomas, el diagnostico se anotara SUA M , Si es por un plipo se
anotara SUA P y as sucesivamente .




Anovulatorias:
Estmulo prolongado de Estrgenos sobre el endometrio en ausencia de
Progesterona.
Ovario poliquistico.
Folculo persistente.
Constituye principal causa de HUD (del total)
Mas frecuente en adolescentes y periodo pre menopasico (ciclos anovulatorios)
Acclica
Clnica cardinal: Sangrado infrecuente

Ovulatoria:

Alt. Fase Folicular: Mayor o menor duracin
Alt. Fase Ltea: Insuficiencia del cuerpo lteo: disminuye produccin de E2 y P2 en
la 2 mitad del ciclo (duracin del ciclo acortada)

302
Causa mas frecuente en mujeres cclicas de edad frtil
Clnica cardinal: Sangrado menstrual frecuente +/- sangrado premenstrual.

5) Cul es la fisiopatologa de este cuadro?

Existen 4 mecanismos tericos responsables de un sangrado uterino anormal de
origen ovrico.
1.Sangrado por deprivacin de estrgenos:


Este hecho puede condicionar una falencia en la sustentacin endometrial y su
consecuente descamacin. Se puede observar al suspender la terapia estrognica
pura en la mujer hipogondica; tambin es la causa de goteo periovulatorio que
ocurre por la cada de estrgenos que precede a la ovulacin en mitad del ciclo ,
sangrado post ooforectomia .

2. Sangrado por disrupcin de estrgenos:


Se observa en mujeres anovulatorias crnicas estrogenizadas, cuyo endometrio
prolifera excesivamente al ser estimulado slo por estrgenos. Al no ovular no hay
sntesis de progesterona, no ocurriendo el efecto opositor a la proliferacin que es
el cambio secretor inducido por la progesterona. Hay crecimiento exagerado del
endometrio y se llega al punto en que los vasos endometriales no logran irrigar la
capa superficial del endometrio y tras un perodo de sagrado menstrual
infrecuente o amenorrea puede haber descamacin endometrial irregular que
condiciona un sangrado profuso. Este es el mecanismo patognico ms importante
y frecuente de las sangrados uterinos anormales por anovulacin que vemos en
clnica, como los descritos en la peri menarquia, peri menopausia y sndrome de
ovarios poli qusticos. En casos extremos el estmulo estrognico sobre el
endometrio puede llegar a producir una hiperplasia endometrial e incluso cncer
de endometrio si la paciente nunca es tratada.

4. Sangrado por deprivacin de progesterona:



La existencia de un sangrado por este mecanismo requiere de la presencia de
estrgenos (endgenos o exgenos) que hagan proliferar el endometrio. Se puede
observar en anovulatorias crnicas bien estrogenizadas a quienes, estando en
amenorrea, se les administra progesterona producindose un sangrado, el que
ocurre posterior a la suspensin de la progestina.
Tambin ocurre por efecto farmacolgico en la suplementacin hormonal de
adolescentes hipo gondicas con esquemas secuenciales que consideran la
administracin permanente de estrgenos y cclica de progestina. Si bien el
estmulo proliferativo del estrgeno es constante, la maduracin endometrial

303
cclica inducida por la progestina y su posterior deprivacin determina la
descamacin endometrial.
La deprivacion de progesterona provoca espasmos de los vasos endometriales y el
endometrio cae por isquemia .

5. Sangrado por disrupcin durante estmulo de progesterona:



Ocurre solamente cuando existe una muy alta proporcin de progesterona en
relacin a estrgenos, en cuyo caso hay una pobre proliferacin endometrial, o
mas bien atrofia endometrial. Esto se observa, por ejemplo, con el uso de
preparados anticonceptivos de progestina pura (levonorgestrel, acetato de
Medroxiprogesterona de depsito, etc.) que pueden provocar goteo intermitente.
Un caso especial y frecuente es el sangrado durante terapia con anticonceptivos
combinados de micro dosis (20-30 ug de etinilestradiol); la baja dosis estrognica
asociada a un alto efecto progestativo desde el primer da de terapia, puede
inducir flujos rojos intermitentes de tipo goteo, aparte del flujo rojo por
deprivacin de estrgeno y progestina esperado al fin de cada ciclo.

6) Qu factores de riesgo buscara?

1. stress
2. entrenamiento fsico intensivo
3. prdida aguda de peso
4. trastornos de alimentacin, anorexia nervosa, bulimia
5. falla en la secrecin de pulsos de GnRH (anovulacin idioptica)
6. Uso de medicamentos hormnales .

7) Cul es la epidemiologia de este cuadro?

Es la causa ms frecuente (90%) de sangrado uterino anormal en el periodo post
menarquia ( 2 a 3 aos). Se produce en general por ciclos anovulatorios dados por
una inmadurez del eje hipotalamo-hipfisis. Al no haber ovulacin, no hay
secrecin de progesterona y el endometrio descama (sangra) segn las
fluctuaciones de los estrgenos.

20 % DE LAS CONSULTAS GINECOLOGICAS
30 % MUJERES PRESENTA EL PROBLEMA ANUALMENTE
2/3 DE LAS HISTERECTOMIAS
25 % DE LAS CIRUGIAS GINECOLOGICAS
50 % DE LAS MUJERES SUFRE ANEMIA FERROPRIVA


304
8) Cmo hacemos el diagnstico clnico?

Grupo Clinica tpica Comentario
Puberes Sangrado uterino Produccin continua y
irregular baja de estrgenos por
un aparato folicular
que no llega a un
desarrollo adecuado,
pero si suficiente para
provocar pequeas
hemorragias (por
privacin hormonal o
por disrupcin)
Edad frtil Polimenorrea
Premenopausia Mujer de 40 50 aos. El 75% de las mujeres
Tras perodos de 2 a 6 presentan trastornos
semanas de hemorrgicos antes
amenorrea presentan de la retirada
una fuerte hemorragia definitiva de la
menstruacin.


9) Que nos interesa preguntar en la anamnesis de esta paciente?

Evaluar caracteristcas del sangrado:

1. Frecuencia
2. Duracin
3. Volumen: frecuencia de cambio de toallas y tipo de toallas ( diarios, nocturnos,
mimi, episodio nico, episodios repetidos.
4. Evaluacin clnica en busca de sntomas y signos de anemia y compromiso
hemodinmico (antecedente de lipotimias ,cansancio, palidez de mucosas, llene
capilar, Taquicardia, hipotensin)
5. Descartar embarazo ( con historia y/o test de embarazo)


Sntomas y/o Signos de anemia y/o compromiso hemodinmico o Sangrado actual
abundante.
Evaluacin clnica en busca de sntomas y signos de anemia y compromiso
hemodinmico (antecedente de lipotimias ,cansancio, palidez de mucosas,
llene capilar, Taquicardia, hipotensin.
Frecuencia de cambio de toallas y tipo de toallas ( diarios, nocturnos, mimi,)
Antecedente de lipotimias ,cansancio, palidez de mucosas, llene capilar,
Taquicardia, hipotensin

305
10) qu buscara dirigida mente en el examen fsico de esta paciente?

Signos de anemia.
Compromiso hemodinmico
Descartar patologa obsttrica

11) Cmo estudiamos este cuadro y que resultados esperara encontrar en estos
exmenes?

CSV , Examen fsico general y ginecologico , especuloscopia, en busca de sangrado
Ver las toallitas, ver la anatoma genital.

Hemograma
Bhcg
Prolactina
PAP
Cultivos cervicales
Ecotv , histerosonografia, Histeroscopia, TAC
Perfil Coagulacion
Biopsia endometrial
Indicaciones de estudio por coagulopatia

Sangrado menstrual abundante desde la menarquia
Uno de los siguientes :
Hemorragia post parto
Sangrado relacionado a procedimiento quirrgico
Sangrado relacionado a procedimiento dental
Dos o mas de los siguientes
Equimosis 1 2 veces al mes
Epistaxis 1 2 veces al mes
Sangrado frecuente de encas
Antecedentes familiares de trastorno de coagulacin



Indicaciones de Biopsia de endometrio

Endometrio normal post menstrual 4 5 mm
Endometrio Pre menstrual 14 mm
Biopsia de endometrio :
< de 45 aos con factor de riesgo
o Anovulatorias
o Falla de tratamiento medico
o SUA con endometrio normal a la Eco TV

306
> de 45 aos sin causa estructural
o Sangrado menstrual abundante irregular
o Factores de riesgo para cncer de endometrio


12) cul es el tratamiento de esta paciente?

Una vez descartada las causas orgnicas se debe iniciar tratamiento
hormonal para corregir el desbalance estrgeno, progesterona, para
estabilizar el endometrio, generar una descamacin endometrial sincrnica
y prevenir que la paciente desarrolle anemia. Esto a su vez previene del
cncer endometrial e hiperplasia endometrial.

Terapia inicial en APS en nias mayores de 17 aos

ACO 30 mcg estrgenos :


Sulfato ferroso 200 mg ,2 comp/ da alejado de comidas por mnimo 1 mes (a
evaluar segn hemograma)

13) qu consecuencias puede traer esta patologa a la mujer / madre/ y al feto?
Para la mujer la consecuencia es la anemia principalmente.

307





Bibliografa
5) Obstetricia, Prez-Sanchez, cuarta edicin, 2011
6) Gua perinatal CEDIP 2011

66. SANGRADO UTERINO ANORMAL (SUA 0 )



Mujer de 18 aos consulta por que desde hace 15 das presenta regla abundante , Su
ultima regla fue hace 3 meses.
Antecedentes Familiares : no refiere.
Antecedentes Personales : no refiere.
Antecedentes Gineco obsttricos : menarquia 13 aos , RM IV/35 60 G0P0A0
Hbitos: cigarro 10 al da.

1Cul es el diagnostico ms probable de esta paciente?
Nuligesta
G0P0A0
Tabquica crnica
Sangrado uterino anormal , probablemente SUA O

2Cules son los diagnsticos diferenciales?
Patologa Obsttrica
Hemorragia primer trimestre
Sangrado uterino origen no estructural
Coagulacin, endometrial, ovulatorio,iatrognico, no clasificado.
Causa no ginecolgica : Endocrinopatas: Trastornos tiroideos,
Hiperprolactinemia, Infecciosas : cervicitis, endometritis, PIP
Tumorales ( cervicales, uterinas u ovricas)
Asociadas a enfermedades sistmicas

308



3Cmo se define esta patologa?
Se define como el flujo rojo uterino irregular y abundante que ocurre como
consecuencia de la anovulacin, en ausencia de patologa estructural o de
enfermedades mdicas.

ES UN DIAGNOSTICO DE EXCLUSION

4Cual es/son la/s etiologa de este cuadro clnico?

Es importante enfatizar que en el ciclo ovulatoria de periodicidad normal el
endometrio que se descama es estable, dado que ha respondido a la secuencia
apropiada de estrgenos y progesterona, evitando as la descamacin irregular.
Adems, el fenmeno de descamacin endometrial es universal; es decir, los
cambios isqumicos ocurren simultneamente en todo el endometrio debido a que
la descamacin ocurre por deprivacin de estrgenos y progesterona, lo que
determina que el flujo menstrual tenga una magnitud y duracin auto limitada. En
concomitancia con el descenso hormonal brusco que induce la isquemia
endometrial universal, se inicia la reparacin endometrial favorecida por el
aumento de la secrecin estrognica del nuevo ciclo. La ausencia de ciclos
ovulatorios normales altera la ocurrencia de los eventos descritos, favoreciendo la
metrorragia disfuncional.

Anovulatorias:
Estmulo prolongado de E2 sobre endometrio en ausencia de P2.
Ovario poli folicular
Folculo persistente
Constituye principal causa de SUA (del total)
Adolescentes y periodo pre menopasico (ciclos anovulatorios)
Acclica

309
Clnica cardinal: Sangrado menstrual infrecuente

Ovulatorias:
Alt. Fase Folicular: Mayor o menor duracin
Alt. Fase Ltea: Insuficiencia del cuerpo lteo: disminuye produccin de E2 y P2 en
la 2 mitad del ciclo (duracin del ciclo acortada)
Causa + frecuente en mujeres de edad frtil
Clnica cardinal: Sangrado menstrual frecuente +/- spotting premenstrual.

5Cul es la fisiopatologa de este cuadro?

Existen 4 mecanismos tericos responsables de un sangrado uterino de tipo
disfuncional.

1.Sangrado por deprivacin de estrgenos:


Este hecho puede condicionar una falencia en la sustentacin endometrial y su
consecuente descamacin. Se puede observar al suspender la terapia estrognica
pura en la mujer hipogondica; tambin es la causa de goteo menstrual
periovulatorio que ocurre por la cada de estrgenos que precede a la ovulacin.

2. Sangrado por disrupcin durante estmulo estrognico:



Se observa en mujeres anovulatorias crnicas estrogenizadas, cuyo endometrio
prolifera excesivamente al ser estimulado slo por estrgenos. Al no ovular no hay
sntesis de progesterona, no ocurriendo el efecto opositor a la proliferacin que es
el cambio secretor inducido por la progesterona. Tras un perodo de sangrado
menstrual infrecuente o amenorrea, puede haber descamacin endometrial
irregular asincrnica que condiciona un sangrado profuso (antigua metrorragia
disfuncional). Este es el mecanismo patognico ms importante y frecuente de los
sangrado uterino anormales que vemos en clnica, como los descritos en la
perimenarquia, peri menopausia y sndrome de ovarios poli qusticos. En casos
extremos el estmulo estrognico sobre el endometrio puede llegar a producir una
hiperplasia endometrial e incluso cncer de endometrio si la paciente nunca es
tratada.

3. Sangrado por deprivacin de progesterona:


La existencia de un sangrado por este mecanismo requiere de la presencia de
estrgenos (endgenos o exgenos) que hagan proliferar el endometrio. Se puede
observar en anovulatorias crnicas bien estrogenizadas a quienes, estando en
amenorrea, se les administra progesterona producindose un sangrado, el que
ocurre posterior a la suspensin de la progestina.

310
Tambin ocurre por efecto farmacolgico en la suplementacin hormonal de
adolescentes hipogondicas con esquemas secuenciales que consideran la
administracin permanente de estrgenos y cclica de progestina. Si bien el
estmulo proliferativo del estrgeno es constante, la maduracin endometrial
cclica inducida por la progestina y su posterior deprivacin determina la
descamacin endometrial.

4. Sangrado por disrupcin durante estmulo de progesterona:


Ocurre solamente cuando existe una muy alta proporcin de progesterona en
relacin a estrgenos, en cuyo caso hay una pobre proliferacin endometrial. Esto
se observa, por ejemplo, con el uso de preparados anticonceptivos de progestina
pura (levonorgestrel, acetato de Medroxiprogesterona de depsito, etc.) que
pueden provocar goteo intermitente. Un caso especial y frecuente es el sangrado
durante terapia con anticonceptivos combinados de micro dosis (20-30 ug de
etinilestradiol); la baja dosis estrognica asociada a un alto efecto progestativo
desde el primer da de terapia, puede inducir flujos rojos intermitentes de tipo
goteo, aparte del flujo rojo por deprivacin de estrgeno y progestina esperado al
fin de cada ciclo.

6Qu factores de riesgo buscara?

stress
entrenamiento fsico intensivo
prdida aguda de peso
trastornos de alimentacin, anorexia nervosa, bulimia
falla en la secrecin de pulsos de GnRH (anovulacin idioptica)

7) Cul es la epidemiologia de este cuadro?
Es la causa ms frecuente (90%) de SUA en el periodo post menarquia ( 2 a 3 aos).
Se produce en general por ciclos anovulatorios dados por una inmadurez del eje
hipotalamo-hipfisis. Al no haber ovulacin, no hay secrecin de progesterona y el
endometrio descama (sangra) segn las fluctuaciones de los estrgenos.

8) Cmo hacemos el diagnstico clnico?

Grupo Clinica tpica Comentario
Puberes Sangrado menstrual Produccin continua y baja de estrgenos por un
irregular aparato folicular que no llega a un desarrollo
adecuado, pero si suficiente para provocar
pequeas hemorragias (por privacin hormonal o
por disrupcin)

Edad frtil Sangrado menstrual


frecuente

311
Premenopausia Tras perodos de 2 a 6 El 75% de las mujeres presentan trastornos
semanas de hemorrgicos antes de la retirada definitiva de la
amenorrea presentan menstruacin.
una fuerte hemorragia

Evaluar cuanta del sangrado:
6. Duracin
7. Volumen: frecuencia de cambio de toallas y tipo de toallas ( diarios, nocturnos,
mimi, episodio nico, episodios repetidos.
8. Evaluacin clnica en busca de sntomas y signos de anemia y compromiso
hemodinmico (antecedente de lipotimias ,cansancio, palidez de mucosas, llene
capilar, Taquicardia, hipotensin)
9. Descartar embarazo ( con historia y/o test de embarazo)

9) Que nos interesa preguntar en la anamnesis de esta paciente?

Sntomas y/o Signos de anemia y/o compromiso hemodinmico o Sangrado actual
abundante.
Evaluacin clnica en busca de sntomas y signos de anemia y compromiso
hemodinmico (antecedente de lipotimias ,cansancio, palidez de mucosas,
llene capilar, Taquicardia, hipotensin.
Frecuencia de cambio de toallas y tipo de toallas ( diarios, nocturnos, mimi,)
Antecedente de lipotimias ,cansancio, palidez de mucosas, llene capilar,
Taquicardia, hipotensin
Descartar embarazo ( con historia y/o test de embarazo).
Descartar coagulopatias.

10) Qu buscara dirigida mente en el examen fsico de esta paciente?

Signos de anemia.
CSV
Especuloscopia : origen del sangrado
Tacto vaginal : descartar patologa estructural

11) Cmo estudiamos este cuadro y que resultados esperara encontrar en estos
exmenes?

Examen fsico y especuloscopia, en busca de metrorragia. Ver las toallitas, ver la
anatoma genital.
Hemograma: en busca de anemia , trombocitopenia y descartar leucemia por la
edad.
Sub unidad beta en orina o test pack para descartar embarazo.
Pruebas de coagulacin para descartar coagulopatias.
TSH para descartar hipotiroidismo

312
Eco trans vaginal.
El diagnstico de anovulacin es esencialmente clnico. El apoyo de laboratorio
tiene el objetivo de buscar factores etiolgicos corregibles que puedan romper el
crculo de la anovulacin. Se debe siempre investigar el nivel de prolactina
plasmtica (PRL) y de hormona tiroestimulante (TSH) , patologas causantes de
anovulacin que no siempre dan otras manifestaciones clnicas.
Ante una paciente hiperandrognica con obesidad central, con o sin acantosis
nigricans, se debe investigar la posibilidad de SOP y resistencia insulnica
.
Ante la duda de si se trata de una anovulacin crnica estrognica o
hipoestrognica, adems de la observacin clnica del trofismo vaginal y mamario,
se puede realizar una prueba de progesterona. Se administra acetato de
medroxiprogesterona 10 mg/da durante 10 das o progesterona oleosa 50 mg
intramuscular en una sola dosis; si el endometrio de la paciente ha sido estimulado
por estrgenos, sta presentar un flujo rojo en un plazo de hasta 14 das posterior
a la progesterona.
Si se trata de una paciente estrognica, lo ms probable es que se trate de un
estado disfuncional ovulatorio tipo SOP.

Si se trata de una paciente hipoestrognica se debe buscar el origen ovrico y
central. Se mide FSH que estar elevada (mayor de 75 mUI/ml) en caso de
hipogonadismo hipergonadotrfico: falla ovrica o menopausia precoz, resistencia
ovrica a las gonadotrofinas, etc. En estos casos, en la adolescente es necesario
realizar un cardiograma ante la posibilidad de una disgenesia gonadal con
cromosoma Y, debido al riesgo oncognico en estas gnadas disgenticas.

En caso de hipogonadismo hipogonadotrfico, adems del estradiol disminuido, la
FSH est baja o normal (inapropiadamente baja para un estradiol disminuido). Se
debe investigar el origen central, siendo lo ms importante el descartar una
Hiperprolactinemia. Se debe buscar otras posibles causas funcionales que son cada
vez ms frecuentes: trastornos de la alimentacin, baja de peso, stress,
entrenamiento fsico excesivo. Si se sospecha un tumor se deber realizar una
tomografa axial computarizada o una resonancia nuclear magntica de silla turca
(por ejemplo, si hay alteraciones visuales u otras manifestaciones neurolgicas, o
en caso de prolactinemia muy alta o de hipogonadismo hipogonadotrfico sin
causa aparente).

12) Cul es el tratamiento de esta paciente?

Una vez descartada las causas estructurales, se debe iniciar tratamiento hormonal
para corregir el desbalance estrgeno, progesterona, para estabilizar el
endometrio, generar una descamacin endometrial sincrnica y prevenir que la
paciente desarrolle anemia. Esto a su vez previene del cncer endometrial e
hiperplasia endometrial.

313

Terapia inicial en APS en nias mayores de 17 aos
ACO: Anulette CD ( 2 caja tomar slo las blancas!)
1 comp cada 8 hrs por 7 das o hasta que ceda metrorragia y luego 1 comprimido al
da hasta terminar 2 cajas
Sugerir uso de domperidona 10 mg ( idon) , 1 comp c/ 8 en caso de muchos
vmitos o nuseas)

Luego la paciente decidir si continua tomando ACO combinado o progestina
cclica por 10 14 das desde da 15 del ciclo .

Sulfato ferroso 200 mg ,2 comp/ da alejado de comidas por mnimo 1 mes (a
evaluar segn hemograma)

13) Qu consecuencias puede traer esta patologa a la mujer
Para la mujer la consecuencia es la anemia principalmente y el ausentismo laboral
o escolar .

Bibliografa
Obstetricia, Prez-Sanchez, cuarta edicin, 2011

Gua perinatal CEDIP 2011

67. POLIPOS ENDOMETRIALES


Mujer de 42 aos quine consulta por que su regla ha sido mas abudante desde
hace 3 meses, con goteo post menstrual .

Antec Familiar : Madre Ca cuello uterino

Antec Personales : nada

Antec Gineco obsttricos : Menarquia 12 aos, G3P3A0 , Usuaria de ACO


1. Definicin : Los plipos corresponden a
crecimiento benignas y localizados de la superficie
endometrial, constituidos por glndulas , estroma
endometrial y pedculo vascular , que forman una
proyeccin ssil o pediculada hacia la superficie del
endometrio. Pueden ser nicos o mltiples y

314
pueden medir desde algunos milmetros a varios centmetros.
2. Epidemiologia : Son raros en mujeres menores de 20 aos. La incidencia aumenta
directamente con la edad, siendo mayor en la quinta dcada de la vida y declina
gradualmente despus de la menopausia. Causa aproximadamente el 25% de casos de
sangrado post menopasicos , El riesgo de desarrollar un cncer a lo largo de su vida es
de un 2-3 %
3. Etiologa :
Se han formulado algunas hiptesis relacionadas con su origen, como:
1. Factores hormonales: Algunos estudios han mostrado una relacin entre la
presencia de plipos endometriales y la expresin de receptores hormonales. Los
niveles de aromatasa, enzima implicada en la produccin local de estrgenos,
tambin se han asociado a la presencia de plipos endometriales. Se encontr
niveles mayores de aromatasas, tanto en el tejido del plipo como en el
endometrio adyacente .Se postula que la patognesis podra ser la sobre-expresin
de la aromatasa endometrial, especialmente en pacientes pre menopusicas.
La exposicin a hormonas exgenas tambin se ha asociado con el riesgo de
aparicin de plipos endometriales.
2. Factores inflamatorios y proliferativo, principalmente por aumento de los
mastocitos.
3. Factores genticos
4. Factores de riesgos son principalmente, edad (Mxima prevalencia 40-50 aos),
hipertensin arterial, obesidad , uso de Tamoxifeno Y DE Tibolona.
5. Clnica : es variada, un 50% de las mujeres es asintomtica y se realiza el diagnostico
por hallazgo ecogrfico, el otro 50% presenta sintomatologa, el principal sntoma
es el sangrado uterino anormal, Sangrado uterino abundante , sangrado pre
menstrual , sangrado menstrual abundante o sangrado postmenopausia. Tambin
pudiesen presentar dismenorrea ya que el plipo acta como cuerpo extrao. Y
por ltimo pudiese presentar algia plvica aguda, por ulceracin y torsin o
expulsin , situacin rara.
El examen fsico generalmente es normal , a la espculoscopia podra observarse
plipos endocervicales concomitantes o rara vez la exteriorizacin de un plipo a
travs de el OCE.
Estudio de la paciente:
Eco TV , donde se puede observar endometrio engrosado
Hieroscopia, procedimiento Gold Estndar cuando se requiere estudiar la cavidad
endometrial.

315
Histerosonografia, que es la modalidad no invasora ms til para la evaluacin de
los plipos en mujeres con metrorragia. Tiene una sensibilidad 93% y una
especificidad de 94%.
Tratamiento, dependiendo de la
sintomatologa, pudiese ser conservador,
en caso de plipos pequeos y
asintomticos o tratamiento quirrgico,
que es la polipectoma, se realiza
mediante histeroscopa quirrgica, es el
Gold estndar, , mediante la cuales se
secciona el plipo a partir de su base o
pedculo.
Consecuencias : sin riesgo de malignidad
( o es maligno desde su origen o es siempre benigno ), se ha descrito una posible
asociacin con infertilidad, ya que los plipos endometriales tienen una alta prevalencia
entre las mujeres estriles (15,6%-32%), pero los datos no son concluyentes que
realmente produzca por s solo infertilidad, por lo tanto sigue en estudio.


Bibliografa
Vivas CA, Ros JJ, Romero HA. Plipos endometriales, fisiopatologa y factores de
riesgo. Rev CES Med 2012
Sridevi Rao Intermenstrual and post-coital bleeding, OBSTETRICS, GYNAECOLOGY
AND REPRODUCTIVE MEDICINE 2011

68. HIPERPLASIA ENDOMETRIAL



Paciente de 48 aos consulta por presentar desde hace 15 das sangrado vaginal
despus de un periodo de amenorrea de 3 meses. El examen ginecolgico muestra un
tero de tamao y consistencia normal, espculoscopia cuello sano con sangrado por
OCE.
Antecedentes Familiares: padre sano, madre diabtica
Antecedentes Personales: dislipidemia.
Antecedentes Gineco obsttricos: menarquia 13 aos, RM VII/35 50 G0P0A0
Hbitos: no refiere

1) CUAL ES EL DIAGNOSTICO MAS PROBABLE DE ESTA PACIENTE ?
Nuligesta
Sangrado uterino infrecuente y prolongado

316
2) CUALES SON LOS DIAGNOSTICOS DIFERENCIALES ?

Hiperplasia endometrial
Miomas
Plipos
Cncer endometrial
Embarazo
Adenomiosis
Alteraciones de la coagulacin

3) COMO SE DEFINE ESTA PATOLOGIA ?

Proliferacin anormal de glndulas endometriales de tamao y forma irregular,
con aumento de la relacin glndula/estroma, principalmente causado por accin
de estrgenos sin efecto compensador de la progesterona. Frecuente en mujeres
con ciclos anovulatorios.

4) CUAL ES/SON LA/S ETIOLOGIA DE ESTE CUADRO CLINICO ?

Principalmente el origen es un dficit en la estimulacin progestativa, lo que
produce una proliferacin poco controlada del endometrio por efecto estrogenico
mantenido . Principalmente por anovulacin y uso externo de estrgenos en la
menopausia.

5) CUAL ES LA FISIOPATOLOGIA DE ESTE CUADRO ?

Hiperplasia endometrial: Se produce por una estimulacin estrogenica sin un
contraposicin de progesterona. En pacientes obesas es ms frecuente por los
altos niveles de estrgenos endgenos producidos por la conversin de
androstenediona a estrona y la aromatizacin de andrgenos a estradiol ocurrido
en el tejido adiposo perifrico. Es tambin frecuente que se de en pacientes con
anovulacin (ej. SOP) ya que no tienen el estimulo de la progesterona y ocurre una
proliferacin poco controlada del endometrio. Otro caso es la terapia hormonal
durante la menopausia cuando se usa sin freno de progesterona (ej. Ca mama).

La hiperplasia endometrial puede clasificarse en:

Simple sin atipia : cambios en la arquitectura de las glndulas, pequeos quistes
(queso suizo)
Compleja sin atipia : aumento del nmero de las glndulas y de su tamao.
Simple con atipias celulares.
Compleja con atipias celulares.

317
Evolucin: hiperplasias sin atipias se malignizan en un 1-3% mientras que las con
atipia 8-29%
Tambin hay factores genticos que participan en la hiperplasia endometrial como es
la expresin del oncogen bcl-2, este inhibe la apoptosis celular prolongando la vida de
esta, este oncogen estara bajo control hormonal. Durante la fase secretora su
expresin esta disminuida, por lo que aumenta la apoptosis celular en el endometrio
en esta fase. A sido demostrado que la expresin del bcl-2 esta aumentada en la
hiperplasia endometrial.

6) QUE FACTORES DE RIESGO BUSCARA ?

Obesidad, diabetes, HTA, anovulacin (ej. SOP), terapia estrogenica sin
contraposicin de progesteronas.

7) CUAL ES LA EPIDEMIOLOGIA DE ESTE CUADRO ?

La hiperplasia endometrial es causante del 5% de las metrorragias en la post
menopausia.
En este punto es importante describir el riesgo de malignizacin segn el tipo de
hiperplasia presente en la paciente, por lo que la probabilidad de malignizacin es la
siguiente:
Simple sin atipias: 1%
Simple con atipias: 8%
Compleja sin atipias: 3%
Compleja con atipias: 29%

8) COMO HACEMOS EL DIAGNOSTICO CLINICO ?

A travs de la historia, clnica y examen fsico podremos sospechar una hiperplasia
endometrial. El diagnostico clnico es difcil ya que la nica manifestacin que
puede existir es metrorragia. Ante la presencia de este cuadro clnico, mas factores
de riesgo o historia acorde, el estudio imagenologico es indispensable para
confirmar el diagnostico y descartar diagnsticos diferenciales.

9) QUE NOS INTERESA PREGUNTAR EN LA ANAMNESIS DE ESTA PACIENTE ?

Cantidad de sangrado, numero de episodios, frecuencia: las caractersticas del
sangrado pueden ser relevantes y apuntar a ciertas causas de la metrorragia, esto
nos servir para descartar algunos diagnsticos diferenciales ya que tienen
distintos patrones de sangrado.

318
Antecedentes de TRH (E sin P), antecedentes de MAC, morbilidades (HTA, DM,
IMC, SOP, anovulacin) por aumento del riesgo de hiperplasia y cncer
endometrial principalmente.

10)QU BUSCARA DIRIGIDAMENTE EN EL EXAMEN FISICO DE ESTA PACIENTE ?

La mayora de las pacientes presenta un examen fsico normal, fuera del sangrado
uterino que puede ser visible. Cuando el sangrado es muy importante pueden
haber signos de anemia.
Por la especuloscopia debo descartar otro origen del sangrado vaginal ) vagina ,
cuello = y por el tacto tratar de descartar alguna causa estructural qie explique el
sangrado.

10)COMO ESTUDIAMOS ESTE CUADRO Y QUE RESULTADOS ESPERARIA ENCONTRAR EN
ESTOS EXAMENES ?

Existen dos examen especialmente tiles para el diagnostico etiolgico y para
definir la conducta a seguir:
Ecografa TV: a travs de esta podremos observar el tero y anexos de forma
completa, en bsqueda de masas o algn hallazgo patolgico. La medicin del
endometrio es lo mas importante en este examen, si el grosor endometrial es >
14 mm, hay que observar si este es homogneo o heterogneo. Si el
endometrio es homogneo hay que seguir el estudio con biopsia, por el
contrario si es heterogneo se puede comenzar el estudio con
histerosonografia, si esta arroja que el endometrio es el engrosado (sin lesin
focal) apunta a hiperplasia, y se requiere una biopsia.
Biopsia endometrial: por lo general se realiza a travs de un Pipelle (cnula
aspirativa) este al tener fenestraciones a lo largo de la cnula toma una biopsia
representativa de toda la cavidad endometrial. Las indicaciones para la
realizacin de biopsia son:
o Grosor endometrial por ecografa TV >14 mm con endometrio
homogneo..
o Endometrio hiperecogenico difuso o focal.
o Si no se logra visualizar de buena manera el endometrio durante la
ecografa TV.
o Sangrado persistente
Existen otros exmenes complementarios que se pueden realizar dependiendo de
caso a caso.
Dilatacin y curetaje: forma de biopsiar endometrio previo al Pipelle ya que
este ultimo demostr tener la misma sensibilidad con la mejora de realizarse
de forma ambulatoria.

11)CUAL ES EL TRATAMIENTO DE ESTA PACIENTE ?

319
El manejo de esta paciente corresponde realizar el estudio para confirmar si se
trata de hiperplasia u otro diagnostico diferencial, a travs de lo planteado
anteriormente. Dependiendo del resultado de la biopsia y el tipo de hiperplasia la
conducta ser:
Depender de la edad de la paciente y del tipo histolgico de la lesin.
o Si es con atipia: principalmente histerectoma por riesgo de
malignizacion.
Con deseo de fertilidad:
Medico: acetato de medroxiprogesterona 100 mg/da o
megestrol 80 mg/da por 3-6 meses
Quirrgico: legrado uterino
o Sin atipia
En edad frtil con deseo de embarazo: acetato de
medroxiprogesterona 10 mg/da por 10 a 14 das por 3 meses.
Edad frtil sin deseos de embarazo: gestagenos en 2do mitad del
ciclo, DIU levonorgestrel, ACO, danazol, anlogos GnRH,
histerectoma.
Peri y post menopausia: Gestagenos durante 3 meses,
histerectoma.
Bibliografa
- Ginecologa; Prez Snchez, 3ra edicin (2007), Capitulo 25 "Patologa benigna del aparato genital femenino" (L. Contreras,
C.L. Alvarado, A. Prez); pag. 440-445

- Manual Obstetricia y Ginecologa; J.A. Carvajal, C. Ralph, 4ta edicin (2013), Capitulo 59 "Patologa endometrial: benigna y
maligna"; pag. 619-628
- Manual Ginecologa y obstetricia CTO; M. Muoz, D.P. Hernndez, 8va edicin (2012), Capitulo 15 "Patologa del cuerpo
uterino y endometrial"; pag. 46-49
- OBSTETRICS, GYNAECOLOGY AND REPRODUCTIVE MEDICINE 20:4; 2010, Endometrial hyperplasia: clinicians review; M.M.
Hannemann, H.M. Alexander, N.J. Cope, N. Acheson, A. Phillips; pag. 116-120

69. METRORRAGIA POST MENOPAUSIA



Paciente de 63 aos consulta por presentar sangrado vaginal desde hace varios das.
Mide 1,60 mts pesa 78 kgs IMC 29. Utero en AVF, mvil, tamao normal. Anexos no se
palpan. La ecografa muestra un endometrio de 10 mm, resto n/e.
Antecedentes Familiares: padre infarto, madre Ca pulmon
Antecedentes Personales: hipertensin e hipotiroidismo
Antecedentes Gineco obsttricos: menarquia 13 aos FUR 50 aos, RM V/25
G2P2A0, no toma TRH.
Habitos: no refiere

320
1) Diagnostico mas probable:
- Multipara de 2
- Menopausica
- Sangrado uterino post menopausia Obs hiperplasia endometrial
- Hipertension arterial
- Hipotiroidismo
- Sobrepeso

2) Diagnsticos diferenciales:
- Atrofia endometrial
- Hiperplasia endometrial
- Miomas
- Polipos
- Cancer endometrial
- Cancer de cuello uterino
- Vulvovaginitis

3) Definicion:
Sangrado uterino post menopausia se define como sangrado uterino (proveniente
desde cavidad endometrial ) posterior al inicio de la menopausia.
La hiperplasia endometrial es la proliferacin anormal de glndulas
endometriales, con aumento en la relacin glndulas/estroma por falta de
estmulo progestativo. Se produce principalmente como consecuencia de
anovulacin y del uso exgeno de estrgenos en mujeres menopusicas, donde
existe un exceso de estrgenos que estimula el endometrio sin oposicin de
progesterona. Se produce mayor vascularizacin y fragilidad, sangrado irregular y
multifocal. El endometrio en estas condiciones es un precursor al cncer
ginecolgico ms comn: cncer de endometrio de histologa endometrioide.

En la edad frtil el endometrio ( medido post


regla ) normal mide entre 8 - 14 mm de
grosor, con un tope mximo de hasta 16
mm. Entre los 40 y 50 aos lo normal es que
el endometrio mida < 12 mm. En
pacientes menopusicas el endometrio debe
medir < 4 mm y si el grosor endometrial es
mayor hay que preguntar por uso de terapia
de reemplazo hormonal (TRH) con lo que
puede llegar hasta 7 mm.

4) Etiologa del cuadro clnico:


Principalmente el sangrado postmenopusico es causado por atrofia endometrial
(80%), seguido por lesiones benignas (plipos, miomas; 15%), hiperplasia
endometrial y luego cncer endometrial.

321

5) Fisiopatologa:
Va a depender de la causa de este. Dentro de las mas frecuetes estn:
- Atrofia endometrial: causada por el hipoestrogenismo que se produce luego de la
menopausia. Esto provoca una atrofia y colapso de la superficie endometrial, por
lo que existe una friccion intracavitaria, lo que provoca microerosiones del epitelio
y una inflamacin crnica de este, generando un sangrado escaso o spotting.
- Polipos endometriales: crecimiento endometrial focal de origen desconocido.
Causa sangrado durante la perimenoupausia o en la postmenopausia temprana. La
terapia estrogenica y el uso de tamoxifeno pueden estimular su crecimiento.
- Miomas uterinos: es un tumor benigno de miometrio, es una causa poco frecuente
de sangrado en la postmenopausia, ya que durante la menopausia los miomas
comienzan a disminuir de tamao por lo que se hacen menos sintomticos debido
al hipoestrogenismo. En los casos de miomas sintomticos durante la
postmenopausia hay que descartar un sarcoma uterino.
- Hiperplasia endometrial: al existir un hipoestrogenismo en la postmenopausia la
hiperplasia endometrial se explica por los altos niveles de estrgenos endgenos
producidos en mujeres obesas provocados por la conversin de androstenediona a
estrona y la aromatizacin de andrgenos a estradiol ocurrido en el tejido adiposo
perifrico. La hiperplasia endometrial puede clasificarse en:
o Simple: cambios en la arquitectura de las glndulas, pequeos quistes
o Compleja: aumento del nmero de las glndulas y de su tamao.
o Simple con atipias celulares.
o Compleja con atipias celulares.
Evolucion: la hiperplasias sin atipias se malignizan en un 1-3% mientras que las
con atipia 8-29% progesan en forma maligna.
- Cancer endometrial: causa menos frecuente de sangrado uterino
postmenopusico (95% de los sangrados postmenopusicos son de origen
benigno), pero siempre hay que descartarla. Sin embargo la metrorragia es el
principal sntoma del cncer endometrial. Tambien pueden producir sangrado
otros canceres como el sarcoma, cncer ovrico, trompas y cervicouterino.

6) Factores de riesgo:
son diferente segn la causa de metrorragia en la postmenopausia.
- Miomas: Raza (negra, prevalencia 50% vs blanca 20-25%), edad (>35-54 aos,
90%), miomas previos (generalmente multiples), TRH, terapia estrogenica /
progestgena.
- Polipos: edad (30-60 aos)
- Hiperplasia endometrial / Cancer endometrial: HONDA
o H: hipertension arterial
o O: obesidad
o N: nuliparidad

322
o D: diabetes mellitus
o A: anovulacion (ej. SOP)

7) Epidemiologa:
El sangrado post menopusico tiene una prevalencia cercana al 5%, en mujeres sin
TRH. El 95% es causa de patologa benigna. Entre las causas de este cuadro la mas
frecuente es la atrofia endometrial que causa el 80% de estos.

8) Diagnstico clnico:
Es un cuadro que puede caracterizarse nicamente por sangrado depues del inicio
de la menopausia. Por lo que se necesita una estudio complementario para
clarecer la etiologa. Se puede asociar a dolor, sntomas de compresin (mioma),
anemia.

9) Importante de preguntar en la anamenesis:
Es relevante hacer una anamenesis acusiosa
- FUR detallada
o cantidad de sangrado
o numero de episodios
o frecuencia
o caractersticas del sangrado: pueden ser relevantes y apuntar a ciertas
causas de la metrorragia, los plipos tienden a sangrar bastante ya que son
ricos en vasos sanguneos, los miomas con menos frecuencia producen
sangrados, mientras que la atrofia endometrial es un sangrado escaso o
spotting.
o sntomas asociados: compresin (mioma), dolor, CEG (anemia, cncer), etc.
o antecedentes de TRH (E, E+P),
o antecedentes de MAC,
o morbilidades: (HTA, DM, IMC, SOP, anovulacin) por aumento del riesgo de
hiperplasia y cncer endometrial principalmente.

10) Que buscar dirigidamente en el examen fsico:

El cuadro podra no dar muchas alteraciones del examen fsico. En algunos casos
se pueden observar signos de sindrome anmico si el sangrado es importante. Con
la espesculoscopa se podra descartar otra causa ginecologica de sangrado y
observar la presencia de sangrado a travs del OCE, cuantificarlo en caso que sea
posible, observar el OCE para verificar precencias de protrusiones (plipos) o
lesiones. En el tacto vaginal descartar presencia de dolor en el cuello y utero, y
palpacin de masas (miomas).

11) Cmo estudiar el cuadro? Qu resultado esperara encontrar en los examenes?:

323
Existen dos examen especialmente tiles para el diagnostico etiolgico del
sangrado y para definir la conducta a seguir:
- Ecografia TV: a travs de esta podremos observar el utero y anexos de forma
completa, en bsqueda de masas o algn hallazgo patolgico. La medicin del
endometrio es lo mas importante en este examen, ya que si el grosor endometrial
es < 4 mm, la causa mas probable del sangrado es la atrofia endometrial, la cual es
autolimitada, por lo que si persiste el sangrado habra que seguir el estudio. La
mayora de las patologas tanto benignas como malignas mantienen un
endometrio >4 mm, por lo que se biopsian.
- Biopsia endometrial: por lo general se realiza a travs de un Pipelle (canula
aspirativa) este al tener fenestraciones a lo largo de la canula toma una biopsia
representativa de toda la cavidad endometrial.
Las indicaciones para la realizacin de biopsia son:
o Grosor endometrial por ecografa TV > 4 mm.
o Endometrio hiperecogenico difuso o focal.
o Si no se logra visualizar de buena manera el endometrio durante la
ecografa TV.
o Sangrado persistente
Existen otros exmenes complementarios que se pueden realizar dependiendo de
caso a caso.
- Histerosonografia: Ante la sospecha de un plipo (ya se clnica, ecogrfica, etc).
- Dilatacion y curetaje: forma de biopsiar endometrio previo al Pipelle ya que este
ultimo demostr tener la misma sensibilidad con la mejora de realizarse de forma
ambulatoria.
- Citologia cervical (PAP): toda mujer con metrorragia post menopusica debe tener
estudio citolgico cervical, ya que el CaCu tiene una presentacin bimodal (35-
39/60-64).
- Histeroscopia : en caso de endometrio irregular o 2 episodio de sangrado con
endometrio atrofico

12) Tratamiento:
El manejo de esta paciente corresponde realizar estudio etiolgico del sangrado
por lo planteado anteriormente con los exmenes mencionados anteriormente.
Dependiendo de la etiologa la conducta cambiara segn esta.
- Atrofia endometrial: es auto limitada, la persistencia del sangrado ante el
diagnostico de atrofia endometrial debiera hacer sospechar otras causas y
continuar el estudio.
- Mioma: el manejo depende de la sintomatologa.
o Expectante: en pequeos, asintomticos y durante embarazo, con revisin
periodica.
o Tratamiento quirrgico: miomectomia (laparotoma, laparoscopia,
histeroscopia), histerectoma.
o Embolizacion: se realiza desde la arteria femoral a travs de la arteria
uterina embolizar la irrigacin del mioma reduciendo su tamao.

324
o Tratamiento medico: anlogos de GnRH.
o Sintomaticos: AINES y antifibrinoliticos (acido tranexamico)
- Polipos endometriales: extirpacin quirrgica mediante histeroscopia,
realizando estudio histolgico para descartar malignidad.
- Hiperplasia endometrial: este depender del tipo histolgico de la lesin, deseo
de fertilidad y edad de la paciente
o Hiperplasia endometrial sin atipia:
- Con deseo de fertilidad futura: hacer BEM de control (el 90% anda bien,
pero en las que persiste hay que buscar otras alternativas de
tratamiento.
Mdico: acetato de medroxiprogesterona 10 mg/da por 10 a 14
das por 3 meses (90% de xito)
Quirrgico: legrado uterino
- Sin deseo de fertilidad futura
Quirrgico: Histerectoma
o Hiperplasia endometrial con atipia:
- Con deseo de fertilidad futura:
o Mdico: acetato de medroxiprogesterona 100mg/da o
Megestrol 80mg/da por 3 a 6 meses. Estas pacientes deben ser
controladas con biopsia cada 3 a 6 meses.
o Quirrgico: legrado uterino
- Sin deseo de fertilidad futura:
o i. Quirrgico: Histerectoma

Bibliografa
Gua CEDIP 2014
Allison KH, Reed SD, VoigtLF, et al. Diagnosing Endometrial Hyperplasia. Am J Surg
Pathol 2008; 32(5):691-8.
Fernndez Parra J. Metrorragia menopasica. En: Guas de prctica clnica del Hospital
Universitario Virgen de las Nieves 2005.
Carvajal J, Ralph C. Manual obstetricia y ginecologa. Cuarta edicin (2013). Patologa
endometrial: benigna y maligna. Capitulo 59.

70. PROLAPSO ORGANOS PELVICOS.



Mujer de 62 aos consulta por bulto en genitales externos desde hace meses, aumenta
con posicin de pie y al final del da le dificulta la miccin. Presenta ITU a repeticin,
perdida de orina con urgencia miccional, frecuencia miccional aumentada y nicturia. Al
examen fsico se aprecia masa vaginal que protruye desde pared anterior.

Antecedentes Familiares : padre infarto , madre diabtica

325
Antecedentes Personales : IMC 28.
Antecedentes Gineco obsttricos : menarquia 13 aos , RM V/25 G5P5A0 menopausia a
los 52 aos .
Hbitos : no refiere
Trabaja en la feria con puesto de verduras

1. Diagnstico:
- Multpara de 5
- Post- menopusica
- Incontinencia urinaria de urgencia
- Prolapso vaginal.
- Sobrepeso

2. Diagnsticos diferenciales:

- Infeccin urinaria: que puede llevar a tenesmo vesical
- Divertculo uretral o abscesos de la glndula vestibular menor: que puede imitar
un cistouretrocele y en el caso de los divertculos pueden ser causa de
incontinencia
- Fistulas: en pacientes con ciruga plvica o radiacin previa

1.Definicin: prolapso vaginal corresponde al descenso de rganos plvicos atraves de la
pared vaginal anterior, apical, posterior o cpula vaginal .
Posee varias clasificaciones:
2.Clsicacion POP Q :

1. Grado 0: no tiene prolapso
2. Grado I: Punto de mayor descenso llega a mas de 1 cm sobre el himen
3. Grado II: Punto de mayor descenso llega a mas de 1 cm bajo el himen y no
sobresale mas de un cm del himen.
4. Grado III: Punto de mayor descenso llega a mas de 1 cm del himen y no es
mayor a 2 cm de la longitud vaginal
5. Grado III: procidencia genital completa, grado mximo, incluye vejiga, recto
y tero.
6.
2) Por contenido:
1. Rectocele: recto
2. Histerocele: tero
3. Colpocele: vagina (caso clnico expuesto)
4. Enterocele: intestino

326
5. Cistocele: vejiga

3) Por defecto de
pared:
1. Anterior
2. Posterior
3. Apical











4) POP-Q: sistema con 9 puntos de reparo que se miden en valsalva con una regla.
Este sistema est ideado para investigacin y unificacin de criterios. Los puntos
de reparo son:
1. Aa: menos de 3 cm del introito pared anterior.
2. Ap: menos de 3 cm del introito pared posterior.
3. Ba: Punto ms alto del tabique vaginal anterior.
4. Bp: punto ms alto posterior.
5. C: extremo distal del cuello uterino.
6. D: fondo de saco Douglas (fornix).
7. Gh: meato uretral a lnea media del himen.
8. Ph: distancia cuerpo perineal hacia orificio anal.
9. Tvl: total largo vaginal

327



3.Etiologa: los rganos del aparato genital se sostienen mediante una compleja
interaccin de msculos (msculos elevadores), fascias (diafragma urogenital,
fascia endopelvica) y ligamentos (uterosacro y cardinales). Cada una de estas
estructuras puede perder su capacidad para proporcionar sostn debido a un
traumatismo obsttrico (Ej. elevacin de presin intraabdominal; debilidades

328
intrnsecas, o atrofias provocadas por el envejecimiento o el hipoestrogenismo)
dando como resultado el prolapso de alguna de las estructuras antes descritas. El
descenso de las estructuras plvicas ocurre por ruptura o desinsercin de los
elementos de sostn .

4.Fisiopatologa: la vagina se sostiene por varios mecanismos de soporte.
- componente seo: La pelvis sea corresponde a un mecanismo fijo, no
modificable.
- componente muscular: Es el principal mecanismo de soporte y est
conformado por el msculo elevador del ano (3 fascculos: puborrectal,
pubococcgeo e ileococcgeo) y el msculo coccgeo.
- Ligamentos: el segundo componente en importancia corresponde a la fascia
endopelvica, estructura de tejido conectivo que discurre por sobre los
msculos del piso plvico. Luego el completo tero-retinculo conformado por
ligamento tero sacro, ligamentos cardinales, ligamento pubovesicouterino.
Finalmente el ligamento redondo y ligamento ancho son los de menor
importancia.

El soporte de los rganos plvico se mantiene por interacciones entre el musculo
elevador del ano, la vagina y el tejido conjuntivo del piso plvico. Cuando el
musculo elevador del ano esta indemne y tiene un tono normal y la vagina tiene la
profundidad adecuada, la parte superior de la vagina tiene una posicin casi
horizontal en la mujer de pie. Esto crea un efecto de vlvula en el que la vagina
se comprime contra el musculo elevador del ano durante los periodos de aumento
en la presin intraabdominal. Se postula la teora de que cuando el musculo
elevador del ano pierde tono, los ligamentos y fascias quedan sometidos a tensin
y al des insertarse o romperse la vagina cae de la posicin horizontal a una
inclinada. Esto ensancha o abre el hiato genital y predispone al prolapso de las
vsceras plvicas. Sin el soporte adecuado del elevador del ano, las inserciones
aponeurticas viscerales del contenido plvico se someten a tensin; estirndose y
finalmente vencindose. Esto puede ocurrir en tres niveles, segn Lancey :









329





5.Factores de riesgo:
- Embarazo , es el principal desencadenante
- Parto vaginal: 2 partos vaginales constituyen riesgo.
-
Gran multiparidad: ms de 5 embarazos.
-
Edad avanzada.
-
Obesidad
-
Parto instrumental: uso de frceps.
-
Hipoestrogenismo: menopausia.
-
Alteraciones del colgeno.
-
Tos crnica.
-
Tabaquismo
-
Constipacin
-
Macrosomia fetal
-
Trabajo pesado

6.Epidemiologia:

330
El prolapso de rganos plvicos es un problema de salud que afecta a millones de
mujeres en todo el mundo. En estados unidos es la tercera indicacin ms
frecuente para histerectoma, adems una mujer tiene un riesgo estimado en toda
la vida de 11% para someterse a una operacin por prolapso o incontinencia.
Entre 27-98% de las mujeres tendran algn grado de prolapso en la vida. Slo 3-
6% de los prolapsos sobrepasan el himen y son sintomticos.
Se calcula que el 20 % de la mujeres deber ser sometida a ciruga de prolapso y de
estas 1/3 ser re operada por recidiva.

7.Diagnostico clnico: el diagnstico es clnico, mediante anamnesis y examen fsico.
Lo ms habitual es la descripcin de sensacin de bulto y peso en relacin a los
genitales externos. Se confirma con el examen fsico ginecolgico observando el
descenso de estructuras con maniobras de valsalva con la paciente de pie y en
litotoma )paciente debe ser examinada con vejiga llena y luego vaca ) .

8.Anamnesis: los principales sntomas que refieren las pacientes y que hay que buscar
dirigida mente en la anamnesis son: perdida o retencin fecal o de orina; presin o
pesadez vaginal; dolor o molestias abdominales, lumbares, vaginales o perineales;
sensacin de bulto; dificultad para caminar, levantar objetos o sentarse;
hipertrofia, excoriacin, ulceras o hemorragias cervicales; dificultad con las
relaciones sexuales, y estrs o miedo relacionados con la ansiedad generalizada del
problema.
Existen dos cuestionarios de uso frecuente para una mejor aproximacin diagnstica
- Inventario de sntomas del piso plvico (PFDI): que valora sntomas urinarios, color
rectales y de prolapso
- Cuestionario de impacto del piso plvico (PFIQ): que valora el impacto del
prolapso en la calidad de vida.

9.Examen fsico: se debe realizar la inspeccin de genitales externos en reposo y en
valsalva, se puede observar masa protruyendo o cuello erosionado, tacto vaginal
explorando descenso de paredes vaginales o de tero con maniobra de valsalva, se
puede pesquisar prdida de orina. Importante determinar el grado de descenso y
la pared comprometida (Anterior, posterior, apical).

10.Estudio de la paciente: la ecografa es el nico estudio sugerido, para descartar quistes
o tumores, visualizar adecuadamente la anatoma del tero, endometrio, residuo
miccional y posible uro dinamia si hay dudas.

11.Tratamiento y conducta:
a. Prevencin (orientado en factores de riesgo prevenibles)
b. Medidas generales: baja de peso, cese del tabaco, mejorar tos crnica,
constipacin, manejo obsttrico adecuado.
c. Hormonoterapia en postmenopusicas para aumentar tonicidad de mucosa
vaginal.

331
d. Kinesioterapia: Ejercicios de Kagel, biofeedback, electro estimulacin. Los
ejercicios de Kegell corresponden a 3 series de 8-12 contracciones del piso
plvico sosteniendo por 6-8 segundos, deben realizarse 3-4 veces por semana
durante 15-20 das hasta 5 meses. El biofeedback corresponde a una tcnica de
re-educacin de los pacientes en la contraccin de los msculos del piso
plvico donde se utiliza una mquina que mide oscilaciones durante la
contraccin, debe ser dirigido por kinesilogo.
e. Tratamiento quirrgico est recomendado para pacientes sintomticas, o para
prolapso POP-Q desde Grado 2 a 4.
f. Existen varias tcnicas quirrgicas:
I. Obliterativas: La colpocleisis corresponde a la amputacin de la vagina,
donde se cierra la vagina, impidiendo que los rganos plvicos desciendan
ms all del introito. Slo se realiza en mujeres que no desean actividad
sexual.
II. Reconstructivas: Colporrafia corresponde a la tcnica de plastia de paredes
vaginales donde se extirpa un fragmento triangular de la mucosa vaginal y
luego se afrontan los extremos de mucosa en la vagina. Esta puede ser con
o sin malla, la malla est justificada en pacientes con recidiva del prolapso
anterior o en ausencia de fascia endopelvica. No se justifica su uso en
prolapso posterior ya que el 30% de las pacientes manifiestan dispareunia.
III. Colposuspensin: corresponde a la fijacin de la cpula vaginal a puntos de
soporte. La sacropexia o promontopexia es la ms utilizada, se fija la cpula
vaginal al promontorio a travs de una malla. Tambin existe mecanismo
de fijacin al ligamento sacro espinoso y al ligamento tero sacro. Las
colposuspensiones slo se realiza en prolapso de cpula vaginal en
paciente histerectomizada, ya que en paciente en edad frtil y con tero
existe riesgo de embarazo con la malla anclada al promontorio, lo que lo
transformara en un embarazo de alto riesgo.
g. El tratamiento quirrgico es correccin sitio especfico.
a. Defecto pared vaginal anterior Plasta anterior (colporrafia anterior
fascia pubocervical )
b. Defecto de pared vaginal posterior Plasta posterior (colporrafia
posterior fascia recto vaginal )
c. Defecto apical colposuspensin a lig tero sacro, sacro espinoso ,
colposacropexia o colpocleisis en pacientes sin vida sexual .

12. Consecuencias para la paciente: 20-30% de los prolapsos recidiva. Pueden aparecer
sntomas de incontinencia urinaria de esfuerzo por hipermobilidad uretral.




Bibliografa:

332
- Reid F. "Assessment of pelvic organ prolapse. Obstetrics, Gynaecology &
Reproductive Medicine, Volumen 21, Issue 7, July 2011, Paginas 190197.
- Hagen S, Thakar R. Conservative management of pelvic organ prolapse.
Obstetrics, Gynaecology & Reproductive Medicine. Volumen 22, Issue 5, May
2012, Pginas 118122.
- Weber A, Richter H. Pelvic Organ Prolapse. Obstetrics & Gynecology.
Volumen 106, Septiembre 2005. Pginas 615-634.
- Pelvic Organ Prolapse. Asociacion internacional de uroginecologa (IUGA).
2011. Disponible en:
http://c.ymcdn.com/sites/www.iuga.org/resource/resmgr/brochures/eng_po
p.pdf

71. INCONTINENCIA URINARIA DE URGENCIA



Mujer de 39 aos consulta por presentar perdida de orina frecuente asociada al deseo
de orinar , no alcanza a llegar al bao. Frecuencia miccional aumentada 10 veces al da y
3 en la noche .No sale por que le dan ganas de ir al bao y no alcanza a llegar .No pierde
orina con la tos.
Antecedentes Familiares : padre infarto , madre diabetica
Antecedentes Personales : esttica
Antecedentes Gineco obsttricos : menarquia 13 aos , RM V/25 G2P2A0
Hbitos : no refiere

1)Cul es el diagnostico mas probable de esta paciente?

1. G2P2A0
2. Urgencia miccional
3. Incontinencia urinaria de urgencia

2)Cuales son los diagnsticos diferenciales?

- Incontinencia urinaria de esfuerzo: escape involuntario de orina durante los
esfuerzos , estornudos o tos. Se produce fuga de orina por el de la presin
abdominal (como, por ejemplo toser, rer, correr o andar) y en ausencia del
musculo detrusor.
- Pseudo incontinencia: Escape involuntario de orina a causa de fstulas, urter
ectpico, tumores, cistitis crnica.
- Vejiga neurognica: alteracin vesico-esfintereana relacionada a una patologia
neurolgica.
- Malformacin congnita: desembocadura anormal del urter directo a uretra:
goteo constante.

333
- Vegiga hiperactiva
- Infeccion urinaria.
- Malos hbitos miccionales.
- mixta

3) Cmo se define esta patologa?

La incontinencia urinaria (IU) es cualquier perdida de orina, es un sntoma, un
signo y un transtorno.
La IE de urgencia (IUU) dificultad para posponer la miccin y casi siempre deben
vaciar la vedija pronto a la primera seal sin demora,.

4)Cual es/son la/s etiologa de este cuadro clnico?

Comportamiento
Exceso consumo de liquido
Cafena
ansiedad

Inflamatoria
ITU
Cistitis insterticial o por radiacin o quimica
Cncer de vejiga
litiasis

Neurolgica
Esclerosis multiple
AVE
Parkinson
Demencia
Tumores SNC
Dao Espina dorsal

Endocrina
Atrofia genital urinaria
diabetes
Anatomica
obstruccion vejiga
masa plvica
divertculo
embarazo

Causas neurolgicas

334

Esclerosis mltiple.
Demencia.
Enfermedad de Parkinson.
Diabetes.
Enfermedad vascular cerebral.


Causas no neurolgicas
Problemas de vejiga.
Tumores.
Alteracin de la pared.
Clculos.
Infeccin.
Llenado rpido.
Dficit estrognico.
Uso de diurticos.
Histerectoma.

5)Cul es la fisiopatologa de este cuadro?

La Teora Integral dice que :
La incontinencia urinaria de esfuerzo, la incontinencia de urgencia y la
disfunciones de vaciamiento vesical son consecuencias de las alteraciones que
ocurren en los elementos de soporte uretral, de los ligamentos y de los musculos
del piso pelvico.Para ello se divide en 3 grupos :

3 Ligamentos: pubouretral
uretroplvico
sacrouterino

3 Grupos musculares
3 Zonas de disfuncin

Incontinencia Urinaria de Urgencia:
Ocurre por Contracciones involuntarias del detrusor no inhibidas durante la fase de
llenado
1. Vejiga hiperactiva(VH): se produce a consecuencia que los nervios envan
seales a la vejiga en el momento incorrecto, causando que se contraigan los
msculos de la pared de la vejiga sin aviso. Varias afecciones crnicas comunes,
tales como depresin, estreimiento, trastornos neurolgicos y disfuncin
erctil, se han asociado de manera significativa a VHA, incluso despus de
ajustar respecto a variables importantes, como la edad, el sexo y el pas .

335
2. Destusar inestable: Afeccin en la que el msculo depresor se contrae de
forma espontnea o provocada, durante el llenado vesical, mientras la persona
intenta inhibir la miccin. La contraccin involuntaria del depresor, es debido a
la contraccin vesical ocurre como consecuencia de la activacin de los
receptores muscarnicos existentes en el detrusor por la acetilcolina.

5) Qu factores de riesgo buscara?

1. Edad
2. Parto y embarazo
3. Menopausia
4. Obesidad
5. Factores uro ginecolgicos
-Prolapso genital
-Debilidad de la musculatura del piso pelviano
-Estreimiento
-Ciruga uro ginecolgica previa
-Histerectoma
6. Otras patologas
-Parkinson
-Esclerosis mltiple
-Radiacin previa
7. Medicamentos: Bloqueador alfa adrenrgico, antihipertensivo neurolpticos,
benzodiacepinas.
8. Raza negra
9. Tabaquismo, Enfermedad pulmonar crnica

6) Cul es la epidemiologa de este cuadro?

La IUU es mas frecuentes en mayores de 75 aos siendo ms frecuente en la mujer
postmenopusica. En la mujer la prevalencia de IUU es entre el 11% y el 20%. En la
mujer joven la mas frecuente es la IUE.

7) Cmo hacemos el diagnostico clnico?

El diagnstico de IUU es clnico, poliaquiria >8 veces al da, urgencia miccional,
grandes fugas de orina, no alcanza a llegar al bao , nicturia usual.

8) Que nos interesa preguntar en la anamnesis de esta paciente?

Los antecedentes de un paciente son sumamente importantes en la evaluacin de
la incontinencia urinaria.

336
- Edad
- Ciruga (plvica, ginecolgica, abdominal, columna vertebral o del tracto urinario).
- Patologa neurolgica (accidente vascular cerebral).
- Patologa sistmica (diabetes, enfermedades degenerativas).
- Frmacos Hbitos del alcoholismo.
- Estilo de vida.
- Incontinencia fecal.
- Estreimiento.
- Traumatismo.
- Antecedentes obsttricos: GPA, peso, frceps
- Historia sexual
- Historia colorectal: constipacin, incontinencia
- En el interrogatorio se tiene que tener en cuenta:
- Consumo de lquido
- Volumen urinario
- Episodios de escape de orina
- Factor causante de incontinencia
- Horario sueo vigilia

10) Qu buscara dirigida mente en el examen fsico de esta paciente?

-Observar: caminar, equilibrio, temblores
-Atrofia del perine
-Diverticulo uretral
-Evaluacin neurolgica: reflejo bulbocavernoso y guio anal
-Soporte plvico: prolapso de organos pelvicos, fuerza musculos pelvicos
-Perdida de orina con Valsalva


11) Cmo estudiamos este cuadro y que resultados esperara encontrar en estos
exmenes?

1. Estudio de orina: orina completa y cultivo de orina, para descartar infeccin
urinaria
2. Volumen residual postmiccional: por ecografa o sondeo transuretral. El
normal es de 50 ml.
3. Estudio Uro dinmico: Permite el ms acabado anlisis de las variables
fisiopatolgicas que intervienen en la acumulacin de orina y la miccin.
Adems, establece con seguridad la presencia de inestabilidad vesical y la
presencia o ausencia de hipocontrabilidad del detrusor, la incompetencia
esfinteriana, disinergia del esfinter .
nico mtodo para detectar dficit intrnseco del esfnter
> 90 cms H20 competente
<60 cms de H20 lesin del esfnter

337
Cistomanometria : para evaluar la funcin del musculo detrusor y determinar
el VLPP ( valsalva pick point pressure) y as determinar la severidad del cuadro.
Uroflujometra: se le pide al paciente que vaci su vedija conectado a un
medidor de flujo, despus de registrar la velocidad mxima de flujo, se coloca
una sonda para medir el volumen residual postmiccional, y se identifica la
retencin urinaria.

12) Cul es el tratamiento de esta paciente?

- Tcnicas conductuales: reentrenamiento vesical (educacin, diario miccional,
refuerzo positivo), entrenamiento de hbitos, dieta (restriccin de OH, caf,
mayor ingesta de lquidos), ejercicios, evitar baos de espumas
- Farmacolgico: fundamental. Reducir actividad no deseada del detrusor a
travs de un bloqueo reversible de los receptores muscarnicos de la unin
neuro muscular del detrusor.
- 5 tipos de receptores muscarinicos , con M2 y M3 los ms importantes en
detrusor
o Oxibutinina 2,5 5 mg da.
o Tolterodina 1 2 mg da .
o Trospium 20 mg c 12 horas.
o Solifenacina. 5 10 mg da.
o Darifenacina 7,5 15 mg da.
- imipramina : anticolinrgico y adrenrgico alfa (mixto)
Efectos colaterales: xerostoma, estreimiento y visin borrosa contriindicado
en glaucoma de angulo estrecho.
- Ejercicios de Kegel: tambin pueden resultar beneficisioso en el alivio de los
sntomas
- Conos vaginales: fortalece los msculos del piso plvico
- Cuando est contraindicado los medicamentos o mala respuesta a ellos, se
puede ofrecer biofeedback, electro estimulacin, estimulacin magntica e
inyeccin de toxina botulnica tipo A en el detrusor. La ciruga es una
alternativa final disponible.

13) Qu consecuencias puede traer esta patologa a la mujer / madre/ y al feto ?

Deterioro significativamente la calidad de vida de quien la padece, ya que
reduce su autoestima y merma su autonoma.

Bibliografa
- Chang M., Dr. Humberto. (2013). Incontinencia Urinaria.
REV.MED.CLIN.CONDES, 24 (2), 219-227.
- Asociacion espaola de ginecologia
http://www.aeu.es/UserFiles/IncontinenciaUrinariaInterna_rev(1).pdf

338
72. INCONTINENCIA URINARIA DE ESFUERZO

Mujer de 45 aos consulta por prdida de orina desde el primer parto hace 7 aos , que
ha aumentado con los otros partos llegando a utilizar toallas absorbentes , aumenta
sobre todo con la tos y la fuerza.
- Antecedentes Familiares : padre infarto , madre diabtica
- Antecedentes Personales: Obesa y tosedora crnica.
- Antecedentes Gineco obsttricos : menarquia 13 aos , RM V/25 G3P3A0 RN
4.100 grs 1 frceps
- Hbitos: no refiere

1. Diagnstico
Multpara de 3
Incontinencia urinaria: de esfuerzos (IUE)
Obesa
Tosedora crnica

2. Diagnsticos diferenciales
o Incontinencia urinaria de urgencia
o Pseudo incontinencia: Prdida involuntaria de orina por causa de fstulas,
urter ectpico, tumores, cistitis crnica.
o Vejiga neurognica: Disfuncin vesico-esfintereana asociada siempre a
trastorno neurolgico, es poco frecuente, pero grave, ya que puede
evolucionar a insuficiencia renal crnica y muerte.
o Fstula vesico-vaginal o vesico-uterina: (post ciruga ginecolgica).
o Malformacin congnita: desembocadura anormal del urter directo a
uretra: goteo constante.
o Leucorrea
o Malos hbitos miccionales.
o Confusin sintomtica femenina (aprox. un 30% de las mujeres no conoce
su anatoma).

3. Definicin

La incontinencia urinaria (IU) es cualquier perdida de orina .
4. Clasificacion

La IU se clasifica en:
o Incontinencia urinaria de esfuerzo (IUE): es la prdida involuntaria de orina
que se asocia al esfuerzo fsico. El ejercicio provoca un aumento de la

339
presin abdominal (como, por ejemplo toser, rer, correr o andar) y, como
consecuencia, se produce un incremento de la presin dentro de la vejiga
(presin intravesical). Cuando esta presin intravesical supera a la presin
del sistema esfinteriano uretral, se ocasiona la incontinencia. Antiguamente
conocida como tipo I y II. Son grados progresivos del mismo fenmeno.
o Incontinencia urinaria de urgencia: contracciones involuntarias del detrusor
no inhibidas durante la fase de llenado. Vejiga hiperactiva. Detrusor
inestable
o Incontinencia urinaria mixta: coexisten ambas incontinencias.
o Incontinencia urinaria por rebosamiento: obstruccin o alteracin
contrctil del detrusor.
o Incontinencia urinaria anatmica: fstula vesico-vaginal

5. Etiologa

o Prdida del soporte del piso plvico:
debilitamiento del piso plvico, y del
ligamento que sostiene la uretra al pubis )
pubo uretral y uretroplevico , lo cual causa
una movilidad exagerada de la misma,
impidiendo el cierre hermtico de la uretra ,
puede ser por :
o Hipermovilidad uretral .
o Deficiencia intrinseca del esfincter .

6. Fisiopatologa

La Disminucin en la transmisin de la presin intra abdominal a la uretra
proximal, debido a la prdida del soporte de los rganos plvicos por dao en
la base msculo aponeurtico del perineo caracterizado por hipermovilidad
uretral.
a) Hipermovilidad uretral: Prdida de sustentacin piso plvico y
desplazamiento ngulo uretro vesical
b) Deficiencia intrnseca esfnter: La uretra es incapaz de generar suficiente
resistencia de salida para mantener la uretra cerrada durante el reposo
con mnimos esfuerzos

Soporte uretral intacto y la uretra abierta por falla en:

o Musculo liso
o Musculo estriado

340
o Inervacin
o Elasticidad tejido conectivo
o Coaptacin mucosa
o Plexo venoso
o Debilidad tisular congnita
o Ciruga previa
o Trauma
o Parto
o Edad

7. Factores de riesgo

o Menopausia o deficiencia de estrgenos
o Partos mltiples
o Obesidad
o Cirugas plvicas previas
o Condiciones neurolgicas existentes:
parkinson, esclerosis mltiples
o Esfuerzo
o Parto traumtico antiguo
o Diabetes Mellitus.

8. Epidemiologa

La IUE es muy prevalente en las mujeres de edad avanzada y supone
tambin la forma ms comn de IU en las mujeres menores de 75 aos,
afecta a casi un 50% de ellas. Se asocia, sobre todo, a la obesidad y a los
partos mltiples, as como a la dificultad funcional (problemas en la
marcha, etc.).

9. Diagnstico clnico

Es generalmente un diagnostico de hallazgo por el medico, muchas
pacientes asumen como normal el orinarse despues de un parto y no lo
relatan

- Anamnesis prxima y remota, examen fsico y estudios complementarios
deben orientarse hacia:
Facilitar la consulta por esta causa
Objetivar y cuantificar la prdida de orina
Determinar el impacto sobre la calidad de vida

341
Establecer hiptesis diagnstica respecto al tipo de incontinencia y
su enfermedad causal
Descartar patologa urolgica asociada
Descartar condiciones generales agravantes o desencadenantes
asociadas
- Historia clnica:
o Diario miccional
o Valoracin de la percepcin e impacto de la
incontinencia sobre la calidad de vida de la
paciente mediante el uso de cuestionaros.
o Exmen fsico y neurolgico bsico:
sensibilidad y reflejos perineales
o Prueba de esfuerzo

(estudios complementarios)
o Anlisis de orina y estudio de residuo post
miccional
o Funcin renal
o Ecografa renal
o Uretrocistoscopa
o Mtodos urodinmicos: para planteo de tto
quirrgico o tras el fracaso de correccin de la
misma.( Esta indicado cuando no se visualiza
perdida de oriuna al Valsalava y la paciente
asi lo relata, en IO recidivado post cirugia y
cuando hay dudas diagnosticas )

10. Anamnesis

Los antecedentes de un paciente son sumamente importantes en la evaluacin de
la incontinencia urinaria.
o Ciruga (plvica, ginecolgica, abdominal,
columna vertebral o del tracto urinario).
o Patologa neurolgica (accidente vascular
cerebral).
o Patologa sistmica (diabetes, enfermedades
degenerativas).
o Frmacos (hipnticos, anticolinrgicos,
diurticos, calcio antagonista, beta
estimulantes o alfa bloqueantes).
o Hbitos del alcoholismo.

342
o Estilo de vida.
o Incontinencia fecal.
o Estreimiento.
o Traumatismo.
o Antecedentes obsttricos: GPA, peso, frceps
o Historia sexual
o Historia colorectal: constipacin,
incontinencia
o Medicamentos:
o Aumentan incontinencia: Bloqueadores alfa
(Prazosin, Dozazosin, Teratozin ( hytrin)
o Mejoran continencia: Agonista alfa (Efedrina,
Fenilefrina, Fenilpropanolamina, Imipramina
(efecto mixto, tambien anticolinrgico))
o En el interrogatorio se tiene que tener en
cuenta:
o Como fue el inicio insidioso o brusco.
o Cuando y circunstancias de presentacin, tos,
risa o esfuerzo.
o Frecuencia.
o Duracin.
o Intensidad.
o Horario diurno o nocturno.
o Sntomas acompaantes.

11. Examen fsico

o Observar: caminar, equilibrio, temblores
o Evaluar obesidad y estrogenismo
o Realizar examen neurolgico: estado mental
y reflejos
o Trofismo vulva vaginal
o Presencia de uretro y/o cistocele (prolapso)
o Hipermovilidad de la uretra
o Descartar masas periuretrales
o Evaluar el estado de la musculatura plvica
o Test Marshall/Bonje: test que objetiva el
escape el escape urinario uretral en relacin a
valsalva y evaluar su cambio en respuesta a la
maniobra de suspender la uretra con
compresin lateral de sta, sin obstruirla, de

343
modo para evaluar si bajo condiciones de
correccin de hipermovilidad, la continencia
mejora.
o Evaluar la presencia de cicatrices en rea
abdominal baja y perineal
o Tacto rectal
o Medicin de residuo postmiccional: ver
mecanismo involucrado

12. Estudio de la paciente

o Orina completa y Urocultivo: siempre hay que descartar infeccin
o Hemograma: puede estar normal
o Glicemia: alterado si paciente es DM
o Estrs test : llenar vejiga 250 ml y hacer toser , pujar ; acostado y de pie:
positiva luego colocar 2 dedos bajo uretra media y sin ascender la vagina
retpetir la prueba para ver si persiste IO .
o Test Bonney: observar prdida de orina con el esfuerzo y luego con
reduccin del prolapso uretral repetir maniobra valsalva. Si hay prolapso es
prueba positiva.
o Ecografa perineal: descenso del cuello vesical
o Urodinamia: Diferencia esfuerzo de urgencia. Es la Presin de fuga
abdominal. nico mtodo para detectar dficit intrnseco del esfnter. > 90
cms H20 competente (urgencia), <60 cms de H20 lesin del esfnter
(esfuerzo).
o Si Pierde orina en posicin supina/pierde orina con vejiga vaca VLPP < 60
cm h20 y MUCP < 20 cm h20: sospecha de deficiencia intrnseca del
esfnter.
*Abdominal LPP o Valsalva LPP: presin abdominal o presin de valsalva en
el punto de incontinencia). Es una medida de la competencia uretral o de la
capacidad de la uretra para resistir la fuerza expulsiva de la presin
abdominal. Esta nos permite clasificar el tipo de incontinencia (0,I o II) y
adems permite evaluar la incontinencia urinaria tipo III o incontinencia por
insuficiencia esfinteriana intrnseca.
*MUCP (Mxima presin uretral de cierre): es la mxima diferencia entre la
presin uretral y la presin intravesical.

13. Tratamiento

A) Rehabilitacin del piso plvico:

344
a. Ejercicios de Kegel: son ejercicios de contraccin voluntaria de los
msculos del piso plvico con el objetivo de fortalecerlos, realizarlo
durante 3 meses y evaluar.
b. Bioretroalimentacin: es un proceso que ayuda a ganar control sobre
las funciones corporales al hacer que tenga ms conocimiento de ellas.
c. Estimulacin elctrica: ayuda a los ejercicios del piso plvico mediante
el aislamiento de los msculos comprometidos.
B) Tratamiento mdico: no es muy efectivo, sin embargo algunos casos
particulares, por dficit estrognico, responden con tratamiento de
reemplazo hormonal local, con cremas u vulos vaginales. Duloxetina

o Ciruga: proporciona resultados ms confiables y permanentes. Existen
diversos procedimientos quirrgicos para la incontinencia urinaria de
esfuerzo, que han evolucionado hasta la creacin en la actualidad de
tcnicas menos agresivas, mnimamente invasivas, que consiste en el
sistema de cinta vaginal, en la que se coloca una malla de polipropileno
macroporo con forma de cinta por debajo de la uretra media con la
finalidad de sustituir el ligamento que sostiene naturalmente a la uretra.Es
por via Vaginal con aguja de TOT: Tcnica fcil, rpida y con menor
morbilidad que la retropbica.
o Abdominales abierta o Laparoscopia : Op Burch

14. Consecuencias para la paciente/feto

No es una enfermedad que ponga en peligro la vida del paciente, pero
deteriora significativamente la calidad de vida de quien la padece, ya que
reduce su autoestima y merma su autonoma. Itu a repeticion
Deterior de la esfera sexual

CLASIFICACIN INCONTINENCIA ORINA (BLAIVAS)

o Tipo 0: Paciente refiere IOE que no es comprobada clnicamente ni por
urodinamia.
o Tipo I: Cuello vesical en reposo sobre borde inferior pubis y desciende hasta
2 cm con valsalva.
o Tipo II: Cuello vesical cerrado en reposo a 2 o + cm del borde inferior del
pubis.
o Tipo III: Cuello y uretra abierta en reposo , pierde orina fcilmente

TOT estaria indicado en los tipos I II y III , para el tipo IV se propone TVT o inyecion
peri uretral .

345

Bibliografa
- Chang M., Dr. Humberto. (2013). Incontinencia Urinaria. REV.MED.CLIN.CONDES,
24 (2), 219-227.

73. PATOLOGA MAMARIA BENIGNA : MASTODINIA.



Mujer de 2 aos que consulta por dolor mamario bilateral de 3 meses de evolucin y de
intensidad creciente. El examen mamario es normal, solo mama sensible ++
Antecedentes Familiares: padre infarto, madre diabtica
Antecedentes Personales: hipotiroidismo
Antecedentes Gineco-obsttricos: menarquia 13 aos, RM V/25, G0P0A0, FUR
Hace 18 DIAS
Hbitos: no refiere

1. Diagnsticos:
- Nulpara
- Mastodinia
- Hipotiroidismo

2. Diagnstico diferencial:
- Radiculopata cervical
- Costo condritis
- Mialgia del pectoral mayor

3. Definicin: dolor de mama. Se puede clasificar en:
- Dolor cclico: El ms frecuente, el dolor aparece y desaparece con el ciclo
menstrual presentndose con mayor dolor en la fase lutenica y cede con el
inicio de la menstruacin. Puede ser bilateral o unilateral, es ms intensa en
los cuadrantes supero externos y coexiste con nodularidad difusa. La
intensidad es variable, suele iniciarse a los 30 aos y sigue un curso de recadas
crnicas hasta la menopausia.
Dolores severos, incapacitantes, que duran ms de una semana podran entrar
en el concepto de enfermedad benigna.
- Dolor no cclico: El segundo en frecuencia, sin relacin con el ciclo menstrual.
Se da en pre y postmenopausia; suele ser unilateral y localizado, sobre todo
retro areolar y en cuadrantes supero externos. La nodularidad es infrecuente.

346
Suele ser transitorio y episdico. A veces se presenta un trigger spot (punto de
gatillo), donde la presin produce el cuadro de dolor. Se asocia
frecuentemente con fenmenos de fibrosis y retraccin ductales.
- Dolor de la pared torcica: en la mayora de los casos es unilateral, localizado,
con buena respuesta a anti-inflamatorios locales. Puede presentarse a
cualquier edad. Hay que considerar aqu las osteocondritis (sndrome de
Tietze), el trauma y el dolor referido.

4. Etiologa: no est bien aclarado.
- Retencin acuosa: Se incrementa en fase premenstrual.
- Factores endocrinos:
Aumento de la secrecin de estrgenos ovricos.
Dficit de secrecin de progesterona (hiperestrogenismo relativo)
En casos de mastalgia cclica s se ha observado un incremento de la
respuesta de la FSH y LH al estmulo de la Gn-RH, siendo normales
sus valores basales.
Hiperprolactinemia: Si bien el incremento de la prolactina se asocia
inicialmente con una inflamacin mamaria, la Hiperprolactinemia
puede persistir y sin embargo normalizarse la sintomatologa
mamaria.
Hormonas tiroideas: En el hipotiroidismo se aprecia una mayor
tendencia a la aparicin de clnica de Mastopata fibroqustica.
- Otros factores:
Ingesta de metilxantinas (caf, t, chocolate, etc.): La mama se
hiperestimulara por la interaccin de estas sustancias sobre la
degradacin del ATP.
Dficit de cidos grasos esenciales: Conducira a una produccin
deficiente de prostaglandina E2.
Psiconeurosis

5. Epidemiologa: ms del 60% de las mujeres refieren dolor en mama, y un 21% de
manera intensa.

6. Anamnesis:
- Carcter: Sensibilidad, pesadez, quemazn, etc.
- Periodicidad: Continuo o intermitente
- Patrn circadiano
- Duracin

347
- Intensidad
- Lado dominante
- Distribucin en la mama e irradiacin
- Factores agravantes: Contacto fsico, movimientos, posturas, etc.
- Factores atenuantes: Analgsicos, calor local, etc.
- Alteracin del estilo de vida: Insomnio, problemas maritales, impide
abrazar y ocuparse de sus nios, etc.

7. Examen fsico: evaluar presencia de mamas, dolor a la palpacin y galactorrea.

8. Estudio de la paciente: los signos de esta patologa son muy difciles de
diagnosticar y cuantificar. Inicialmente hay que descartar otros trastornos
mamarios haciendo un interrogatorio adecuado, exploracin fsica y
cuando est indicado exmenes complementarios como mamografa y
ecografa mamaria .
Ej: mamografa, ultrasonografa, aspiracin con aguja fina y biopsia abierta.
En caso de duda, solicitar pruebas tiroideas, prolactina, FSH, etc. (No es de
rigor)

9. Tratamiento y conducta:
- Tranquilizar a la paciente y descartar cncer: ello basta para prcticamente
el 85% de las pacientes. Las guas de prctica clnica de la Sociedad
Canadiense de Gineco-Obstetricia del 2006, hacen una recomendacin I-A,
para el uso de ste tipo de intervenciones como parte integral y de primera
lnea del manejo de la mastalgia.
- Reglas higinicas: uso de sujetador sin elementos traumticos. Eliminacin
de las metilxantinas (t, chocolate, caf, otros) de la dieta
- Diurticos: No hay buena evidencia que soporte su uso en el manejo de la
mastalgia.
- Progestgenos: basados en la hiptesis de la insuficiencia ltea han
demostrado una actividad dudosa en estudios aleatorizados frente a
placebo.
- Bromocriptina: como antiprolactnico a dosis de 2.5 mg al da ha
demostrado gran efectividad en pacientes con mastalgia cclica en dos
estudios clnicos controlados, aleatorizados, con una reduccin absoluta en
las escalas anlogas del dolor de un 40 45%. No obstante cabe contar con
sus molestos efectos secundarios como cefalea, mareo, hipotensin
postural, nusea entre otros, que han limitado su uso. No aprobado por la
FDA para sta indicacin

348
- Danazol: como antigonadotrofnico, a dosis inicial de 200 mg al da, para
disminurla a 100 mg al da como mantenimiento, ha demostrado ser el
frmaco ms potente en el manejo de la mastalgia. Su eficacia se ha
establecido en un alivio significativo de la mastalgia en un 70% a un 93% de
los casos. Los efectos adversos que incluyen aumento de peso,
irregularidades menstruales, virilizacin, flushing facial entre otros, ha
limitado su uso. Para minimizar sus efectos se propone darlo nicamente
en la fase ltea. Es el nico medicamento aprobado por la FDA para el
tratamiento de la mastodinea
- Tamoxifeno: como anti estrgeno, a dosis de 10-20 mg al da, ha mostrado
eficacia en el tratamiento de la mastalgia, pero sus efectos adversos como
el riesgo de eventos tromboemblicos, cncer endometrial y la posibilidad
de inducir tumores hepticos ha recomendado relegarlo a una segunda
lnea. No aprobado por la FDA para esta indicacin.
- Aceite de onagra o prmuro: Rico en EFA (72% de cido linoleico y 7% de
cido gamma-linolnico) se ha utilizado en la mastalgia cclica. Tiene la
ventaja de ofrecer a la paciente un producto natural.
- AINES Tpicos
- Piridoxina (Vitamina B6): Se basa en que favorece la decarboxilacin de
dopa a dopamina, se supone que inhibira los niveles de prolactina. Se ha
usado a dosis de 200 mg al da, sin resultados clnicos significativos.

Tratamiento de la mastalgia NO cclica
- AINES
- Vitamina A: evidencia insuficiente para realizar una recomendacin.
- Vitamina E: Se han descrito algunos resultados con 600 UI al da de acetato
de tocoferol sinttico.
- Tranquilizantes: Si se asocia un cuadro de ansiedad, pero deben darse con
precaucin para evitar secundariamente cuadros de galactorrea y tensin
mamaria iatrgenicas.
- Semilla de linaza: un estudio canadiense examin los efectos de la linaza
en forma dietaria para el manejo de la mastalgia cclica severa, pacientes
recibieron una dosis de 25 mg de semilla de linaza al da, con una
importante mejora de la mastodinea.

10. Consecuencias para la paciente:
- De causa benigna, es importante descartar otro tipo de patologas.
- Ansiedad: Tranquilizar a pacientes y advertir curso benigno.

349

Bibliografa:
- Prez A, Donoso E. Prez Snchez Obstetricia. Cuarta edicin (2011).
Enfermedades benignas de la mama. Capitulo 46. Paginas 818 835.
- Beckmann C. Obstetricia y Ginecologa. Sexta edicin (2010). Desorders od the
breast. Capitulo 31. Paginas 283 294.
- Rungruang B, Kellley J. Benign Breast Diseases: Epidemiology, Evaluation, and
Management. Clinical obstetrics and gynecology. Volumen 54, Numero1, 110
124 .
- Onstad M, Stuckey A. Benign Breast Disorders. Obstetrics and Gynecology
Clinics of North America. Volumen 40. Publicacin 3. Septiembre 2013, Paginas
459473.

74. PATOLOGA MAMARIA BENIGNA : MASTOPATA


FIBRO-QUISTICA)

Mujer de 41 aos consulta por dolor y ndulo en mama izda. Desde hace 5 meses nota dolor en
ambas mamas, principalmente en CSEMD que no se relaciona con la regla. Se palpan mamas
congestivas y sensibles, se palpa nodulo de 3x2 cm en CSEMD de consistencia elstica y sensible,
mvil y sin elementos inflamatorios. Examen axilar normal.
Antecedentes Familiares: padre infarto, madre diabetica
Antecedentes Personales: mastodinea hace 10 aos
Antecedentes Gineco obsttricos: menarquia 13 aos, RM V/25 G2P2A0 usa DIU.
Hbitos: no refiere

1) Diagnsticos:
- Multipara de 2
- Mastalgia / Mastopatia fibroquistica

2) Diagnsticos diferenciales
- Fibroadenoma mamario
- Adenoma
- Quiste mamario
- Tumor filodes
- Cancer mamario

3) Definicin: la mastopata fibroqustica (MFQ), tambin llamada displasia mamaria, es una
enfermedad benigna y crnica, que consiste en la proliferacin del estroma y del
parnquima mamario desarrollando tumores o quistes palpables.

350
4) Etiologa: se desconoce la causa; se ha postulado la influencia hormonal, con
hiperestrogenismo, lo que no permanece constante en todos los casos.

5) Fisiopatologa: no es clara, ya que algunos la consideran cambios normales en el desarrollo de
la mama. Se postula la influencia de un desbalance hormonal, principalmente un
hiperestrogenismo que estimulara la proliferacin del parnquima mamario
constituyendo quistes y tumores, pero este desbalance hormonal no se repite en todas las
pacientes con MFQ.

7) Epidemiologa: es una condicin tan frecuente que algunos no la clasifican como enfermedad,
sino que sera una alteracin del normal desarrollo de la mama (ANDI). Por lo que su
prevalencia no es clara por su alta frecuencia, y porque en un gran nmero de casos es
asintomtica y subclinica.

8) Diagnostico clnico: el sntoma mas frecuente es el dolor mamario, que en su mayora es
premenstrual y mejora con la regla (mastodinia), es bilateral. Ademas puede aparecer
ndulos mamarios palpables e induracin mamaria. El embarazo y la lactancia producen
un estado de reposo en el que la sintomatologa disminuye.

9) Anamnesis: por la clnica que presenta siempre hay que preguntar por antecedentes familiares
de cncer mamario u otro relacionado. Historia personal de cncer de mama.

10) Examen fsico: el examen fsico puede ser normal en muchas pacientes con MFQ. Dentro de
los hallazgos mas frecuentes se encuentra el nodulo palpable, uni o bilateral, induracin
mamaria, dolor a la palpacin. Por lo general el nodulo palpado es de consitencia gomosa,
regular, elstico, no adherido, y en ocaciones doloroso.

10) Cmo estudiar el cuadro? qu resultados espera encontrar en los exmenes?: el
hallazgo se basa en la clnica principalmente, una vez sospechado se necesita el estudio
con imgenes para descartar malignidad de la patologa. Por lo que se realiza:
- Mamografia y/o Ecografia mamaria: esto va a depender de edad, consistencia de la mama,
y resultado no definido de alguno, principalmente para descartar signos de malignidad.
- Citologia e histologa: esto no se realiza de rutina, y va a dpender del resultado de la
imagen solicitada con anterioridad. Si existe algn signo sospechoso de malignidad o poca
certeza en dichos estudios, se deber realizar este estudio:
Puncion-aspirativa: en las MFQ que predomina la forma nodular-quistica.
Biopsia: ante la duda en la puncion o en la naturaleza benigna del proceso.

11) Tratamiento: por lo general la displasia mamaria (MFQ) no requiere tratamiento. Solo con
explicar a la paciente la benignidad de su cuadro la sintomatologa tiende a disminuir en
muchos de los casos.
Como tratamiento medico en casos aislados se pueden usar:
- Progestagenos: en la segunda fase del ciclo para compensar el hiperestrogenismo, en los
casos en que se asocia a este.
- Progestagenos en gel: sin mucho estudios, parecieran ser utiles para realizar tratamiento
local.
- Sintomaticos: principalmente AINES en dosis bajas y por pocos das.

351

12) Consecuencias para la mujer: en casos aislados tienden a malignizarse. Teniendo el
estudio cito-histolgico la probabilidad de malignizacin depende de el tipo,
siendo la proliferativa con atipias la con mayor riesgo, permaneciendo este bajo.
Del total de MFQ estudiadas la distribucin es la siguiente aproximadamente:
- No proliferativa (68%)
- Proliferativa sin atipia (26%)
- Proliferativa con atipia (4%)

Bibliografa:
- Prez A, Donoso E. Prez Snchez Obstetricia. Cuarta edicin (2011).
Enfermedades benignas de la mama. Capitulo 46. Paginas 818 835.
- Beckmann C. Obstetricia y Ginecologa. Sexta edicin (2010). Desorders od the
breast. Capitulo 31. Paginas 283 294.
- Rungruang B, Kellley J. Benign Breast Diseases: Epidemiology, Evaluation, and
Management. Clinical obstetrics and gynecology. Volumen 54, Numero1, 110
124 .
- Onstad M, Stuckey A. Benign Breast Disorders. Obstetrics and Gynecology Clinics of
North America. Volumen 40. Publicacin 3. Septiembre 2013, Paginas 459473.

75. PATOLOGA BENIGNA DE MAMA :


FIBROADENOMA.

Mujer de 21 aos consulta por que baando se encontr ndulo en mama izquierda
hace 7 das.
Antecedentes Familiares: no hay
Antecedentes Personales: sana
Antecedentes Gineco obsttricos: menarquia 13 aos, RM V/25 G0PA0 MAC:
ACO
Hbitos: no refiere

1.Diagnstico apropiado: Nuligesta
Nodulo mamario Izquierdo, probablemente Fibroadenoma.
2. Diagnstico diferencial:
Tumor filoides, Hamartoma.Lipoma, CA MAMA
3. Definicin:
Fibroadenoma: Son tumores benignos comunes de la mama, compuestos de
epitelio y estroma del conducto terminal de la unidad lobular ductal de la mama.

352
4. Etiologa:
No hay una etiologa definida para este cuadro
5. Fisiopatologa:
Son tumores dependientes de estrgenos, por lo que suelen presentar variaciones
con el ciclo menstrual. En la menopausia suelen disminuir de tamao.
6. Factores de riesgo:
El uso de anticonceptivos antes de los 20 aos est asociado con el riesgo de
Fibroadenoma. Raza negra tiene tendencia a desarrollar Fibroadenoma en mayor
cantidad y en edades ms jvenes de presentacin.
7. Epidemiologa:
Son ms frecuentes en mujeres jvenes de entre 20 y 40 aos de edad, aunque
pueden presentarse en mujeres de cualquier edad. (1)
8. Diagnstico clnico:
Habitualmente es un ndulo identificado por la propia paciente, generalmente
nico, blando, desplazable, indoloro y bien delimitado. Macroscpicamente es
blanco, granuloso o fasciculado, mide entre 1 y 4 cm. Entre el 10 y 20% son
bilaterales (1).
9. Anamnesis:
Presencia de otros ndulos mamarios, tiempo desde que la masa ha estado
presente, localizacin de la masa, relacin con planos profundos, dolor o
sensibilidad del ndulo con variacin hormonal, descarga por el pezn, cambios en
la piel (eritema, calor local, retraccin).
Antecedentes familiares de cncer de mama, uso de ACO previo a los 20 aos,
tabaco, exposicin a radiacin o quimioterapia.
10. Examen fsico:
Se deben buscar dirigida mente las caractersticas del ndulo: el Fibroadenoma es
de consistencia blanda, de bordes bien delimitados, indoloro, mvil, sin adherencia
a planos profundos y hasta el 20% de los casos son bilaterales. Evaluar presencia
de adenopatas axilares.
11. Estudio de la paciente:
A la ecografa de mama se evidencia ndulo lobulado o elptico, generalmente ms
ancho que alto, de contenido slido. Mamogrficamente se observa como un
ndulo de contornos bien delimitados.
El diagnstico puede confirmarse con una puncin con aguja fina.
12. Tratamiento:
En estas pacientes no es necesaria la extirpacin del Fibroadenoma en caso de
tener certeza del diagnostico, ya sea por clnica o por biopsia. Se debe mantener
control peridico con examen fsico y ecografas para descartar cambios de estas
lesiones. Se recomiendan controles cada 6 meses durante el primer ao y luego
anuales.
Tratamiento quirrgico: si hay duda diagnostica, sintomtico, postmenopausia,
historia familiar de cncer de mama, deseo de la paciente.
13. Consecuencias para la paciente:

353
El riesgo de cncer de mama posterior es levemente mayor slo si el
Fibroadenoma es complejo, si hay enfermedad proliferativa adyacente, o si hay
una historia familiar de cncer de mama (1,5 a 2 veces mayor).
Para la mayora de las mujeres con Fibroadenoma simples, no hay mayor riesgo de
desarrollar cncer de mama. (2)

Referencias
1. Benign Breast Disorders Onstad, Michaela et al. Obstetrics and Gynecology Clinics ,
Volume 40 , Issue 3 , 459 473
2. Overview of benign breast disease Michael S Sabel, MD En: Up to date (Consultado en abril de
2015)

76. CNCER DE MAMA


Mujer de 44 aos acude por que se palpo ndulo indoloro en mama izquierda hace 5
das. Al examen mamas simtricas y se palpa ndulo de 1,5 cm que se delimita mal en
mama izquierda, se palpa ndulo de 1 cm en axila izquierda.
Antecedentes Familiares: Madre Ca mama
Antecedentes Personales: No refiere
Antecedentes Gineco obsttricos: menarquia 13 aos, RM V/25 G4P3A1
Hbitos: no refiere. No se ha practicado controles desde hace 3 aos

1. Diagnstico apropiado:
Multipara de 3
Nodulo mmario, Obs Cncer de Mama

2. Diagnstico diferencial:
Los diagnsticos diferenciales de un ndulo mamario son patologa mamaria
benigna como fibroadenoma, macroqiustes, cambios fibroqusticos mamarios.
3. Definicin:
Proliferacin acelerada e incontrolada de clulas del epitelio glandular mamario
que invaden y presentan capacidad de metastizar.
4. Etiologa:
Se conocen numerosos factores que influyen en la aparicin de este cncer pero
no se ha encontrado uno claramente dominante. Se cree que los estrgenos y la
mayor exposicin a estos tienen un rol fundamental en la aparicin de este cncer.
5. Fisiopatologa:

354
El crecimiento celular es un proceso extremadamente regulado. Debido a causas
tanto exgenas como endgenas, los controles que regulan la multiplicacin
celular se alteran y una clula empieza a crecer sin control. Cuando los
descendientes de sta heredan la tendencia a crecer sin responder a regulacin
alguna, el resultado es un clon celular (teora del origen clonal) capaz de
expandirse ilimitadamente. Finalmente este clon de clulas puede formar una
masa o tumor.
No existe una causa, sino un grupo de factores cuyos efectos actan
sinrgicamente y predisponen al cncer. Pueden ser mutaciones espontneas
debidas a fallas en procesos biolgicos endgenos naturales que ocurren en la
clula como es el caso de la reparacin del ADN; por herencia, es decir, transmisin
de mutaciones en genes supresores de tumores.
Se plantea que los tumores de mama son progresivos, es decir, comienzan siendo
una alteracin de las clulas llamada hiperplasia tpica que es una lesin benigna,
luego, si se dan las condiciones podrn transformarse en una hiperplasia atpica y
de ah evolucionar a un carcinoma in situ o invasor.
En la mama existen dos estructuras primordiales: los conductos y los lobulillos. La
mayora de los carcinomas se localizan en los conductos. Existen dos tipos de
carcinomas los invasores y los in situ. Los primeros son aquellos que en su
crecimiento superan las paredes de la estructura en la que se encuentran y se
expanden a los tejidos circundantes, que llegarn a las axilas por los vasos
linfticos. Los carcinomas in situ nunca superan las paredes del sitio donde se
desarrollan.
6. Factores de riesgo:
- Factores de riesgo Mayores: mutacin BRCA 1-2, historia familiar de 1 o 2 con
cncer bilateral <50 aos en dos generaciones, cncer de mama y ovario, hombres
con cncer de mama, radioterapia en <30aos, lesiones histolgicas precursoras
(hiperplasia ductal atpica o lobulillar atpica, cncer ductal o lobulillar in situ),
antecedente personal de cncer de mama.
- Factores de riego Menores: edad, familia de 1, 2 o 3 y mayores de 60 aos,
menarquia precoz-menopausia tarda, nuliparidad o primer parto >30 aos,
enfermedad mamaria benigna proliferativa, sobrepeso despus de la menopausia,
alcohol crnico, TRH combinada.
7. Epidemiologa:
Es la enfermedad maligna no dermatolgica ms frecuente en mujeres. Su
incidencia ha ido en aumento. Se diagnostican 500.000 casos nuevos al ao, se

355
estima que 1-2 de cada 10 mujeres desarrollara cncer de mama. Este cncer es la
primera causa de muerte a nivel mundial en mujeres.
En Chile en 2008 la tasa de mortalidad era de 14,5 por 100.000 mujeres. La
incidencia nacional en 2009 fue cercana a 3.100 casos nuevos en el sistema
pblico. Ha ido aumentando el diagnostico en etapas precoces (I y II) y disminuido
en etapas tardas (IV).
8. Diagnstico clnico:

- Lesin palpable sospechosa de malignidad: masa distinta al resto del tejido
mamario con bordes irregulares, no doloroso (>1cm), consistencia firme, aspecto
slido, irregular, escaso desplazamiento, unilateral, adenopatas axilares, piel
retrada, piel de naranja, ulcerada, edema o eritema.
9. Anamnesis:
Es importante evaluar la historia de la paciente, factores de riesgo, historia
familiar,
10. Examen fsico:
Hay que buscar dirigidamente las caractersticas de la masa (consistencia,
contornos, superficie, adherencia a planos profundos, tamao, localizacin),
presencia de adenopatas ipsi y contra-laterales, axilares, supraclaviculares y en
lnea mamaria interna. A dems de examen pulmonar, abdominal.
11. Estudio de la paciente:
- Tamizaje y Screening:
- Mamografa es la base del tamizaje. Se realiza en doble proyeccin crneo-caudal y
medio-lateral-oblicua. Cada lugar presenta su propio programa de screening. El
ministerio de salud de Chile recomienda realizar seguimiento bianual a toda mujer
entre 50 y 74 aos con mamografa. Pacientes de alto riesgo realizar cribado 10
aos antes de aparicin familiar.
o BIRADS 0: Insuficiente
o BIRADS 1: Normal
o BIRADS 2: Lesiones mamarias de aspecto benigno
o BIRADS 3: Hallazgos probablemente benignos, repetir en 6 meses (<2%)
o BIRADS 4: Anormalidad sospechosa de malignidad (<20%), realizar biopsia
diagnostica.
o BIRADS 5: Alta probabilidad de malignidad (95%),realizar biopsia
o BIRADS 6: Cncer de mama diagnosticado y/o tratado en control

356
- Ecografa: mtodo complementario y fundamental para diferenciar lesiones solidas
de qusticas. Es un estudio fundamental en mujeres menores de 30 aos y con
mamas densas.
Confirmacin diagnstica:
- PAAF ( Citologia por puncion ): riesgo de falsos negativos, sensibilidad 87%, no
diferencia carcinoma in situ de invasor.
- Biopsia core: puncin son aguja gruesa, proporciona detalles histolgicos. Se usa
en ndulos slidos, palpables y no palpables sospechosos, BIRADS 4-5
- Biopsia esterotxica: obtiene un corte de tejido, diagnostica lesiones benignas y
diferencia carcinomas ductales invasivo. Se usa en microcalcificaciones, asimeti de
densidad BIRADS 4-5.
- Citologa: de telorragia
Estudio de Metastasis: presenta metstasis microscpicas desde el comienzo en
sangre y medula sea. Mayor frecuencia en pulmn, hgado, hueso y sistema
nervioso central.
- radiografa de trax, ecografa heptica, hemograma, funcin heptica,
funcin renal, y marcadores tumorales. Cintigrafa sea, tac toraco-
abdominal.
Etapificacin: es necesario para eleccin de tratamiento, estimar pronstico. Se
usa TNM determinando extensin clnica (TNMc) o por anatoma patolgica
(TNMp)
T N M
1 tumor <2cm 1 Linfonodos axilares ipsi laterales mviles 1 metstasis
2 tumor 2-5cm 2 Linfonodos axilares fijos
3 tumor >5 cm 3 Linfonodos Mamarios internos o
supraclaviculares
4 extensin a piel o pared

12. Tratamiento:
Hay distintas lneas de tratamiento y va a depender de mltiples factores cual es el
ms adecuado para la paciente. La decisin la toma el equipo mdico en conjunto con
la paciente tomando en cuenta la histologa, etapificacin, presencia de receptores
hormonales, edad de la paciente.

- Ciruga: Las distintas tcnicas incluyen la mastectoma parcial, mastectoma total,
mastectoma radical modificada, biopsia de ganglio centinela, diseccin axilar, y
reconstruccin mamaria. La mastectoma parcial significa extirpar una parte de la
mama y se realiza en tumores pequeos; la mastectoma total y radical implican

357
extirpar la mama completa y se practica en tumores grandes o en el caso que haya
ms de un tumor en la mama; la biopsia de ganglio centinela se refiere a extirpar
slo los ganglios de la axila que estn ms cerca de la mama y la diseccin axilar
consiste en extraer todos los ganglios de la axila. La reconstruccin mamaria se
puede realizar de diferentes maneras. La eleccin del tipo de ciruga se define
segn las caractersticas de la paciente y de su tumor.
- Radioterapia: Consiste en el uso de radiacin para destruir clulas cancerosas. Los
tratamientos generalmente se administran a travs de visitas diarias durante 5 6
semanas. Cada tratamiento dura unos pocos minutos, no es doloroso y no produce
cada del pelo. Se puede usar como coadyuvante a otros tratamientos.
- Quimioterapia: Consiste en el uso de drogas para destruir las clulas cancerosas.
stas se pueden administrar por va oral o intravenosa. La quimioterapia produce
efectos secundarios. La duracin de la quimioterapia depender de cada caso en
particular.
- Hormonoterapia: Es el uso de medicamentos antiestrognicos para bloquear o
disminuir los efectos de los estrgenos propios en las clulas cancerosas y se
administran por va oral. En general, son bien tolerados.

13. Consecuencias para la paciente:
La principal consecuencia es el riesgo en la vida de la mujer. En segundo lugar las
consecuencias psicologas y fsicas del cncer, y consecuencias estticas.

72. Cncer de cuello uterino



Mujer de 39 aos consulta por sinusorragia y flujo vaginal aumentado de mal olor desde
hace 3 meses. A la especuloscopa se visualiza lesin sangrante en labio anterior que se
introduce al canal vaginal.

Antecedentes personales: tonsilectoma
Hbitos: no refiere
Antecedentes familiares:
- padre infarto
- madre falleci a los 30 aos (no sabe a causa)
Antecedentes Gineco-obsttricos:
- menarquia 13 aos
- RM V/25
- G2 P2 A0
- Antecedente de PIP con tratamiento ambulatorio

358

1. Diagnostico mas probable:

Multpara de 2
Leucorrea insepecifica
Lesion cuello uterino Obs Ca Cuello.

2. Diagnsticos diferenciales:

Cervicitis infecciosa
Quiste de Naboth
Cervicitis por VIH
Plipo endocervical

3. Definicin:

Proliferacin anormal de las clulas q se encuentran en el cuello del tero

4. Etiologa del cuadro clnico:
El cncer de cuello uterino es causado por el VPH. Los de mayor riesgo son los
VPH tipo 16, 18, 31 y 45.
El VPH infecta las clulas de cuello. La zona de transformacin epitelial es la
regin donde se originan mas del 90% de los canceres de cuello uterino,
pasando de ser epitelio cilndrico (glandular) a estratificado no queratinizado.

5. Fisiopatologa:

Por accin de las hormonas y la acidificacin del medio vaginal durante la
pubertad se produce una eversin del cuello, por lo que se genera una zona
de transicin (ZT) epitelial de cilndrico a estratificado.
Las clulas metaplsicas de la ZT son las clulas mas recientes y menos
maduras del cuello uterino, y se cree que son las mas vulnerables a las
alteraciones oncognicas.
El AND del VPH se integra a las clulas de epitelio cervical, las cuales de a poco
van alterando su morfologa.

6. Factores de riesgo:

Edad media de la vida, sexualmente activas
Inicio precoz de vida sexual
Elevado numero de parejas sexuales
Tabaquismo
Bajo nivel socio-economico

359
Antecedentes de ETS
Inmunosupresin
Malnutricin
Uso prolongado (mayor de 5 aos) de anticonceptivos orales
Multipariedad

7. Epidemiologa:

6to lugar en muertes por cncer en la mujer
Mortalidad 7,5/100.000 mujeres
El 48,4% de las muertes se produce en mujeres entre 35 y 64 aos
Del cncer invasor la histologa ms frecuentes es el epidermoide (75%), y
luego el adenocarcinoma (25%)

8. Cmo hacer el diagnstico clnico?:

La sospecha diagnostica se realiza con el PAP, segn el resultado se toman las
conductas correspondientes (repetir, derivar, confirmacin diagnostica, etc)
La confirmacin diagnstica se debe realizar mediante el informe histolgico
positivo de lesin pre-invasora o cncer de cuello uterino.

9. Que es importante preguntar en la anamnesis?:

antecedentes personales de ca. de cuello uterino
antecedentes familiares de ca. de cuello uterino
sangrado vaginal anormal
sinusorragia (sangrado durante relaciones sexuales)
flujo genital de mal olor (infeccin)

10. Qu buscar dirigidamente al examen fsico?:

Especuloscopa: visualizacin de la lesin y estimacin del tamao
Tacto vaginal: evaluar la consitencia del cuello
Tacto rectvaginal: para evaluar el cuello y parametrios.

11. Cmo estudiar el cuadro? qu resultados espera encontrar en los exmenes?:

Colposcopa: espero encontrar en la regin de la lesin (de menor a mayor
grado de malignidad): epitelio blanco, punteado base, mosaico, vasos atpicos.
Al utilizar acido actico las lesiones se van a teir blancas. Su uso lugol el cuello
sano se va a teir y las lesiones no (no captan el lugol).
Biopsia: lesin de alto grado con presencia de clulas de morfologa atpica.

360
Legrado endocervical: en caso que la colposcopa sea insatisfactoria, cuando la
lesin se extiende hacia el canal endocervical, cuando no existe una lesin
identificable que explique la citologa alterada o cuando la alteracin citolgica
es una atipia glandular
Conizacin Cervical: en los casos que se sospeche una micro-invasin o
disociacin colpo-cito-histolgica.

12. Tratamiento:

Consideraciones del tratamiento:
- Evaluar el grado de la lesin
- Deseo de paridad futura (riesgo de parto prematuro y abortos por
incompetencia cervical)
- Evaluacin en conjunto de las patologas asociadas para una resolucin
definitiva. Ej miomas uterinos

En caso de ser carcinoma invasor de debe hacer tratamiento quirrgico hasta los 4
cm, tumores mayores a ese valor se tratan con radioterapia y quimioterapia.
La etapificacin es clnica, y segn eso se planifica el tratamiento (la evaluacin es
con especuloscopa, tacto vaginal con revisin de parametrios y consistencia del
cuello).
Si se sospecha compromiso de vejiga o recto se debe realizar una cistoscopia y/o
rectoscopia.
La ciruga para el cncer de cuello es la histerectoma radical, incluyendo los
parametrios, tercio superior de la vagina y los ganglios plvicos bilaterales. Una
excepcin es el cncer de cuello microinvasor, donde es vlido plantear una
histerectoma extrafascial.
El pacientes con deseo de paridad futura traquelectomia radical (extirpacin de
cuello, parametrios y ganglios, con preservacin del cuerpo del utero)

361

You might also like